Вы находитесь на странице: 1из 371

JEE SPRINT

JEE MAIN
2019
Y A
S H
K
MATHEMATICS
JEE Main 2019

MATHEMATICS
Syllabus

JEE Main Mathematics Syllabus


UNIT 1 : SETS, RELATIONS AND FUNCTIONS:

Sets and their representation; Union, intersection and complement of sets and their
algebraic properties; Power set; Relation, Types of relations, equivalence relations,
functions;. one-one, into and onto functions, composition of functions.

UNIT 2 : COMPLEX NUMBERS AND QUADRATIC EQUATIONS:

Complex numbers as ordered pairs of reals, Representation of complex numbers in the


form a+ib and their representation in a plane, Argand diagram, algebra of complex
numbers, modulus and argument (or amplitude) of a complex number, square root of a
complex number, triangle inequality, Quadratic equations in real and complex number
system and their solutions. Relation between roots and co-efficients, nature of roots,
formation of quadratic equations with given roots.

UNIT 3 : MATRICES AND DETERMINANTS:


Matrices, algebra of matrices, types of matrices, determinants and matrices of order two
and three. Properties of determinants, evaluation of determinants, area of triangles
using determinants. Adjoint and evaluation of inverse of a square matrix using
determinants and elementary transformations, Test of consistency and solution of
simultaneous linear equations in two or three variables using determinants and
matrices.

UNIT 4 : PERMUTATIONS AND COMBINATIONS:

Fundamental principle of counting, permutation as an arrangement and combination as


selection, Meaning of P (n,r) and C (n,r), simple applications.

UNIT 5 : MATHEMATICAL INDUCTION:

Principle of Mathematical Induction and its simple applications.

UNIT 6 : BINOMIAL THEOREM AND ITS SIMPLE APPLICATIONS:

Binomial theorem for a positive integral index, general term and middle term, properties
of Binomial coefficients and simple applications.

UNIT 7 : SEQUENCES AND SERIES:

Arithmetic and Geometric progressions, insertion of arithmetic, geometric means


between two given numbers. Relation between A.M. and G.M. Sum upto n terms of
special series: S n, S n2, Sn3. Arithmetico – Geometric progression.

UNIT 8 : LIMIT, CONTINUITY AND DIFFERENTIABILITY:

Real – valued functions, algebra of functions, polynomials, rational, trigonometric,


logarithmic and exponential functions, inverse functions. Graphs of simple functions.
Limits, continuity and differentiability. Differentiation of the sum, difference, product and
quotient of two functions. Differentiation of trigonometric, inverse trigonometric,
logarithmic, exponential, composite and implicit functions; derivatives of order upto two.
Rolle’s and Lagrange’s Mean Value Theorems. Applications of derivatives: Rate of change
of quantities, monotonic – increasing and decreasing functions, Maxima and minima of
functions of one variable, tangents and normals.

UNIT 9 : INTEGRAL CALCULUS:

Integral as an anti – derivative. Fundamental integrals involving algebraic,


trigonometric, exponential and logarithmic functions. Integration by substitution, by
parts and by partial fractions. Integration using trigonometric identities.

Evaluation of simple integrals of the type


Integral as limit of a sum. Fundamental Theorem of Calculus. Properties of definite
integrals. Evaluation of definite integrals, determining areas of the regions bounded by
simple curves in standard form.

UNIT 10: DIFFERENTIAL EQUATIONS:

Ordinary differential equations, their order and degree. Formation of differential


equations. Solution of differential equations by the method of separation of variables,
solution of homogeneous and linear differential equations of the type:

dy/dx+p(x)y=q(x)

UNIT 11: CO-ORDINATE GEOMETRY:

Cartesian system of rectangular co-ordinates 10 in a plane, distance formula, section


formula, locus and its equation, translation of axes, slope of a line, parallel and
perpendicular lines, intercepts of a line on the coordinate axes.

Straight lines

Various forms of equations of a line, intersection of lines, angles between two lines,
conditions for concurrence of three lines, distance of a point from a line, equations of
internal and external bisectors of angles between two lines, coordinates of centroid,
orthocentre and circumcentre of a triangle, equation of family of lines passing through
the point of intersection of two lines.

Circles, conic sections

Standard form of equation of a circle, general form of the equation of a circle, its radius
and centre, equation of a circle when the end points of a diameter are given, points of
intersection of a line and a circle with the centre at the origin and condition for a line to
be tangent to a circle, equation of the tangent. Sections of cones, equations of conic
sections (parabola, ellipse and hyperbola) in standard forms, condition for y = mx + c to
be a tangent and point (s) of tangency.

UNIT 12: THREE DIMENSIONAL GEOMETRY:


Coordinates of a point in space, distance between two points, section formula, direction
ratios and direction cosines, angle between two intersecting lines. Skew lines, the
shortest distance between them and its equation. Equations of a line and a plane in
different forms, intersection of a line and a plane, coplanar lines.

UNIT 13: VECTOR ALGEBRA:

Vectors and scalars, addition of vectors, components of a vector in two dimensions and
three dimensional space, scalar and vector products, scalar and vector triple product.

UNIT 14: STATISTICS AND PROBABILITY:

Measures of Dispersion: Calculation of mean, median, mode of grouped and ungrouped


data calculation of standard deviation, variance and mean deviation for grouped and
ungrouped data.

Probability: Probability of an event, addition and multiplication theorems of probability,


Baye’s theorem, probability distribution of a random variate, Bernoulli trials and
Binomial distribution.

UNIT 15: TRIGONOMETRY:

Trigonometrical identities and equations. Trigonometrical functions. Inverse


trigonometrical functions and their properties. Heights and Distances.

UNIT 16: MATHEMATICAL REASONING:

Statements, logical operations and, or, implies, implied by, if and only if. Understanding
of tautology, contradiction, converse and contrapositive.
Topic Index
1. LOGARITHMS
2. TRIGONOMETRIC RATIOS AND IDENTITIES
3. TRIGONOMETRIC EQUATIONS
4. PROPERTIES OF TRIANGLES
5. MATHEMATICAL REASONING

6. STATISTICS

7. QUADRATIC EQUATIONS

8. SEQUENCE AND SERIES

9. COMPLEX NUMBERS

A
10. BINOMIAL THEOREM

11. PRINCIPLE OF MATHEMATICAL INDUCTION

Y
12. PERMUTATION AND COMBINATIONS

13. PROBABILITY

H
14. SETS

15. RELATIONS

16. FUNCTIONS

S
17. INVERSE TRIGONOMETRIC FUNCTIONS

18. LIMITS

K
19. CONTINUITY

20. DIFFERENTIABILITY

21. METHODS OF DIFFERENTIATION

A
22. TANGENT AND NORMALS

23. MONOTONICITY

L
24. MAXIMA AND MINIMA

25. VECTOR ALGEBRA

26. 3D GEOMETRY

27. DETERMINANTS

28. MATRICES

29. STRAIGHT LINES

30. CIRCLES

31. PARABOLA

32. ELLIPSE

33. HYPERBOLA

34. INDEFINITE INTEGRATION

35. DEFINITE INTEGRATION

36. AREA UNDER THE CURVES

37. DIFFERENTIAL EQUATION

JEE Main 2018 and 2017 Offline and Online Paper Questions
JEE Mains Topicwise Archive Questions and Solutions
Weightage Analysis for Mathematics JEE (Main) 2002 to 2017
Total No.
Topic 2002 2003 2004 2005 2006 2007 2008 2009 2010 2011-I 2011-II 2012 2013 2014 2015 2016 2017 of
Questio
No. of Questions 75 75 75 75 40 40 35 30 30 30 30 30 30 30 30 30 30
n
Co-ordinate Geometry (2-D) 9 9 10 10 6 5 4 4 3 3 4 6 5 5 6 5 3
Straight line 4 5 4 3 2 2 1 1 1 1 2 2 2 2 1 1 1 19
Circle 1 2 4 4 2 1 1 2 1 1 1 1 1 1 2 2 0 16
Parabola 1 1 1 1 1 1 1 - 1 - - 1 1 1 2 1 0 10
Ellipse 2 1 1 1 - 1 1 - 1 - 2 1 1 1 0 1 10
Hyperbola 1 1 1 - 1 - - - - 1 - 0 0 1 1 4
Algebra 19 19 21 20 11 11 14 10 11 10 10 9 9 10 10 10 12
Complex number 3 4 3 3 2 1 1 1 2 2 - 1 1 1 1 1 1 15
Binomial Theorem 2 3 4 5 2 2 1 1 1 1 - 1 1 1 1 1 1 14
Sequence & Series 5 1 3 1 2 1 1 1 1 1 1 2 1 2 1 2 3 19
Permution & Combination 2 2 1 1 1 2 1 1 1 1 1 1 0 2 1 1 14
Quadratic equation 2 3 3 3 1 1 1 1 - - 2 - 1 2 1 1 0 11
Matries & Determinants 1 3 3 4 2 3 3 2 3 2 3 2 2 2 2 2 2 30
Probability 6 3 3 2 1 2 2 2 2 2 1 1 1 1 1 1 3 20
Mathematical induction 1 - - 1 - - - 1 0 0 0 0 2
Mathematical Reasoning - - - 2 1 1 1 1 1 1 1 1 1 1 12
Trigonometry 25 2 3 2 3 2 2 1 2 1 1 1 3 2 2 2 2
Trigo. Ratio & identities 5 2 1 2 - - 1 2 1 - 1 1 0 0 1 2 11
Trigonometric equation 2 1 - - - - - 1 - 1 1 0 1 0 3
Inverse Trigonometric function 2 1 1 - 1 1 - - - - - 1 0 1 0 0 3
Height & distance 1 1 - 1 1 - - - - - 0 1 1 0 0 2
Differential Calculus 16 13 13 13 6 7 5 7 6 6 6 5 4 5 4 5 6
Sets & Relation 1 1 1 - 1 1 1 1 1 1 1 1 1 0 0 10
Functions 4 5 4 2 1 1 1 2 - 1 1 - 0 1 0 1 1 7
Limit of function 4 3 2 2 - - - - 1 1 1 - 1 1 1 1 0 4
Continuity & Derivability 1 2 1 3 1 2 1 1 - 1 2 1 0 0 1 1 1 12
Differentiation (MOD) 3 1 1 1 - - 1 1 1 - - 1 0 0 1 1 5
Application of Derivatives (AOD) 4 2 4 5 2 4 2 2 3 1 1 3 1 2 1 1 3 26
Integral Calculus 6 6 6 5 4 6 5 5 6 5 5 8 7
Indefinite integration 1 2 1 - 1 1 - - - - 1 1 2 1 1 1 9
Definite integration 5 6 4 3 4 2 1 1 1 2 1 1 2 0 2 1 1 19
Area under curve 1 1 1 3 - 1 1 1 1 1 1 1 1 1 0 1 1 11
Differential equation 3 2 2 2 1 1 2 1 1 2 2 1 1 1 1 1 1 16
Statistics 1 2 2 3 1 1 1 2 1 1 1 1 1 1 1 1 0 13
Solution of Triangle 3 3 1 2 1 0 0 0 0 0 0 0 0 0 0 0 0 2
Exponential & Log Series 0 1 0 0 0 0 0 0 0 0 0 0 0 2
Statics & Dynamics 6 6 5 3 3 0 0 0 0 0 0 0 0 0 0 0 12
Vector 5 6 5 7 2 2 2 2 2 2 2 2 1 1 1 1 1 23
Co-ordinate Geometry (3-D) 3 6 5 4 2 3 2 1 2 2 2 2 2 2 2 2 2 25
All The Best

From

Team
MT Lakshya

Now It's time to Start your Revision ......


JEE Sprint Mathematics
LOGARITHMS
SELECT THE CORRECT ALTERNATIVE (ONLY ONE CORRECT ANSWER)

1 The value of x, satisfying the inequality log0.3(x2 + 8) > log0.39x, lies in


(A) 1 < x < 8 (B) 8 < x < 13
(C) x > 8 (D) none these

 1 
2. If a = log1/ 2 0.125 and b = log3   then
 24  17 
(A) a> 0, b > 0 (B) a < 0, b < 0
(C) a > 0, b < 0 (D) a < 0 , b > 0

3. The number log27 is

T
(A) rational number (B) irrational number
(C) a prime number (D) an integer

N
4. The value of x, satisfying 3 4log9  x 1  22log2 x  3 , is

I
(A) x = 0 (B) x = 1
(C) x = 2 (D) x = 3

R
1 1
5. The value of  is
log3  log4 

P
(A) 2 (B) less than 2
(C) more than 2 (D) none of these

S
6. The value of |logba + logab| where a and b are positive numbers is always
(A)  2 (B)  2

E
(C) = 2 (D) none of these

7. If a > 0, c > 0, b = ac , a  1, c  1, ac  1 and n > 0, then the value of

E
loga n  logb n

J
is equal to
logb n  logc n
loga n logn a
(A) (B)
logc n logn c
(C) logca (D) none of these

16 25 81
8. The value of 7 log  5 log  3 log is
15 24 80
(A) log2 (B) log 3
(C) 0 (D) none of these

9. Values of x satisfying the equation


   
log 2x 3  6x 2  23x  21  4  log 3x 7  4x 2  12x  9 are
1 1
(A) 1,  (B) 2, 
3 4
1 1
(C) 1,  (D) 2, 
4 3

Page 1
JEE Sprint Mathematics

10. Value of x, satisfying the equation


 x 
6 loga x. log10 a.loga 5 log10  
a 3  10   9log100 x  log4 2 is
5
(A) 50 (B) 100
(C) 150 (D) 200

1 1 1 1
11.    .......  is equal to
log2 n log3 n log4 n log43 n
logn 1
(A) (B)
log(43!) log 43! n

T
(C) log43! n (D) none of these

12. If log0.3(x  1) < log0.09(x  1), then x will lie in the interval

N
(A) (2, ) (B) (1, 2)

I
(C) (2, 1) (D) none of these

loga  logb a 

R
13. The value of is
logb  loga b 

P
(A) logab (B) logba
(C)  logab (D) none of these

S
14. The values of x, satisfying the equation for  a > 0
2 logx a  logax a  3 loga2 x a  0 are

E
(A) a2, a1 (B) a1/2, a1
(C) a3, a1 (D) a4/3, a1/2

E
15. The value of x, satisfying the equation

J
 3 2
log10 98  x  x  12x  36  2 is 
(A) 3 (B) 4
(C) 5 (D)  6

16. If 4log9 3  9log2 4  10logx 83 , then x is equal to


(A) 2 (B) 10
(C) 9 (D) none of these

1 1 1
17. If x > 1, y > 1, z > 1 are in G.P, then , , are in
1  ln x 1  ln y 1  ln z
(A) A.P (B) G.P
(C) H.P (D) none of these

18. 
log3 log2 log  3 
81 is equal to

(A) 2 (B) 1
(C) 3 (D) none of these

Page 2
JEE Sprint Mathematics

ln x ln y ln z
19. For   , xyz is equal to
bc c a ab
(A) 1 (B) abc
(C) 0 (D) none of these

20. The number of solution of log4 (x – 1) = log2 (x – 3) is


(A) 3 (B) 1
(C) 2 (D) 0

N T
R I
S P
J E E
Answers

1.
4.
7.
10.
A
B
A
B
2.
5.
8.
11.
A
C
A
B
3.
6.
9.
12.
B
A
B
A
13. C 14. D 15. B
16. B 17. C 18. B
19. A 20. B

Page 3
JEE Sprint Mathematics
TRIGONOMETRIC RATIOS AND IDENTITIES
SELECT THE CORRECT ALTERNATIVE (ONLY ONE CORRECT ANSWER)
   3   7 
tan  x   .cos   x   sin 3   x
 2  2   2 
1. The expression simplifies to -
   3 
cos  x   . tan   x
 2  2 
(A) (1 + cos2x) (B) sin2x (C) – (1 + cos2x) (D) cos2x
2. Exact value of cos2 73° + cos2 47° – sin2 43° + sin2 107° is equal to -
(A) 1/2 (B) 3/4 (C) 1 (D) none
sin 22  cos 8   cos158  cos 98 
3. The expression when simplified reduces to -
sin 23  cos 7   cos157  cos 97 

T
(A) 1 (B) – 1 (C) 2 (D) none
 3   3 
4. The two legs of right triangle are sin + sin    and cos – cos    . The length of its hypotenuse is
2   2 

N
(A) 1 (B) 2 (C) 2 (D) some function of 

I
a
5. If tan  = where a, b are positive reals then the value of sin sec7 + cos cosec7 is -
b

R
3 4 4 3 4 4 3 4 4
(a  b) (a  b ) (a  b) (a  b ) (a  b) (b  a ) (a  b) (a  b 4 )
3 4
(A) (B) (C) (D) –
(ab)7 / 2 (ab)7 / 2 (ab)7 / 2 (ab)7 / 2

P
sin(   )  sin(   )
6. The expression is –
cos(  )  cos(  )

S
(A) independent of  (B) independent of  (C) independent of  (D) independent of  and 
7. The tangents of two acute angles are 3 and 2. The sine of twice their difference is -
(A) 7/24 (B) 7/48 (C) 7/50 (D) 7/25

E
sin 2   sin 3   sin 4 
8. If = tan k is an identity then the value of k is equal to -
cos 2   cos 3   cos 4 

E
(A) 2 (B) 3 (C) 4 (D) 6

J
9. Exact value of cos 20° + 2 sin2 55° – 2 sin 65° is -
1
(A) 1 (B) (C) 2 (D) zero
2
10. If cos ( + ) = mcos( – ), then tan  is equal to -
1  m  1  m  1  m  1  m 
(A)   tan  (B)   tan  (C)   cot  (D)  1  m  cot 
1m 1  m  1  m 
11. If sin  + cosec  = 2, then the value of sin8 + cosec8  is equal to -
(A) 2 (B) 28 (C) 24 (D) none of these
12. If the expression 4 sin 5 cos 3 cos 2 is expressed as the sum of three sines then two of them are sin 4 and
sin 10. The third one is -
(A) sin 8 (B) sin 6 (C) sin 5 (D) sin 12
 4  3      
13. The expression, 3 sin      sin 4 (3    )  – 2 sin 6      sin 6 (5    )  when simplified is equal to -
  2    2  
(A) 0 (B) 1 (C) 3 (D) sin 4 + cos 6
1  1
14. If cos  =  a  a  then cos 3 in terms of ‘a’ =
2
1  3 1  3 1 1  3 1
(A)  a  3  (B) 4  a  3  (C)  a  3  (D) none
4  a  a  2  a

Page 4
JEE Sprint Mathematics
1 1
15. + =
cos 290 3 sin 250
2 3 4 3
(A) (B) (C) 3 (D) none
3 3
16. The product cot 123°. cot 133° . cot 137° . cot 147°, when simplified is equal to -
(A) – 1 (B) tan 37° (C) cot 33° (D) 1
1
17. Given sin B = sin (2A + B) then, tan (A + B) = k tan A, where k has the value equal to -
5
(A) 1 (B) 2 (C) 2/3 (D) 3/2
 C C A B
18. If A + B + C =  & sin  A   = k sin , then tan tan =
2 2 2 2

T
k 1 k 1 k k 1
(A) (B) (C) (D)
k 1 k 1 k 1 k
1  4 sin 10 sin 70
19. The value of the expression is –

N
2 sin 10
(A) 1/2 (B) 1 (C) 2 (D) none of these

I
20. Which of the following number ( s ) is / are rational ?
(A) sin15° (B) cos15° (C) sin15°cos15° (D) sin15°cos75°

R
21. If  and  are two positive acute angles satisfying  –  = 15°and sin = cos 2 then the value of
 +  is equal to-
(A) 35° (B) 55° (C) 65° (D) 85°

P
22. If  = 2, then -
           

S
(A) tan  tan  tan  tan tan tan (B) tan tan  tan tan  tan tan  1
2 2 2 2 2 2 2 2 2 2 2 2
           
(C) tan  tan  tan   tan tan tan (D) tan tan  tan tan  tan tan  0
2 2 2 2 2 2 2 2 2 2 2 2

E
23. The value of sin10° + sin20° + sin30° +......+ sin360° is -
(A) 1 (B) 0 (C) –1 (D) none of these
3

E
24. If A and C are two angles such that A  C  , then (1+cotA)(1+cotC) equals -
4

J
(A) 1 (B) 2 (C) -1 (D) -2
25. log t1 (4 sin 9  cos 9 ) ; where t1= 4sin63° cos63°, equals -

5 1 5 1
(A) (B) (C) 1 (D) none of these
4 4
2  y
 cot 2 x.cos 2 x  2 cos 2 
26. l  and m  a
log
a
 
, at y = 4, then l2 + m2 is equal to -
 cot 2 x  cos 2 x 
(A) 4 (B) 16 (C) 17 (D) none of these
sin(2 )
27. If (a + b) tan() = (a – b) tan(+), then is equal to -
sin(2 )
a b
(A) ab (B)
(C) (D) a 2 b 2
b a
SELECT THE CORRECT ALTERNATIVES (ONE OR MORE THAN ONE CORRECT ANSWERS)
28. If  is internal angle of n sided regular polygon, then sin is equal to -

 2  n
(A) sin (B) sin (C) sin (D) sin
n n 2n 

Page 5
JEE Sprint Mathematics
29. If sin   sin   sin   sin   ......  sec 4  , then sin is equal to -
(1  cos 2  ) (1  cos 2  )
(A) sec2 tan2 (B) 2 (C) 2 (D) cot 2  cosec 2 
(1  cos 2  ) 2 (1  cos 2  )2

30. If tan  cosec – sin  , then -
2
2 
(A) sin  2 sin 2 18  (B) cos 2   2 cos   1  0
2
2 
(C) sin  4 sin 2 18  (D) cos2 + 2cos – 1 = 0
2
3
31. If cos(A – B) = & tanAtanB = 2, then -
5

T
1 2 1 2
(A) cos A cos B  (B) sin A sin B   (C) cos(A  B )   (D) sin A sin B 
5 5 5 5
32. Factors of cos4 – cos4 are -

N
(A) (cos + cos) (B) (cos – cos ) (C) (cos + sin) (D) (cos – sin )

I
33. For the equation sin 3   cos 3   1  sin 2  -

R
 
(A) tan   1 is possible (B) cos   0 is possible (C) tan  1 is possible (D) cos  0 is possible
2 2

P
34. If 2tan10° + tan50° = 2x, tan20° + tan50° = 2y, 2tan10° + tan70° = 2w and tan20° + tan70° = 2z, then which
of the following is/are true -

S
(A) z > w > y > x (B) w = x + y (C) 2y = z (D) z + x = w + y
35. If (3 – 4sin21)(3 – 4sin23)(3 – 4sin232) ...... (3 – 4sin2(3n – 1)) = sina/sinb, where n  N & a, b are integers in
radian, then the digit at the unit place of (a + b) may be-

E
(A) 4 (B) 0 (C) 8 (D) 2

JE
ANSWER KEY
Que. 1 2 3 4 5 6 7 8 9 10 11 12 13 14 15
Ans. B C A B A C D B A C A B B C B
Que. 16 17 18 19 20 21 22 23 24 25 26 27 28 29 30
Ans. D D A B C C A B B D C B B A,B A,D
Que. 31 32 33 34 35
Ans. A,C,D A,B,C,D A,B,C A,B,C,D A,B,C,D

Page 6
JEE Sprint Mathematics
EXTRA PRACTICE QUESTIONS ON TRIGONOMETRY

SELECT THE CORRECT ALTERNATIVES (ONE OR MORE THAN ONE CORRECT ANSWERS)

1. Let m = tan 3 and n = sec 6, then which of the following statement(s) does/do not hold good ?
(A) m & n both are positive(B) m & n both are negative
(C) m is positive and n is negative (D) m is negative and n is positive

1  sin A sin A 1
2. If  , for all permissible values of A, then A belongs to -
1  sin A cos A cos A

(A) first quadrant (B) second quadrant (C) third quadrant (D) fourth quadrant

T
3
3. If  < 2 < , then 2  2  2 cos 4  equals -
2

N
(A) – 2 cos  (B) – 2 sin  (C) 2 cos  (D) 2 sin 

I
sin 3   cos 3  cos 
4. – – 2 tan cot  = – 1 if -
sin   cos  1  cot 2 

R
     3   3 
(A)    0,  (B)    ,  (C)    ,  (D)    ,2 
2 2 2 2

P
17 5
5. If sec A = and cosecB = then sec (A + B) can have the value equal to -
8 4

S
85 85 85 85
(A) (B) – (C) – (D)
36 36 84 84
6. Which of the following when simplified reduces to unity ?

E
1  2 sin 2  sin(    )
(A) (B)  cos(    )
    
2 cot     cos 2    sin   cos  tan

E
4  4  2

J
1 (1  tan 2  )2 1  sin 2 
(C) + (D)
2
4 sin  cos  2
4 tan 2  (sin   cos  )2
2
3 sin(   )  cos(   )

4 cos
7. It is known that sin = & 0 <  <  then the value of 6 is -
5 sin 
5
(A) independent of  for all  in (0, ) (B) for tan  < 0
3
3 (7  24 cot  )
(C) for tan  > 0 (D) none
15
8. In a triangle ABC, angle A is greater than angle B. If the measures of angles A and B satisfy the equation
2 tanx – k (1 + tan2x) = 0, where k  (0, 1), then the measure of the angle C is -
  5 
(A) (B) (D) (D)
6 3 12 2

sin 3  11 
9. If = then tan can have the value equal to -
sin  25 2
(A) 2 (B) 1/2 (C) – 2 (D) – 1/2

Page 7
JEE Sprint Mathematics
m m
 cos A  cos B   sin A  sin B 
10. The expression  +  where m  N, has the value -
 sin A  sin B   cos A  cos B 

 A  B
(A) 2 cotm  , if m is odd (B) 0, if m is odd
2 
 A  B
(C) 2 cotm  , if m is even (D) 0, if m is even
2 
11. If cos(A – B) = 3/5, and tanA tanB = 2, then -
1 2 1
(A) cos A cos B  (B) sin A sin B  (C) cos( A  B )  (D) none of these
5 5 5

12. If A  B  and cosA + cosB = 1, then -

T
3

2
(A) cos(A – B) = 1/3 (B) | cosA – cosB | =
3

N
1 1

I
(C) cos( A  B )   (D) | cos A  cos B | 
3 2 3
13. I f A an d B a r e a c u t e p o s i t i ve a n g l e s s at i s f y i n g t he e q u a t i o n s 3 s i n 2A + 2 s i n 2B = 1 a n d

R
3sin2A – 2sin2B = 0 then A + 2B is-
  2
(A) (B) (C) (D) none

P
3 2 3
14. If A + B – C = 3, then sinA + sinB – sinC is equal to -

S
A B C A B C A B C A B C
(A) 4 sin sin cos (B) – 4 sin sin cos (C) 4 cos cos cos (D) – 4 cos cos cos
2 2 2 2 2 2 2 2 2 2 2 2
15. 2 sin11° 15' is equal to –

E
2 2 2 2 2 2
(A) 2 2 2 (B) 2 2 2 (C) (D)
2 2

JE
16. If tan 3   cot 3   52 , then the value of tan 2   cot 2  is equal to -
(A) 14 (B) 15 (C) 16 (D) 17
2
17. If 60° +  & 60° –  are the roots of sin x + bsinx + c = 0, then -
(A) 4b2 + 3 = 12c (B) 4b + 3 = 12 c (C) 4b2 – 3 = –12c (D) 4b2 – 3 = 12c
18. If  B 1 OA 1 = 60° & radius of bigge st circle is r. According to figure trapezium A 1 B 1 D 1 C 1 , C 1 D 1 D 2 C 2 ,
C2D2D 3C 3........ and so on are obtained. Sum of areas of all the trapezium is -
B1

r 2
9r 2 D1
(A) (B) D2
D3
2 3 2 3
O
C3
9r 2 r2 C2
(C) (D) C1
3 9 3
A1

1 1
19. If    are acute angles & sin   , cos   , then the value of    belongs to the interval -
2 3

     2   2 5   5 
(A)  ,  (B)  ,  (C)  ,  (D)  , 
3 2 2 3   3 6   6 

Page 8
JEE Sprint Mathematics
20. The maximum value of log 20 (3sinx – 4cosx + 15) -
(A) 1 (B) 2 (C) 3 (D) 4
21. If x2 + y2 = 9 & 4a2 + 9b2 = 16, then maximum value of 4a2x2 + 9b2y2 – 12abxy is -

(A) 81 (B) 100 (C) 121 (D) 144


22. Let A,B,C are 3 angles such that cosA + cosB + cosC = 0 and if cosA cosB cosC = (cos3A + cos3B + cos3C),
then  is equal to -

1 1 1 1
(A) (B) (C) (D)
3 6 9 12

sin x cos x
23. ƒ(x)   is constant in which of following interval -
2
1  tan x 1  cot 2 x

T
     3   3 
(A)  0,  (B)  ,   (C)  , (D)  ,2  
 2 2   2   2 

N
n
24. Let n be an odd integer. If sin n    b r sin r  , for every value of  , then -

I
r 0
(A) b 0  1, b1  3 (B) b 0  0, b1  n
(C) b 0  1, b1  n (D) b 0  0, b1  n 2  3n  3

R
 
25. For a positive integer n, let fn ( )   tan  (1  sec  )(1  sec 2  )(1  sec 4 )....(1  sec 2 n  ) . Then [JEE 99, 3M]
 2

P
        
(A) f2    1 (B) f3    1 (C) f4    1 (D) f5  1
 16   32   64   128 

E S
JE
ANSWER KEY
Que. 1 2 3 4 5 6 7 8 9 10
Ans. A, B, C A,D D B A,B,C,D A , B , D D D A,B,C,D B,C
Que. 11 12 13 14 15 16 17 18 19 20
Ans. A,C B,C B D A A D C B A
Que. 21 22 23 24 25
Ans. D D B,D B A,B, C,D

Page 9
JEE Sprint Mathematics
TRIGONOMETRIC EQUATIONS

SELECT THE CORRECT ALTERNATIVE (ONLY ONE CORRECT ANSWER)

sec x 1
1. The number of solutions of the equation  in [0, 2] is equal to -
1  cos x 1  cos x
(A) 3 (B) 2 (C) 1 (D) 0
2. The number of solutions of equation 2 + 7tan2 = 3.25 sec 2  (0° <  < 360°) is -
(A) 2 (B) 4 (C) 6 (D) 8
3. The number of solutions of the equation tan2x – sec10x + 1 = 0 in (0, 10) is -

(A) 3 (B) 6 (C) 10 (D) 11

T
4. If (cos + cos2)3 = cos3 + cos32 then the least positive value of  is equal to -

   
(A) (B) (C) (D)

N
6 4 3 2

I
5. The number of solution(s) of sin2x + cos4x = 2 in the interval (0, 2) is -
(A) 0 (B) 2 (C) 3 (D) 4
2
6. The complete solution of the equation 7cos x + sin x cos x – 3 = 0 is given by -

R
 
(A) n   ; (n   ) (B) n   ; (n   )
2 4

P
14 3 4
(C) n   tan ; (n   ) (D) n   , k   tan 1 ; (n, k   )
3 4 3

S
7. If cos(sinx) = 0, then x lies in -

     FG   , 0IJ FG  , 3 IJ
(A)  ,    ,   (B) H 4 K (C) H 2K (D) null set

E
4 2  2 
8. If 0     90° and tan(  ) = 3 and tan( – ) = 2 then value of sin2 is -

E
1 1 1
(A)  (B) (C) (D) none of these

J
2 2 2
9. 2 2
If tanA and tanB are the roots of x –2x –1 = 0, then sin (A+B) is -

1 1
(A) 1 (B) (C) (D) 0
2 2

cosec x
10. If cos2x – 3cosx + 1 = , then which of the following is true ?
cot x  cot 2x

(A) x  (2n  1) , n  I (B) x  2n  , n  
2
1  2 
(C) x  2n   cos   , n  I (D) no real x
5
11. The solutions of the equation sinx + 3sin2x + sin3x = cosx + 3cos2x + cos3x in the interval 0 < x < 2, are ;
 5 2  5  9  13  4  9  2  13   5 9 4
(A) , , (B) , , , (C) , , , (D) , , ,
8 8 3 8 8 8 8 3 3 3 8 8 8 3 3
 5 5 
12. If x    ,  , then the greatest positive solution of 1 + sin4 x = cos2 3x is -
 2 2 
5
(A)  (B) 2 (C) (D) none of these
2

Page 10
JEE Sprint Mathematics
2 2
x x
13. Number of values of ‘x’ in (– 2, 2) satisfying the equation 2 sin + 4.2 cos = 6 is -
(A) 8 (B) 6 (C) 4 (D) 2
14. General solution for |sin x| = cos x is -

  
(A) 2n + ,n  I (B) 2n ± , n  I (C) n + ,nI (D) none of these
4 4 4

1
15. The most general solution of tan = –1, cos = is -
2

7 n 7 7
(A) n  , n  (B) n  ( 1) , n  (C) 2n  , n  (D) none of these

T
4 4 4

SELECT THE CORRECT ALTERNATIVES (ONE OR MORE THAN ONE CORRECT ANSWERS)

N
16. The solution(s) of the equation cos2x sin6x = cos3x sin5x in the interval [0, ] is/are -

I
  2 5
(A) (B) (C) (D)
6 2 3 6

R
2
17. The equation 4 sin x  2( 3  1) sin x  3  0 has -

P
(A) 2 solutions in (0, ) (B) 4 solutions in (0, 2) (C) 2 solutions in (– ) (D) 4 solutions in (– )
18. If cos 22x + 2cos 2x = 1, x  (– ), then x can take the values -

S
  3
(A)  (B)  (C)  (D) none of these
2 4 4
19. The solution(s) of the equation sin 7x + cos 2x = –2 is/are -

E
2k  3   
(A) x   , k I (B) x  n   , n I (C) x  2n   , n  I (D) none of these
7 14 4 2

JE
20. Set of values of x in ( ) for which |4sinx – 1|< 5 is given by -

  3    3    3    3 
(A)  ,  (B)   ,  (C)  ,   (D)   ,  
 10 10   10 10   10 10   10 10 

ANSWER KEY
Que. 1 2 3 4 5 6 7 8 9 10
Ans. D B A B A D D B C D
Que. 11 12 13 14 15 16 17 18 19 20
Ans. B B C B C A,B,D B,D A, B, C C B

Page 11
JEE Sprint Mathematics
EXTRA PRACTICE QUESTIONS ON TRIGONOMETRIC EQUATIONS

SELECT THE CORRECT ALTERNATIVES (ONE OR MORE THAN ONE CORRECT ANSWERS)

1. If cos 2 x + cos 2 2x + cos 2 3x = 1 then -

  
(A) x = (2n + 1) ,nI (B) x = (2n + 1) ,nI (C) x = n± ,nI (D) none of these
4 2 6

2. If 4cos2 + 3 = 2( 3 + 1)cos, then  is -


  
(A) 2n  ,nI (B) 2n  ,nI (C) 2n  ,nI (D) none of these
3 4 6
Set of values of '' in [0, 2] for which m  log F
GH x  1x IJK
3. (2 sin   1)  0 , is -

T
LM  , 5 OP FG  , 5 IJ FG 0 ,  IJ  FG 5 , IJ FG 5 , 7 IJ
(A)
N6 6 Q (B) H6 6 K (C) H 6K H 6 K (D) H 6 6K

N
4. If (a + 2)sin + (2a – 1)cos = (2a + 1), then tan =

I
2a 2a
(A) 3/4 (B) 4/3 (C) (D)
a2  1 a2  1

R
5. If 1, 2, 3, 4 are the roots of the equation sin(  ) = k sin2, no two of which differ by a multiple of
2, then  1 +  2 +  3 +  4 is equal to -

P
(A) 2n, n  Z (B) (2n + 1), n  Z (C) n, nZ (D) none of these

e j FGH 1 IJ , in the interval FG   , 3 IJ , is -


2K H 4 4K

S
6. The number of solution(s) of the equation cos 2  2  1 cos  

(A) 4 (B) 1 (C) 2 (D) 3


7. The value(s) of  lying between 0 & 2 satisfying the equation : r sin  = 3 & r + 4sin = 2( 3 + 1)

E
is/are -
  2 5

E
(A) (B) (C) (D)
6 3 3 6

J
8. The value(s) of , which satisfy 3 – 2cos – 4sin – cos2 + sin2 = 0 is/are -
 
(A)  = 2n ; n  I (B) 2n + ; n  I (C) 2n – ; n  I (D) n ; n  I
2 2
3 1
9. Given that A, B are positive acute angles and 3 sin2A = sin2B & 3 sin2A + sin2B = , then A or B
2
may take the value(s) -
(A) 15° (B) 30° (C) 45° (D) 75°
10. 2 2
The solution(s) of 4cos x sinx – 2sin x = 3sinx is/are -

(A) n ; n  I (B) n + (–1) n ; n  I
10

(C) n + (–1) n   3   ; n  I (D) none of these


 10 

 1  a sin x  1  2a sin x
11. If   = 1, where a  R then -
 1  a sin x  1  2a sin x

(A) x   (B) x  R  a

(C) a = 0, x  R (D) a  R, x  n, where n  I

Page 12
JEE Sprint Mathematics
12. The general solution of the following equation : 2(sinx – cos2x) – sin2x(1 + 2sinx) + 2cosx = 0 is/are -

 
(A) x = 2n ; n  I (B) n + (–1) n    ; n  I
 2

 
(C) x = n + (–1) n ; n  I (D) x = n + (–1) n ; n  I
6 4

13. The value(s) of , which satisfy the equation : 2cos 33 + 3cos3 + 4 = 3sin 23 is/are -

2n  2  2n  2  2n  2  2n  2 
(A)  , nI (B)  , nI (C)  , n I (D)  ,n I
3 9 3 9 5 5 5 5

k 2

T
14. If x  , k  I and (cos x) sin x  4 sin x  3  1 , then all solutions of x are given by -
2

n   
(A) n   ( 1) ;nI (B) 2n   ;nI (C) (2n+1)  ; n  I (D) none of these

N
2 2 2

I
15. Using four values of  satisfying the equation 8 cos4 + 15 cos2 – 2 = 0 in the interval (0,4), an arithmetic

progression is formed, then :

R
(A) The common difference of A.P. may be  (B) The common difference of A.P. may be 2

(C) Two such different A.P. can be formed. (D) Four such different A.P. can be formed.

S P
JE E
ANSWER KEY
Que. 1 2 3 4 5 6 7 8 9 10
Ans. A,B,C A,C B B,D B C A,B,C,D A,B A,B A,B,C
Que. 11 12 13 14 15
Ans. C,D A, B, C A,B D A,D

Page 13
JEE Sprint Mathematics
PROPERTIES OF TRIANGLES

SELECT THE CORRECT ALTERNATIVE (ONLY ONE CORRECT ANSWER)

c2  a 2
1. In a triangle A  55 and B  15 , then is equal to -
ab
(A) 4 (B) 3 (C) 2 (D) 1
2. In a triangle ABC a : b : c = 3 : 1 : 1, then the triangle is -
(A) right angled triangle (B) obtuse angled triangle
(C) acute angled triangle, which is not isosceles (D) Equilateral triangle

T
2 2
3. The sides of a triangle ABC are x, y, x  y  xy respectively. The size of the greatest angle in radians
is -

N
2  
(A) (B) (C) (D) none of these
3 3 2

I
 a2 b2 c2  A B C
4. In a ABC     . sin sin sin simplifies to -
 sin A sin B sin C  2 2 2

R
 
(A) 2 (B)  (C) (D)
2 4

P
(where  is the area of triangle)
5. If p 1, p 2, p 3 are the altitudes of a triangle from its vertices A, B, C and , the area of the triangle ABC,

S
1 1 1
then + – is equal to -
p1 p2 p 3

s sc sb sa

E
(A) (B) (C) (D)
   
6. If in a triangle ABC angle B = 90° then tan 2 A/2 is -

E
b c b c b c b c

J
(A) (B) (C) (D)
a b c b c a

a3  b3  c3
7. In a triangle ABC, if  343 , the diameter of the circle circumscribing the triangle
sin A  sin 3 B  sin 3 C
3

is -
(A) 7 units (B) 14 units (C) 21 units (D) none of these
B C
8. In a ABC if b + c = 3a then cot . cot has the value equal to -
2 2
(A) 4 (B) 3 (C) 2 (D) 1
a
9. If = K, then the area of ABC in terms of K and sines of the angles is -
sin A
K2 K2
(A) sinAsinB sinC (B) sinA sinB sinC
4 2
(C) 2K 2 sinA sinBsin(A + B) (D) none

10. In a  ABC, C  60  & A  75  . If D is a point on AC such that the area of the  BAD is 3
times the area of the  BCD, then the ABD =
(A) 60 ° (B) 30 ° (C) 90° (D) none of these

Page 14
JEE Sprint Mathematics
11. In a ABC, a semicircle is inscribed, whose diameter lies on the side c. Then the radius of the semicircle
is (Where  is the area of the triangle ABC)

2 2 2 c
(A) (B) (C) (D)
ab a bc s 2
12. In a triangle ABC, right angled at B, the inradius is -

AB  BC  AC AB  AC  BC AB  BC  AC
(A) (B) (C) (D) none
2 2 2

13. In triangle ABC where A, B, C are acute, the distances of the orthocentre from the sides are in the proportion
(A) cos A : cos B : cos C (B) sin A : sin B : sin C

T
(C) sec A : sec B : sec C (D) tan A : tan B : tan C

a cos A  b cos B  c cos C


14. In a ABC, the value of is equal to -
a b c

I N
r R R 2r
(A) (B) (C) (D)
R 2r r R
15. If the orthocentre and circumcentre of a triangle ABC be at equal distances from the side BC and lie on

R
the same side of BC then tanB tanC has the value equal to -

1 1

P
(A) 3 (B) (C) – 3 (D) –
3 3

S
16. In an equilateral triangle, inradius r, circumradius R & ex-radius r1 are in -
(A) A.P. (B) G.P. (C) H.P. (D) none of these

1 1 1 1 1 1 K R3

E
17. With usual notation in a ABC     r  r   r  r   2 2 2 then K has value equal to -
 r1 r2   2 3  3 1 a b c
(A) 1 (B) 16 (C) 64 (D) 128

E
r1  r2

J
18. In a triangle ABC, is equal to -
1  cos C
(A) 2ab/c (B) (a + b)/c (C) abc/2 (D) abc/ 2
19. With usual notations in a triangle ABC, if r 1 = 2r 2 = 2r 3 then -
(A) 4a = 3b (B) 3a = 2b (C) 4b = 3a (D) 2a = 3b

20. If r 1, r 2, and r 3 be the radii of excircles of the triangle ABC, then


r 1
is equal to -
r r
1 2

A A B A A
(A)  cot 2 (B)  cot 2 cot 2 (C)  tan 2 (D)  tan 2
21. If in a triangle PQR, sin P, sin Q, sin R are in A.P., then -

(A) the altitudes are in A.P. (B) the altitudes are in H.P.

(C) the medians are in G.P. (D) the medians are in A.P.

22. In ABC, if r : r 1 : R = 2 : 12 : 5, where all symbols have their usual meaning, then -

(A) ABC is an acute angled triangle (B) ABC is an obtuse angled triangle

(C) ABC is right angled which is not isosceles (D) ABC is isosceles which is not right angled

Page 15
JEE Sprint Mathematics

 3 
23. In a ABC, A = and b : c = 2 : 3. If tan   , 0    , then -
3 5 2
(A) B  60    (B) C  60    (C) B  60    (D) C  60   
24. In a triangle ABC, points D and E are taken on sides BC such that DB = DE = EC. If ADE = AED = ,
then -
6 tan  2 A
(A) tan = 3tanB (B) tan = 3tanC (C) tan A  (D) 9 cot  tan 2 
tan 2   9 2
25. If a, b, A are given in a triangle and c1 and c2 are two possible values of third side such that c12  c1 c 2  c 22  a 2 ,
then A is equal to -
(A) 30° (B) 60° (C) 90° (D) 120°

T
26. In a ABC, AD is the bisector of the angle A meeting BC at D. If I is the incentre of the triangle, then
AI : DI is equal to -
(A) (sinB + sinC) : sinA (B) (cosB + cosC) : cosA

N
B C  B C  B C  B C

I
(C) cos   : cos  2  (D) sin   : sin  2 
 2     2   

P R
E S
JE
ANSWER KEY
Que. 1 2 3 4 5 6 7 8 9 10
Ans. D B A B B B A C B B
Que. 11 12 13 14 15 16 17 18 19 20
Ans. A A C A A A C C C C
Que. 21 22 23 24 25 26
Ans. B C B,C A,B, C,D B A,C

Page 16
JEE Sprint Mathematics
EXTRA PRACTICE QUESTIONS ON PROPERTIES OF TRIANGLES

SELECT THE CORRECT ALTERNATIVES (ONE OR MORE THAN ONE CORRECT ANSWERS)
1. If A, B, C are angles of a triangle which of the following will not imply it is equilateral -
(A) tanA + tanB + tanC = 33 (B) cotA + cotB + cotC = 3
(C) a + b + c = 2R (D) a2 + b2 + c2 = 9R2
s
2. In a ABC, is equal to -
R
A B C s
(A) sinA + sinB + sinC (B) 4 cos cos cos (C) 4sinA sinBsinC (D)
2 2 2 abc
3. If cosA + cosB + 2cosC = 2 then the sides of the ABC are in-

T
(A) A.P. (B) G.P. (C) H.P. (D) none

x y A
4. The line   1 cuts the co-ordinate axis at A & B. If O is origin, then  sin for the triangle OAB is -

N
6 8 2

I
(A) 5/6 (B) 1/10 (C) 5/4 (D) none of above

C 1 1 1 
5. In a triangle ABC, CD is the bisector of the angle C. If cos has the value and (CD) = 6, then   

R
2 3 a b
has the value equal to -

P
1 1 1
(A) (B) (C) (D) none
9 12 6

S
6. In the triangle ABC, CH and CM are the lengths of the altitude and median to the base AB. If a = 10,
b = 26, c = 32 then length HM is -

(A) 5 (B) 7 (C) 9 (D) none

E
7. D, E, F are the foot of the perpendiculars from vertices A, B, C to sides BC, CA, AB respectively, and H is the

E
orthocentre of acute angled triangle ABC; where a, b, c are the sides of triangle ABC, then

J
(A) H is the incentre of triangle DEF
(B) A, B, C are excentres of triangle DEF
(C) Perimeter of DEF is acosA + bcosB + c cosC

R
(D) Circumradius of triangle DEF is , where R is circumradius of ABC.
2
abc
8. If x, y and z are the distances of incentre from the vertices of the triangle ABC respectively then is
xyz
equal to -

A A A A
(A)  tan 2 (B)  cot 2 (C)  tan 2 (D)  cot 2
9. The medians of a ABC are 9 cm, 12 cm and 15 cm respectively. Then the area of the triangle is -

(A) 96 sq cm (B) 84 sq cm (C) 72 sq cm (D) 60 sq cm

10. In an isosceles ABC , if the altitudes intersect on the inscribed circle then the cosine of the vertical angle ‘A’ is :

1 1 2
(A) (B) (C) (D) none
9 3 3

Page 17
JEE Sprint Mathematics
11. In triangle ABC, cosA + 2cosB + cosC = 2, then -

A C A C A C B A C
(A) tan tan  3 (B) cot cot  3 (C) cot  cot  2 cot (D) tan tan  0
2 2 2 2 2 2 2 2 2
12. If in a triangle ABC p, q, r are the altitudes from the vertices A, B, C to the opposite sides, then which
of the following does not hold good ?

 1  1  1  1
(A) (p)    = (a)    (B) (p) (a) =      
 p   a  p  a
 1 1 1 1
(C) ( p) (pq) (a) = ( a) (ab) (p) (D)         a 2 = 16R 2
 p p q r 
13. AD, BE and CF are the perpendiculars from the angular points of a ABC upon the opposite sides. The

T
perimeters of the DEF and ABC are in the ratio -
2r r r r
(A) (B) (C) (D)
R 2R R 3R

N
Where r is the inradius and R is circum-radius of the ABC

I
14. If ‘O’ is the circum centre of the ABC and R1, R2 and R3 are the radii of the circumcircles of triangles OBC,
a b c
OCA and OAB respectively then R  R  R has the value equal to -

R
1 2 3

abc 3 4 abc
R
(A) 3 (B) (C) 2 (D)
2R abc R R3

P
15. In a triangle ABC, (r1 – r) (r2 – r) (r3 – r) is equal to -

S
4 abc. 
(A) 4Rr2 (B)
(a  b  c ) 2

E
3 A B C
(C) 16R3(cosA + cosB + cosC – 1) (D) r cos ec cos ec cos ec
2 2 2

E
16. Two rays emanate from the point A and form an angle of 43° with one another. Lines L 1, L 2 and L 3 (no
two of which are parallel) each form an isosceles triangle with the original rays. The largest angle of the

J
triangle formed by lines L 1, L 2 and L 3 is -
(A) 127° (B) 129° (C) 133° (D) 137°

ANSWER KEY
Que. 1 2 3 4 5 6 7 8 9 10
Ans. C A,B A B A C A,B, C,D B,D C A
Que. 11 12 13 14 15 16
Ans. B,C B C C,D A,B,D B

Page 18
JEE Sprint Mathematics
MATHEMATICAL REASONING

1. The inverse of the statement (p  ~q)  r is- 13 If statements p, q, r have truth values T, F, T
(1) ~(p ~q) ~r (2) (~p q) ~r respectively then which of the following statement
(3) (~p q) ~r (4) None of these is true-
(1) (p q) r (2) (p q) ~r
2. (~p  ~q) is logically equivalent to-
(3) (p q) (q r) (4) (p q) r
(1) p q (2) ~p q (3) p ~q (4) ~p ~q
14. If statement p (q r) is true then the truth values
3. The equivalent statement of (p  q) is-
of statements p, q, r respectively-
(1) (p q) (p q) (2) (p q) (q p) (1) T, F, T (2) F, T, F
(3) (~p q) (p ~q) (4) (~p q) (p ~q) (3) F, F, F (4) All of these
4. If the compound statement p  (~p  q) is false 15. Which of the following statement is a contradiction-

T
then the truth value of p and q are respectively- (1) (p q) (~(p q)) (2) p (~p q)
(1) T, T (2) T, F (3) F, T (4) F, F (3) (p q) p (4) ~p ~q
5. The statement (p  ~p)  (~p  p) is- 16. The negative of the statement "If a number is

N
(1) a tautology divisible by 15 then it is divisible by 5 or 3"

I
(2) a contradiction (1) If a number is divisible by 15 then it is not divisible
by 5 and 3
(3) neither a tautology nor a contradiction
(2) A number is divisible by 15 and it is not divisible

R
(4) None of these
by 5 or 3
6. Negation of the statement (p  r)  (r  q) is- (3) A number is divisible by 15 or it is not divisible
(1) ~(p r) ~(r q) (2) (~p ~r) (r q)

P
by 5 and 3
(3) (p r) (r q) (4) (p r) (~r ~q) (4) A number is divisible by 15 and it is not divisible
7. The dual of the statement ~p [~q (p q)  ~r] by 5 and 3

S
is- 17. Which of the following is a statement-
(1) ~p [~q (p q) ~r] (1) Open the door
(2) p [q (~p ~q) r] (2) Do your home work

E
(3) ~p [~q (p q) ~r] (3) Hurrah! we have won the match
(4) Two plus two is five
(4) ~p [~q (p q) ~r]

E
18. The negation of the statement "2 + 3 = 5 and
8. Which of the following is correct-
8 < 10" is-

J
(1) (~p ~q) (p q)
(1) 2 + 3  5 and 8  10 (2) 2 + 3 5 or 8 > 10
(2) (p q) (~q ~p) (3) 2 + 3 5 or 8  10 (4) None of these
(3) ~(p ~q) (p ~q) 19. For any three simple statement p, q, r the statement
(4) ~(p  q) (p q) (q p) (p q) (q r) is true when-
9. The contrapositive of p  (~q  ~r) is- (1) p and r true and q is false
(1) (~q r) ~p (2) (q r) ~p (2) p and r false and q is true
(3) (q ~r) ~p (4) None of these (3) p, q, r all are false
10. The converse of p  (q  r) is- (4) q and r true and p is false
20. Which of the following statement is a tautology-
(1) (q ~r) p (2) (~q r) p
(1) (~p ~q) (p ~q) (2) (~p ~q) (p ~q)
(3) (q ~r) ~p (4) (q ~r) p
(3) ~p (~p ~q) (4) ~q (~p ~q)
11. If p and q are two statement then (p  ~q) is true
21. Which of the following statement is a contradiction-
when -
(1) (~p ~q) (p ~q) (2) (p q) (p ~q)
(1) p and q both are true (2) p and q both are false
(3) (~p q) (~q) (4) (~p q) (~q)
(3) p is false and q is true (4) None of these 22. The negation of the statement q  (p  ~r) is
12. Statement (p  q)  p is- equivalent to-
(1) a tautology (2) a contradiction (1) ~q (p r) (2) ~q ~(p r)
(3) neither (1) nor (2) (4) None of these (3) ~q (~p r) (4) None of these

Page 19
JEE Sprint Mathematics
23. Which of the following is not a statement- 30. If p is any statement, t is a tautology and c is a
(1) every set is a finite set contradiction then which fo the following is not
(2) every square is a rectangle correct-

(3) The sun is a star (1) p (~c) p


(4) Shut the window (2) p (~t) p
24. The statement ~(p  q)  (~p  ~q) is- (3) t c p t
(1) a tautology (4) (p t) (p c) (t c)
(2) a contradiction 31. If p, q, r are simple statement with truth values
(3) neither a tautology nor a contradiction T, F, T respect ively then the tr uth value of
((~p q) ~r) p is-
(4) None of these
(1) True (2) False
25. Which of the following is equivalent to (p  q)
(3) True if r is false (4) True if q is true

T
(1) p ~q (2) ~(~p  ~q)
(3) ~(p  ~q) (4) None of these 32. Which of the following is wrong-

26. The dual of the following statement "Reena is (1) p  ~p is a tautology

N
healthy and Meena is beautiful" is-
(2) ~(~p)  p is a tautology

I
(1) Reena is beaufiful and Meena is healthy
(3) p  ~p is a contradiction
(2) Reena is beautiful or Meena is healthy
(3) Reena is healthy or Meena is beutiful (4) ((p  p)  q) p is a tautology

R
(4) None of these 33. 2
The statement "If 2 = 5 then I get first class" is
27. If p is any statement, t and c are a tautology and a logically equivalent to-

P
contradiction respectively then which of the
(1) 22 = 5 and I donot get first class
following is not correct-

S
(1) p t p (2) p c c (2) 22 = 5 or I do not get first class
(3) p t c (4) p c p
(3) 22  5 or I get first class
28. If S*(p, q) is the dual of the compound statement

E
S(p, q) then S*(~p, ~q) is equivalent to- (4) None of these
(1) S(~p, ~q) (2) ~S(p, q) 34. If statement (p ~r) (q r) is false and statement

E
(3) ~S*(p, q) (4) None of these q is true then statement p is-

J
29. Which of the following is a statement-
(1) true (2) false
(1) I am Lion
(2) Logic is an interesting subject (3) may be true or false (4) None of these
(3) A triangle is a circle and 10 is a prime number 35. Which of the following statement are not logically
(4) None of these equivalent-
(1) ~(p ~q) and (~p q) (2) ~(p q) and (p ~q)
(3) (p q) and (~q ~p) (4) (p q) and (~p q)

ANSWER KEY
Que. 1 2 3 4 5 6 7 8 9 10 11 12 13 14 15
Ans. 3 3 4 2 2 4 3 2 1 1 3 1 4 4 1
Que. 16 17 18 19 20 21 22 23 24 25 26 27 28 29 30
Ans. 4 4 3 4 1 3 1 4 3 3 3 3 2 3 4
Que. 31 32 33 34 35
Ans. 1 4 3 3 4

Page 20
JEE Sprint Mathematics
STATISTICS
SELECT THE CORRECT ALTERNATIVE (ONLY ONE CORRECT ANSWER)
12. In a frequency dist. , if di is deviation of variates
1. Mean of the first n terms of the A.P. fid i
a, (a + d), (a + 2d), ........ is- from a number  and mean =  + , then  is :-
fi
nd (n  1)d (1) Lower limit (2) Assumed mean
(1) a  (2) a  (3) Number of observation(4) Class interval
2 2
(3) a + (n – 1) d (4) a + nd 13. The A.M. of n observation is x . If the sum of n – 4
2. The A.M. of first n even natural number is - observations is K, then the mean of remaining
n+1 n observations is-
(1) n(n + 1) (2) (3) (4) n + 1
2 2 x K nx  K
(1) (2)

T
3. n n n
The A.M. of C0 , C1 , C2, ..... Cn is - n 4 n 4
nx  K nx  (n  4)K
2n 2 n+1 2n 2 n+1 (3) (4)
4 4
(1) (2) (3) (4)
n n n+1 n+1

N
1 1 1
14. The mean of values 1, , ,...... which have
4. If the mean of numbers 27, 31, 89, 107, 156 is 82, 2 3 n

I
then the mean of numbers 130, 126, 68, 50, 1 frequencies 1, 2, 3, ........ n resp., is :-
will be- 2n  1 2 n 1 2
(1) 80 (2) 82 (3) 75 (4) 157 (1) (2) (3) (4)

R
3 n 2 n 1
5. If the mean of n observations x1, x2, .......xn is x , 15. The sum of squares of deviation of variates from
then the sum of deviations of observations from their A.M. is always :-

P
mean is :- (1) Zero (2) Minimum
(3) Maximum (4) Nothing can be said
(1) 0 (2) nx 16. If the mean of following feq. dist. is 2.6, then the

S
x value of f is :-
(3) (4) None of these
n xi 1 2 3 4 5
6. The mean of 9 terms is 15. if one new term is
fi 5 4 f 2 3

E
added and mean become 16, then the value of
new term is :- (1) 1 (2) 3
(1) 23 (2) 25 (3) 27 (4) 30 (3) 8 (4) None of these

E
7. If the mean of first n natural numbers is equal to 17. The weighted mean (W.M.) is computed by the
formula ?

J
n 7
, then n is equal to- x i wi
3
(1) W.M. = (2) W.M. 
(1) 10 (2) 11 w i x i
(3) 12 (4) none of these
w i x i w i x i
8. The mean of first three terms is 14 and mean of (3) W.M. = (4) W.M.=
next two terms is 18. The mean of all the five terms x i w i
is- 18. The weighted mean of first n natural numbers when
(1) 15.5 (2) 15.0 (3) 15.2 (4) 15.6 their weights are equal to corresponding natural
number, is :-
9. If the mean of five observations x, x + 2 , x + 4,
n 1 2n  1
x+ 6 and x + 8 is 11, then the mean of last three (1) (2)
obsevations is- 2 3
(1) 11 (2) 13 (3) 15 (4) 17 (n  1)(2n  1)
(3) (4) None of these
10. The mean of a set of numbers is x . If each number 6
is decreased by , the mean of the new set is- 19. The average income of a group of persons is x
(1) x (2) x   (3)   x (4) x   and that of another group is y . If the number of
11. The mean of 50 observations is 36. If its two persons of both group are in the ratio 4 : 3, then
average income of combined group is :-
observations 30 and 42 are deleted, then the mean
x+ y 3x + 4y
of the remaining observations is- (1) (2)
7 7
(1) 48 (2) 36
4x + 3y
(3) 38 (4) none of these (3) (4) None of these
7
Page 21
JEE Sprint Mathematics
20. In a group of students, the mean weight of boys is 28. A boy goes to school from his home at a speed of
65 kg. and mean weight of girls is 55 kg. If the x km/hr. and comes back at a speed of y km/hr.
mean weight of all students of group is 61 kg, then then the average speed of the boy is :-
the ratio of the number of boys and girls in the xy
group is :- (1) km / hr (2) xy km / hr
2
(1) 2 : 3 (2) 3 : 1 (3) 3 : 2 (4) 4 : 3
2xy xy
Geometric mea n, Harmonic mea n (3) km / hr (4) km / hr
xy 2xy
21. The G.M. of n positive terms x1, x2,..... xn is :- Median, Mode

1 29. The median of an arranged series of n even


(1) (x1 × x2 × ..... × xn)n (2) (x × x2 × ... × xn) observations, will be :-
n 1
(3) (x1 × x2 × ... × xn)1/n (4) None of these
 n  1

T
(1)   th term
m
22. The G.M. of numbers 4, 5, 10, 20, 25 is :- 2 
(1) 12.8 (2) 10  n
(2)   th term
m
(3) 7.8 (4) None of these 2

N
23. The geometric mean of the first n terms of the n 

I
series a, ar, ar2, ...., is- (3)   1 th term
m
2 
(1) arn/2 (2) arn (3) ar(n–1)/2 (4) arn–1
 n n 
24. If G1 and G2 are geometric mean of two series of (4) Mean of   th and   1 th terms

R
ms
sizes n1 and n2 resp. and G is geometric mean of
2 2
30. The median of the numbers 6, 14, 12, 8, 10, 9,
their combined series, then log G is equal to :-
11, is :-

P
(1) log G1 + log G2 (2) n1 log G1 + n2log G2 (1) 8 (2) 10 (3) 10.5 (4) 11
31. Median of the following freq. dist.
logG 1  logG 2 n 1 logG 1  n 2 log G 2

S
(3) (4) xi 3 6 10 12 7 15
n1  n2 n1  n 2
fi 3 4 2 8 13 10
25. The Harmonic mean of 3, 7, 8, 10, 14 is-
(1) 7 (2) 10

E
3  7  8  10  14
(1) (3) 8.5 (4) None of these
5

E
32. Median is independent of change of :-
5

J
(2) (1) only Origin
3  7  8  10  14
(2) only Scale
1 1 1 1 1 (3) Origin and scale both
   
3 7 8 10 14 (4) Neither origin nor scale
(3)
5
33. A series which have numbers three 4's, four 5's,
5 five 6's, eight 7's, seven 8's and six 9's then the
(4)
1 1 1 1 1 mode of numbers is :-
   
3 7 8 10 14 (1) 9 (2) 8 (3) 7 (4) 6
26. The H.M. of the numbers 2, 3, 4 is :- 34. Mode of the following freqency distribution

36 13 x: 4 5 6 7 8
(1) 3 (2) 2(3) 1/3 (3) (4) f: 6 7 10 8 3
13 36
27. The H.M. of following freq. dist. is :- (1) 5 (2) 6 (3) 8 (4) 10
35. The mode of the following freq. dist is :-
xi 3 6 9 12
Class 110 11 20 21 30 31  40 41 50
fi 1 2 3 4
fi 5 7 8 6 4
(1) 9 (2) 3
(1) 24 (2) 23.83
(3) 7.5 (4) None of these
(3) 27.16 (4) None of these

Page 22
JEE Sprint Mathematics
Symmetric and asymmetric distribution, Range Varia nce a nd Sta ndard De v iat ion

36. For a normal dist :- x a


44. The variate x and u are related by u  then
(1) mean = median h
(2) median = mode correct relation between x and u is :-
(3) mean = mode
(1)  x  h  u (2)  x  h   u
(4) mean = median = mode
37. The relationship between mean, median and mode (3)  u  h  x (4)  u  h   x
for a moderately skewed distribution is-
45. The S.D. of the first n natural numbers is-
(1) mode = median – 2 mean
(2) mode = 2 median – mean
n2  1 n2  1
(3) mode = 2 median – 3 mean (1) (2)
2 3

T
(4) mode = 3 median – 2 mean

38. The range of observations 2, 3, 5, 9, 8, 7, 6, 5, 7, n2  1 n2  1


(3) (4)

N
4, 3 is :- 4 12

I
(1) 6 (2) 7 (3) 5.5 (4) 11
46. The variance of observations 112, 116, 120, 125,
Mean Deviation 132 is :-

R
(1) 58.8 (2) 48.8
39. The mean deviation of a frequency dist. is equal to :-
(3) 61.8 (4) None of these
d i  di

P
10 10
(1) f (2) f
i i 47. If  (x i  15)  12 and  (x i  15)2  18 then the
i 1 i 1

S
fi d i fi d i S.D. of observations x1, x2 ............ x10 is :-
(3) fi (4) fi 2 3
(1) (2)
5 5

E
40. Mean deviation from the mean for the observation
4
–1, 0, 4 is- (3) (4) None of these
5

E
14 2
(1) (2) 48. The S.D. of 7 scored 1, 2, 3, 4, 5, 6, 7 is-

J
3 3
(1) 4 (2) 2
(3) 2 (4) none of these
41. Mean deviation of the observations 70, 42, 63, (3) 7 (4) none of these
34, 44, 54, 55, 46, 38, 48 from median is :-
49. The variance of series a, a + d, a + 2d, ....., a + 2nd
(1) 7.8 (2) 8.6
is :-
(3) 7.6 (4) 8.8
42. Mean deviation of 5 observations from their mean n(n  1) 2 n(n  1) 2
(1) d (2) d
3 is 1.2, then coefficient of mean deviation is :- 2 3
(1) 0.24 (2) 0.4
(3) 2.5 (4) None of these n(n  1) 2 n(n  1) 2
(3) d (4) d
43. The mean deviation from median is 6 12
(1) greater than the mean deviation from any other
50. Variance is independent of change of-
central value
(2) less than the mean deviation from any other (1) only origin
central value (2) only scale
(3) equal to the mean deviation from any other (3) origin and scale both
central value
(4) none of these
(4) maximum if all values are positive

Page 23
JEE Sprint Mathematics
51. If the coefficient of variation and standard deviation 54. If each observation of a dist., whose variance is 2 ,
of a distribution are 50% and 20 respectively, then is multiplied by , then the S.D. of the new new
its mean is- observations is-
(1) 40 (2) 30 (1)  (2) 
(3) 20 (4) None of these (3) || (4) 2 
55. The standard deviation of variate x i is . Then
52. If each observation of a dist. whose S.D. is , is
ax i  b
increased by , then the variance of the new standard deviation of the variate ,
c
observations is - where a , b, c are constants is-
(1)  (2)  +  a a
(1)    (2) 
(3) 2 (4) 2 +  c c

T
53. The variance of 2, 4, 6, 8, 10 is-
 a2 
(3)  2  (4) None of these
(1) 8 (2) 8 c

N

I
(3) 6 (4) none of these

P R
E S
JE
ANSWER-KEY STATISTICS

Que. 1 2 3 4 5 6 7 8 9 10 11 12 13 14 15 16 17 18 19 20
Ans. 2 4 3 3 1 2 2 4 2 4 2 2 3 4 2 1 4 2 3 3
Que. 21 22 23 24 25 26 27 28 29 30 31 32 33 34 35 36 37 38 39 40
Ans. 3 2 3 4 4 3 3 3 4 2 3 4 3 2 2 4 4 2 4 3
Que. 41 42 43 44 45 46 47 48 49 50 51 52 53 54 55
Ans. 2 2 2 1 4 2 2 2 2 1 1 3 1 3 2

Page 24
JEE Sprint Mathematics
SELECT THE CORRECT ALTERNATIVE (ONLY ONE CORRECT ANSWER)
EXTRA QUESTIONS ON STATISTICS
1. The A.M. of the series 1, 2, 4, 8, 16, ......, 2n is- 9. The observations 29, 32, 48, 50, x, x + 2, 72,
78, 84, 95 are arranged in ascending order and
2n  1 2 n 1  1 their median is 63 then the value of x is :-
(1) (2)
n n 1 (1) 61 (2) 62 (3) 62.5 (4) 63
2n  1 2 n 1  1 10. If the mode of a distribution is 18 and the mean is
(3) (4)
n 1 n 24, then median is-
2. If the mean of n observations 12, 22, 32, ....... n2 is (1) 18 (2) 24 (3) 22 (4) 21
11. If the mean and S.D. of n observations x1, x2,....xn
46n
, then n is equal to- are x and  resp, then the sum of squares of
11

T
observations is :-
(1) 11 (2) 12
(3) 23 (4) 22 (1) n (2 + x 2 ) (2) n ( 2 – x 2)

3. The weighted mean of first n natural numbers (3) n ( x 2 – 2 ) (4) None of these

N
whose weights are equal, is :- 12. The variance of observations 8, 12, 13, 15, 22, is

I
:-
n 1 2n  1
(1) (2) (1) 21 (2) 21.2
2 2

R
(3) 21.4 (4) None of these
2n  1 (2n  1) (n  1)
(3) (4) 13. If the mean of a set of observations x1, x2,......., x10
3 6

P
is 20, then the mean of x1+ 4, x2+ 8, x3+12,...,x10+ 40
4. The average age of a group of men and women is
is-
30years. If average age of men is 32 and that of (1) 34 (2) 42 (3) 38 (4) 40

S
women is 27, then the percentage of women in 14. The mean of values 0, 1, 2, ......, n when their
the group is- weights are 1, nC1, nC2, .... , nCn , resp., is

E
(1) 60 (2) 50
2n n 1 2 n 1 n
(3) 40 (4) 30 (1) (2) (3) (4)
n 1 2 n(n  1) 2

E
5. The geometric mean of the observations 2, 4, 8, 15. The G.M. of first n natural numbers is :-
16, 32, 64 is-

J
n 1
(1) 25/2 (2) 27/2 (1) (2) (n!)n
2
(3) 33 (4) None of these (3) (n!)1/n (4) None of these
6. The H.M. of the reciprocal of first n natural 7
16. If a variable takes the discrete values  + 4,  – ,
numbers is :- 2
5 1 1
n 1 n – ,  – 3,  – 2,  + ,  – ,  + 5( > 0),
(1) (2) 1 1 1 2 2 2
2  then the median of these values-
 1    ....  
2 3 n
5 1
2 (1)   (2)  
4 2
(3) (4) None of these
n 1 5
7. Product of n positive numbers is unit. The sum of (3)  – 2 (4)  
4
these numbers can not be less than- 17. The S.D. of first n odd natural numbers is :-
(1) 1 (2) n
n2  1 n2  1
(3) n 2
(4) none of these (1) (2)
2 3
8. The A.M. of first n terms of the series
1.3.5, 3.5.7, 5.7.9,....., is-
n2  1 n2  1
(1) 3n3 + 6n2 + 7n – 1 (2) n3 + 8n2 + 7n – 1 (3) (4)
6 12
3 2
(3) 2n + 8n – 7n – 2 (4) 2n3 + 8n2 + 7n – 2

Page 25
JEE Sprint Mathematics
18. If the sum and sum of squares of 10 observations 27. The S.D. of the following freq. dist. :-
are 12 and 18 resp., then, The S.D. of observations
Class 0  10 10  20 20  30 30  40
is :-
fi 1 3 4 2
1 2 3 4 (1) 7.8 (2) 9
(1) (2) (3) (4)
5 5 5 5 (3) 8.1 (4) 0.9
19. The mean of n values of a distribution is x . If its 28. The mean of a dist. is 4. if its coefficient of variation
first value is increased by 1 , second by 2, .... then is 58%. Then the S.D. of the dist. is :-
the mean of new values will be- (1) 2.23 (2) 3.23
(1) x  n (2) x  n / 2 (3) 2.32 (4) None of these
 n 1  29. The mean of a set of observations is x . If each
(3) x +   (4) None of these observation is divided by , (  0) and then is
 2 

T
increased by 10, then the mean of the new set is
20. The mean of the series x1, x2, ....., xn is X . If x2 is
x x  10
replaced by , then the new mean is- (1) (2)
 

N
X  x2   nX  x 2   x  10  x  10
(1) (2) (3) (4)

I
n n  

(n  1)X   nX  x 2   30. The average age of a teacher and three students is


(3) (4)

R
n n 20 years. If all students are of equal age and the
21. Let G1 and G2 be the geometric means of two series difference between the age of the teacher and that
x1, x2, ......, xn and y1, y2 ..... yn respectively. If G is

P
of a student is 20 years, then the age of the teacher
the geometric mean of series xi/yi, i = 1, 2,..., n, is-
then G is equal to-

S
(1) 25 years (2) 30 years
(1) G1 – G2 (2) log G1/log G2
(3) log (G1/G2) (4) G 1/G 2 (3) 35 years (4) 45 years
22. The mean deviation of the numbers 1, 2, 3, 4, 5 31. If a , b, c are any three positive numbers, then the

E
is- 1 1 1 
(1) 0 (2) 1.2 least value of (a + b+ c)     is-
a b c
(3) 2 (4) 1.4

E
(1) 3 (2) 6
23. If mean = (3 median – mode) x, then the value of x

J
is- (3) 9 (4) None of these
2n 2n 2n
(1) 1 (2) 2 (3) 1/2 (4) 3/2 32. Median of C0, C 1, C2 ,....,2nC n
24. A man spends equal ammount on purchasing three (when n is even) is-
kinds of pens at the rate 5 Rs/pen, 10 Rs/pen, 2n
(1) C n 1 (2)
2n
Cn
20 Rs/pen, then average cost of one pen is :- 2 2
35 2n
(1) 10 Rs (2) Rs (3) C n 1 (4) None of these
3
2
60 33. The mean deviation from mean of observations
(3) Rs (4) None of these
7 5, 10, 15, 20, .........85 is :-
25. The median of 21 observation is 40. if each
observations greater than the median are increased (1) 43.71 (2) 21.17
by 6, then the median of the observations will be-
(3) 38.7 (4) None of these
(1) 40 (2) 46
(3) 46 + 40/21 (4) 46 – 40/21 34. If standard deviation of variate xi is 10, then variance
26. The coefficient of range of the following distribution of the variate ( 50 + 5xi) will be-
10, 14, 11, 9, 8, 12, 6 (1) 50 (2) 250
(1) 0.4 (2) 2.5 (3) 500 (4) 2500
(3) 8 (4) 0.9

Page 26
JEE Sprint Mathematics
35. The S.D. of the numbers 31, 32, 33, .... 47 is- 39. The median of 19 observations of a group is 30. If
two observations with values 8 and 32 are further
(1) 2 6 (2) 4 3
included, then the median of the new group of
21 observation will be
47 2  1
(3) (4) None of these (1) 28 (2) 30
12
(3) 32 (4) 34
36. The sum of the squares of deviation of 40. The coefficient of mean deviation from median of
10 observations from their mean 50 is 250, then observations 40, 62, 54, 90, 68, 76 is :-
coefficient of variation is- (1) 2.16 (2) 0.2
(1) 10% (2) 40% (3) 5 (4) None of these
(3) 50% (4) None of these 41. A group of 10 observations has mean 5 and S.D.
37. The median and standard deviation (S.D.) of a 2 6 . another group of 20 observations has mean

T
distribution will be, If each term is increased by 2 -
5 and S.D. 3 2 , then the S.D. of combined group
(1) median and S.D. will increased by 2
of 30 observations is :-
(2) median will increased by 2 but S.D. will remain

N
same (1) 5 (2) 2 5

I
(3) median will remain same but S.D. will increased (3) 3 5 (4) None of these
by 2
42. For the values x 1 , x 2 ...... x 101 of a distribution
(4) median and S.D. will remain same x1 < x2 < x3 < ..... < x100 < x101 . The mean deviation

R
38. If X 1 and X 2 are the means of two series such of this distribution with respect to a number k will
be minimum when k is equal to-

P
that X 1 < X 2 and X is the mean of the combined (1) x1 (2) x51
series, then-
x 1  x 2  .....  x 101

S
(1) X < X 1 (2) X > X 2 (3) x50 (4)
101
43. In any discrete series (when all the value are not
X  X2 same) the relationship between M.D. about mean
(3) X 1 < X < X 2 (4) X = 1

E
2 and S.D. is-
(1) M.D. = S.D. (2) M.D. > S.D.

E
(3) M.D. < S.D. (4) M.D.  S.D.

J
ANSWER-KEY
Que. 1 2 3 4 5 6 7 8 9 10 11 12 13 14 15 16 17 18 19 20
Ans. 2 1 1 3 2 3 2 4 2 3 1 2 2 4 3 1 2 3 3 4
Que. 21 22 23 24 25 26 27 28 29 30 31 32 33 34 35 36 37 38 39 40
Ans. 4 2 3 3 1 1 2 3 3 3 3 2 2 4 1 1 2 3 2 2
Que. 41 42 43
Ans. 2 2 3

Page 27
JEE Sprint Mathematics
QUADRATIC EQUATIONS
SELECT THE CORRECT ALTERNATIVE (ONLY ONE CORRECT ANSWER)

1. The roots of the quadratic equation (a + b – 2c) x2 – (2a – b – c) x + (a – 2b + c) = 0 are -


(A) a + b + c & a – b + c (B) 1/2 & a – 2b + c
(C) a – 2b + c & 1/(a + b – 2c) (D) none of these
8 8
2. If the A.M. of the roots of a quadratic equation is and A.M. of their reciprocals is , then the quadratic
5 7
equation is -
(A) 5x2 – 8x + 7 = 0 (B) 5x2 – 16x + 7 = 0 (C) 7x2 – 16x + 5 = 0 (D) 7x2 + 16x + 5 = 0
3. If sin  & cos  are the roots of the equation ax2 + bx + c = 0 then -

T
(A) a2 – b2 + 2ac = 0 (B) a2 + b2 + 2ac = 0
(C) a2 – b2 – 2ac = 0 (D) a2 + b2 – 2ac = 0

N
a
4. If one root of the quadratic equation px 2 + qx + r = 0 (p  0) is a surd , where p, q, r; a, b

I
a  a b
are all rationals then the other root is -

R
b a(a  b)
(A) (B) a +
a  a b b

P
a  a a  b a  a b
(C) (D)
b b

S
FG  IJ
5. A quadratic equation with rational coefficients one of whose roots is tan
H 12 K is -

E
(A) x2 – 2x + 1 = 0 (B) x2 – 2x + 4 = 0 (C) x2 – 4x + 1 = 0 (D) x2 – 4x – 1 = 0
6. ax 2 + bx + c = 0 has real and distinct roots  and ( > ). Further a > 0, b < 0 and c < 0, then -

E
b
(A) 0 <  < || (B) 0 <  <  (C)  +  < 0 (D) || + | =

J
a
7. If the roots of (a2 + b2) x2 – 2b (a + c) x + (b2 + c2) = 0 are equal then a, b, c are in

(A) A.P. (B) G.P. (C) H.P. (D) none of these

8. If a (b – c) x2 + b (c – a) x + c (a – b) = 0 has equal root, then a, b, c are in

(A) A.P. (B) G.P. (C) H.P. (D) none of these

9. Let p, q  {1, 2, 3, 4}. Then number of equation of the form px2 + qx + 1 = 0, having real roots, is

(A) 15 (B) 9 (C) 7 (D) 8


10. If the roots of the quadratic equation ax 2 + bx + c = 0 are imaginary then for all values of a, b, c and
x  R, the expression a 2 x 2 + abx + ac is -
(A) positive (B) non-negative
(C) negative (D) may be positive, zero or negative

11. If x, y are rational number such that x + y + (x – 2y) 2 = 2x – y + (x – y – 1) 6 , then

(A) x and y connot be determined (B) x = 2, y = 1


(C) x = 5, y = 1 (D) none of these

Page 28
JEE Sprint Mathematics
12. Graph of the function f(x) = Ax2 – BX + C, where
A = (sec – cos) (cosec – sin)(tan + cot),
B = (sin + cosec)2 + (cos + sec)2 – (tan2 + cot2) &
C = 12, is represented by
y y y y

(A) x (B) x (C) x (D) x

13. The equation whose roots are the squares of the roots of the equation ax 2  bx  c  0 is -

(A) a 2 x 2  b 2 x  c 2  0  
(B) a 2 x 2  b 2  4ac x  c 2  0

T
 
(C) a 2 x 2  b 2  2ac x  c 2  0 (D) a 2 x 2  b 2

 ac x  c 2  0
14. If , then the equation whose roots are & is

N
(A) x2 + 5x – 3 = 0 (B) 3x2 + 12x + 3 = 0 (C) 3x2 – 19x + 3 = 0 (D) none of these

I
1 1
15. If   are the roots of the equation x2 – 3x + 1 = 0, then the equation with roots , 
will be
 2  2

R
(A) x2 – x – 1 = 0 (B) x2 + x – 1 = 0 (C) x2 + x + 2 = 0 (D) none of these

16. If x 2  11x  a and x 2  14 x  2a have a common factor then 'a' is equal to

P
(A) 24 (B) 1 (C) 2 (D) 12
x 5

S
17. The smallest integer x for which the inequality 2 > 0 is satisfied is given by -
x  5 x  14
(A) – 7 (B) – 5 (C) – 4 (D) – 6
x 2 (3x  4) 3 (x  2) 4

E
18. The number of positive integral solutions of the inequation  0 is -
(x  5) 5 (2x  7) 6
(A) 2 (B) 0 (C) 3 (D) 4

E
19. The value of ‘a’ for which the sum of the squares of the roots of 2x2 – 2 (a – 2) x – a – 1 = 0 is least is -

J
(A) 1 (B) 3/2 (C) 2 (D) – 1
20. If the roots of the quadratic equation x2 + 6x + b = 0 are real and distinct and they differ by atmost 4 then the
least value of b is -
(A) 5 (B) 6 (C) 7 (D) 8
x 2  2x  1
21. The expression lies in the interval ; (x  R) -
x 2  2x  7
(A) [0, –1] (B) (  , 0]  [1 , ) (C) [0, 1) (D) none of these
2 2
22. If the roots of the equation x – 2ax + a + a – 3 = 0 are real & less than 3 then -
(A) a < 2 (B) 2  a  3 (C) 3  a  4 (D) a > 4
2 2
23. The number of integral values of m, for which the roots of x – 2mx + m – 1 = 0 will lie between – 2 and 4 is -
(A) 2 (B) 0 (C) 3 (D) 1
3 2
24. If the roots of the equation, x + Px + Qx – 19 = 0 are each one more than the roots of the equation,
x3 – Ax2 + Bx – C = 0, where A, B, C, P & Q are constants then the value of A + B + C =
(A) 18 (B) 19 (C) 20 (D) none
1 1 1 1
25. If     are roots of x4 – 100x3 + 2x2 + 4x + 10 = 0, then        is equal to -

2 1 2
(A) (B) (C) 4 (D) 
5 10 5

Page 29
JEE Sprint Mathematics
26. Number of real solutions of the equation x4 + 8x2 + 16 = 4x2 – 12x + 9 is equal to -
(A) 1 (B) 2 (C) 3 (D) 4
27. The complete solution set of the inequation x  18  2  x is -
(A) [–18, –2] (B) (–, –2)  (7, ) (C) (–18, 2)  (7, ) (D) [–18, –2)

3x  1
28. If log1 / 3 is less than unity then x must lie in the interval -
x 2

(A) (–, –2)  (5/8, ) (B) (–2, 5/8)


(C) (–, –2)  (1/3, 5/8) (D) (–2, 1/3)
2
29. Exhaustive set of value of x satisfying log |x|(x + x + 1)  0 is -

T
(A) (–1, 0) (B) (–, 1)  (1, )
(C) (– ) – {–1, 0, 1} (D) (–, –1)  (–1, 0)  (1, )
30. Solution set of the inequality, 2 – log 2(x 2 + 3x)  0 is -

N
(A) [–4, 1] (B) [–4, –3)  (0, 1] (C) (–, –3)  (1, ) (D) (–, –4)  [1, )

I
SELECT THE CORRECT ALTERNATIVES (ONE OR MORE THAN ONE CORRECT ANSWERS)

R
31. If  is a root of the equation 2x(2x + 1) = 1, then the other root is -
(A) 33 – 4 (B) –2( + 1) (C) 43 – 3 (D) none of these
32. If b2  4ac for the equation ax 4 + bx 2 + c = 0, then all roots of the equation will be real if -

P
(A) b > 0, a < 0, c > 0 (B) b < 0, a > 0, c > 0
(C) b > 0, a > 0, c > 0 (D) b > 0, a < 0, c < 0

S
33. Let   be the roots of x2 – ax + b = 0, where a & b  R. If  + 3 = 0, then -
(A) 3a2 + 4b = 0 (B) 3b2 + 4a = 0 (C) b < 0 (D) a < 0
2
34. For x  [1, 5], y = x – 5x + 3 has -

E
(A) least value = –1.5 (B) greatest value = 3
5  13

E
(C) least value = –3.25 (D) greatest value =
2

J
2
35. Integral real values of x satisfying log 1/2(x – 6x + 12) > –2 is -
(A) 2 (B) 3 (C) 4 (D) 5
1
36. If < log 0.1 x < 2, then -
2
1 1 1
(A) the maximum value of x is (B) x lies between and
10 100 10
1 1 1
(C) x does not lie between and (D) the minimum value of x is
100 10 100

ANSWER KEY
Que. 1 2 3 4 5 6 7 8 9 10
Ans. D B A C C B B C C A
Que. 11 12 13 14 15 16 17 18 19 20
Ans. B B C C A A D C B A
Que. 21 22 23 24 25 26 27 28 29 30
Ans. C A C A D A D A D B
Que. 31 32 33 34 35 36
Ans. B,C B,D A,C B,C A, B, C A,B,D

Page 30
JEE Sprint Mathematics
EXTRA PRACTICE QUESTIONS ON QUADRATIC EQUATIONS

SELECT THE CORRECT ALTERNATIVES (ONE OR MORE THAN ONE CORRECT ANSWERS)

1. The equation whose roots are sec2  & cosec2 can be -


(A) 2x2 – x – 1 = 0 (B) x2 – 3x + 3 = 0 (C) x2 – 9x + 9 = 0 (D) x2 + 3x + 3 = 0
2. If cos  is a root of the equation 25x2 + 5x – 12 = 0, – 1 < x < 0, then the value of sin 2 is -
(A) 12/25 (B) – 12 / 25 (C) – 24 / 25 (D) 24 / 25
1 1 1
3. If the roots of the equation   are equal in magnitude and opposite in sign, then -
x p x q r
(A) p + q = r (B) p + q = 2r

T
1 2
(C) product of roots =  (p  q 2 ) (D) sum of roots = 1
2
y
4. Graph of y = ax2 + bx + c = 0 is given adjacently. What conclusions can be drawn

N
from this graph -

I
O x
(A) a > 0 (B) b < 0

R
Vertex
(C) c < 0 (D) b2 – 4ac > 0
5. If a, b, c are real distinct numbers satisfying the condition a + b + c = 0 then the roots of the quadratic

P
equation 3ax 2 + 5bx + 7c = 0 are -

S
(A) positive (B) negative (C) real and distinct (D) imaginary
y
6. The adjoining figure shows the graph of y = ax2 + bx + c. Then - Vertex

E
(A) a > 0 (B) b > 0
x1 x2 x
(C) c > 0 (D) b2 < 4ac

E
7. If x2 + Px + 1 is a factor of the expression ax3 + bx + c then -

J
(A) a2 + c2 = – ab (B) a2 – c2 = – ab (C) a2 – c2 = ab (D) none of these
8. The set of values of ‘a’ for which the inequality (x – 3a) (x – a – 3) < 0 is satisfied for all x in the interval 1  x  3
(A) (1/3, 3) (B) (0, 1/3) (C) (–2, 0) (D) (–2, 3)
3 2
9. Let p(x) be the cubic polynomial 7x – 4x + K. Suppose the three roots of p(x) form an arithmetic progression.
Then the value of K, is -
4 16 16 128
(A) (B) (C) (D)
21 147 441 1323
2
10. If the quadratic equation ax + bx + 6 = 0 does not have two distinct real roots, then the least value of
2a + b is -
(A) 2 (B) – 3 (C) – 6 (D) 1
11. If p & q are distinct reals, then 2 {(x – p) (x – q) + (p – x) (p – q) + (q – x) (q – p)} = (p – q)2 + (x – p)2 + (x – q)2
is satisfied by -
(A) no value of x (B) exactly one value of x (C) exactly two values of x (D) infinite values of x

12. The value of 'a' for which the expression y = x2 + 2a a 2  3 x + 4 is perfect square, is -
(A) 4 (B) ± 3

(C) ± 2 (D) a  (–, – 3 ]  [ 3 , )

Page 31
JEE Sprint Mathematics
13. Set of values of 'K' for which roots of the quadratic x2 – (2K – 1)x + K(K – 1) = 0 are -
(A) both less than 2 is K  (2, ) (B) of opposite sign is K  (–, 0)  (1, )

(C) of same sign is K  (–, 0)  (1, ) (D) both greater than 2 is K  (2, )
14. The correct statement is / are -
x1 x 2
(A) If x1 & x2 are roots of the equation 2x2 – 6x – b = 0 (b > 0), then   2
x 2 x1
(B) Equation ax2 + bx + c = 0 has real roots if a < 0, c > 0 and b  R
(C) If P(x) = ax2 + bx + c and Q(x) = –ax2 + bx + c, where ac  0 and a, b, c  R, then P(x).Q(x) has at least
two real roots.
(D) If both the roots of the equation (3a + 1)x2 – (2a +3b)x + 3 = 0 are infinite then a = 0 & b R

T
15. If 1<2<3 < 4 < 5 < 6, then the equation (x –1)(x–3)(x–5)+3(x – 2)(x – 4)(x – 6)=0 has -
(A) three real roots (B) no real root in (–, 1)

N
(C) one real root in (1, 2) (D) no real root in (5, 6)
2x2

I
16. Equation – 2(2a + 1)x + a(a + 1) = 0 has one root less than 'a' and other root greater than 'a', if
(A) 0 < a < 1 (B) –1 < a < 0 (C) a > 0 (D) a < –1
17. The value(s) of 'b' for which the equation, 2log 1/25 (bx + 28) = – log 5 (12 – 4x – x 2) has coincident roots,

R
is/are -
(A) b = – 12 (B) b = 4 (C) b = 4 or b = – 12 (D) b = –4 or b = 12

P
8 5 2
18. For every x  R, the polynomial x – x + x – x + 1 is -

S
(A) positive (B) never positive
(C) positive as well as negative (D) negative
19. If ,  are the roots of the quadratic equation (p + p + 1) x2 + (p – 1) x + p2 = 0 such that unity lies between
2

E
the roots then the set of values of p is -
(A)  (B) p  (– , – 1)  (0,  (C) p  (– 1, 0) (D) (– 1, 1)

E
4 3 2
20. Three roots of the equation, x – px + qx – rx + s = 0 are tanA, tanB & tanC where A, B, C are the
angles of a triangle. The fourth root of the biquadratic is -

J
p r p r p r p r
(A) (B) (C) (D)
1q s 1 q s 1q s 1 q s
 2x 
21. If log  x 2 12 x  30   log 2  > 0 then x belongs to interval -
   5
 10 

5 5
(A) ( , 6 + 6) (B) ( ,6– 6) (C) (6, 6 + 6) (D) (10, )
2 2

ANSWER KEY
Que. 1 2 3 4 5 6 7 8 9 10
Ans. C C,D B,C A,B, C,D C B,C C B D B
Que. 11 12 13 14 15 16 17 18 19 20
Ans. D C C A, B, C A, B, C A ,C , D B A C A
Que. 21
Ans. B,D

Page 32
JEE Sprint Mathematics
SEQUENCE AND SERIES
SELECT  THE  CORRECT  ALTERNATIVE  (ONLY  ONE  CORRECT  ANSWER)
1. The  maximum  value  of  the  sum  of  the  A.P.  50,  48,  46,  44,  ....................  is  -
(A) 325 (B) 648 (C) 650 (D) 652

2. Let    T r     be  the  r th   term  of  an  A.P.  for  r  =  1,  2,  3,  ...........  If  for  some  positive  i ntegers  m,  n  we

1 1
have  Tm  & Tn  , then  Tmn  equals  -
n m  

1 1 1
(A)  (B)   (C)    1 (D) 0
mn m n

T
3. The  interior  angles  of  a  convex  polygon  are  in  AP  .  The  smallest  angle  is  120°  &  the  common  difference
is  5°.    Find  the  number  of  sides  of  the  polygon  -

I N
(A) 9 (B) 16 (C) 12 (D) none  of  these

4. The  first  term  of  an  infinitely  decreasing  G.P.  is  unity  and  its  sum  is  S.  The  sum  of  the  squares  of  the  terms  of

R
the  progression  is  -

S S2 S

P
(A)  (B)  (C)  (D) S2
2S  1 2S  1 2S

5. A  particle  begins  at  the  origin  and  moves  successively  in  the  y

S
following  manner  as  shown,  1  unit  to  the  right,  1/2  unit  up,  1/4  unit  to 1/4
the  right,  1/8  unit  down,  1/16  unit  to  the  right  etc.  The  length  of  each

1/8
1/2
move  is  half  the  length  of  the  previous  move  and  movement  continues  in 1/16

E
1
x
the  ‘zigzag’  manner  indefinitely.  The  co-ordinates  of  the  point  to  which 0

the  ‘zigzag’  converges  is  -

E
(A) (4/3,  2/3) (B) (4/3,  2/5) (C) (3/2,  2/3) (D) (2,  2/5)

J
100 100
6. Let an  be the  nth  term of  a  G.P. of  positive  numbers. Let   a 2n =    &  a 2n 1 =   such  that       . Then  thhe
n 1 n 1
common  ratio  of  the  G.P.  is  -

   
(A) (B) (C)  (D) 
   

7. If  p,  q,  r  in  harmonic  progression  and  p  &  r  be  different  having  same  sign  then  the  roots  of  the  equation
px2  +  qx  +  r  =  0  are -
(A) real  and  equal (B) real  and  distinct (C) irrational (D) imaginary

1 1 1
8. If  x  >  1,  y  >  1,  z  >1  are  in  G.P.,  then   ,  ,   are  in
n  -
1  n x 1  n y 1  n z

(A) A.P. (B) H.P. (C) G.P. (D) none  of  above

9. If  ln  (a  +  c)  ,  ln  (c  –  a),  ln  (a  –  2b  +  c)  are  in  A.P.,  then  :
(A) a,  b,  c  are  in  A.P. (B) a2,  b2,  c2  are  in  A.P
(C) a,  b,  c  are  in  G.P. (D) a,  b,  c  are  in  H.P.

Page 33
JEE Sprint Mathematics
10. If the  (m + 1)th , (n +1)th  &  (r + 1)th terms of an AP are  in GP & m, n, r are in HP, then  the ratio of the common
difference  to  the  first  term  of  the  AP  is  -

1 2 2
(A)  (B)  (C)   (D)  none  of  these
n n n

11. The  sum  of  roots  of  the  equation  ax2  +  bx  +  c  =  0  is  equal  to  the  sum  of  squares  of  their  reciprocals.  Then
bc2  ,  ca2  and  ab2  are  in  -
(A) AP (B) GP (C) HP (D) none  of  these
12. The  quadratic  equation  whose  roots  are  the  A.M.  and  H.M.  between  the  roots  of  the  equation,
2x2  –  3x  +  5  =  0  is  -
(A) 4x2  –  25x  +  10  =  0 (B) 12x2  –  49x  +  30  =  0

T
(C) 14x2  –  12x  +  35  =  0 (D) 2x2  +  3x  +  5  =  0
13. If  the  sum  of  the  first  n  natural  numbers  is  1/5  times  the  sum  of  the  their  squares,  then  the  value  of  n  is  -
(A) 5 (B) 6 (C) 7 (D) 8

N
14. Suppose  p  is  the  first  of  n(n  >  1)  AM's  between  two  positive  numbers  a  and  b,  then  value  of  p  is  -

I
na  b na  b nb  a nb  a
(A)  (B)  (C)  (D) 
n 1 n 1 n 1 n 1

R
1 1 1 1
15. If      0   and  a,  b,  c  are  not  in  A.P.,  then  -
a a  2b c c  2b

P
b b
(A) a,  b,  c  are  in  G.P. (B) a, , c   are  in  A.P.. (C)  a, , c   are  in  H.P.. (D) a,  2b,  c  are  in  H.P.

S
2 2

3 5 7
16. The  sum  to  n  terms  of  the  series  2
 2 2
 2  .........   is  -
1 1 2 1  22  32

E
3n 6n 9n 12n
(A)  (B)  (C)  (D) 

E
n 1 n 1 n 1 n 1

J
1 1 1 4 1 1 1
17. If  4
 4  4  ........ + to    ,  then  4  4  4  ...... + to    is  equals  to  -
1 2 3 90 1 3 5

4 4 89 4
(A)  (B)  (C)  (D) none  of  these
96 45 90

n s
  3
18. If    r   an  bn 2  cn ,  then  find  the  value  of  a  +  b  +  c.
s 1 r 1

(A) 1 (B) 0 (C) 2 (D) 3

 5c   3b   a 
19. If  a,  b,  c  are  positive  numbers  in  G.P.  and  log   , log     and  log     are  in  A.P.,  then  a,  b,  c  forms  the
he
 a   5c   3b 
sides  of  a  triangle  which  is  -
(A) equilateral (B) right  angled (C) isosceles (D) none  of  these
SELECT  THE  CORRECT  ALTERNATIVES  (ONE  OR  MORE  THAN  ONE  CORRECT  ANSWERS)

20. If  sum  of  n  terms  of  a  sequence  is  given  by  Sn  =  3n2  –  5n  +  7  &  tr  represents  its  rth  term,  then  -

(A) t7  =  34 (B) t2  =  7 (C) t10  =  34 (D) t8  =  40

Page 34
JEE Sprint Mathematics
1
21. If  10  harmonic  means    H1,  H2,  H3  .........  H10   are  inserted  between  7  and  –  ,  then  -
3

3 1 7
(A) H1  =  –7 (B) H2  = (C) H1  =  – (D) H10  = 
7 7 19

22. If  tn  be  the  nth  term  of  the  series  1  +  3  +  7  +  15  +  ........,  then  -

(A) t5  +  1  =  32 (B) t7  =  27  +  1 (C) t10  =  210  –  1 (D) t100  =  250  +  1

  

23. Indicate  the  correct  alternative(s),  for  0    ,   if  x   cos 2n , y   sin 2n  and z   cos 2n  sin 2 n  ,
2 n 0 n 0 n 0

T
then  -

(A) xyz  =  xz  +  y (B) xyz  =  xy  +  z (C) xyz  =  x  +  y  +  z (D) xyz  =  yz  +  x

R I N
S P
JE E
ANSWER  KEY
Que. 1 2 3 4 5 6 7 8 9 10
Ans. C C A B B A D B D C
Que. 11 12 13 14 15 16 17 18 19 20
Ans. A B C A D B A A D A,D
Que. 21 22 23
Ans. A,D A,C B,C

Page 35
JEE Sprint Mathematics
EXTRA PRACTICE QUESTIONS ON SEQUENCE AND SERIES
SELECT  THE  CORRECT  ALTERNATIVES  (ONE  OR  MORE  THAN  ONE  CORRECT  ANSWERS)
1. Consider  an  A.P.  with  first  term  ‘a’  and  the  common  difference  d.  Let  Sk  denote  the  sum  of  the  first  K  terms.
S kx
Let  S   is  independent  of  x,  then  -
x

(A) a  =  d/2 (B)  a  =  d (C)  a  =  2d (D)  none  of  these


3 2
2. Let  ,  ,    be  the  roots  of  the  equation  x   +  3ax   +  3bx  +  c  =  0.  If  ,  ,    are  in  H.P.  then    is  equal  to  -
(A) –  c/b (B) c/b (C) –  a (D) a


FG 9 IJ r

 H 11K

T
3. (2r  1)   is  equal  to  -
r 1

(A) 45 (B) 55

N
(C) sum  of  first  nine  natural  numbers (D) sum  of  first  ten  natural  numbers

I
4. For  the  A.P.  given  by  a1,  a2,  .............,  an,  ........,  with  non-zero  common  difference,  the  equations  satisfied
are-

R
(A) a1  +  2a2  +  a3  =  0 (B) a1  –  2a2  +  a3  =  0

P
(C) a1  +  3a2  –  3a3  –  a4  =  0 (D) a1  –  4a2  +  6a3  –  4a4  +  a5  =  0

5. If  a,  a1,  a2,.....,a10  ,  b  are  in  A.P.  and  a,  g1,  g2,.....g10  ,  b  are  in  G.P.  and  h  is  the  H.M.  between  a  and  b,  then

S
a1  a 2  .....  a 10 a 2  a 3  .....  a 9 a  a6
  .......  5
g1 g10 g2 g9 g 5 g 6 is -

E
10 15 30 5
(A)  (B)  (C)  (D) 
h h h h

E
n(n  1) 2
6. The  sum  of  the  first  n  terms  of  the  series  12  +  2.22  +  32  +  2.42  +  52  +  2.62  +  .......  is  ,  when  n  is

J
2

even.  When  n  is  odd,  the  sum  is  -

2
n (n  1) n(n  1)(2n  1) n(n  1) 2 n 2 (n  1)2
(A)  (B)  (C)  (D) 
2 6 2 2

7. If  (1  +  3  + 5  +...+ a)  + (1  +  3  + 5  +...+  b)  =  (1  +  3  +  5  +  ....  +  c),  where  each  set  of  parentheses
contains  the  sum  of  consecutive  odd  integers  as  shown  such  that  -  (i)  a  +  b  +  c  =  21,  (ii)  a  >  6
If  G  =  Max{a,  b,  c}  and  L  =  Min{a,  b,  c},    then  -
(A) G  –  L  =  4 (B) b  –  a  =  2 (C) G  –  L  =  7 (D) a  –  b  =  2

8. If  a,  b  and  c  are  distinct  positive  real  numbers  and  a2  +  b2  +  c2  =  1,  then  ab  +  bc  +  ca  is  -
(A) equal  to  1 (B)  less  than  1 (C)    greater  than  1 (D) any  real  number
9. Let  p,  q,  r    R+  and  27  pqr    (p  +  q  +  r)3  and  3p  +  4q  +  5r  =  12  then  p3  +  q4  +  r5  is  equal  to  -
(A) 2 (B) 6 (C) 3 (D) none  of  these
10. The  sum  of  the  first  100  terms  common  to  the  series  17,  21,  25,  .........  and  16,  21,  26,  ...............is  -
(A) 101100 (B) 111000 (C) 110010 (D) 100101

Page 36
JEE Sprint Mathematics
If  a,  b,  c  are  positive  such  that  ab 2 c 3   =  64  then  least  value  of 
FG 1  2  3 IJ
11.
Ha b cK is  -

(A) 6 (B) 2 (C) 3 (D) 32

12. If  a1,  a2,..................an    R+  and  a1.a2.......  an  =  1  then  the  least  value  of (1  a 1  a 12 )(1  a 2  a 22 ).....(1  a n  a 2n )

is -
(A) 3n (B) n3n (C) 33n (D) data  inadequate
13. Let  a 1 ,  a 2 ,  a 3 ,........  and  b 1 ,  b 2 ,  b 3 ,........  be  arithmetic  progression  such  that  a 1   =  25,  b 1   =  75  and
a100  +  b100  =  100,  then  -
(A) The common difference in progression 'ai' is equal but opposite in sign to the common difference in progression

T
'bj'.
(B) an  +  bn  =  100  for  any  n.
(C) (a1  +  b1),  (a2  +  b2),  (a3  +  b3),  .......  are  in  A.P.

N
100

I
(D)   (a r  b r )  10 4
r 1

14. If  the  AM  of  two  positive  numbers  be  three  times  their  geometric  mean  then  the  ratio  of  the  numbers  is  -

R
2
(A) 3  2 2 (B)  2  1 (C) 17  12 2 (D) 3  2 2 

P
15. If  first  and  (2n  –  1)th  terms  of  an  A.P.,  G.P.  and  H.P.  are  equal  and  their  nth  terms  are  a,  b,  c  respectively,  then -

S
(A) a  +  c  =  2b (B) a  b  c (C) a  +  c  =  b (D) b2  =  ac
16. Let  a,  x,  b  be  in  A.P.  ;    a,  y,  b  be  in  G.P.  and  a,  z,  b  be  in  H.P.  If  x  =  y  +  2  and  a  =  5z  then  -

9 1

E
(A) y2  =  xz (B) x  >  y  >  z (C) a  =  9,  b  =  1 (D) a  = ,  b  = 
4 4

17. The  pth  term  Tp  of  H.P.  is  q(q  +  p)  and  qth  term  Tq   is  p(p  +  q)  when  p  >  1,  q  >  1,  then  -

E
(A) Tp  +  q  =  pq (B) Tpq    =  p  +  q (C) Tp  +  q  >  Tpq (D) Tpq  >  Tp+q

J
18. a,  b,  c  are  three  distinct  real  numbers,  which  are  in  G.P.  and  a  +  b  +  c  =  xb,  then  -
(A) x  <  –1 (B) –1  <  x  <  2 (C) 2  <  x  <  3 (D) x  >  3
19. Let  a1,  a 2,  .......,  a 10  be  in  A.P.  &  h 1,  h 2,  .......h 10  be  in  H.P. .  If  a1= h 1 = 2  &  a 10  =  h 10  =  3  then  a 4h 7  is  -
     

(A) 2 (B) 3 (C) 5 (D) 6

ANSWER  KEY
Que. 1 2 3 4 5 6 7 8 9 10
Ans. A A A,C B,D C A A,D B C A
Que. 11 12 13 14 15 16 17 18 19
Ans. C A A,B,C,D C,D B,D A,B,C A,B,C A,D D

Page 37
38
JEE Sprint Mathematics
COMPLEX NUMBERS

SELECT  THE  CORRECT  ALTERNATIVE  (ONLY  ONE  CORRECT  ANSWER)


13
1. The  value  of  the  sum  i n

 i n 1   ,  where  i  1   ,  equals [JEE  98]
n 1

(A) i (B) i – 1 (C) –i (D) 0

2. The  sequence  S  =  i  +  2i 2  +  3i 3  +  ......  upto  100  terms  simplifies  to  where  i  = 1   -


(A) 50(1  –  i) (B) 25i (C) 25(1  +  i) (D) 100(1  –  i)

3. Let  i  1 .  The  product  of  the  real  part  of  the  roots  of  z 2  –  z  =  5  –  5i  is  -
(A) –25 (B) –6 (C) –5 (D) 25

T
1 1
4. If z1  =  , a    0  and  z2 = ,  b    0  are such  that  z 1  z2   then  -
ai 1  bi
(A) a  =  1,  b  =  1 (B)  a  =  1,  b  =  –1 (C) a  =  –1,  b  =  1 (D) a  =  –1,  b  =  –1

N
5. The  inequality  |z  –  4|  <  |z  –  2|  represents  the  following  region  -

I
(A) Re(z)  >  0 (B) Re(z)  <  0 (C) Re(z)  >  2 (D) none  of  these
2
6. If  (1  +  i)  (  1  +  2i)  (  1  +  3i)  ....  (1  +  ni)  =    +  i  then  2  .  5  .  10  ...  (1  +  n )  =

R
(A)  – i (B) 2 – 2 (C)  2  +  2 (D)  none  of  these
7. In the  quadratic equation  x2  +  (p +iq)  x  +  3i =  0  ,  p &  q  are  real.  If the  sum  of  the squares  of  the  roots is  8  then :

P
(A) p  =  3,  q  =  –1 (B) p  =  –3,  q  =  –1
(C) p  =  3,  q  =  1  or  p  =  –3,  q  =  –1 (D) p  =  –3,  q  =  1

S
8. The  curve  represented  by  Re(z2)  =  4  is  -
(A) a  parabola (B) an  ellipse
(C) a  circle (D) a  rectangular  hyperbola

E
i
9. Real  part  of  e e is  -
(A) ecos    [cos  (sin  )] (B) ecos    [cos  (cos  )] (C) esin   [sin  (cos  )] (D) esin    [sin  (sin  )]

E
10. Let  z  and    are  two  non-zero  complex  numbers  such  that  |z|  =  ||  and  arg  z  +  arg    =  ,  then  z  equal

J
to  -
(A)  (B) –   (C)  (D) –  
11. Number  of  values  of  x  (real  or  complex)  simultaneously  satisfying  the  system  of  equations
1  +  z  +  z 2  +  z 3  +  ........  +  z 17  =  0  and  1  +  z  +  z 2  +  z 3  +  .........  +  z 13  =  0  is  -
(A) 1 (B) 2 (C) 3 (D) 4
12. If  |z1|  =  1,  |z2|  =  2,  |z3|  =  3  and  |9z1z2 +  4z1z3 +  z2z3|  =  12  then  the  value  of  |z1 +  z2 +  z3|  is  equal  to-
(A) 2 (B) 3 (C) 4 (D) 6
13. A  point  ‘z’  moves  on  the  curve  |  z  –  4  –  3i  |=  2  in  an  argand  plane.  The  maximum  and  minimum  values  of  |z|
are  -
(A) 2, 1 (B) 6,  5 (C) 4, 3 (D) 7,  3
14. The  set  of  points  on  the  complex  plane  such  that  z2  +  z  +  1  is  real  and  positive  (where  z  =  x  +  iy,  x, y  R )  is-
(A) Complete  real  axis  only
(B) Complete  real  axis  or  all  points  on  the  line  2x  +  1  =  0
 1 3  1 3
(C) Complete  real  axis  or  a  line  segment  joining  points   ,    &    2 ,  2    excluding  both.
 2 2   
(D) Complete  real  axis  or  set  of  points  lying  inside  the  rectangle  formed  by  the  lines.

2x + 1 = 0 ; 2x – 1 = 0 ;  2y  3  0  &   2y  3  0

Page 38
39
JEE Sprint Mathematics
15. If     is  an  imaginary  cube  root  of  unity,  then    (1     2 )7   equals [JEE  98]
(A) 128 (B) –128 (C) 1282 (D) –1282
334 365
 1 i 3  1 i 3
16. If  i  1   , then 4  5     3   is  equal to  : [JEE  99]
 2 2   2 2 

(A) 1  i 3 (B) 1  i 3 (C) i 3 (D)  i 3



17. The  set  of  points  on  an  Argand  diagram  which  satisfy  both  |z|   4  &  Arg z  are  lying  on  -
3
(A) a  circle  &  a  line (B) a  radius  of  a  circle (C) a  sector  of  a  circle (D) an  infinite  part  line


18. If Arg  (z –  2 –  3i) =  ,  then the  locus of  z is  -
4

T
y y
(2,3) (2,3)

(A)  (B)  (C)  x (D)  x

N
(–2,–3)

I
(–2,–3)
19. The  origin  and  the  roots  of  the  equation  z2  +  pz  +  q  =  0  form  an  equilateral  triangle  if  -
(A) p2  =  2q (B) p2  =  q (C) p2  =  3q (D) q2  =  3p

R
20. Points    z 1  &  z 2  are  adjacent  vertices  of  a  regular  octagon.  The  vertex  z 3  adjacent  to    z 2  (z 3    z 1)  can  be

P
represented  by    -
1 1
(A) z 2  (1  i)(z 1  z 2 ) (B)  z 2  ( 1  i)(z 1  z 2 )

S
2 2
1
(C) z 2  ( 1  i)(z 2  z 1 ) (D)    none  of  these
2

E
6 6 5 5
 1  i 3   1  i 3   1  i 3   1  i 3 
21.           is  equal  to  -
 2   2   2   2 

JE
(A) 1 (B) –1 (C) 2 (D) none  of  these
22. If  z  and    are  two  non-zero  complex  numbers  such  that  |z|  =  1,  and  Arg  (z)  –  Arg()  =  /2,  then  z    is
equal  to  -
(A) 1 (B) –1 (C) i (D) –i

SELECT  THE  CORRECT  ALTERNATIVES  (ONE  OR  MORE  THAN  ONE  CORRECT  ANSWERS)
23. For  two  complex  numbers  z1  and  z2  :  (az1  bz1 )(cz 2  dz2 )  (cz1  dz1 )(az 2  bz2 )   if  (a,  b,  c,  d    R)  -
a c a b
(A)   (B)   (C) | z1 | | z 2 | (D) arg(z1)  =  arg(z2)
b d d c
24. Which  of  the  following,  locii  of  z  on  the  complex  plane  represents  a  pair  of  straight  lines  ?
(A) Re(z2)  =  0 (B) Im(z2)  =  0 (C) |z|  +  z  =  0 (D) |z  –  1|  =  |z  –  i|
25. If  the  complex  numbers  z1,  z2,  z3  represents  vertices  of  an  equilateral  triangle  such  that    |z 1|=|z2|=|z 3|,  then
which  of  following  is  correct  ?
(A) z1  +  z2  +  z3    0 (B) Re(z1  +  z2  +  z3)  =  0 (C) Im(z1  +  z2  +  z3)  =  0 (D) z1  +  z2  +  z3  =  0
| x  1  2i| 2
26. If  S  be  the  set  of  real  values  of  x  satisfying  the  inequality  1  –  log2  0 ,  then  S  contains  -
2 1
(A) [–3,  –1) (B) (–1,  1] (C) [–2,  2] (D) [–3,  1]

Page 39
40
JEE Sprint Mathematics
27. If  amp  (z1z2)  =  0  and  |z1|  =  |z2|  =  1,  then  :-
(A) z1  +  z2  =  0 (B) z1z2  =  1 (C) z1  =  z2 (D) none  of  these

28. If  the  vertices  of  an  equilateral  triangle  are  situated  at  z  =0,  z=z1,  z  =z2,  then  which  of  the  following  is/are  true -
(A) |z 1 |  =  |z 2 | (B) |z 1  –  z 2|  =  |z 1|
(C) |z 1  +  z 2|  =  |z 1 |  +  |z 2 | (D) | arg z1  –  arg  z2|=  /3
29. Value(s)  of  (–i)1/3  is/are  -

3 i 3 i  3 i  3 i
(A)  (B)  (C)  (D) 
2 2 2 2
30. If  centre  of  square  ABCD  is  at  z=0.  If  affix  of  vertex  A  is  z1,  centroid  of  triangle  ABC  is/are  -
    

T
z1
(A)  (cos    +  i  sin  ) (B) 4  cos   i  sin  
3  2 2 
z 1      z 1     
 cos   i  sin    cos   i  sin  

N
(C)  (D) 
3  2 2  3  2 2 

I
x 1  2
31. If    is  an  imaginary  cube  root  of  unity,  then  a  root  of  equation   x  2 1 =  0,  can  be  :-

R
2
 1 x 2

(A) x  =  1 (B) x  =   (C) x  =  2 (D) x  =  0

S P
JE E
ANSWER  KEY
Que. 1 2 3 4 5 6 7 8 9 10
Ans. B A B B D C C D A D
Que. 11 12 13 14 15 16 17 18 19 20
Ans. A A D B D C C A C B
Que. 21 22 23 24 25 26 27 28 29 30
Ans. A D A,D A,B B,C,D A,B B.C A,B,D A,C C,D
Que. 31
Ans. D

Page 40
41
JEE Sprint Mathematics
EXTRA PRACTICE QUESTIONS ON COMPLEX NUMBERS
SELECT  THE  CORRECT  ALTERNATIVES  (ONE  OR  MORE  THAN  ONE  CORRECT  ANSWERS)

1. On  the  argand  plane,  let    2  3z   ,    2  3z   &  |  z  |  =  1.  Then  the  correct  statement  is  -

(A)   moves  on  the  circle,  centre  at  (–2,0)  and  radius  3
(B)   &    describe  the  same  locus
(C)   &    move  on  different  circles
(D)   –    moves  on  a  circle  concentric  with  |z|=1

2. The  value  of  in  +  i–n,  for  i  1   and  n  I   is  -

2n (1  i) 2 n (1  i) 2 n (1  i) 2 n (1  i) 2 n 2n 2n 2n

T
(A)   (B)   (C)  (D) 
(1  i)2 n 2n 2n 2n 2n (1  i)2 n (1  i) 2 n (1  i) 2 n

3. The  common  roots  of  the  equations  z3  +  (1 +  i)z2  +  (1  +  i)z  +  i  =  0,  (where  i  = 1 ) and  z1993  +  z1994  +  1  = 0  are -

N
(where    denotes  the  complex  cube  root  of  unity)
(A) 1 (B)  (C) 2 (D) 981

I
 
4. If  x r  CiS  r   for  1  r  n ; r, 
,  n  N  then -
2 

R
 n   n   n   n 
(A)  Lim Re   x r   1 (B) Lim Re   x r   0 (C) Lim Im   x r   1 (D) Lim Im   x r   0
n   r 1  n   r 1  n   r 1  n   r 1 

P
5. Let  z 1 ,  z 2  be  two  complex  numbers  represented  by  points  on  the  circle  |z 1|  =  1  and  |z 2|=2  respectively,
then  -

S
1
(A) max|2z 1 +  z 2 |  =  4 (B) min  |z1  –  z2|  =  1 (C)  z 2  3 (D)    none  of  these
z1

E

6. If  ,    be  any  two  complex  numbers  such  that   1 ,  then  which  of  the  following  may  be  true  -
1 

E
(A) |  | 1 (B)  | | 1 (C)    e i ,   R (D)     e i ,   R

J
7. Let  z,  z  and  z  +  z  represent  three  vertices  of  ABC,  where    is  cube  root  unity,  then  -
2 2
(A) centroid  of  ABC  is (z  z) (B) orthocenter  of  ABC  is (z  z)
3 3
(C) ABC  is  an  obtuse  angled  triangle (D) ABC  is  an  acute  angled  triangle
8. Which  of  the  following  complex  numbers  lies  along  the  angle  bisectors  of  the  line  -
L1  :  z  =  (1  +  3)  +  i(1  +  4)
L2  :  z  =  (1  +  3)  +  i(1  –  4)
11 3i
(A)  i (B) 11  +  5i (C) 1– (D) 5  –  3i
5 5
9. Let  z  and    are  two  complex  numbers  such  that  |z|    1,  ||    1  and  |z  +  i|  =  |z  –  i  |  =  2,  then  z
equals  -
(A) 1  or  i (B) i  or  –i (C) 1  or  –1 (D) i  or  –1
10. If  g(x)  and  h(x)  are  two  polynomials  such  that  the  polynomial  P(x)  =  g(x3)  +  xh(x3)  is  divisible  by  x2  +  x  +  1,  then -
(A) g(1)  =  h(1)  =  0 (B) g(1)  =  h(1)    0 (C) g(1)  =  –h(1) (D) g(1)  +  h(1)  =  0

ANSWER  KEY
Que. 1 2 3 4 5 6 7 8 9 10
Ans. A,B,D B,D B,C A,D A, B, C A,B, C,D A,C A,C C A ,C , D

Page 41
42
JEE Sprint Mathematics
BINOMIAL THEOREM
SELECT THE CORRECT ALTERNATIVE (ONLY ONE CORRECT ANSWER)

n
 x
1. If the coefficients of x7 & x8 in the expansion of 2   are equal , then the value of n is -
 3
(A) 15 (B) 45 (C) 55 (D) 56
n
 1
2. The sum of the binomial coefficients of 2 x   is equal to 256 . The constant term in the expansion
 x
is -
(A) 1120 (B) 2110 (C) 1210 (D) none
3. The sum of the coefficients in the expansion of (1  2x + 5x2)n is ' a ' and the sum of the coefficients in the

T
expansion of (1 + x)2n is b . Then -
(A) a = b (B) a = b2 (C) a2 = b (D) ab = 1
4. Given that the term of the expansion (x1/3  x1/2)15 which does not contain x is 5 m where m  N , then m is

N
equal to -

I
(A) 1100 (B) 1010 (C) 1001 (D) none
  1  4 x  1 7  1  4 x  1 7 
1

R
5. The expression      is a polynomial in x of degree -
4 x  1   2 
 
 2  

P
(A) 7 (B) 5 (C) 4 (D) 3
1/3 1/3 n
6. In the binomial (2 +3 ) , if the ratio of the seventh term from the beginning of the expansion to the

S
seventh term from its end is 1/6 , then n is equal to -
(A) 6 (B) 9 (C) 12 (D) 15
11
2  3
7. The term independent of x in the product (4  x  7x )  x  

E
is -
 x
(A) 7. 11 C 6 (B) 36. 11C6 (C) 35. 11C5 (D) –12. 211

E
8. If ‘a’ be the sum of the odd terms & ‘b’ be the sum of the even terms in the expansion of (1 + x)n , then

J
(1  x²)n is equal to -
(A) a²  b² (B) a² + b² (C) b²  a² (D) none
9. The sum of the coefficients of all the even powers of x in the expansion of (2x2  3x + 1)11 is -
(A) 2 . 610 (B) 3 . 610 (C) 611 (D) none
10. The greatest terms of the expansion (2x + 5y)13 when x = 10, y = 2 is -
13
(A) C5 . 208 . 105 (B) 13
C6 . 207 . 104 (C) 13
C4 . 209 . 104 (D) none of these
100
11. Number of rational terms in the expansion of  2 43  is -
(A) 25 (B) 26 (C) 27 (D) 28


p  n
12. If    0 for p < q, where p, q  W, then   2r  
q  r 0  

(A) 2n (B) 2n–1 (C) 22n–1 (D) 2n


Cn

 47  5  52  j   x  x
13.         , then y =
 4  j 1  3   y 

(A) 11 (B) 12 (C) 13 (D) 14

Page 42
43
JEE Sprint Mathematics
1 1 1 1
14. If n  N & n is even , then    ......  =
1. (n  1) ! 3 ! (n  3) ! 5 ! (n  5) ! (n  1) ! 1 !
2 n 1
(A) 2n (B) (C) 2n n ! (D) none of these
n!

15. Let R  (5 5  11) 31  I  ƒ , where I is an integer and ƒ is the fractional part of R, then R · ƒ is equal to -
(A) 231 (B) 3 31 (C) 262 (D) 1

10
 10  15 
16. The value of    is equal to -
r  0  r  14  r 

(A) 25C (B) 25C (C) 25C (D) 25 C


12 15 10 11

C0 C C C

T
17.  1  2  ......  10 is equal to (here Cr = 10
C r)
1 2 3 11
211 211  1 311 311  1
(A) (B) (C) (D)

N
11 11 11 11

I
n n
1 r
18. If a n   n
Cr
, then  n
Cr
equals - [JEE 98]
r 0 r 0

R
(A) ( n -1) an (B) n an (C) n an /2 (D) none of these
19. The last two digits of the number 3400 are -
(A) 81 (B) 43 (C) 29 (D) 01

P
SELECT THE CORRECT ALTERNATIVES (ONE OR MORE THAN ONE CORRECT ANSWERS)

S
20. If the coefficients of three consecutive terms in the expansion of (1 + x)n are in the ratio of 1 : 7 : 42, then
n is divisible by -
(A) 9 (B) 5 (C) 3 (D) 11

E
20
 1 
21. In the expansion of  3 4  4  -
 6

E
(A) the number of irrational terms = 19 (B) middle term is irrational

J
(C) the number of rational terms = 2 (D) 9th term is rational
22. If (1 + x + x 2 + x 3)100 = a0 + a1x + a2x 2 + ......... + a 300x300, then -
(A) a0 + a1 + a2 + a3 +.......+ a300 is divisible by 1024
(B) a0 + a2 + a4 +.......+ a 300 = a1 + a 3 + ....... + a 299
(C) coefficients equidistant from beginning and end are equal
(D) a1 = 100
23. The number 101100  1 is divisible by -
(A) 100 (B) 1000 (C) 10000 (D) 100000
n
24. If 9  80  = I+f where I , n are integers and 0 < f < 1 , then -
(A) I is an odd integer (B) I is an even integer
n
(C) (I + f) (1  f) = 1 (D) 1  f = 9  80 
30
 1 
25. In the expansion of  x 2 / 3   , a term containing the power x13 -
 x
(A) does not exist (B) exists and the co-efficient is divisible by 29
(C) exists and the co-efficient is divisible by 63 (D) exists and the co-efficient is divisible by 65

Page 43
44
JEE Sprint Mathematics
2n
26. The co-efficient of the middle term in the expansion of (1 + x) is -
1.3.5 .7......(2 n  1)
(A) 2n (B) 2n
Cn
n!

(n  1) (n  2) (n  3) .... (2n  1) (2n) 2 .6 .10 .14 ...... (4n  6) (4n  2)


(C) (D)
1.2 .3.......... (n  1) n 1.2.3 .4 .....(n  1) . n

N T
R I
S P
JE E
ANSWER KEY
Que. 1 2 3 4 5 6 7 8 9 10
Ans. C A A C D B B A B C
Que. 11 12 13 14 15 16 17 18 19 20
Ans. B B C B C D B C D B,D
Que. 21 22 23 24 25 26
Ans. A,B, C,D A,B, C,D A, B, C A ,C , D B,C,D A,B, C,D

Page 44
45
JEE Sprint Mathematics
EXTRA PRACTICE QUESTIONS ON BINOMIAL THEOREM
SELECT THE CORRECT ALTERNATIVES (ONE OR MORE THAN ONE CORRECT ANSWERS)
1. The coefficient of xr (0  r  n  1) in the expression :
(x + 2)n1 + (x + 2)n2 . (x + 1) + (x + 2)n3 . (x + 1)² + ...... + (x + 1)n1 is -
n
(A) Cr (2r  1) (B) n
Cr (2nr  1) (C) n
Cr (2r + 1) (D) n
Cr (2nr + 1)
2. If (1 + x + x²)25 = a0 + a1x + a2x² + ..... + a50 . x50 then a0 + a2 + a4 + ..... + a50 is -
(A) even (B) odd & of the form 3n
(C) odd & of the form (3n  1) (D) odd & of the form (3n + 1)
4 2 12
3. The co-efficient of x in the expansion of (1  x + 2x ) is -
12 13 14 12 13 14
(A) C3 (B) C3 (C) C4 (D) C3 + 3 C3 + C4

T
22 n 2
4. Let (1 + x ) (1 + x) = A0 + A1 x + A2 x + ...... If A0, A1, A2 are in A.P. then the value of n is -
(A) 2 (B) 3 (C) 5 (D) 7
nr

N
5. If  nk
Cr = xCy then -

I
k 1

(A) x = n + 1 ; y = r (B) x = n ; y = r + 1
(C) x = n ; y = r (D) x = n + 1 ; y = r + 1

R
t m1 m2
6. Co-efficient of  in the expansion of ( + p) + ( + p) ( + q) + ( + p)m  3 ( + q)2 +...... ( + q)m  1 where
 q and p  q is -

P
(A)
m

Ct pt  qt  (B)
m

C t pm  t  q m  t  (C)
m

Ct pt  qt  (D)
m

C t pm  t  q m  t 
p q p q p q pq

S
7. The co-efficient of x401 in the expansion of (1 + x + x2 + ...... + x9) 1 , (x < 1) is -
(A) 1 (B)  1 (C) 2 (D)  2

E
8. Number of terms free from radical sign in the expansion of (1 + 31/3 + 71/7)10 is -
(A) 4 (B) 5 (C) 6 (D) 8

E
 30r  15r    30r 1  151   .......   300  15r  is maximum is/are

J
9. The value r for which -

(A) 21 (B) 22 (C) 23 (D) 24


n
3 x
10. If the 6 th
term in the expansion of    when x = 3 is numerically greatest then the possible integral
 2 3
value(s) of n can be -
(A) 11 (B) 12 (C) 13 (D) 14
11. In the expansion of (1 + x)n (1 + y)n (1 + z)n , the sum of the co-efficients of the terms of degree ' r ' is -

(B) n C
3
(A) n C r (C) 3n
Cr (D) 3 . 2nCr
r3

 35  10  45  r   x 
12.         , then x – y is equal to -
 6  r 0  5   y 
(A) 39 (B) 29 (C) 52 (D) 40
s n
13. The value of   n C ss C r is -
r 0 s 1
r s

(A) 3 n – 1 (B) 3 n + 1 (C) 3 n (D) 3(3 n – 1)

Page 45
46
JEE Sprint Mathematics
11
 3  log 2 x3 
14. In the expansion of  x  3 .2  -

(A) there appears a term with the power x2
(B) there does not appear a term with the power x2
(C) there appears a term with the power x 3
1
(D) the ratio of the co-efficient of x3 to that of x 3 is
3
15. The sum of the series (1² + 1).1! + (2² + 1).2! + (3² + 1).3! + ..... + (n² + 1).n! is -
(A) (n + 1) . (n + 2)! (B) n . (n + 1)! (C) (n + 1) .(n + 1)! (D) none of these
3n
 1 
16. The binomial expansion of  x k  2 k  , n  N contains a term independent of x -
 x 

T
(A) only if k is an integer (B) only if k is a natural number
(C) only if k is rational (D) for any real k

N
n 2 n
17. Let n  N. If (1 + x) = a0 + a1x + a2x + ........+ anx and an–3, an–2, an–1 are in AP, then -
(A) a1, a2, a3 are in AP (B) a1, a2, a3 are in HP

I
(C) n = 7 (D) n = 14
18 18 18 20
18. Set of values of r for which, Cr  2 + 2 . Cr  1 + Cr  C13 contains -

R
(A) 4 elements (B) 5 elements (C) 7 elements (D) 10 elements

S P
JE E
ANSWER KEY
Que. 1 2 3 4 5 6 7 8 9 10
Ans. B A D A,B B B B C B,C B,C,D
Que. 11 12 13 14 15 16 17 18
Ans. C D A B,C,D B D A,C C

Page 46
47
JEE Sprint Mathematics
SELECT THE CORRECT ALTERNATIVE (ONLY ONE CORRECT ANSWER)
PRINCIPLE OF MATHEMATICAL INDUCTION
n
1. The sum of n terms of 12 + (12 + 22) + (12 + 22 + 32)  n 1 
13. If n is a natural number then    n! is true
+ .... is-  2 
n(n  1) (2n  1) n(n  1) (2n  1) when-
(1) (2)
6 6 (1) n > 1 (2) n  1 (3) n > 2 (4) Never
1 1
(3) n(n + 1)2 (n + 2) (4) n2 (n + 1)2 14. For natural number n, 2n (n – 1) ! < nn , if-
12 12
2. T he greatest positive i nteger. which divide s (1) n < 2 (2) n > 2 (3) n  2 (4) never
(n + 16) (n + 17) (n + 18) (n + 19), for all n N, is- 15. For every positive integer
(1) 2 (2) 4 (3) 24 (4) 120 n n5 2n 3 n
3. Let P(n) : n2 + n is an odd integer. It is seen that n, + + – is-

T
7 5 3 105
truth of P(n)  the truth of P(n + 1). Therefore, (1) an integer
P(n) is true for all- (2) a rational number

N
(1) n > 1 (2) n (3) a negative real number
(4) an odd integer

I
(3) n > 2 (4) None of these
4. For every natural number n- 16. For positive integer n, 3n < n! when-
(1) n > 2n (2) n < 2n (3) n  2n (4) n  2n (1) n  6 (2) n > 7 (3) n  7 (4) n  7

R
5. If n  N, then x 2n–1
+ y 2n–1
is divisible by- a 1
17. If A =   , then for any n  N, An equals-
(1) x + y (2) x – y (3) x2 + y2 (4) x2 + xy 0 a 

P
6. The inequality n! > 2n–1 is true-  na n  an na n 1 
(1)   (2)  
(1) for all n > 1 (2) for all n > 2 0 na  0 an 

S
(3) for all n  N (4) None of these  na 1   an n
(3)   (4)  
7. 2 2 2
1.2 + 2.3 + 3.4 + ..... upto n terms, is equal to- 0 na  0 an 
1 18. The sum of n terms of the series

E
(1) n(n + 1) (n + 2) (n + 3)
12 1 2 2 3 3 4
. . .
1 2 2 + 2 2 + 2 2

E
(2) n(n + 1) (n + 2) (n + 5) + ....... is-
12 13 13  2 3 13  2 3  3 3

J
1 1 n n 1 n 1
(3) n(n + 1) (n + 2) (3n + 5) (1) (2) (3) (4)
12 n(n  1) n 1 n n 2
(4) None of these
19. For all n  N, 72n – 48n – 1 is divisible by-
8. The sum of the cubes of three consecutive natural
numbers is divisible by- (1) 25 (2) 26 (3) 1234 (4) 2304

(1) 2 (2) 5 (3) 7 (4) 9 20. The nth term of the series
9. If n  N, then 11n+2 + 122n+1 is divisible by- 4 + 14 + 30 + 52 + 80 + 114 + ..... is-
(1) 113 (2) 123 (1) 5n – 1 (2) 2n2 + 2n (3) 3n2 + n (4) 2n2 + 2
(3) 133 (4) None of these 21. n
If 10 + 3.4 n+2
+  is exactly divisible by 9 for all
4n+2 2n+1
10. If n  N, then 3 + 5 is a multiple of- n  N, then the least positive integral value of  is-
(1) 14 (2) 16 (3) 18 (4) 20 (1) 5 (2) 3 (3) 7 (4) 1
2n+1
11. For each n  N, 10 + 1 is divisible by- 22. The sum of n terms of the series
(1) 11 (2) 13 1 + (1 + a) + (1 + a + a2) + (1 + a + a2 + a3) +.....,
(3) 27 (4) None of these is-
12. The difference between an +ve integer and its cube n a(1  a n ) n a(1  a n )
(1) – (2) +
is divisible by- 1a (1  a )2 1a (1  a )2
(1) 4 (2) 6 n a(1  a n ) n a(1  a n )
(3) 9 (4) None of these (3) + (4) – +
1a (1  a) 2 1a (1  a )2

Page 47
48
JEE Sprint Mathematics
23. For all n  N, n4 is less than- 1 1 1
31. + + + ..... upto n terms is-
(1) 10 n (2) 4n 1.3 3.5 5.7
(3) 10 10 (4) None of these 1 n 1 2n
(1) (2) (3) (4)
24. For all n  N, n 2n  1 2n  1 2n –1 3(n  1)
n–2
32. For positive integer n, 10 > 81n when-
(2n  1) 2 (2n  1) 2
(1) < (2) > (1) n < 5 (2) n > 5 (3) n  5 (4) n > 6
8 8
33. If P is a prime number then np – n is divisible by p
(2n  1) 2
(3) = (4) None of these when n is a
8
(1) natural number greater than 1
25. For all n  N, cos cos2 cos4 ...... cos 2 n – 1 
(2) odd number
equals to-
(3) even number
sin 2 n  sin 2 n 

T
(1) (2) (4) None of these
2 n sin  sin  34. 1 + 3 + 6 + 10 + ...... upto n terms is equal to-
cos 2 n  cos 2 n  1 1
(3) n (4) n

N
2 cos 2  2 sin  (1) n(n + 1)(n + 2) (2) n(n + 1)(n + 2)
3 6
26. For all positive integral values of n, 32n – 2n + 1 is

I
1 1
divisible by- (3) n(n + 2)(n + 3) (4) n(n + 1)(n + 2)
12 12
(1) 2 (2) 4 (3) 8 (4) 12 35. A student was asked to prove a statement by

R
2 2 2 2 2 induction. He proved
12 1 2 1 2 3
27. + + + ... upto n terms is- (i) P(5) is true and
1 1 2 1 2 3

P
(ii) Truth of P(n)  truth of p(n + 1), n  N
1 1 2 On the basis of this, he could conclude that P(n) is

S
(1) (2n + 1) (2) n true for
3 3
(1) no n  N (2) all n  N
1 1 (3) all n  5 (4) None of these
(3) (n + 2) (4) n(n + 2)

E
3 3 36. The sum of the series
3 5 7
28. The smallest positive integer for which the
2 + 12  2 2 + 1 2  2 2  3 2 + ..... upto n terms
ms

E
statement 3n+1 < 4n holds is- 1
3n

J
(1) 1 (2) 2 (3) 3 (4) 4 2n 3n 6n
(1) (2) (3) (4)
29. Sum of n terms of the series n 1 n 1 2(n  1) n 1
1 1 1 1 3 7 15
+ + + ...... is- 37. + + + + ... upto n terms equal to-
o-
1 1 2 1 2 3 2 4 8 16

n 2 2n 2(n  1) 1 1
(1) (2) (3) (4) (1) n + n (2) 2n +
n 1 n(n  1) n 1 n2 2 2n
30. For every natural number n, n(n + 3) is always- 1 1
(3) n – 1 + n (4) n + 1 +
(1) multiple of 4 (2) multiple of 5 2 2n
(3) even (4) odd

ANSWER KEY
Que. 1 2 3 4 5 6 7 8 9 10 11 12 13 14 15
Ans. 3 3 4 2 1 2 3 4 3 1 1 2 2 2 1
Que. 16 17 18 19 20 21 22 23 24 25 26 27 28 29 30
Ans. 3 2 2 4 3 1 1 1 1 1 1 4 4 3 3
Que. 31 32 33 34 35 36 37
Ans. 2 3 1 2 3 4 3

Page 48
49
JEE Sprint Mathematics
SELECT THE CORRECT ALTERNATIVE (ONLY ONE CORRECT ANSWER)
PREVIOUS YEAR QUESTIONS PRINCIPLE OF MATHEMATICAL INDUCTION EXERCISE-II
2
1. Let S(k) = 1 + 3 + 5 + ...... + (2k – 1) = 3 + k , 4. Statement –1 : For every natural number n 2
then which of the following is true ? [AIEEE-2004]

(1) S(1) is true 1 1 1


+ +... + > n
(2) S(k)  S(k + 1) 1 2 n

(3) S(k) 
 S(k + 1) Statement –2 : For every natural number n  2 ,
(4) Principle of mathematical Induction can be used n  n  1  < n+1. [AIEEE-2008]
to prove that formula
(1) Statement –1 is false, Statement –2 is true
2. The sum of first n terms of the given series (2) Statement–1 is true, Statement–2 is false

T
(3) Statement–1 is true, Statement–2 is true;
n(n  1)2 Statement–2 is a correct explanation for
12 + 2.22 + 32 + 2.42 + 52 + 2.62 + .... is ,
2 Statement–1

N
when n is even. When n is odd, then sum will be- (4) Statement–1 is true, Statement–2 is true;

I
[AIEEE-2004] Statement–2 is not a correct explanation for
Statement–1
n(n  1)2 1 2
(1) (2) n (n + 1) 5. Statement - 1: For each natural number n,
2

R
2
(n + 1) 7 – n 7 –1 is divisible by 7.
(3) n(n + 1)2 (4) None of these Statement - 2: For each natural number n, n7 – n

P
1 0  1 0  is divisible by 7. [AIEEE-2011]
3. If A =   and I =   , then which one of (1) Statement-1 is false, statement-2 is true.
1 1  0 1 
(2) Statement- 1 i s tr u e , s t a t e m e n t - 2 i s t r u e ;

S
the following holds for all n  1, (by the principal S t a t e m e n t - 2 i s c o r r e c t e x p l a n a t i o n for
of mathematical induction) [AIEEE-2005] statement-1.
(1) An = nA + (n – 1)I (2) An = 2n–1 A + (n + 1)I (3) Statement- 1 i s tr u e , s t a t e m e n t - 2 i s t r u e ;

E
Statement-2 is not a correct explanation for
(3) An = nA – (n – 1)I (4) An = 2n–1 A – (n – 1)I statement-1.
(4) Statement-1 is true, statement-2 is false.

JE
ANSWER KEY
Que. 1 2 3 4 5
Ans. 2 2 3 3 2

Page 49
50
JEE Sprint Mathematics
PERMUTATION AND COMBINATIONS
SELECT THE CORRECT ALTERNATIVE (ONLY ONE CORRECT ANSWER)
1. The total number of words which can be formed using all the letters of the word "AKSHI" if each word begins
with vowel or terminates with vowel -
(A) 84 (B) 12 (C) 48 (D) 60
2. The number of different seven digit numbers that can be written using only three digits 1, 2 & 3 under the
condition that the digit 2 occurs exactly twice in each number is -
(A) 672 (B) 640 (C) 512 (D) none of these
3. Out of seven consonants and four vowels, the number of words of six letters, formed by taking four consonants

T
and two vowels is (Assume that each ordered group of letter is a word) -
(A) 210 (B) 462 (C) 151200 (D) 332640

N
4. A 5 digit number divisible by 3 is to be formed using the numerals 0, 1, 2,3,4 & 5 without repetition. The total

I
number of ways this can be done is -
(A) 3125 (B) 600 (C) 240 (D) 216

R
5. The number of ways in which 5 different books can be distributed among 10 people if each person can get at
most one book is -

P
(A) 252 (B) 105 (C) 510 (D) 10
C5 . 5!
6. Number of ways in which 9 different prizes can be given to 5 students, if one particular student receives 4 prizes

S
and the rest of the students can get any numbers of prizes is -
(A) 9C4 . 210 (B) 9C5 . 54 (C) 4 . 45 (D) none of these

E
7. Boxes numbered 1, 2, 3, 4 and 5 are kept in a row and they are necessarily to be filled with either a red
or a blue ball such that no two adjacent boxes can be filled with blue balls. How many different arrangements

E
are possible, given that the balls of a given colour are exactly identical in all respects ?

J
(A) 8 (B) 10 (C) 13 (D) 22
8. Ten different letters of alphabet are given. Words with four letters are formed from these letters, then the
number of words which have at least one letter repeated is -
(A) 104 (B) 10 P 4 (C) 10 C 4 (D) 4960
9. If all the letters of the word “QUEUE” are arranged in all possible manner as they are in a dictionary, then the
rank of the word QUEUE is -
(A) 15th (B) 16th (C) 17th (D) 18th
10. Number of ways in which 9 different toys can be distributed among 4 children belonging to different age groups in
such a way that distribution among the 3 elder children is even and the youngest one is to receive one toy more is -

(5 !) 2 9! 9!
(A) (B) (C) (D) none of these
8 2 3!(2!) 3
11. The number of ways of arranging the letters AAAAA, BBB, CCC, D, EE & F in a row if no two 'C's are
together :

13
12 ! 13 ! 14 ! 13!
(A) C3 . (B) (C) (D) 11.
5 !3 !2 ! 5 !3 !3 !2 ! 5 !3 !2 ! 6!

Page 50
51
JEE Sprint Mathematics
12. Number of numbers greater than a million and divisible by 5 which can be formed by using only the digits
1, 2, 1, 2, 0, 5 & 2 is -
(A) 120 (B) 110 (C) 90 (D) none of these
13. A set contains (2n + 1) elements. The number of subset of the set which contain at most n elements is : -
(A) 2n (B) 2 n+1 (C) 2n – 1 (D) 2 2n
14. The maximum number of different permutations of 4 letters of the word “EARTHQUAKE” is -

(A) 2910 (B) 2550 (C) 2190 (D) 2091


15. The number of ways in which we can arrange n ladies & n gentlemen at a round table so that 2 ladies or 2
gentlemen may not sit next to one another is -

T
(A) (n – 1)! (n – 2)! (B) (n)! (n – !)! (C) (n + 1)! (n)! (D) none of these

16. The number of proper divisors of apbqcrds where a, b, c, d are primes & p, q, r, s  N is -

N
(A) pqrs (B) (p + 1) (q + 1) (r + 1) (s + 1) – 4

I
(C) pqrs – 2 (D) (p + 1) (q + 1) (r + 1) (s + 1) – 2

17. The sum of all numbers greater than 1000 formed by using the digits 1, 3, 5, 7 such that no digit is being

R
repeated in any number is -

(A) 72215 (B) 83911 (C) 106656 (D) 114712

P
18. The number of way in which 10 identical apples can be distributed among 6 children so that each child receives

S
atleast one apple is -

(A) 126 (B) 252 (C) 378 (D) none of these

E
19. Number of ways in which 25 identical pens can be distributed among Keshav, Madhav, Mukund and Radhika
such that at least 1, 2, 3 and 4 pens are given to Keshav, Madhav, Mukund and Radhika respectively, is -

E
(A) 18 C 4 (B) 28 C 3 (C) 24 C 3 (D) 18 C 3

J
SELECT THE CORRECT ALTERNATIVES (ONE OR MORE THAN ONE CORRECT ANSWERS)

20. There are (p + q) different books on different topics in Mathematics. (p  q)

If L = the number of ways in which these books are distributed between two students X and Y such that
X get p books and Y gets q books.

M = The number of ways in which these books are distributed between two students X and Y such that one
of them gets p books and another gets q books.

N = The number of ways in which these books are divided into two groups of p books and q books then -

(A) L = N (B) L = 2M = 2N (C) 2L = M (D) L = M

21. Number of dissimilar terms in the expansion of (x1 + x2 + ...... + xn)3 is -

n 2 (n  1)2 n(n  1)(n  2) n+1 C n+1 C


n 3  3n 2
(A) (B) (C) 2 + 3 (D)
4 6 4

22. A persons wants to invite one or more of his friend for a dinner party. In how many ways can he do so if he has
eight friends : -

(A) 28 (B) 28 – 1 (C) 82 (D) 8C 1 + 8C2 + .....+ 8C 8

Page 51
52
JEE Sprint Mathematics
23. If P(n, n) denotes the number of permutations of n different things taken all at a time then P(n, n) is also

identical to :-

(A) n.P(n – 1, n – 1) (B) P(n, n – 1) (C) r! . P(n, n – r) (D) (n – r) . P(n, r)

where 0  r  n

24. Which of the following statement(s) is/are true :-

(A) 100C is not divisible by 10


50

(B) n(n – 1)(n – 2) .........(n – r + 1) is always divisible by r! (n  N and 0  r  n)

(C) Morse telegraph has 5 arms and each arm moves on 6 different positions including the position of rest.

T
Number of different signals that can be transmitted is 56 – 1.

(D) There are 5 different books each having 5 copies. Number of different selections is 65 –1.

R IN
S P
JE E
ANSWER KEY
Que. 1 2 3 4 5 6 7 8 9 10
Ans. A A C D D A C D C C
Que. 11 12 13 14 15 16 17 18 19 20
Ans. A B D C B D C A D A,C
Que. 21 22 23 24
Ans. B,C B,D A, B, C A,B,D

Page 52
53
JEE Sprint Mathematics
EXTRA PRACTICE QUESTIONS ON PERMUTATION AND COMBINATIONS

SELECT THE CORRECT ALTERNATIVES (ONE OR MORE THAN ONE CORRECT ANSWERS)

1. 5 Indian & 5 American couples meet at a party & shake hands. If no wife shakes hands with her own husband
& no Indian wife shakes hands with a male, then the number of hand shakes that takes place in the party is -
(A) 95 (B) 110 (C) 135 (D) 150
2. The number of ways in which a mixed double tennis game can be arranged from amongst 9 married couple if
no husband & wife plays in the same game is -
(A) 756 (B) 3024 (C) 1512 (D) 6048
3. There are n identical red balls & m identical green balls. The number of different linear arrangements consisting

T
of "n red balls but not necessarily all the green balls" is xCy then -
(A) x = m + n, y = m (B) x = m + n + 1, y = m

N
(C) x = m + n + 1, y = m + 1 (D) x = m + n, y = n

I
4. Number of different words that can be formed using all the letters of the word “DEEPMALA” if two vowels are
together and the other two are also together but separated from the first two is -

R
(A) 960 (B) 1200 (C) 2160 (D) 1440
5. In a unique hockey series between India & Pakistan, they decide to play on till a team wins 5 matches. The

P
number of ways in which the series can be won by India, if no match ends in a draw is -

(A) 126 (B) 252 (C) 225 (D) none of these

S
6. A road network as shown in the figure connect four cities. In how many ways
can you start from any city (say A) and come back to it without travelling

E
on the same road more than once ?

(A) 8 (B) 12

E
(C) 9 (D) 16

J
7. The number of ways of choosing a committee of 2 women & 3 men from 5 women & 6 men, if Mr. A refuses to
serve on the committee if Mr. B is a member & Mr. B can only serve, if Miss C is the member of the committee,
is -

(A) 60 (B) 84 (C) 124 (D) none of these

8. Six persons A, B, C, D, E and F are to be seated at a circular table. The number of ways this can be done if A
must have either B or C on his right and B must have either C or D on his right is -

(A) 36 (B) 12 (C) 24 (D) 18


9. Sum of all the numbers that can be formed using all the digits 2, 3, 3, 4, 4, 4 is -
(A) 22222200 (B) 11111100 (C) 55555500 (D) 20333280
10. N = 22 . 33.5 4.7, then -

(A) Number of proper divisors of N(excluding 1 & N) is 118

(B) Number of proper divisors of N(excluding 1 & N) is 120

(C) Number of positive integral solutions of xy = N is 60

(D) Number of positive integral solutions of xy = N is 120

Page 53
54
JEE Sprint Mathematics
11. Sameer has to make a telephone call to his friend Harish, Unfortunately he does not remember the 7 digit
phone number. But he remembers that the first three digits are 635 or 674, the number is odd and there
is exactly one 9 in the number. The maximum number of trials that Sameer has to make to be successful
is -
(A) 10,000 (B) 3402 (C) 3200 (D) 5000
12. Let Pn denotes the number of ways in which three people can be selected out of ‘n’ people sitting in a row, if no
two of them are consecutive. If Pn+1 – Pn = 15 then the value of ‘n’ is -
(A) 7 (B) 8 (C) 9 (D) 10
13. The number of solutions of x1 + x2 + x3 = 51 (x1, x2, x3 being odd natural numbers) is : -
(A) 300 (B) 325 (C) 330 (D) 350

T
14. The number of positive integral solutions of the equation x1x2x3 = 60 is : -
(A) 54 (B) 27 (C) 81 (D) none of these

N
15. Total number of even divisors of 2079000 which are divisible by 15 are -

I
(A) 54 (B) 128 (C) 108 (D) 72
16. The number of five digit numbers that can be formed using all the digits 0, 1, 3, 6, 8 which are -

R
(A) divisible by 4 is 30
(B) greater than 30,000 and divisible by 11 is 12

P
(C) smaller than 60,000 when digit 8 always appears at ten's place is 6
(D) between 30,000 and 60,000 and divisible by 6 is 18.

S
17. All the 7 digit numbers containing each of the digits 1, 2, 3, 4, 5, 6, 7 exactly once and not divisible by 5 are
arranged in the increasing order. Then -
(A) 1800th number in the list is 3124567 (B) 1897th number in the list is 4213567

E
(C) 1994th number in the list is 4312567 (D) 2001th number in the list is 4315726

JE
ANSWER KEY
Que. 1 2 3 4 5 6 7 8 9 10
Ans. C C B D A B C D A A,D
Que. 11 12 13 14 15 16 17
Ans. B B B A C A,B,D B,D

Page 54
55
JEE Sprint Mathematics
PROBABILITY
SELECT THE CORRECT ALTERNATIVE (ONLY ONE CORRECT ANSWER)
1. A quadratic equation is chosen from the set of all quadratic equations which are unchanged by squaring their
roots. The chance that the chosen equation has equal roots is -
(A) 1/2 (B) 1/3 (C) 1/4 (D) 2/3
2. 5 persons entered the lift cabin on the ground floor of an 8 floor building. Suppose that each of them independently
and with equal probability, can leave the cabin at any other floor, starting from the first. The probability that
all 5 persons leave at different floors is -

5 8 8
5  C5 5! C5 5 !
(A)   (B) 5 (C) 5 (D)
8  8 85

T
8

3 If the integers m and n are chosen at random between 1 and 100, then the probability that a number of the
form 7m + 7n is divisible by 5 equals - [JEE 99]

IN
1 1 1 1
(A) (B) (C) (D)
4 7 8 49

4. There are ten prizes, five A's, three B's and two C's, placed in identical sealed envelopes for the top ten

R
contestants in a mathematics contest. The prizes are awarded by allowing winners to select an envelope at
random from those remaining. When the 8th contestant goes to select the prize, the probability that the remaining

P
three prizes are one A, one B and one C, is -
(A) 1/4 (B) 1/3 (C) 1/12 (D) 1/10

S
5. A & B are two independent events such that P(A )  0.7, P(B)  a & P(A  B)  0.8 , then a =

(A) 5/7 (B) 2/7 (C) 1 (D) none

E
6. A determinant is chosen at random from the set of all determinant of order 2 with elements 0 or 1 only. The
probability that the determinant chosen has the value non negative is -

E
(A) 3/16 (B) 6/16 (C) 10/16 (D) 13/16

J
7. A license plate is 3 letters (of English alphabets) followed by 3 digits. If all possible license plates are equally
likely, the probability that a plate has either a letter palindrome or a digit palindrome (or both), is -

7 9 8
(A) (B) (C) (D) none
52 65 65

8. Two cubes have their faces painted either red or blue. The first cube has five red faces and one blue face. When
the two cubes are rolled simultaneously, the probability that the two top faces show the same colour is 1/2.
Number of red faces on the second cube, is -
(A) 1 (B) 2 (C) 3 (D) 4
9. A is one of the 6 horses entered for a race and is to be ridden by one of two jockeys B or C. It is 2 to 1 that B
rides A, in which case all the horses are equally likely to win, if C rides A, his chance is trebled, Then the odds
against his winning are -

5 18 13 13
(A) (B) (C) (D)
13 5 18 5

10. Lot A consists of 3G and 2D articles. Lot B consists of 4G and 1D article. A new lot C is formed by taking 3
articles from A and 2 from B. The probability that an article chosen at random from C is defective, is -
(A) 1/3 (B) 2/5 (C) 8/25 (D) none

Page 55
56
JEE Sprint Mathematics
11. 'A' and 'B' each have a bag that contains one ball of each of the colours blue, green, orange, red and violet. 'A'
randomly selects one ball from his bag and puts it into B's bag. 'B' then randomly selects one ball from his bag
and puts it into A's bag. The probability that after this process the contents of the two bags are the same, is -
(A) 1/2 (B) 1/3 (C) 1/5 (D) 1/6
12. A bowl has 6 red marbles and 3 green marbles. The probability that a blind folded person will draw a red marble
on the second draw from the bowl without replacing the marble from the first draw, is -
(A) 2/3 (B) 1/4 (C) 5/12 (D) 5/8
13. Two cards are drawn from a well shuffled pack of 52 playing cards one by one. If
A : the event that the second card drawn is an ace and
B : the event that the first card drawn is an ace card.
then which of the following is true ?

T
4 1 1 1
(A) P(A) = ; P(B) = (B) P(A) = ; P(B) =
17 13 13 13

N
1 1 16 4
(C) P(A) = ; P(B) = (D) P(A) = ; P(B) =
13 17 221 51

I
14. An Urn contains 'm' white and 'n' black balls. All the balls except for one ball, are drawn from it. The probability
that the last ball remaining in the Urn is white, is -

R
m n 1 mn
(A) (B) (C) (D)
m n m n (m  n)! (m  n)!

P
15. Three numbers are chosen at random without replacement from {1, 2, 3,....., 10}. The probability that the

S
minimum of the chosen numbers is 3 or their maximum is 7 is -
(A) 1/2 (B) 1/3 (C) 1/4 (D) 11/40
16. If atleast one child in a family with 3 children is a boy then the probability that 2 of the children are boys, is -

E
3 4 1 3
(A) (B) (C) (D)
7 7 3 8

E
17. 7 persons are stopped on the road at random and asked about their birthdays. If the probability that 3 of them

J
K
are born on Wednesday, 2 on Thursday and the remaining 2 on Sunday is , then K is equal to -
76

(A) 15 (B) 30 (C) 105 (D) 210


18. A box has four dice in it. Three of them are fair dice but the fourth one has the number five on all of its faces.
A die is chosen at random from the box and is rolled three times and shows up the face five on all the three
occasions. The chance that the die chosen was a rigged die, is -

216 215 216


(A) (B) (C) (D) none
217 219 219

19. Mr. Dupont is a professional wine taster. When given a French wine, he will identify it with probability 0.9
correctly as French and will mistake it for a Californian wine with probability 0.1. When given a Californian wine,
he will identify it with probability 0.8 correctly as Californian and will mistake it for a French wine with probability
0.2. Suppose that Mr. Dupont is given ten unlabelled glasses of wine, three with French and seven with Californian
wines. He randomly picks a glass, tries the wine and solemnly says : "French". The probability that the wine he
tasted was Californian, is nearly equal to -
(A) 0.14 (B) 0.24 (C) 0.34 (D) 0.44

Page 56
57
JEE Sprint Mathematics
SELECT THE CORRECT ALTERNATIVES (ONE OR MORE THAN ONE CORRECT ANSWERS)
20. For two events A & B, which of the following is/are correct -
(A) (A  B)  B = B (B) (A  B) C  BC = (A  B)C
(C) AC  (A  B)  BC =  (D) (A C  B)  (A  B) = B
21. From a pack of 52 playing cards, face cards and tens are removed and kept aside then a card is drawn at
random from the remaining cards. If
A : The event that the card drawn is an ace
H : The event that the card drawn is a heart
S : The event that the card drawn is a spade
then which of the following holds ?
(A) 9P(A) = 4P(H) (B) P(S) = 4P(A  H) (C) 4P(H) = 3P(A  S) (D) P(H) = 9P(A  S)

T
22. Before a race the chance of three runners A, B & C were estimated to be proportional to 5, 3 & 2 respectively
but during the race A meets with an accident which reduces his chance to 1/3. If the respective chances of B

N
and C are P(B) and P(C) then -

I
2 4 2 4
(A) P(B) = (B) P(C) = (C) P(C) = (D) P(B) =
5 15 5 15

R
23. If E & F are events with P ( E)  P (F) & P  E  F  > 0 , then - [JEE 98]

P
(A) occurrence of E  occurrence of F . (B) occurrence of F  occurrence of E
(C) non – occurrence of E  non – occurrence of F (D) none of the above implications holds.

S
1 1
24. If A and B are two independent events such that P(A) = and P(B) = , then -
2 5

E
3 1
(A) P  A  B   (B) P  A / B  
5 2

JE
5
(C) P  A / A  B   (D) P  A  B / A ' B '   0
6

ANSWER KEY
Que. 1 2 3 4 5 6 7 8 9 10
Ans. A D A A B D A C D C
Que. 11 12 13 14 15 16 17 18 19 20
Ans. B A B A D A B C C A, B, C
Que. 21 22 23 24
Ans. A ,C , D A,B D A,B, C,D

Page 57
58
JEE Sprint Mathematics
EXTRA PRACTICE QUESTIONS ON PROBABILITY

SELECT THE CORRECT ALTERNATIVES (ONE OR MORE THAN ONE CORRECT ANSWERS)
1. If two of the 64 squares are chosen at random on a chess board, the probability that they have a side in common
is -
(A) 1/9 (B) 1/18 (C) 2/7 (D) none of these
2. Let 0 < P (A) < 1, 0 < P (B) < 1 and P (A  B) = P (A) + P (B)  P (A) P (B). Then -

B
(A) P   = P (B) – P (A) (B) P (AC  BC) = P (AC) + P (BC)
A

A
(C) P((A  B)C) = P (AC) P (BC) (D) P   = P (A)

T
B
3. 15 coupons are numbered 1,2,3,.........,15 respectively. 7 coupons are selected at random one at a time with
replacement. The probability that the largest number appearing on a selected coupon is 9 is -

N
6 7 7
 9   8   3 97  87

I
(A)   (B)   (C)   (D)
 16   15   5 15 7
4. A child throws 2 fair dice. If the numbers showing are unequal, he adds them together to get his final score. On

R
the other hand, if the numbers showing are equal, he throws 2 more dice & adds all 4 numbers showing to get
his final score. The probability that his final score is 6 is -

P
145 146 147 148
(A) (B) (C) (D)
1296 1296 1296 1296

S
5. If E1 and E2 are two events such that P(E1)=1/4, P(E2/E1)=1/2 and P(E1/E2) = 1/4 then -
(A) E1 and E2 are independent
(B) E1 and E2 are exhaustive

E
(C) E2 is twice as likely to occur as E1
(D) probabilities of the events E1  E2, E1 and E2 are in G.P..

E
6. Two numbers a and b are selected from the set of natural number then the probability that a2 + b2 is divisible by

J
5 is -
9 7 11 17
(A) (B) (C) (D)
25 18 36 81
7. If a, b and c are three numbers (not necessarily different) chosen randomly and with replacement from the set
{1, 2, 3, 4, 5}, the probability that (ab + c) is even, is -
50 59 64 75
(A) (B) (C) (D)
125 125 125 125
8. For any two events A & B in a sample space :

A P (A )  P (B)  1
(A) P    , P(B)  0 is always true
B P (B)

 
(B) P A  B = P (A) – P (A  B)

(C) P (A  B) = 1 – P  A  P  B  , if A & B are independent


C C

(D) P (A  B) = 1 – P  A  P  B  , if A & B are disjoint


C C

Page 58
59
JEE Sprint Mathematics
1
9. In a horse race there are 18 horses numbered from 1 to 18. The probability that horse 1 would win is , horse
6
1 1
2 is and 3 is . Assuming a tie is impossible, the chance that one of the three horses wins the race, is -
10 8

143 119 47 1
(A) (B) (C) (D)
420 120 120 5
10. The probability that a radar will detect an object in one cycle is p. The probability that the object will be detected
in n cycles is -
(A) 1–pn (B) 1–(1–p)n (C) pn (D) p(1 – p)n–1

11. Two real numbers, x & y are selected at random. Given that 0  x  1; 0  y  1 . Let A be the event that

T
y 2  x ; B be the event that x 2  y , then -

1
(A) P(A  B )  (B) A & B are exhaustive events

N
3

I
(C) A & B are mutually exclusive (D) A & B are independent events.
12. A Urn contains 'm' white and 'n' black balls. Balls are drawn one by one till all the balls are drawn. Probability that

R
the second drawn ball is white, is -

m m(n  1)

P
(A) (B)
m n (m  n)(m  n  1)

m(m  1) mn

S
(C) (D)
(m  n)(m  n  1) (m  n)(m  n  1)

13. Two buses A and B are scheduled to arrive at a town central bus station at noon. The probability that bus A will

E
be late is 1/5. The probability that bus B will be late is 7/25. The probability that the bus B is late given that bus
A is late is 9/10. Then the probabilities

E
(i) neither bus will be late on a particular day and

J
(ii) bus A is late given that bus B is late, are respectively
(A) 2/25 and 12/28 (B) 18/25 and 22/28 (C) 7/10 and 18/28 (D) 12/25 and 2/28
14. If A & B are two events such that P(B)  1, BC denotes the event complementary to B, then -

P(A )  P(A  B)
(A) P(A/BC) = (B) P(A  B)  P(A) + P(B) –1
1  P(B)

(C) P(A) >< P(A/B) according as P(A/BC) >< P(A) (D) P(A/BC) + P(AC/BC) = 1
15. The probabilities of events, A  B, A, B & A  B are respectively in A.P. with probability of second term equal
to the common difference. Therefore the events A and B are -
(A) compatible (B) independent
(C) such that one of them must occur (D) such that one is twice as likely as the other
16. From an urn containing six balls, 3 white and 3 black ones, a person selects at random an even number of balls
(all the different ways of drawing an even number of balls are considered equally probable, irrespective of their
number). Then the probability that there will be the same number of black and white balls among them -

4 11 11 2
(A) (B) (C) (D)
5 15 30 5

Page 59
60
JEE Sprint Mathematics
17. A pair of fair dice having six faces numbered from 1 to 6 are thrown once, suppose two events E and F are
defined as -
E : Product of the two numbers appearing is divisible by 5.
F : At least one of the dice shows up the face one.
Then the events E and F are
(A) mutually exclusive (B) independent
(C) neither independent nor mutually exclusive (D) are equiprobable
18. Shalu brought two cages of birds : Cage-I contains 5 parrots
and 1 owl and Cage-II contains 6 parrots, as shown. One day Cage-I Cage-II
Shalu forgot to lock both cages and two birds flew from Cage-
I to Cage-II. Then two birds flew back from Cage-II to Cage-I.

T
Assume that all birds have equal chance of flying, the
probability that the Owl is still in Cage-I, is -

N
(A) 1/6 (B) 1/3
Birds like to fly

I
(C) 2/3 (D) 3/4
19. In a maths paper there are 3 sections A, B & C. Section A is compulsory. Out of sections B & C a student has
to attempt any one. Passing in the paper means passing in A & passing in B or C. The probability of the student

R
passing in A, B & C are p, q & 1/2 respectively. If the probability that the student is successful is 1/2 then,
which of the following is false -

P
(A) p = q = 1 (B) p = q = 1/2 (C) p = 1, q = 0 (D) p = 1, q = 1/2
20. Sixteen players s1, s2,.........,s 16 play in a tournament. They are divided into eight pairs at random. From each

S
pair a winner is decided on the basis of a game played between the two players of the pair. Assume that all the
players are of equal strength. The probability that "exactly one of the two players s 1 & s2 is among the eight
winners" is -

E
4 7 8 9
(A) (B) (C) (D)
15 15 15 15

E
21. The number 'a' is randomly selected from the set {0, 1, 2, 3,.........98, 99}. The number 'b' is selected from the

J
same set. Probability that the number 3a + 7b has a digit equal to 8 at the units place, is -

1 2 4 3
(A) (B) (C) (D)
16 16 16 16

22 If E & F are the complementary events of events E & F respectively & if 0 < P (F) < 1 , then - [JEE 98]

(A) P(E F )  P( E F )  1 (B) P(E F )  P(E F )  1 (C) P(E F )  P(E F )  1 (D) P(E F )  P(E F )  1

ANSWER KEY
Que. 1 2 3 4 5 6 7 8 9 10
Ans. B C,D D D A ,C , D A B A, B, C C B
Que. 11 12 13 14 15 16 17 18 19 20
Ans. A,B A C A,B, C,D D B C,D D A, B, C C
Que. 21 22
Ans. D A,D

Page 60
61
JEE Sprint Mathematics
SETS
SELECT THE CORRECT ALTERNATIVE (ONLY ONE CORRECT ANSWER)

1. If A and B are two sets, then A  (A  B)' is equal 12. If A and B are not disjoint, then n(A  B) is equal
to- to-
(1) A (2) B (1) n(A) + n(B)
(3)  (4) none of these (2) n(A) + n(B) – n(A  B)
2. If A is any set, then- (3) n(A) + n(B) + n(A  B)
(1) A A' =  (2) A  A' = U (4) n(A).n(B)
(3) A A' = U (4) none of these 13. If A = {2, 4, 5}, B = {7, 8, 9} then n(A × B) is equal
3. If A, B be any two sets, then (A  B)' is equal to- to-
(1) A' B' (2) A' B' (1) 6 (2) 9 (3) 3 (4) 0

T
(3) A  B (4) A  B 14. Let A and B be two sets such that n(A) = 70,
4. If A and B be any two sets, then (A  B)' is equal n(B) = 60 and n(A  B) = 110. Then n(A  B) is
to - equal to-

N
(1) A'  B' (2) A' B' (3) A  B (4) A B (1) 240 (2) 20 (3) 100 (4) 120

I
5. Let U = {1, 2, 3, 4, 5, 6, 7, 8, 9, 10}, A = {1, 2, 5}, 15. Which set is the subset of all given sets ?
B = {6, 7} then A B' is- (1) {1, 2, 3, 4, ....} (2) {1}
(1) B' (2) A (3) A' (4) B. (3) {0} (4) { }

R
6. If A and B are two sets, then A  B = A  B iff-
 1 
(1) A B (2) B A 16. If Q =  x : x  , where y  N  , then-

P
 y 
(3) A = B (4) none of these
7. Let A and B be two sets in the universal set. Then

S
2
A – B equals- (1) 0  Q (2) 1  Q (3) 2  Q (4) Q
3
(1) A B' (2) A'  B
(3) A B (4) none of these 17. A = {x : x x} represents-

E
8. Two sets A, B are disjoint iff- (1) {0} (2) { } (3) {1} (4) {x}
(1) A  B =  (2) A  B  18. Which of the following statements is true ?

E
(3) A  B =  (4) A – B = A (1) 3 {1, 3, 5} (2) 3 {1, 3, 5}

J
9. Which of the following is a null set ? (3) {3} {1, 3, 5} (4) {3, 5} {1, 3, 5}
(1) {0} 19. Which of the following is a null set ?
(2) {x : x > 0 or x < 0}
(1) A = {x : x > 1 and x < 1]
(3) {x : x2 = 4 or x = 3}
(2) B = {x : x + 3 = 3}
(4) {x : x2 + 1 = 0, x R}
(3)
3 = {}
10. If A  B, then A  B is equal to-
(1) A (2) B (3) A' (4) B' (4) D = {x : x 1 and x 1}

11. If A and B are any two sets, then A  (A  B) is 20. P(A) = P(B) 
equal to- (1) A B (2) B A
(1) A (2) B (3) A' (4) B' (3) A = B (4) none of these

ANSWER KEY
Que. 1 2 3 4 5 6 7 8 9 10 11 12 13 14 15
Ans. 3 2 2 2 2 3 1 3 4 1 1 2 2 2 4
Que. 16 17 18 19 20
Ans. 2 2 2 1 3

Page 61
62
JEE Sprint Mathematics
RELATIONS
SELECT THE CORRECT ALTERNATIVE (ONLY ONE CORRECT ANSWER)
1. If R is a relation from a finite set A having m 10. Let a relation R is the set N of natural numbers be
elements to a finite set B having n elements, then defined as (x, y) R if and only if x2 – 4xy + 3y2 = 0
the number of relations from A to B is- for all x, y  N. The relation R is-
mn mn n
(1) 2 (2) 2 –1 (3) 2mn (4) m (1) Reflexive
2. In the set A = {1, 2, 3, 4, 5}, a relation R is defined (2) Symmetric
by R = {(x, y) | x, y A and x < y}. Then R is- (3) Transitive
(1) Reflexive (2) Symmetric (4) An equivalence relation
(3) Transitive (4) None of these 11. Let A = {2, 3, 4, 5} and let R = {(2, 2), (3, 3),
3. For real numbers x and y, we write (4, 4), (5, 5), (2, 3), (3, 2), (3, 5), (5, 3)} be a relation

T
x R y  x – y + 2 is an irrational number. Then in A. Then R is-
the relation R is- (1) Reflexive and transitive
(1) Reflexive (2) Symmetric (2) Reflexive and symmetric

N
(3) Transitive (4) none of these (3) Reflexive and antisymmetric

I
4. Let X = {1, 2, 3, 4} and Y = {1, 3, 5, 7, 9}. Which (4) none of these
of the following is relations from X to Y- 12. If A = {2, 3} and B = {1, 2}, then A × B is equal to-
(1) R1 = {(x, y) | y = 2 + x, x X, y Y} (1) {(2, 1), (2, 2), (3, 1), (3, 2)}

R
(2) R2 = {(1, 1), (2, 1), (3, 3), (4, 3), (5, 5)} (2) {(1, 2), (1, 3), (2, 2), (2, 3)}
(3) R3 = {(1, 1), (1, 3), (3, 5), (3, 7), (5, 7)} (3) {(2, 1), (3, 2)}

P
(4) R4 = {(1, 3), (2, 5), (2, 4), (7, 9)} (4) {(1, 2), (2, 3)}
5. Let L denote the set of all straight lines in a plane. 13. Let R be a relation over the set N × N and it is

S
Let a relation R be defined by  R     , defined by (a, b) R (c, d)  a + d = b + c. Then R
, L. Then R is- is-
(1) Reflexive (2) Symmetric (1) Reflexive only

E
(3) Transitive (4) none of these (2) Symmetric only
6. Let R be a relation defined in the set of real numbers (3) Transitive only
by a R b  1 + ab > 0. Then R is- (4) An equivalence relation

E
(1) Equivalence relation (2) Transitive 14. Let N denote the set of all natural numbers and R

J
(3) Symmetric (4) Anti-symmetric be the relation on N × N defined by (a, b) R (c, d) if
7. Which one of the following relations on R is ad (b + c) = bc(a + d), then R is-
equivalence relation- (1) Symmetric only
(1) x R1 y  |x| = |y| (2) x R2 y  x  y (2) Reflexive only
(3) x R3y  x | y (4) x R4 y  x < y (3) Transitive only
8. Two poi nt s P and Q in a plane are related if (4) An equivalence relation
OP = OQ, where O is a fixed point. This relation 15. If A = {1, 2, 3}, B = {1, 4, 6, 9} and R is a relation
is- from A to B defined by 'x is greater than y'. Then
(1) Reflexive but symmetric range of R is-
(2) Symmetric but not transitive (1) {1, 4, 6, 9} (2) {4, 6, 9}
(3) An equivalence relation (3) {1} (4) none of these
(4) none of these 16. Let L be the set of all straight lines in the Euclidean
9. The relation R defined in A = {1, 2, 3} by a R b if plane. Two lines 1 and 2 are said to be related by
|a2 – b2|  5. Which of the following is false- the relation R if 1 is parallel to 2. Then the relation
(1)R ={(1, 1), (2, 2), (3, 3), (2, 1), (1, 2), (2, 3), (3, 2) R is-
–1
(2) R = R (1) Reflexive (2) Symmetric
(3) Domain of R = {1, 2, 3} (3) Transitive (4) Equivalence
(4) Range of R = {5}

Page 62
63
JEE Sprint Mathematics
17. A and B are two sets having 3 and 4 elements 24. Let P = {(x, y) | x2 + y2 = 1, x, y R} Then P is-
respectively and having 2 elements in common. (1) reflexive (2) symmetric
The number of relations which can be defined from (3) transitive (4) anti-symmetric
A to B is- 25. Let X be a family of sets and R be a relation on X
(1) 25 (2) 210 – 1 defined by 'A is disjoint from B'. Then R is-
12
(3) 2 – 1 (4) none of these (1) reflexive (2) symmetric
18. For n, m  N, n|m means that n is a factor of m, (3) anti-symmetric (4) transitive
the relation | is- 26. In order that a relation R defined in a non-empty
(1) reflexive and symmetric set A is an equivalence relation, it is sufficient that R
(2) transitive and symmetric (1) is reflexive
(3) reflexive, transitive and symmetric
(2) is symmetric

T
(4) reflexive, transitive and not symmetric
(3) is transitive
19. Let R = {(x, y) : x, y  A, x + y = 5} where
(4) possesses all the above three properties
A = {1, 2, 3, 4, 5} then
27. If R be a relation '<' from A = {1, 2, 3, 4} to

N
(1) R is not reflexive, symmetric and not transitive
B = {1, 3, 5} i.e. (a, b) R iff a < b, then ROR–1 is-

I
(2) R is an equivalence relation
(1) {(1, 3), (1, 5), (2, 3), (2, 5), (3, 5), (4, 5)}
(3) R is reflexive, symmetric but not transitive
(2) {(3, 1), (5, 1), (3, 2), (5, 2), (5, 3), (5, 4)}
(4) R is not reflexive, not symmetric but transitive

R
(3) {(3, 3), (3, 5), (5, 3), (5, 5)}
20. Let R be a relation on a set A such that R = R–1
(4) {(3, 3), (3, 4), (4, 5)}
then R is-
28. If R is an equivalence relation in a set A, then R–1

P
(1) reflexive
(2) symmetric is-

S
(3) transitive (1) reflexive but not symmetric
(4) none of these (2) symmetric but not transitive
21. Let x, y  I and suppose that a relation R on I is (3) an equivalence relation
(4) none of these

E
defined by x R y if and only if x  y then
(1) R is partial order ralation 29. Let R and S be two equivalence relations in a set
(2) R is an equivalence relation A. Then-

E
(3) R is reflexive and symmetric (1) R  S is an equivalence relation in A

J
(4) R is symmetric and transitive (2) R  S is an equivalence relation in A
22. Let R be a relation from a set A to a set B, then- (3) R – S is an equivalence relation in A
(1) R = A  B (2) R = A  B (4) none of these
(3) R  A × B (4) R  B × A 30. Let A = {p, q, r}. Which of the following is an
23. Given the relation R = = {(1, 2), (2, 3)} on the set equivalence relation in A ?
A = {1, 2, 3}, the minimum number of ordered (1) R1 = {(p, q), (q, r), (p, r), (p, p)}
pairs which when added to R make it an equivalence (2) R2 = {(r, q) (r, p), (r, r), (q, q)}
relation is- (3) R3 = {(p, p), (q, q), (r, r), (p, q)}
(1) 5 (2) 6 (3) 7 (4) 8 (4) none of these

ANSWER KEY
Que. 1 2 3 4 5 6 7 8 9 10 11 12 13 14 15
Ans. 1 3 1 1 2 3 1 3 4 1 2 1 4 4 3
Que. 16 17 18 19 20 21 22 23 24 25 26 27 28 29 30
Ans. 4 4 4 1 2 1 3 3 2 2 4 3 3 2 4

Page 63
64
JEE Sprint Mathematics
EXTRA PRACTICE QUESTIO NS ON RELATIONS
SELECT THE CORRECT ALTERNATIVE (ONLY ONE CORRECT ANSWER)

1. Let R = {(1, 3), (4, 2), (2, 4), (2, 3), (3, 1)} be a 5. Let R be the set of real numbers.
Statement-1:
releation on the set A = {1, 2, 3, 4}. The relation
A = {(x, y)  R × R : y – x is an integer} is an
R is- [AIEEE - 2004] equivalence relation on R. [AIEEE - 2011]
(1) transitive (2) not symmetric Statement-2:
B = {(x, y) R × R : x = y for some rational number
(3) reflexive (4) a function
} is an equivalence relation on R.
2. Let R = {(3, 3), (6, 6), (9, 9), (12, 12), (6, 12), (1) Statement-1 is true, Statement-2 is false.
(3, 9), (3, 12), (3, 6)} be relation on the set (2) Statement-1 is false, Statement-2 is true
A = {3, 6, 9, 12). The relation is- [AIEEE - 2005] (3) Statement-1 is true, Statement-2 is true;
Statement-2 is a correct explanation for

T
(1) rflexive and transitive only Statement-1
(2) reflexive only (4) Statement-1 is true, Statement-2 is true;
(3) an equilvalence relation Statement-2 is not a correct explanation for

N
Statement-1.
(4) reflexive and symmetric only 6. Consider the following relation R on the set of real

I
3. Let W denote the words in the English dictionary. square matirces of order 3.
Define the relation R by : R = {(x, y) W × W| the R={(A, B)|A=P–1 BP for some invertible matrix P}.
Statement - 1:

R
words x and y have at least one letter in common}.
R is an equivalence relation.
Then R is- [AIEEE - 2006] Statement - 2:

P
(1) reflexive, symmetric and not transitive For any two invertible 3 × 3 martices M and N,
(MN) –1 = N –1 M –1 [AIEEE - 2011]
(2) reflexive, symmetric and transitive
(1) Statement-1 is false, statement-2 is true.

S
(3) reflexive, not symmetric and transtive (2) Statement- 1 i s tr u e , s t a t e m e n t - 2 i s
(4) not reflexive, symmetric and transitive true; Statement-2 is correct
e x p l a n a t i o n for statement-1.
4. Consider the following relations :-

E
(3) Statement- 1 i s tr u e , s t a t e m e n t - 2 i s
R = {(x, y) | x, y are real numbers and x = wy for t r u e ; Statement-2 is not a correct
some rational number w} ; explanation for statement-1.

E
(4) Statement-1 is true, statement-2 is false.
m p

J
S = {( , ) | m, n, p and q are integers such that
n q

n, q  0 and qm = pn}.
Then : [AIEEE - 2010]
(1) R is an equivalence relation but S is not an
equivalence relation
(2) Neither R nor S is an equivalence relation
(3) S is an equivalence relation but R is not an
equivalence relation
(4) R and S both are equivalence relations

ANSWER KEY
Que. 1 2 3 4 5 6
Ans. 2 1 1 3 1 1

Page 64
65
JEE Sprint Mathematics

FUNCTIONS

SELECT THE CORRECT ALTERNATIVE (ONLY ONE CORRECT ANSWER)

1. Solution of 0 < |x-3|  5 is


(A) [-2, 8] (B) [-2,3) U (3,8]
(C) [-2, 9) (D) none of these

 x  3  x  5  x  7 
2. Solution of  0 is
| x  4 | (x  6)
(A) (-6,-5] U [3,7] (B) [3,7]

T
(C) (-6,-5]  [3, 4)  (4,7] (D) none of these

3. If f(x) =  
log1 2 x2  2x  2 , then domain of f(x) is

IN
(A) R (B) {1}
(C) R+ (D) {2}

R
4. If the function f: [1, )  (1, ) is defined by f (x) = 2x(x – 1), then f –1 (x) is
x(x 1)
 1 1
(A)   (B) 1  1  4 log2 x 

P
 2 2 
1
(C) 
1  1  4 log2 x  (D) not defined

S
2

 sin x 
5. If f(x) = cos x    , (where [.] denotes the greatest integer function), then

E
 2 
f(x) is

E
(A) even (B) odd
(C) even and odd simultaneously (D) none of these

J
6. The number of solutions of the equation |sinx| = |x| is
(A) 2 (B) 0
(C) 1 (D) 4

7. If f : R  [-1,1], where f(x) = sin /2 [x] (where [.] denotes the greatest integer function),
then the range of f(x) is
(A) [-1, 1] (B) {-1, 1}
(C) {-1, 0, 1} (D) none of these

8. Let f : (2,4) (1,3) where f(x) = x – [x/2] (where [.] denotes the greatest integer function).
Then f -1(x) is
(A) not defined (B) x – 1
(C) x + 1 (D) none of these

9. The fundamental period of cos 2x + sinx is


(A)  (B) 2
(C) /4 (D) /2

Page 65
66
JEE Sprint Mathematics

10. The range of the function f(x) =



sin   x 2  1
   , where [.] denotes the greatest integer
x4  1
function, is
(A) [0, 1] (B) [–1, 1]
(C) 0 (D) none of these

|x| -x
11. Let f : R  R, where f(x) = 2 – 2 . Then f(x) is
(A) one-one (B) many-one
(C) periodic (D) none of these

T
12. If f(x) = logx 2 x , then the domain of f(x) is
(A) R+ (B) R – {1}
+

N
(C) R – {1} (D) none of these

I
1
13. If f(x) = , then the domain of f(x) is
x
(x  1) (e  1) (x  4) (x  5) (x  6)

R
(A) (-,-5)U (-1,4) U (6, ) (B) (-,-5) U (-1,0) U (0,4) U (6, )
(C) (-5,-1) U (0,4) U (6, ) (D) none of these

P
3x  2
14. If f(x) = , then

S
5x  3
(A) f –1(x) = f(x) (B) f –1(x) = – f(x)
(C) f(f(x)) = – x (D) none of these

E
15. The function f : R  R; defined by f(x) = 2x + |cosx|, is

E
(A) one-one and into (B) one-one and onto
(C) many-one and into (D) many-one and onto

J 1.
3.
B
B
2.
4.
C
B
5. A 6. C
7. C 8. C
9. B 10. C
11. B 12. C
13. C 14. A
15. B

Page 66
67
JEE Sprint Mathematics
EXTRA PRACTICE QUESTIONS ON FUNCTIONS
SELECT THE CORRECT ALTERNATIVE (ONLY ONE CORRECT ANSWER)

 x2 
1. If f(x) = sin-1   then the range of f(x) is
 1  x2
 
(A) [-/2,/2] (B) [0,/2]
(C) [0,/2) (D) [-/2,0)

2. If [x]2 – 5[x] + 6 = 0 (where [.] denotes the greatest integer function), then x belongs to
(A) [2,4) (B) [2,4) – {3}
(C) {3} (D) {2}

T
3. Let f(x) be a function whose domain is [-5, 7] and g(x) = |2x + 5|. Then the domain of
fog(x) is

N
(A) [-5, 1] (B) [-4, 0]

I
(C) [-6, 1] (D) none of these

4. If f(x) and g(x) be two given functions with all real numbers as their domain, then

R
h(x) = {f(x) + f(-x)} {g(x) – g(-x)} is
(A) an even function (B) an odd function

P
(C) even as well as odd function (D) none of these

S
5. If f(x) = x3 + 3x2 + 12x – 2 sin x, where f : R R, then
(A) f(x) is many-one and onto (B) f(x) is one-one and onto
(C) f(x) is one-one and into (D) f(x) is many-one and into

E
 x2  e 
6. If f(x) = n  2 , then range of f(x) is
 x  1 

E
 

J
(A) (0, 1) (B) [0, 1]
(C) [0, 1) (D) (0, 1]

7. y = f(x) be a real valued function with domain as all real numbers. If the graph of the
function is symmetrical about the line x = 1, then for all R
(A) f() = f( + 1) (B) f( – 1) = f()
(C) f( – 1) = f( + 1) (D) f(1- ) = f(1 + )

 2 x 
8. If f(x) = sec 1  , then the domain of f(x) is
 4 
 
(A) [-2, 2] (B) [-6, 6]
(C) (-, -6] U [6, ) (D) [-6, -2] U [2, 6]

9. If f: R  R, where f(x) = ax + cos x and f(x) is bijective, then


(A) a R (B) a R+
-
(C) a R (D) a R – (-1,1)

Page 67
68
JEE Sprint Mathematics

10. If f(x) = {x} + sin ax (where { } denotes the fractional part function) is periodic, then
(A) ‘a’ is a rational multiple of  (B) ‘a’ is a natural number
(C) ‘a’ is any real number (D) ‘a’ is any positive real number

11. The values of b and c for which the identity f(x + 1) – f(x) = 8x + 3 is satisfied,
2
where f(x) = bx + cx + d, are
(A) 4, 1 (B) 4, –1
(C) –1, 4 (D) none of these

12. Let 2 f(x2) + 3 f(1/x2) = x2 – 1 for all xR – {0}. Then f(x) equals
3  x  2x 2 3  x  2x 2

T
(A) (B)
5 x 15 x
5  x  2x 2
(C) (D) none of these

N
5 x

I
1
(log2 x 2 )
13. If y = esin , then y is real if

R
(A) x R – {0} (B) x    2, 2 
 1   1   1   1 


P
(C) x  , , (D) x    2,  , 2
 2  2   2  2 

S
 x  x0

14. If f(x) = 1   x  x  0 and x  I , then

E
 x x  0 and x  I
(A) f(x) is periodic function (B) f(x) is many-one

E
(C) f(x) is one-one (D) none of these

J
15. Let f : R  R be a function such that f(x) = x3 – 6x2 + 11x – 6. Then f(x) is
(A) one-one and into (B) one-one and onto
(C) many-one and into (D) many-one and onto

1. C 2. A
3. C 4. B
5. B 6. D
7. D 8. C
9. D 10. A
11. B 12. A
13. D 14. B
15. D

Page 68
69
JEE Sprint Mathematics
INVERSE TRIGONOMETRIC FUNCTIONS
SELECT  THE  CORRECT  ALTERNATIVE  (ONLY  ONE  CORRECT  ANSWER)

1. The  value  of    sin–1  (  3 / 2)   is  -


(A) –/3 (B) –2/3 (C) 4/3 (D) 5/3
  1
2. cos  2 tan 1      equals  -
  7

(A) sin(4 cot 1 3) (B) sin(3 cot 1 4) (C) cos(3 cot 1 4) (D) cos(4 cot 1 4)

 
3. The  value  of  sec sin 1   sin 50    cos 1 cos   31      is  equal  to  -
  9   9 

T
10  
(A)  sec (B) sec (C) 1 (D) –1
9 9
  8  8

N
4. cos  cos 1 cos    tan 1 tan      has  the  value  equal  to  -
  7   7 

I

(A) 1 (B) –1 (C) cos (D) 0
7

R
1 2 1 2 1 1 2 2 2
5. (sin x)  (sin y )  2(sin x)(sin y)   ,  then  x +y   is  equal  to  -
(A) 1 (B) 3/2 (C) 2 (D) 1/2

P
6. cot–1  [  (cos  )1/2  ]  –  tan–1  [  (cos)1/2  ]  =  x  ,  then  sin  x  =

2   2    

S
(A) tan   (B) cot   (C) tan   (D) cot  
2 2 2
7. tan(cos–1  x)  is  equal  to

E
x 1  x2 1  x2
(A)  (B)  (C)  (D)  1  2 x
1  x2 x x

E
1  1   1 1  x 
8. If  x  =  2cos–1   +  sin–1     +  tan–1  3 and  y  =  cos  sin  sin   then  which  of  the  following  state-

J
2 2 2 2 
ments  holds  good  ?
3 5
(A)  y  cos (B)  y  cos (C)   x  4 cos 1 y (D) none  of  these
16 16

 1 1 1 1  1 
9. If  x  =  tan–1  1–cos–1     sin ; y  cos  cos 1      then  -
2 2 2  8

(A) x  =  y (B) y  =  x (C) tan  x  =  –(4/3)y (D) tan  x  =  (4/3)y


10. tan–12+  tan–13  =  cosec–1x,  then  x  is  equal  to  -
(A)  4 2 (B)  (C)  2 (D) none  of  these
11. The  number  k  is  such  that  tan  {arc  tan(2)  +  arc  tan(20k)}=k.  The  sum  of  all  possible  values  of  k  is  -
19 21 1
(A)   (B)   (C)  0 (D) 
40 40 5

1 
12. If   sin–1  x  +  cot–1     = ,    then    x  is  -
 2 2

1 2 3
(A) 0 (B)  (C)  (D)
5 5 2

Page 69
70
JEE Sprint Mathematics
13. If  tan(cos–1x)  =  sin  (cot–1  1/2)  then  x  is  equal  to  -
(A) 1/ 5 (B)  2 / 5 (C) 3 / 5 (D)  5 / 3

14. sin 1 (2x 1  x 2 )  2 sin 1 x  is true  if  -

 1 1   1 1  3 3
(A) x  [0,1] (B)    ,  (C)    ,  (D)    2 , 2 
 2 2  2 2  
2
15. Domain  of  the  explicit  form  of  the  function  y  represented  implicitly  by  the  equation  (1+x)  cosy  –  x   =  0  is  -
 1 5 1  5 1  5   1 5
(A) (–1,1] (B)   1, 2  (C)   2 , 2  (D)  0, 2 
     

1 1 y
16. If  cos x  cos   ,  then  4x2  –  4xy  cos  +  y2  is  equal  to  -
2

T
(A) –4sin2 (B) 4sin2 (C) 4 (D) 2  sin  2
–1 –1 –1
17. If  cos   x  +  cos   y  +  cos   z  =    ,  then  -
(A) x2+  y2+  z2+  xyz  =  0 (B) x2+  y2+  z2+  xyz  =  1 (C) x2+  y2+  z2+  2xyz  =  0 (D) x2+  y2+  z2+  2xyz  =  1

N
18. If  tan 1 x   , x  N ,  then  the  maximum  value  of  x  is  -

I
 3
(A) 2 (B) 5 (C) 7 (D) none  of  these
The  solution  of  the  inequality  (tan 1 x)2  3 tan 1 x  2  0   is  -

R
19.

(A)  , tan 1    tan 2,  (B)   , tan 1  (C)  ,  tan1   tan 2,  (D)  tan 2, 

P
2 –1
20. The  set  of  values  of  x,  satisfying  the  equation  tan (sin x)  >  1  is  -
 2 2  2 2  2 2

S
(A)    [–1,1] (B)     2 , 2  (C)  ( 1,1)    2 , 2  (D) [–1,1]    2 , 2 
     
SELECT  THE  CORRECT  ALTERNATIVES  (ONE  OR  MORE  THAN  ONE  CORRECT  ANSWERS)
4 2 a

E

21. If  numerical  value  of  tan cos 1  tan 1    is  ,  then    -
 5 3 b
(A) a  +  b  =  23 (B) a  –  b  =  11 (C) 3b  =  a  +  1 (D) 2a  =  3b

E
1 1   14    

J
22. The  value  of  cos  cos  cos        is/are  -
2 5  

 7     2  3
(A)  cos    (B) sin   (C)  cos   (D) –cos  
5 10 5 5

1  2 
23. tan 1    tan 1     equals  to
4
  9 

1 3  1 1  3  1 3 1  1 
(A)  cos 1   (B)  sin   (C)  tan 1   (D)  tan  
2 5  2 5  2 5  2 

3x 4x
24. sin 1  sin 1  sin 1 x ,  then  roots  of  the  equation  are  -
5 5
(A) 0 (B) 1 (C) –1 (D) –2

ANSWER  KEY
Que. 1 2 3 4 5 6 7 8 9 10 11 12 13 14 15
Ans. A A D B C A C A C D A B D B C
Que. 16 17 18 19 20 21 22 23 24
Ans. B D B B C  A,B,C B,C,D A, D A,B,C

Page 70
71
JEE Sprint Mathematics
EXTRA PRACTICE QUESTIONS ON INVERSE TRIGONOMETRIC FUNCTIONS

SELECT  THE  CORRECT  ALTERNATIVES  (ONE  OR  MORE  THAN  ONE  CORRECT  ANSWERS)
1. cos–1x  =  tan–1x  then  -

2
 5 1 2
 5  1
(A)  x    (B)  x   
2  2 

1
 5  1 1
 5  1
(C)  sin(cos x)  
 2  (D)  tan(cos x )   
2 

1  3 1  3

T
2. The  value  of  sin  cot 1      cos  cot 1     is/are  equal  to  -
2  4 2  4

3 2
(A) 1 (B) 

N
10

I
1 1  3  1   1 4
(C)  2 sin  cot     cot (1) (D)  2 sin    tan 1 (1)  tan 1 
2 4  2 3

R
1  1  1 1 3
3. The  value  of  tan  tan 2A   tan (cot A )  tan (cot A )   for  0  <  A  <  (/4)  is  -
2

P
(A) 4 tan–1(1) (B)  2 tan –1(2) (C)  0 (D)  none
–1 –1 –1 3
4. For  the  equation  2x  =  tan(2tan a)  +  2tan(tan a+tan a ),  which  of  the  following  is/are  invalid  ?

S
(A) a2x  +  2a  =  x (B) a 2  +  2ax  +1=  0 (C) a  0 (D) a  1, 1
1
   1 1  a     1 1  a   
5. The  value  of   tan   sin     tan   sin     ,  where  (  0  <  a  <  b),  is  -
 4 2  b 4 2  b  

E
b a b2  a 2 b2  a 2
(A)  (B)  (C)  (D) 
2a 2b 2b 2a

E
6. Identify  the  pair(s)  of  functions  which  are  identical  -

J
1  x2 1
(A) y  =  tan  (cos–1x)  ;  y  = (B) y  =  tan  (cot–1x)  ;  y  =
x x
x
(C) y  =  sin  (arc  tan  x)  ;  y  = (D) y  =  cos  (arc  tan  x)  ;  y  =  sin  (arc  cot  x)
2
1x
 2x 2  1 
7. Which  of  the  following,  satisfy  the  equation  2 cos 1 x  cot 1  
 4 x 2  4x 4 
 1 1 
(A) (–1,  0) (B) (0,  1) (C)   – , (D) [–1,  1]
2 2 
 1  x2 
8. The  solution  set  of  the  equation  sin 1 1  x 2  cos 1 x  cot 1   – sin–1x is -
 x 

(A) [–1, 1]–{0} (B) (0,1] U {–1} (C) [–1,0) U {1} (D) [–1,1]


2
1x
9. If  0  <  x  <  1,  then  tan–1   is  equal  to  -
1x
1 1x 1x 1 1x
(A)  cos 1 x (B)  cos
1
(C)  sin
1
(D)  tan 1
2 2 2 2 1x

Page 71
72
JEE Sprint Mathematics

10. The  number  of  real  solutions  of  tan–1  x(x  1)  sin 1 x 2  x  1    is  - [JEE  99]
2
(A) zero (B) one (C) two (D) infinite
–1 –1
11. If  [sin x]  +  [cos x]  =  0,  where  ‘x’  is  a  non  negative  real  number  and  [.]  denotes  the  greatest  integer  function,
then  complete  set  of  values  of  x  is  -
(A) (cos1,  1) (B) (–1,  cos1) (C) (sin1,  1) (D) (cos1,  sin1)

12. Value  of  k  for  which  the  point  (,  sin–1)( > 0)  lies  inside  the  triangle  formed  by  x  +  y  =  k  with  co-ordinate  axes  is -

          
(A)  1  ,  (B)    1   ,  1    (C)   , 1   (D) (–1–sin1,  1+sin1)
2  2 2  2

 2x 2  3  5
13. Solution  set  of  the  inequality  sin 1  sin 2     2   is  -

T
 x 1 

(A) (  , 1)  (1 , ) (B) [–1,  1] (C) (–1,  1) (D) ( , 1]  [1 , )

N
1
14. Consider  two  geometric  progressions  a1,a2,a3.......an    &  b1,  b 2,  b 3,.....b n  with  ar  =   2 r 1 and another se-
br

I
n
quence  t1,t2,t3.......tn   such  that  tr  =  cot–1  (2ar  +  br)  then  nlim

 t r is    -

R
r 1

(A) 0 (B)  / 4 (C) tan–12 (D)  / 2


15. The  sum  of  the  infinite  terms  of  the  series  -

P
 3  3  3
cot 1  12    cot 1  2 2    cot 1  3 2    ...........  is  equal  to  -
 4   4   4

S
3
(A) tan–1(1) (B) tan–1(2) (C) tan–1(3) (D)   tan 1 3
4

JE E
ANSWER  KEY
Que. 1 2 3 4 5 6 7 8 9 10
Ans. A,C B,C,D A B,C C A,B, C,D B C A, B, C C
Que. 11 12 13 14 15
Ans. D A B B B,D

Page 72
73
JEE Sprint Mathematics
LIMITS
SELECT THE CORRECT ALTERNATIVE (ONLY ONE CORRECT ANSWER)

1  cos x
1. lim is equal to
x 0 x2
(A) 3/4 (B) 1/4
(C) 1/2 (D) 1

16 x  2x
2. If f(x) = , then lim f(x) is
10 x  5 x x 

T
(A) 2 (B) 8
(C) 3 (D) none of these

N
x2  1
3. lim is equal to

I
x  2x  1
(A) 1 (B) 0

R
(C) -1 (D) 1/2

2 cos x 1

P
4. lim is equal to
x 
4
cot x  1

S
1 1
(A) (B)
2 2
1
(C) (D) 1

E
2 2

E
5. lim is equal to
x 0 tan1 2x

J
1
(A) 0 (B)
2
(C) 1 (D) 

1  cos 2x  sin 5x
6. lim is equal to
x 0 x2 sin 3x
10 3
(A) (B)
3 10
6 5
(C) (D)
5 6

 1 2 n 
7. lim   .....   is equal to
n  1  n 2 1 n 2
1 n 2

(A) 0 (B) –1/2


(C) 1/2 (D) none of these

Page 73
74
JEE Sprint Mathematics
[(a  x)  tan x] sin(nx)
8. If lim  0 , where n is nonzero real number, then a is equal to
x 0 x2
1
(A) 0 (B) 1 +
n
1
(C) n (D) n +
n

1
9. lim x x equals
x 

(A) 0 (B) 1
(C) e (D) 

 1
x
10. lim , where [x] is the greatest integer function, is equal to

T
x 2

(A) 1 (B) –1
(C) 1 (D) none of these

IN
x 3
x 3
11. lim   is given by
x   x 1

R
(A) 1 (B) e3
(C) e (D) e4

P
2x sin1 x
12. The value of f(0) so that the function f(x) = , is continuous at each point in

S
2x  tan1 x
its domain, is equal to
(A) 2 (B) 1/3

E
(C) 2/3 (D) –1/3

 x  1 if x 1

E
13. Let f(x) =  2
. The value of a for which f(x) is continuous is
3  ax if x  1

J
(A) 1 (B) 2
(C) -1 (D) -2

sin( cos2 x)
14. lim equals
x 0 x2
(A)   (B) /2
(C)  (D) 1

2 2
15. The function f (x) = [x] – [x ] (where [.] denotes the greatest integer function) is
discontinuous at
(A) all integers (B) all integers except 0 and 1
(C) all integers except 1 (D) all integers except 0

1. C 2. C 3. D 4. B 5. B 6. A

7. B 8. D 9. B 10. D 11. D 12. B 13. A 14. C 15. C

Page 74
75
JEE Sprint Mathematics
CONTINUITY
SELECT THE CORRECT ALTERNATIVE (ONLY ONE CORRECT ANSWER)

 x  2 , when x  1

1. If f  x   4 x  1 , when 1  x  3 , then correct statement is -
 2
 x  5 , when x  3

(A) lim f  x   lim f  x  (B) f(x) is continuous at x = 3


x 1 x 3

(C) f(x) is continuous at x = 1 (D) f(x) is continuous at x = 1 and 3

 1

T
 , x0
2. If f  x    e 1 / x  1 , then -
0 , x0

N
(A) xlim f x  1 (B) xlim f x  0

I
0  0 

(C) f(x) is discontinuous at x = 0 (D) f(x) is continuous

R
1x  31x
3. If function f(x) = , is continuous function, then f(0) is equal to -
x
(A) 2 (B) 1/4 (C) 1/6 (D) 1/3

P
 x 2   a  2  x  2a

S
 , x 2
4. If f  x    x 2 is continuous at x = 2, then a is equal to -
2 , x 2

(A) 0 (B) 1 (C) –1 (D) 2

E
 log(1  2ax)  log(1  bx)
, x 0
If f(x) = 

E
5.  x , is continuous at x = 0 , then k is equal to -


J
k , x 0

(A) 2a + b (B) 2a – b (C) b – 2a (D) a + b

[x]  [  x], x  2
6. If f(x )   , f is continuous at x = 2 then  is (where [.] denotes greatest integer) -
 , x  2

(A) –1 (B) 0 (C) 1 (D) 2


 1  cos 4 x , x0
 x2

7. If f(x) =  a , x  0 , then correct statement is -

 x
, x 0
 16  x  4

(A) f(x) is discontinuous at x = 0 for any value of a


(B) f(x) is continuous at x = 0 when a = 8
(C) f(x) is continuous at x = 0 when a = 0
(D) none of these

Page 75
76
JEE Sprint Mathematics
1
8. Function f(x) = is discontinuous at -
log | x|

(A) one point (B) two points (C) three points (D) infinite number of points
9. Which of the following functions has finite number of points of discontinuity in R (where [.] denotes greatest
integer)
(A) tan x (B) |x| / x (C) x + [x] (D) sin [ x]

1  tan x   
10. If f(x) = , x  , x  0,  is a continuous functions, then f(/4) is equal to -
4x   4  2
(A) –1/2 (B) 1/2 (C) 1 (D) –1

T
a 2  ax  x 2  a 2  ax  x 2
11. The value of f(0), so that function, f(x) = becomes continuous for all x, is given
ax  ax

N
by -

I
(A) a a (B) – a (C) a (D) –a a

x  e x  cos 2x

R
12. If f(x)  , x  0 is continuous at x = 0, then -
x2

P
5
(A) f(0) = (B) [f(0)] = –2 (C) {f(0)} = –0.5 (D) [f(0)].{f(0)}= –1.5
2

S
where [x] and {x} denotes greatest integer and fractional part function.

x(1  a cos x)  b sin x


13. Let f(x) = , x  0 and f(0) = 1. The value of a and b so that f is a continuous function are -
x3

E
(A) 5/2, 3/2 (B) 5/2, –3/2 (C) –5/2, –3/2 (D) none of these

bg

E
14. ‘f’ is a continuous function on the real line. Given that x 2  (f x  2) x  3 . f ( x)  2 3  3  0 . Then the value of

J
f e 3 j is -
2( 3  2)
(A) (B) 2 (1  3 ) (C) zero (D) cannot be determined
3

SELECT THE CORRECT ALTERNATIVES (ONE OR MORE THAN ONE CORRECT ANSWERS)

e sin x
15. The value(s) of x for which ƒ (x) = is continuous, is (are) -
4  x2  9
(A) 3 (B) –3 (C) 5 (D) all x  (–, –3]  [3, )
16. Which of the following function(s) not defined at x = 0 has/have removable discontinuity at the origin ?

1  sin x 
(A) f(x)  (B) f(x)  cos  
1  2 cot x  x 

 1
(C) f(x)  x sin (D) f(x) = n x
x

Page 76
77
JEE Sprint Mathematics
17. Function whose jump (non-negative difference of LHL & RHL) of discontinuity is greater than or equal to one,
is/are -

 (e1 / x  1)  x1 / 3  1
 1/ x ; x 0  1/2 ; x 1
 
(A) ƒ (x)   (e  1) (B) g(x) =  x 1
 (1  cos x)  nx ;
1
 x 1
 ; x 0  (x  1)
x 2

 sin 1 2x  1
 1
; x   0,  log 3 (x  2) ; x 2
  2
(C) u(x) =  tan 3x (D) v(x) =  2
| sin x| log1 / 2 (x  5) ; x  2

T
 x ; x 0

N
1  2 
18. If ƒ (x)  2 , then ƒ   is discontinuous at x =
x  17x  66  x 2 

I
7 24
(A) 2 (B) (C) (D) 6,11

R
3 11

 0; x  Z

P
19. Let ƒ (x) = [x] & g(x)   2 , then (where [.] denotes greatest integer function) -
x ; x  R  Z

S
(A) Lim
x 1
g(x) exists, but g(x) is not continuous at x= 1.

(B) Lim
x 1
f(x) does not exist and ƒ (x) is not continuous at x=1.

E
(C) gof is continuous for all x.
(D) fog is continuous for all x.

JE
ANSWER KEY
Que. 1 2 3 4 5 6 7 8 9 10
Ans. C C C A A A B C B A
Que. 11 12 13 14 15 16 18 18 19
Ans. B D C B A,B B,C,D A ,C , D A, B, C A, B, C

Page 77
78
JEE Sprint Mathematics
EXTRA PRACTICE QUESTIONS ON CONTINUITY
SELECT THE CORRECT ALTERNATIVES (ONE OR MORE THAN ONE CORRECT ANSWERS)

 x if x  0

1. Consider the piecewise defined function f(x)  0 if 0  x  4 choose the answer which best describes
x  4 if x  4

the continuity of this function -
(A) the function is unbounded and therefore cannot be continuous
(B) the function is right continuous at x = 0
(C) the function has a removable discontinuity at 0 and 4, but is continuous on the rest of the real line
(D) the function is continuous on the entire real line

T
2. f(x) is continuous at x=0, then which of the following are always true ?
(A) Lim f(x)  0 (B) f(x) is non continuous at x=1
x 0

N
(C) g(x) = x2f(x) is continuous at x = 0 (D) Lim (f(x)  f(0))  0

I
x 0

3. Indicate all correct alternatives if, f x  bg


x
2
 1 , then on the interval [0,]

R
1 1
(A) tan (f (x)) &
f xbg are both continuous (B) tan (f(x)) &
f xbg
are both discontinuous

P
1
(C) tan (f (x))& f –1
(x) are both continuous bg
(D) tan (f(x)) is continuous but f x is not

S
4. If f(x) = sgn(cos2x – 2 sinx + 3), where sgn ( ) is the signum function, then f(x) -
(A) is continuous over its domain (B) has a missing point discontinuity
(C) has isolated point discontinuity (D) has irremovable discontinuity.

E
2 cos x  sin 2x e  cos x  1
5. f(x)  2
; g(x) 
(   2x) 8x  4 

E
h(x) = f(x) for x</2
= g(x) for x>/2

J
then which of the followings does not holds ?
(A) h is continuous at x = /2 (B) h has an irremovable discontinuity at x=/2
     
(C) h has a removable discontinuity at x = /2 (D) f    g  
2 2

6. The number of points where f(x) = [sinx + cosx] (where [ ] denotes the greatest integer function), x  (0, 2  )
is not continuous is -
(A) 3 (B) 4 (C) 5 (D) 6
  1 1
(x  1)e  |x| x  (x  0)
7. On the interval I = [–2, 2], the function f(x) = 
0 (x  0)
then which one of the following hold good ?
(A) is continuous for all values of x  I (B) is continuous for x  I –(0)
(C) assumes all intermediate values from f(–2) & f(2) (D) has a maximum value equal to 3/e

  
8. If f(x) = cos   cos   x  1  ; where [x] is the greatest integer function of x, then f(x) is continuous at -
x  2
(A) x = 0 (B) x = 1 (C) x = 2 (D) none of these

Page 78
79
JEE Sprint Mathematics

 
3

x 
3  cot 1 2 x 23 
  for x 0
9. Given f(x)   where { } & [ ] denotes the fractional part and the integral part
  
 x 2 cos e1 / x  for x0

functions respectively, then which of the following statement does not hold good -
(A) f (0– ) = 0 (B) f(0 +)=3
(C) f(0)=0  continuity of f at x = 0 (D) irremovable discontinuity of f at x = 0

n n n 2 n2
10. Let ‘f’ be a continuous function on R. If f (1 / 4 )  (sin e )e  then f(0) is -
n2  1
(A) not unique (B) 1

T
(C) data sufficient to find f(0) (D) data insufficient to find f(0)
11. Given f(x) = b ([x]2 + [x]) + 1 for x  1
= sin ( (x  a)) for x < – 1

N
where [x] denotes the integral part of x, then for what values of a, b the function is continuous at x = – 1 ?

I
(A) a  2n  (3 / 2); b  R ; n I (B) a  4n  2 ; b  R ; n I

(C) a  4n  (3 / 2) ; b  R  ; (D) a  4n  1 ; b  R  ;

R
n I n I

x[x]2log(1+x) 2 for

P
–1 < x < 0

12. Consider f(x) = e 2


ln ex  2 {x} j where [*] & {*} are the greatest integer function &

S
for 0< x< 1
tan x

E
fractional part function respectively, then -
(A) f(0) = ln2  f is continuous at x = 0 (B) f(0) = 2  f is continuous at x = 0
(C) f(0) = e2   f is continuous at x = 0 (D) f has an irremovable discontinuity at x = 0

JE
a sin 2 n x for x  0 and n  
13. Let f(x)   then -
2m
 b cos x  1 for x  0 and m  

(A) f(0–)  f(0+) (B) f(0+)  f(0) (C) f(0–)  f(0) (D) f is continuous at x = 0

14. bg
Consider f x  Lim
n 
x n  sin x n
x n  sin x n
for x  0, x  1 f(1)=0 then -

(A) f is continuous at x = 1
(B) f has a finite discontinuity at x = 1
(C) f has an infinite or oscillatory discontinuity at x = 1
(D) f has a removable type of discontinuity at x=1

ANSWER KEY
Que. 1 2 3 4 5 6 7 8 9 10
Ans. D C,D C,D C A ,C , D C B,C,D B,C B,D B,C
Que. 11 12 13 14
Ans. A,C D A B

Page 79
80
JEE Sprint Mathematics
DIFFERENTIABILITY

SELECT THE CORRECT ALTERNATIVE (ONLY ONE CORRECT ANSWER)

1. Let f(x + y) = f(x). f(y), for all x and y. If f(5) = 2 and f 0 = 3, then f 5 is equal to
(A) 5 (B) 6
(C) 0 (D) none of these

2. lim ([x] | x |) , (where [.] denotes the greatest integer function)


x 1

(A) is 0 (B) is 1
(C) does not exit (D) none of these

T
3. Number of points at which f (x) = |x2 + x| + |x – 1| is non-differentiable is
(A) 0 (B) 1

N
(C) 2 (D) 3

I
x 2n  1
4. Let f(x) = lim . Then
x2n  1
n

R
(A) f(x) = 1 for x > 1
(B) f(x) = - 1 for x < 1

P
(C) f(x) is not defined for any value of x
(D) f(x) = 1 for x =1.

S
 1  n2 
5. If lim  an   = b, a finite number, then

E
n  1 n
 
(A) a = 1, b = 1 (B) a = 1, b = 0

E
(C) a = –1, b = 1 (D) none of these

J
 cos1 x
6. lim is equal to
x 1 x 1
(A) 1/ (B) 1/(2)
(C) 1 (D) 0

7. If [x] denotes the greatest integer less than or equal to x,


then the value of lim (1- x + [x –1] + [1 – x]) is
x 1

(A) 0 (B) 1
(C) -1 (D) none of these

sin1 x  tan1 x
8. lim is equal to
x 0 x2
(A) 1/2 (B) – 1/2
(C) 0 (D) none of these

Page 80
81
JEE Sprint Mathematics

9. Let h (x) = min {x, x2}, x  R. Then h (x) is


(A) differentiable everywhere
(B) non- differentiable at three values of x
(C) non- differentiable at two values of x
(D) none of these

10. If f(x) = lim (sin x)2n ,then f is


n
 
(A) continuous at x = , (B) discontinuous at x =
2 2
(C) discontinuous at x =  (D) none of these

T
11. If f (x) = x, x  1 and f (x) = x2 + bx + c (x > 1) and f (x) exists finitely for all x  R, then
(A) b = –1, c  R (B) c = 1, b  R

N
(C) b = 1, c = –1 (D) b = –1, c = 1

I
 Ax  B , x  1

12. If the function f(x) = 3x , 1  x  2 be continuous at x = 1 and discontinuous at

R
 2
Bx  A , x  2

P
x = 2, then
(A) A = 3 + B, B  3 (B) A = 3 + B, B = 3

S
(C) A = 3 + B (D) none of these

2
ax  b , x  1
13. If (x) =  b  0, then f(x) is continuous and differentiable at x=1

E
2
bx  ax  c , x  1
if

E
(A) c = 0, a = 2b (B) a = b, c  R
(C) a = b, c = 0 (D) a = b, c  0.

J
x3 , x0

14. If f (x) = 0, x  0 , then
 3
x , x0
(A) f is derivable at x = 0
(B) f is continuous, but not derivable at x = 0
(C) LHD at x = 0 is 1
(D) none of these

15. Let f  (x) be continuous at x = 0 and f  (0) = 4.


2 f  x   3f  2 x   f  4 x 
Then lim is equal to
x 0 x2
(A) 11 (B) 2
(C) 12 (D) none of these

Page 81
82
JEE Sprint Mathematics

1. B 2. C
3. D 4. A
5. A 6. B
7. C 8. C
9. C 10. B
11. D 12. A
13. A 14. A
15. C

N T
R I
S P
J E E

Page 82
83
JEE Sprint Mathematics
METHODS OF DIFFERENTIATION

SELECT THE CORRECT ALTERNATIVE (ONLY ONE CORRECT ANSWER)

sec x  tan x dy
1. If y  then equals -
sec x  tan x dx

(A) 2 sec x (sec x – tan x) (B) –2sec x (sec x – tan x)2

(C) 2 sec x (sec x + tan x)2 (D) –2 sec x (sec x + tan x)2

1  x2  x4
dy
2. If y = = ax + b, then values of a & b are -
2 and
1 x  x dx
(A) a = 2 , b = 1 (B) a = –2 , b = 1 (C) a = 2 , b = –1 (D) a = –2 , b = –1

T
d
3. Which of the following could be the sketch graph of y =  x nx  ?
dx
y
y y y

N
1

I
x' x x' x
(A) (B) x'
0 1
x (C) 0 1/e (D) 0 e
x' x
0
y'

R
y' y' y'
4. Let f(x) = x +3 ln(x – 2) & g(x) = x + 5 ln(x - 1), then the set of x satisfying the inequality f'(x) < g'(x) is -
 7 7 7
(A)  2,  1, 2    2 , 


(D)  , 

P
(B)  (C) (2, )
2 2
1 1 1
   m  n    m  n   m  n    m  n

S
5. Differential coefficient of  x m  n  .  x n   .  x  m  w.r.t. x is -
     

(A) 1 (B) 0 (C) –1 (D) xlmn

E
1 1 1 dy np
6. If y  n m p m
 m n p n
 m p n p then at x = e m is equal to -
1x x 1x x 1x x dx

E
mnp mn / p np / m
(A) e (B) e (C) e (D) none of these

J
 x2  y2  dy
7. If cos–1  2  = log a then =
 x  y2  dx
x y y x
(A)  (B)  (C) (D)
y x x y

100
n 101  n  f 101 
8. If f(x) =  x  n ; then =
n 1 f ' 101 

1 1
(A) 5050 (B) (C) 10010 (D)
5050 10010

FG  IJ is -
H 4K
|sin x|
9. If f(x)  | x| , then f ' 

F I F 2 log 4  2 2 I F I 1/ 2
F 2 log 4  2 2 I
(B) GH JK
1/ 2
(A) G J GH 2   JK GH 2   JK
H 4K 4

F I 1/ 2
F 2 log   2 2 I F I 1/ 2
F 2 log   2 2 I
(C) GH JK GH 2 4  JK (D) GH JK GH 2 4  JK
4 4

Page 83
84
JEE Sprint Mathematics
x x x x x x dy
10. If y = ......... then -
ababab dx
a b a b
(A) (B) (C) (D)
ab  2ay ab  2by ab  2by ab  2ay
x 2 dy
11. If y  x then =
dx
2 2 2 2
x x 1 x 1
(A) 2 n x.x x (B) (2 n x  1).x (C) ( n x  1).x (D) x .n (ex 2 )
dy
12. If xm . yn = (x + y)m+n, then is -
dx
x y xy
(A) xy (B) (C) (D)
y x xy

T
dy
13. If x (1  y )  y (1  x)  0 , then equals -
dx

N
1 1 1 1
(A) (B)  (C)  (1  x)  (D) none of these

I
2 2
(1  x) (1  x) (1  x)2
dy
14. If x2 ey + 2xyex + 13 = 0, then equals -
dx

R
y x
2xe  2y(x  1) 2xe x  y  2y(x  1) 2xe xy
 2y(x  1)
(A)  y x (B) y x (C)  (D) none of these
x(xe  2) x(xe  2) x(xe y  x  2)

P
y  ...........to  dy
15. If x  e y  e , x > 0 then is -
dx

S
x 1 x 1 x 1
(A) (B) (C) (D)
1 x x x x
1 1 dy
16. If x =  – and y =  + , then =

E
  dx
x y x y
(A) (B) (C) (D)
y y

E
x x

J
   2
x 1 1  x
17. The derivative of sin 1   w.r.t. cos   , (x > 0) is -
 1  x2   1  x2 
1 1
(A) 1 (B) 2 (C) (D) 
2 2
x 10
18. Let g is the inverse function of f & f '(x)  . If g (2) = a then g'(2) is equal to -
1  x  2

5 1  a2 a 10 1  a 10
(A) (B) (C) (D)
2 10 a 10 1  a2 a2
d2 F
19. Let f(x) = sinx ; g(x) = x2 & h(x)=loge x & F(x) = h[g(f(x))] then is equal to -
dx 2
(A) 2 cosec3x (B) 2 cot (x2)–4x2 cosec2 (x2)
(C) 2x cot x2 (D) –2 cosec2x
x 1
20. If ƒ (x) = x 2  1 , g(x) = and h(x) = 2x – 3, then ƒ '(h'(g'(x)) =
x2  1
1 2 x
(A) 0 (B) 2 (C) (D) 2
x 1 5 x 1

Page 84
85
JEE Sprint Mathematics
21. If ƒ & g are the functions whose graphs are as shown, let u(x) = ƒ (g(x)); w(x) = g(g(x)), y
then the value of u'(1) + w'(1) is - 5
ƒ (2,4)
4
(6,3)
1 3 3
(A)  (B) 
2 2 2
g
1
5
(C)  (D) does not exist 0 1 2 3 4 5 6 x
4
22. f'(x) = g(x) and g'(x) = - f(x) for all real x and f(5) = 2 = f'(5) then f2 (10) + g2 (10) is -
(A) 2 (B) 4 (C) 8 (D) none of these

f '(1) f ''(1) f '''(1) ( 1) n f '''''''''.......(n times ) (1)


23. If f(x) = xn, then the value of f(1)     .......  -
1! 2! 3! n!

T
(A) 2n – 1 (B) 0 (C) 1 (D) 2n
24. A function y = f(x) has second order derivative f"(x) = 6(x – 1). If its graph passes through the point (2, 1) and at
that point the tangent to the graph is y = 3x – 5, then the function is -

N
(A) (x + 1)3 (B) (x + 1)2 (C) (x – 1)2 (D) (x – 1)3

I
x2 x 3 xn
25. If ƒ (x) = x +   ........ , then ƒ (0) + ƒ '(0) + ƒ ''(0) + ........ + ƒ''''...... n times(0) is equal to -
1! 2 ! (n  1)!

R
2
n(n  1) (n 2  1)  n(n  1)  n(n  1)(2n  1)

P
(A) (B) (C)   (D)
2 2  2  6

S
cos x x 1
2 f '(x)
2 sin x x 2x
26. Let f (x) = . Then Limit 
tan x x 1 x 0 x

E
(A) 2 (B) –2 (C) –1 (D) 1

E
f(4)  f(x 2 )

J
27. If f is differentiable in (0, 6) & f'(4) =5 then Lim =
x 2 2x
(A) 5 (B) 5/4 (C) 10 (D) 20

f(x )  g  x 
28. If f(4) = g(4) = 2 ; f’ (4) = 9 ; g’ (4) = 6 then Limit is equal to -
x 4 x 2

3
(A) 3 2 (B) (C) 0 (D) none of these
2

SELECT THE CORRECT ALTERNATIVES (ONE OR MORE THAN ONE CORRECT ANSWERS)
29. The slope(s) of common tangent(s) to the curves y = e–x & y = e–x sinx can be -


(A) e  / 2 (B) e  (C) (D) 1
2
30. If y + n(1 + x) = 0, which of the following is true ?

1
(A) ey = xy' + 1 (B) y' =  (C) y' + ey = 0 (D) y' = ey
(x  1)

Page 85
86
JEE Sprint Mathematics
x
31. If y = 2 3 , then y' equals -
x x
(A) 3x n3 n2 (B) y(log2y) n3 n2 (C) 2 3 . 3x n6 (D) 2 3 . 3x n3 n2

d2 y
32. If y = 3t2 & x = 2t then equals-
dx 2

3
(A) 3t (B) 3 (C) (D) None of these
2
33. If g is inverse of ƒ and ƒ (x) = x2 + 3x – 3 (x > 0) then g'(1) equals-

1 1 ƒ '(1)

T
(A) (B) –1 (C) (D)
2 g(1)  3 5 (ƒ(1)) 2

R IN
S P
JE E
ANSWER KEY
Que. 1 2 3 4 5 6 7 8 9 10 11 12 13 14 15
Ans. B C C D B D C B A D D C B A C
Que. 16 17 18 19 20 21 22 23 24 25 26 27 28 29 30
Ans. A C B D C B C B D A B D A A , B A,B,C
Que. 31 32 33
Ans. B , D C A,C

Page 86
87
JEE Sprint Mathematics
EXTRA PRACTICE QUESTIONS ON METHODS OF DIFFERENTIATION
SELECT THE CORRECT ALTERNATIVES (ONE OR MORE THAN ONE CORRECT ANSWERS)

1. If y = fofof (x) and f (0) = 0, f '(0) = 2, then find y'(0) -

(A) 6 (B) 7 (C) 8 (D) 9

d  3 d2 y 
2. If y2 = p(x) is a polynomial of degree 3, then 2 y is equal to -
dx  dx 2 
(A) p'''(x) . p'(x) (B) p''(x) . p'''(x) (C) p(x) . p'''(x) (D) none of these

d2 y dy
3. If y is a function of x then y  0 . If x is a function of y then the equation becomes -

T
2 dx
dx
2 3 2 2
d x dx d2 x
 dx  d2 x  dx  d2 x  dx 
(A) x 0 (B)  y   0 (C)  y   0 (D)  x   0
dy 2 dy dy 2  dy  dy 2  dy  dy 2  dy 

N
dy

I
4. If y = tanx tan 2x tan 3x then is equal to-
dx
(A) 3 sec2 3x tan x tan 2x + sec2x tan 2x tan 3x + 2 sec2 2x tan 3x tan x

R
(B) 2y (cosec 2x + 2 cosec 4x + 3 cosec 6x)
(C) 3 sec2 3x – 2 sec2 2x – sec2 x

P
(D) sec2x + 2 sec2 2x + 3 sec2 3x

x x dy
5. If y  e  e then equals -

S
dx
x x
e  e e x
 e x 1 1
(A) (B) (C) y2  4 (D) y2  4
2 x 2x 2 x 2 x

E
dy
6. Let y  x  x  x  ...... then -
dx

E
1 x 1 y
(A) (B) (C) (D)

J
2y  1 x  2y 1  4x 2x  y
dy
7. If 2x + 2y = 2 x+y then has the value equal to -
dx
2y 1 2 x (1  2 y )
(A)  (B) (C) 1–2 y (D)
2x 1  2x 2 y (2 x  1)
8. The functions u = ex sin x ; v = ex cos x satisfy the equation -

du dv d2 u d2 v
(A) v u  u2  v2 (B) 2
 2v (C)  2u (D) none of these
dx dx dx dx 2

9. Two functions f & g have first & second derivatives at x = 0 & satisfy the relations,

2
f(0)  , f'(0) = 2 g'(0) = 4g (0), g''(0) = 5 f''(0) = 6 f(0) = 3 then -
g(0)

f x 15
(A) if h (x) = then h'(0) = (B) if k(x) = f(x) . g(x) sinx then k'(0) = 2
g x 4

g 'x 1
(C) Limit  (D) none of these
x 0 f 'x 2

Page 87
88
JEE Sprint Mathematics
dy
10. If y2 + b2 = 2xy, then equals -
dx

1 y xy  b 2 xy  b 2
(A) (B) (C) 2 (D)
xy  b 2 yx y  x y

dy
11. If y  x  y  x  c , then is equal to -
dx

2x x c2
y  y2  x2
(A) 2 (B) 2 2 (C) (D)
c y y x x 2y

12. 
Lim  x x
x
  xx  is -

T
x 0

(A) equal to 0 (B) equal to 1 (C) equal to –1 (D) non existent


13. Select the correct statements -

IN
2x 2  3 for x  1
(A) The function f defined by f(x) =  is neither differentiable nor continuous at x = 1.
3x  2 for x  1

R
(B) The function f(x) = x2|x| is twice differentiable at x = 0

(C) If f is continuous at x = 5 and f(5) = 2 then Lim f(4x2–11) exists.

P
x 2

(D) If Lim (f(x)+g(x)) = 2 and Lim (f(x) – g(x)) = 1 then Lim f(x). g(x) may not exist.
x a x a x a

S
n
14. Let   Lim x m  nx  where m, n  N then -
x 0

(A)  is independent of m and n (B)  is independent of m and depends on m

E
(C)  is independent of n and depends on m (D)  is dependent on both m and n
log sin 2 x cos x

E
15. Lim has the value equal to -
x 0 x
log cos

J
x
sin 2 2
2
(A) 1 (B) 2 (C) 4 (D) none of these

ANSWER KEY
Que. 1 2 3 4 5 6 7 8 9 10
Ans. C C C A,B,C A,C A,C,D A,B,C,D A,B,C A,B,C B,C
Que. 11 12 13 14 15
Ans. A,B,C C B,C A C

Page 88
89
JEE Sprint Mathematics
TANGENT AND NORMALS

SELECT  THE  CORRECT  ALTERNATIVE  (ONLY  ONE  CORRECT  ANSWER)

1. If  a  variable  tangent  to  the  curve  x 2y  =c 3  makes  intercepts    a,  b  on  x  and  y  axis  respectively,
then  the  value  of  a 2b  is  -

4 3 27 3 4 3
(A) 27c 3 (B)  c (C)  c (D)  c
27 4 9

x4
2. The  number  of  values  of  c  such  that  the  straight  line  3x  +  4y  =  c    touches  the  curve  x  y   is  -
2

T
(A) 0 (B) 1 (C) 2 (D) 4
3. Let  ƒ  (x)  =  x3  +  ax  +  b  with  a    b  and  suppose  the  tangent  lines  to  the  graph  of  ƒ    at  x  =  a  and  x  =  b  have  the
same  gradient.  Then  the  value  of  ƒ  (1)  is  equal  to  -

IN
1 2
(A) 0 (B) 1 (C) – (D) 
3 3
4. The tangent to the curve 3xy 2 – 2x 2y = 1 at (1,1) meets the curve again at the point -

R
 16 1   16 1   1 16   1 16 
(A)   , (B)    ,  (C)   , (D)    ,

P
 20   
 5 20   5  20 5   20 5 

The  curve  y  –  e xy   +  x  =  0  has  a  vertical  tangent  at  -

S
5.
(A) (1,1) (B) (0,1) (C) (1,0) (D) no  point
6. Suppose  ƒ    and  g  both  are  linear  function  with  ƒ  (x)  =  –2x  +  1  and  ƒ  (g(x))  =  6x  –  7,  then  slope  of  line  y  =  g(x)

E
is -
(A) 3 (B) –3 (C) 6 (D) –2

E
7. A  curve  with  equation  of  the  form  y  =  ax 4+  bx 3   +  cx  +  d  has  zero  gradient  at  the  point  (0,1)  and  also

J
touches  the  x-axis  at  the  point  (–1,0)  then  the  values  of  x  for  which  the  curve  has  a  negative  gradient  are -
(A) x  >  –1 (B) x  <  1 (C) x  <  –1 (D) –1   x   1


8. The  line  which  is  parallel  to  x-axis  and  crosses  the  curve  y  x   at  an  angle  of    is  -
4
1 1 1 1
(A)  y   (B)  x  (C)  y  (D)  y 
2 2 4 2
3 2 4 3 2
9. The  lines  tangent  to  the  curves  y   –  x y  +  5y  –  2x  =  0    and    x   –  x y   +5x  +2y  =  0  at  the  origin  intersect
at  an  angle   equal  to  -

   
(A)  (B)  (C)  (D) 
6 4 3 2
10. The  angle  of  intersection  of  the  curves  2y  =  x3  and  y2  =  32x  at  origin  is  -

   
(A)  (B)  (C)  (D) 
6 3 2 4

11. The  angle  of  intersection  of  x  =  y   and  x3  +  6y  =  7  at  (1,  1)  is  -

   
(A)  (B)  (C)  (D) 
5 4 3 2

Page 89
90
JEE Sprint Mathematics
subnormal
12. At  any  point  of  a  curve    is  equal  to  -
sub tan gent

(A) the  abscissa  of  that  point (B) the  ordinate  of  that  point
(C) slope  of  the  tangent  at  that  point (D) slope  of  the  normal  at  that  point
13. The  length  of  the  tangent  to  the  curve  x  =  a(  +  sin),  y  =  a(1  –  cos)  at   points  is  -


(A) 2asin (B) asin (C) 2asin (D) acos
2

14. The  length  of  the  subnormal  of  the  curve  y2  =  8ax  (a  >  0)  is  -

T
(A) 2a (B) 4a (C) 6a (D) 8a
15. A 13 ft. ladder is leaning against a wall when its base starts to slide away. At the instant when the base is 12 ft.
away from the wall, the base is moving away from the wall at the rate of 5 ft/sec. The rate at which the angle

N
 between the ladder  and the  ground is   changing is  -

I
12 13 10
(A)   rad / sec . (B)  –1  rad/sec. (C)   rad / sec . (D)   rad / sec .
13 12 13

R
16. Water  is  poured  into  an  inverted  conical  vessel  of  which  the  radius  of  the  base  is    2m  and  height    4m  ,
at  the  rate  of  77  litre/minute.  The  rate  at  which  the  water  level  is  rising  at  the  instant  when  the  depth  is

P
70  cm  is  - (use    =  22/7)

S
(A) 10  cm/min (B) 20  cm/min (C) 40  cm  /min (D) none
17. A  point  is  moving  along  the  curve  y3  =  27x.  The  interval  in  which  the  abscissa  changes  at  slower  rate  than
ordinate,  is  -

E
(A) (–3,  3) (B) (–,) (C) (–1,  1) (D) (  , 3)  (3 , )

18. A  particle  moves  along  the  curve  y  =  x3/2  in  the  first  quadrant  in  such  a  way  that  its  distance  from  the  origin

E
dx

J
increases  at  the  rate  of  11  units  per  second.  The  value  of    when  x  =  3  is  -
dt

9 3 3
(A) 4 (B)  (C)  (D) none  of  these
2 2

SELECT  THE  CORRECT  ALTERNATIVES  (ONE  OR  MORE  THAN  ONE  CORRECT  ANSWERS)

19. Which  of  the  following  pair(s)  of  curves  is/are  orthogonal.

(A) y 2   =  4ax  ;  y  =  e –x/2a (B) y 2   =  4ax  ;  x 2   =  4ay

(C) xy  =  a 2   ;  x 2  –  y 2   =  b 2 (D) y  =  ax  ;  x 2   +  y 2   =  c 2

x y K
20. If    1   is  a  tangent  to  the  curve  x  =  Kt, y  , K  0   then  :
a b t

(A) a  >  0,  b  >  0 (B) a  >  0,  b  <  0 (C) a  <  0  ,  b  >  0 (D) a  <  0  ,  b  <  0

x 3 5x 2
21. The  coordinates  of  the  point(s)  on  the  graph  of  the  function,  f(x)    7x  4   where  the  tangent  drawn
3 2
cut  off  intercepts  from  the  coordinate  axes  which  are  equal  in  magnitude  but  opposite  in  sign,  is  -
(A) (2,8/3) (B) (3,7/2) (C) (1,5/6) (D) none

Page 90
91
JEE Sprint Mathematics
22. For  the  curve  represented  parametrically  by  the  equations,  x  =  2  n  cot  t  +  1  and  y  =  tan t  +  cot t

(A) tangent  at  t  =  /4  is  parallel  to  x-axis

(B) normal  at  t  =  /4  is  parallel  to  y-axis

(C) tangent  at  t  =  /4  is  parallel  to  the  line  y  =  x

(D) normal  at  t  =  /4  is  parallel  to  the  line  y  =  x

23. Consider  the  curve  f(x)  =  x 1/3,  then -

(A) the  equation  of  tangent  at  (0,  0)  is  x  =  0 (B) the  equation  of  normal  at  (0,  0)  is  y  =  0

(C) normal  to  the  curve  does  not  exist  at  (0,  0) (D) f(x)  and  its  inverse  meet  at  exactly  3  points.
24. Equation  of  common  tangent(s)  of  x2  –  y2  =  12  and  xy  =  8  is  (are)  -

T
(A) y  =  3x  +  4 6 (B) y  =  –3x  +  4 6 (C) 3y  =  x  +  4 6 (D) y  =  –3x  –  4 6

R IN
S P
JE E
ANSWER  KEY
Que. 1 2 3 4 5 6 7 8 9 10
Ans. C B B B C B C D D C
Que. 11 12 13 14 15 16 17 18 19 20
Ans. D C A B B B C A A,C,D A,D
Que. 21 22 23 24
Ans. A,B A,B A,B,D B,D

Page 91
92
JEE Sprint Mathematics
EXTRA PRACTICE QUESTIONS ON TANGENT AND NORMALS

SELECT  THE  CORRECT  ALTERNATIVES  (ONE  OR  MORE  THAN  ONE  CORRECT  ANSWERS)
1. The  angle  at  which  the  curve  y  =  Ke Kx   intersects  the  y-axis  is  -

1  1 
(A) tan –1k 2 (B) cot–1(k 2 ) (C)  sin  (D) sec 1  1  k 4 
 1  k 4 

2. The  coordinates  of  points(s)  at  each  of  which  the  tangents  to  the  curve  y  =  x 3  –  3x 2  –  7x  +  6  cut  off  on
the  positive  semi  axis  OX  a  line  segment  half  that  on  the  negative  semi  axis  OY  is/are  given  by  :
(A) (–1,9) (B) (3,–15) (C) (1,–3) (D) none

3. The  abscissa  of  the  point  on  the  curve  xy  a  x ,  the  tangent  at  which  cuts  off  equal  intercepts  from

T
the  co-ordinate  axes  is  (a  >  0)
a a

N
(A)  (B) – (C) a 2 (D) – a 2
2 2

I
4. A  cubic  polynomial  f(x)  =  ax3  +  bx2  +  cx  +  d  has  a  graph  which  is  tangent
to  the  x-axis  at  2,  has  another  x-intercept  at  –1,  and  has  y-intercept

R
at  –2  as  shown.  The  values  of,  a  +  b  +  c  +  d  equals -     

(A) –2

P
(B) –1

S
(C) 0

(D) 1

E

5. Equation  of  a  tangent  to  the  curve  y cot x  y 3 tanx  at  the  point  where  the  abscissa  is    is  -
4

E
(A) 4x  +  2y  =   +  2 (B) 4x  –  2y  =    +  2 (C) x  =  0 (D) y  =  0

J
6. Consider  the  curve  represented  parametrically  by  the  equation
x  =  t 3  –  4t 2  –  3t  and    y  =  2t 2   +  3t  –  5  where  t   R
If  H  denotes  the  number  of  point  on  the  curve  where  the  tangent  is  horizontal  and  V  the  number  of  point
where  the  tangent  is  vertical  then-
(A) H  =  2  and  V  =  1 (B) H  =  1  and  V  =  2 (C) H  =  2  and  V  =  2 (D) H  =  1  and  V  =  1
7. If  y  =  f(x)  be  the  equation  of  a  parabola  which  is  touched  by  the  line  y  =  x  at  the  point  where  x  =  1.  Then -
(A) f '(1)  =  1 (B) f '(0)  =  f '(1) (C) 2f(0)  =  1  –  f '(0) (D) f(0)  +  f '(0)  +  f ''(0)  =  1

8. At  the  point  P(a,  an)  on  the  graph  of  y  =  x n (n    N)  in  the  first  quadrant  a  normal  is  drawn.  The  normal

1
intersects  the  y-axis  at  the  point  (0,  b),  If  lim b  =  ,  then  n  equals  -
a 0 2

(A) 1 (B) 3 (C) 2 (D) 4


9. A horse runs along a circle with a speed of 20 km/hr. A lantern is at the centre of the circle. A fence is along
the tangent to the circle at the point at which the horse starts. The speed with which the shadow of the horse
move  along the  fence at the  moment when it covers  1/8 of  the  circle in  km/hr is  -
(A) 20 (B) 60 (C) 30 (D) 40

Page 92
93
JEE Sprint Mathematics
x 2
10. Equation  of  the  line  through  the  point  (1/2,2)  and  tangent  to  the  parabola  y   2   and  secant  to  the
2

curve  y  4  x 2   is  -
(A) 2x  +  2y  –  5  =  0 (B) 2x  +  2y  -  9  =  0
(C) y  –  2  =  0 (D) none
2 3
11. If  the  tangent  at  P  of  the  curve  y   =  x   intersects  the  curve  again  at  Q  and  the  straight  lines  OP,OQ  make
angles  ,    with  the  x-axis  where  ‘O’  is  the  origin  then  tan/tan  has  the  value  equal  to  -

(A) –1 (B) – 2 (C) 2 (D)  2


12. Let  ƒ  (x)  be  a  nonzero  function  whose  all  successive  derivative  exist  and  are  nonzero.  If  ƒ  (x),  ƒ '(x)  and  ƒ ''(x)  are

T
in  G.P.  and  ƒ  (0)  =  1  ,  ƒ '(0)  =  1,  then  -
(A) ƒ '(x) < 0   x  R (B) ƒ ''(x) < 1   x  R
(C) ƒ ''(0)  ƒ '''(0) (D) ƒ ''(x) > 0    x  R

N
13. If  the  line  ax  +  by  +  c  =  0  is  a  normal  to  the  curve  xy  =  1,  then  - [JEE 1986]

I
(A) a  >  0,  b  >  0 (B) a  >  0,  b  <  0 (C) a  <  0,  b  >  0 (D) a  <  0,  b  <  0

P R
E S
JE
ANSWER  KEY
Que. 1 2 3 4 5 6 7 8 9 10
Ans. B,C B A,B B A,B,D B A,C C D A
Que. 11 12 13
Ans. B D B,C

Page 93
94
JEE Sprint Mathematics
MONOTONICITY

SELECT THE CORRECT ALTERNATIVE (ONLY ONE CORRECT ANSWER)


2
1. Function f  x   x 2  x  2  is -

(A) increasing in  0,1   2,  (B) decreasing in  0,1   2, 

(C) decreasing function (D) increasing function

2. The function f  x   tan x  x

(A) always increases (B) always decreases

T
(C) never decreases (D) sometimes increases and sometimes decreases

3. The function f, defined by f(x )  (x  2)e  x is -

N
(A) decreasing for all x (B) decreasing in ( , 1) and increasing in ( 1, )

I
(C) increasing for all x (D) decreasing in ( 1,  ) and increasing in ( , 1)

R
4. Function f  x   x 3  6 x 2   9  2k  x  1 is increasing function if

3 3 3 3
(A) k  (B) k  (C) k  (D) k 

P
2 2 2 2

5. The function f(x)  cos x  2px is monotonically decreasing for

S
1 1
(A) p  (B) p  (C) p < 2 (D) p > 2
2 2

E
6. The value of 'a' for which the function f(x) = sinx – cosx – ax + b decreases for all real values of x, is -

E
(A) a   2 (B) a   2 (C) a  2 (D) a  2

J
7. Let f(x) be a quadratic expression which is positive for all real values of x. If g(x) = f(x) + f'(x) +f''(x), then for any
real x -
(A) g(x) < 0 (B) g(x) > 0 (C) g(x) = 0 (D) g(x)  0

 1 1 
8. f(x) =   2  1  x 2
  dx then f is -
2
1x 

(A) increasing in (0, ) and decreasing in ( –, 0) (B) increasing in (–, 0) and decreasing in (0, )

(C) increasing in (– ) (D) decreasing in ( –, )

 4x
 for 0 < x < 4
2  x
9. Let f(x) = 
4 for x4
16  3x for 4  x  6

Which of the following properties does f have on the interval (0, 6) ?

I. n f(x) exists ; II. f is continuous III. f is monotonic

(A) I only (B) II only (C) III only (D) none

Page 94
95
JEE Sprint Mathematics
10. The length of largest continuous interval in which function f (x) = 4x – tan2x is monotonic, is -
(A) /2 (B) /4 (C) /8 (D) /16
11. The largest set of real values of x for which ln (1 + x)  x is
(A) (–1, ) (B) ( 1, 0)  (0, ) (C) [0, ) (D) (0, )
12. The true set of real values of x for which the function, f(x)  x n x  x  1 is positive is
(A) (1, ) (B) (1 / e, ) (C) [e, ) (D) (0,1)  (1, )

 
13. Number of solution of the equation 3tan x + x3 = 2 in  0,  is
 4
(A) 0 (B) 1 (C) 2 (D) 3

T
14. Rolle's theorem in the indicated intervals will not be valid for which of the following function-

sin x
L x x0 LM 1  cos x
x0
f ( x)  M
x
MMN MMN 0

N
(A) ; x  [  1, 1 ] (B) g ( x)  ; x [  2, 2 ]

I
1 x0 x0

LM
1  cos x
F 1I
LM x sinGH x JK
x0
x2 x0

R
h(x)  M k( x )  M
L 1 1 OP
x  M ,
(C)
MM 1 ; x  [  2 , 2 ] (D)
MN 0
;
N  2 Q
N2

P
x0 x0

S
 x 3  2x 2  5 x  6
if x 1
15. Consider the function for x = [–2, 3], f(x)   x 1 , then

 6 if x 1

E
(A) f is discontinuous at x = 1  Rolle's theorem is not applicable in [–2, 3]

(B) f(–2)  f(3)  Rolle's theorem is not applicable in [–2, 3]

JE
(C) f is not derivable in (–2, 3)  Rolle's theorem is not applicable

(D) Rolle's theorem is applicable as f satisfies all the conditions and c of Rolle's theorem is 1/2

16. If the function f(x) = 2x2 + 3x + 5 satisfies LMVT at x = 2 on the closed interval [1, a] then the value of 'a' is
equal to -
(A) 3 (B) 4 (C) 6 (D) 1

17. Consider the function f(x) = 8x2 – 7x + 5 on the interval [–6, 6]. The value of c that satisfies the conclusion of the
mean value theorem, is -

(A) –7/8 (B) –4 (C) 7/8 (D) 0

18. Let f be a function which is continuous and differentiable for all real x. If f(2) = – 4 and f '( x)  6 for all

x  [2, 4] then
(A) f(4) < 8 (B) f(4)  8 (C) f(4)  12 (D) none of these
19. If f(x) = x 3+ 7x – 1 then f(x) has a zero between x = 0 and x = 1. The theorem which best describes this, is -
(A) Rolle's theorem (B) mean value theorem
(C) maximum-minimum value theorem (D) intermediate value theorem

Page 95
96
JEE Sprint Mathematics

20. Consider f(x) =|1 – x|, 1  x  2 and g(x) = f(x) + b sin x, 1  x  2
2
then which of the following is correct ?
3
(A) Rolles theorem is applicable to both f, g and b =
2
1
(B) LMVT is not applicable to f and Rolles theorem if applicable to g with b =
2
(C) LMVT is applicable to f and Rolles theorem is applicable to g with b = 1
(D) Rolles theorem is not applicable to both f, g for any real b.

T
SELECT THE CORRECT ALTERNATIVES (ONE OR MORE THAN ONE CORRECT ANSWERS)

21. Let h(x) = f(x) – (f (x))2 + (f(x))3 for every real number x. Then.
(A) h is increasing whenever f is increasing (B) h is increasing whenever f is decreasing

N
(C) h is decreasing whenever f is decreasing (D) nothing can be said in general

I
x
22. Let f(x) = e (x  1)(x  2)dx . Then 'f' increases in the interval -
(A) (–, –2) (B) (–2, –1) (C) (1, 2) (D) (2, )

R
2
3x  12x  1 , 1  x  2
23. If f(x) =  then -
37  x , 2x3

P
(A) f(x) is increasing on (–1, 2) (B) f(x) is continuous on [–1, 3]

S
(C) f'(2) does not exist (D) f(x) has the maximum value at x = 2
24. Let f(x) = 8x3 – 6x2 – 2x + 1, then -
(A) f(x) = 0 has no root in (0, 1) (B) f(x) = 0 has at least one root in (0, 1)

E
(C) f'(c) vanishes for some c  (0,1) (D) none
25. Let f and g be two functions defined on an interval I such that f(x)  0 and g(x)  0 for all x  I and f is strictly

E
decreasing on I while g is strictly increasing on I then -

J
(A) the product function fg is strictly increasing on I (B) the product function fg is strictly decreasing on I
(C) fog(x) is monotonically increasing on I (D) fog(x) is monotonically decreasing on I

ANSWER KEY
Que. 1 2 3 4 5 6 7 8 9 10
Ans. A A D A B D B C B B
Que. 11 12 13 14 15 16 17 18 19 20
Ans. A D B D D A D B D C
Que. 21 22 23 24 25
Ans. A,C A,B,D A,B, C,D B,C A,D

Page 96
97
JEE Sprint Mathematics
EXTRA PRACTICE QUESTIONS ON MONOTONICITY

SELECT THE CORRECT ALTERNATIVES (ONE OR MORE THAN ONE CORRECT ANSWERS)

1. A differentiable function f(x) is strictly increasing  x  R , Then -

(A) f '( x )  0  x  R .

(B) f '( x )  0  x  R , provided it vanishes at finite number of points.

(C) f '( x )  0  x  R provided it vanishes at discrete points though the number of these
discrete points may not be finite.

T
(D) f '( x )  0  x  R provided it vanishes at discrete points and the number of these discrete points must be
infinite.

N
2x  1
2. The function y  (x  2) -

I
x 2

(A) is its own inverse (B) decreases for all values of x

R
(C) has a graph entirely above x-axis (D) is bound for all x
3. If f(0) = f(1) = f(2) = 0 & function f(x) is twice differentiable in (0, 2) and continuous in [0, 2]. Then which of the

P
following is/are definitely true -

(A) f ''(c)  0 ;  c  (0 , 2) (B) f '(c)  0 ; for atleast two c  (0, 2)

S
(C) f '(c)  0 ; for exactly one c  (0, 2) (D) f '' (c )  0 ; for atleast one c  (0, 2)

E

 x sin for x  0
4. Consider the function f(x)   x then the number of points in (0, 1) where the derivative f'(x)
0 for x  0

E
vanishes, is -
(A) 0 (B) 1 (C) 2 (D) infinite

J
5. Let (x) = (f(x))3 – 3(f(x))2 + 4f(x) + 5x + 3sinx + 4 cosx  x  R, then -
(A)  is increasing whenever f is increasing (B)  is increasing whenever f is decreasing
(C)  is decreasing whenever f is decreasing (D)  is decreasing if f'(x) = –11
x x
6. If f(x )  and g(x )  , where 0 < x  1, then in this interval -
sin x tan x
(A) both f(x) and g(x) are increasing functions (B) both f(x) and g(x) are decreasing functions
(C) f(x) is an increasing function (D) g(x) is an increasing function
7. f(x) > x ;  x  R. Then the equation f (f(x)) – x = 0 has–
(A) Atleast one real root
(B) More than one real root
(C) no real root if f(x) is a polynomial & one real root if f(x) is not a polynomial
(D) no real root at all

1 1
8. A function y  f(x) is given by x  2
& y for all t > 0 then f is :
1t t(1  t 2 )
(A) increasing in (0,3/2) & decreasing in (3/2, ) (B) increasing in (0,1)
(C) increasing in (0,) (D) decreasing in (0,1)

Page 97
98
JEE Sprint Mathematics
9. Suppose that f is differentiable for all x such that f'(x)  2 for all x. If f(1) = 2 and f(4) = 8 then f(2) has the value
equal to -
(A) 3 (B) 4 (C) 6 (D) 8
n(   x)
10. The function f(x)  is -
n(e  x)
(A) increasing on (0, ) (B) decreasing on (0, )
(C) increasing on (0, e), decreasing on (/e, ) (D) decreasing on (0, e), increasing on (/e, )

11. Number of solution(s) satisfying the equation, 3x 2  2x 3  log 2 (x 2  1)  log 2 x is -

(A) 1 (B) 2 (C) 3 (D) none

T
12. Let g (x) = 2f (x/2) + f (1 – x) and f''(x) < 0 in 0  x  1 then g (x) -

(A) decreases in [0,2/3) (B) decreases in (2/3,1] (C) increases in [0,2/3) (D) increases in (2/3,1]

N
|x|

If f(x )  a   ; g(x)  a a


a| x| sgn x sgn x 
13. for a > 0, a  1 and x  R , where { } & [ ] denote the fractional part and

I
integral part functions respectively, then which of the following statements holds good for the function h (x),
where (  n a)h(x)  (  n f(x)   n g(x)) -

R
(A) ‘h’ is even and increasing for a > 1 (B) ‘h’ is odd and decreasing for a < 1

P
(C) ‘h’ is even and decreasing for a < 1 (D) ‘h’ is odd and increasing for a > 1

14. Number of roots of the equation x 2 .e 2 |x|  1 is -

S
(A) 2 (B) 4 (C) 6 (D) infinite
1
15. Equation  3x  sin x = 0 has -

E
(x  1) 3
(A) no real root (B) two real and distinct roots

E
(C) exactly one negative root (D) exactly one root between –1 and 1

J
 p4 
16. The values of p for which the function f(x)    1 x 5  3x  n 5 decreases for all real x is
 1p 

 3  21   5  27 
(A) ( , ) (B)  4,   (1,  ) (C)  3,   (2,  ) (D) [1, )
 2   2 

ANSWER KEY
Que. 1 2 3 4 5 6 7 8 9 10
Ans. C A,B B,D D A,D C D B B B
Que. 11 12 13 14 15 16
Ans. A B,C D B B,C,D B

Page 98
99
JEE Sprint Mathematics
MAXIMA AND MINIMA
SELECT THE CORRECT ALTERNATIVE (ONLY ONE CORRECT ANSWER)

1. On the interval [0,1] the function x 25 (1–x) 75 takes its maximum value at the point -
(A) 0 (B) 1/3 (C) 1/2 (D) 1/4
2. The value of 'a' so that the sum of the squares of the roots of the equation x2 – (a – 2)x – a + 1 = 0 assume the
least value is -
(A) 2 (B) 0 (C) 3 (D) 1
3. The slope of the tangent to the curve y = –x 3 + 3x 2 + 9x – 27 is maximum when x equals -
(A) 1 (B) 3 (C) 1/2 (D) –1/2
4. The real number x when added to it's reciprocal gives the minimum value of the sum at x equal to -

T
(A) 1 (B) – 1 (C) – 2 (D) 2
5. If the function f(x) = 2x 3 – 9ax 2 + 12a2x + 1 where a > 0, attains it’s maximum and minimum at p and q

N
respectively such that p2 = q then ‘a’ equals -

I
(A) 1 (B) 2 (C) 1/2 (D) 3
6. If f(x) = 1 + 2x2 + 4x4 + 6x6 + .....+ 100x100 is a polynomial in a real variable x, then f(x) has -

R
(A) neither a maximum nor a minimum (B) only one maximum
(C) only one minimum (D) none

P
7. For all a, b  R the function f(x) = 3x4 – 4x3 + 6x2 + ax + b has -
(A) no extremum (B) exactly one extremum

S
(C) exactly two extremum (D) three extremum

LM sin 2x , 0  x 1

MMN3  2 x ,

E
8. Let f ( x )  then :
x 1

E
(A) f(x) has local maxima at x = 1 (B) f(x) has local minima at x = 1

J
(C) f(x) does not have any local extrema at x = 1 (D) f(x) has a global minima at x = 1
9. Two sides of a triangle are to have lengths ‘a’ cm & ‘b’ cm. If the triangle is to have the maximum area, then the
length of the median from the vertex containing the sides ‘a’ and ‘b’ is -

1 2 2a  b a 2  b2 a  2b
(A) a  b2 (B) (C) (D)
2 3 2 3

2 x
10. The difference between the greatest and the least value of f(x)  cos sin x, x  [0, ] is -
2

3 3 3 3 1
(A) (B) (C) (D)
8 8 8 2 2
11. Equation of a straight line passing through (1,4) if the sum of its positive intercepts on the coordinate axis is the
smallest is -
(A) 2x + y – 6 = 0 (B) x + 2y – 9 = 0 (C) y + 2x + 6 = 0 (D) none
12. A rectangle has one side on the positive y-axis and one side on the positive x-axis. The upper right hand vertex
nx
on the curve y  . The maximum area of the rectangle is -
x2
(A) e–1 (B) e–1/2 (C) 1 (D) e1/2

Page
Page100
99
JEE Sprint Mathematics
13. A solid rectangular brick is to be made from 1 cu feet of clay. The brick must be 3 times as long as it is wide. The
width of brick for which it will have minimum surface area is a. Then a3 is -
1/3
2  2 8 3
(A)   (B) (C) (D)
9  9 729 2
14. Let h be a twice continuously differentiable positive function on an open interval J. Let
g(x) = n(h(x) for each x J
Suppose (h'(x))2 > h''(x)h(x) for each x J. Then
(A) g is increasing on J (B) g is decreasing on J
(C) g is concave up on J (D) g is concave down on J
15. Function f(x), g(x) are defined on [–1, 3] and f''(x) > 0, g''(x) > 0 for all x  [–1, 3], then which of the following is
always true ?

T
(A) f(x) – g(x) is concave upwards on (–1, 3) (B) f(x) g(x) is concave upwards on (–1, 3)
(C) f(x) g(x) does not have a critical point on (–1, 3) (D) f(x) + g(x) is concave upwards on (–1, 3)
16. If the point (1,3) serves as the point of inflection of the curve y = ax3 + bx2 then the value of ‘a’ and ‘b’ are -

N
(A) a = 3/2 & b = – 9/2 (B) a = 3/2 & b = 9/2

I
(C) a = –3/2 & b = – 9/2 (D) a = – 3/2 & b = 9/2
ax 3
17. The set of value (s) of ‘a’ for which the function f(x )   (a  2)x 2  (a  1) x  2 possess a negative point of

R
3
inflection -
(A) ( , 2)  (0, ) (B) {–4/5} (C) (–2,0) (D) empty set

P
18. Which of the following statements is true for the general cubic function ƒ (x) = ax3
+ bx2 + cx + d (a  0)
I. If the derivative ƒ '(x) has two distinct real roots then cubic has one local maxima and one local minima.

S
II. If the derivative ƒ '(x) has exactly one real root then the cubic has exactly one relative extremum.
III. If the derivative ƒ '(x) has no real roots, then the cubic has no relative extrema
(A) only I & II (B) only II and III (C) only I and III (D) all I, II, III are correct.

E
SELECT THE CORRECT ALTERNATIVES (ONE OR MORE THEN ONE CORRECT ANSWERS)
19. If y = an|x| + bx 2 + x has its extremum values at x = –1 and x = 2, then :-

E
1 1

J
(A) a = 2, b = – 1 (B) a = 2, b = – (C) a = –2, b = (D) none of these
2 2
20. Let S be the set of real values of parameter  for which the function ƒ (x) = 2x3 – 3(2 + )x2 + 12x has exactly
one local maxima and exactly one local minima. Then the subset of S is -
(A) (5, ) (B) (–4, 4) (C) (3, 8) (D) (–, –1)
3 1
 x  cos a, 0  x  1
21. The value of 'a' for which the function ƒ (x)   has a local minimum at x = 1, is -
2
 x , x 1
1
(A) –1 (B) 1 (C) 0 (D) 
2
22. If a continuous function ƒ (x) has a local maximum at x = a, then -
(A) ƒ '(a+) may be 0 (B) ƒ '(a+) may be –
+
(C) ƒ '(a ) may be non-zero finite real number (D) ƒ '(a–) may be –

ANSWER KEY
Que. 1 2 3 4 5 6 7 8 9 10 11 12 13 14 15
Ans. D D A A B C B A A A A A B D D
Que. 16 17 18 19 20 21 22
Ans. D A C B A ,C , D A , D A, B, C

Page 100
101
JEE Sprint Mathematics
EXTRA PRACTICE QUESTIONS ON MAXIMA AND MINIMA
SELECT THE CORRECT ALTERNATIVES (ONE OR MORE THEN ONE CORRECT ANSWERS)

1. The set of values of p for which the points of extremum of the function,
f(x) = x3 –3px2 + 3 (p2 –1)x + 1 lie in the interval (–2,4) is -
(A) (–3,5) (B) (–3,3) (C) (–1,3) (D) (–1,5)

 Min {f(t) : 0  t  x} ; 0  x  1 1  3 5


2. If f (x)= 4x3 - x2 - 2x +1 and g(x )   then g    g    g  
3  x ; 1  x  2 4 4 4
has the value equal to :
7 9 13 5
(A) (B) (C) (D)
4 4 4 2

T
3. The function ‘f’ is defined by f(x)  x p (1  x) q for all x  R, where p, q are positive integers, has a maximum
value, for x equal to :

N
pq p
(A) (B) 1 (C) (D) 0

I
pq pq
x2  x  2
4. If the point of minima of the function , f(x) =1 + a2x – x3 satisfy the inequality 2  0 , then ‘a’ must lie

R
x  5x  6
in the interval :
(A)  3 3, 3 3  (C)  2 3, 3 3 

P
(C) 2 3, 3 3  (D)  3 3, 2 3   2 3, 3 3 

S
x
5. The function f(x)   1  t 4 dt is such that :
0
(A) it is defined on the interval [–1,1] (B) it is an increasing function

E
(C) it is an odd function (D) the point (0,0) is the point of inflection
sin(x  a)
6. The function has no maxima or minima if -

E
sin(x  b)

J
(A) b – a = n n  I (B) b – a = (2n+1) n  I

(C) b – a = 2n  n  I (D) none of these.


7. The coordinates of the point P on the graph of the function y = e–|x| , where area of triangle made by tangent
and the coordinate axis has the greatest area, is -
 1  1
(A)  1,  (B)  1,  (C) (e, e  e ) (D) none
e e
4 1
8. The least value of ‘a’ for which the equation   a has atleast one solution on the interval (0,  / 2)
sin x 1  sin x
is -
(A) 3 (B) 5 (C) 7 (D) 9
9. Read of the following mathematical statements carefully :
I. A differentiable function 'f' with maximum at x = c  f''(c) < 0.
II. Antiderivative of a periodic function is also a periodic function.
T T

III. If f has a period T then for any a  R,  f(x)dx   f(x  a)dx


0 0

IV. If f(x) has a maxima at x = c, then 'f' is increasing in (c – h, c) and decreasing in (c, c + h) as h  0 for
h>0

Page 101
102
JEE Sprint Mathematics
Now indicate the correct alternative.
(A) exactly one statement is correct (B) exactly two statements are correct
(C) exactly three statements are correct (D) all the four statements are correct
10. The lateral edge of a regular rectangular pyramid is ‘a’ cm long. The lateral edge makes an angle  with the
plane of the base. The value of  for which the volume of the pyramid is greatest, is -
 1 2 1 
(A) (B) sin (C) cot 2 (D)
4 3 3
11. P and Q are two points on a circle of centre C and radius , the angle PCQ being 2 then the radius of the circle
inscribed the triangle CPQ is maximum when -
3 1 5 1 5 1 5 1
(A) sin   (B) sin   (C) sin   (D) sin  
2 2 2 2 4

T
12. In a regular triangular prism the distance from the centre of one base to one of the vertices of the other base
is  . The altitude of the prism for which the volume is greatest -

N
(A) /2 (B)  / 3 (C) /3 (D) /4

I
13. Let P(x) = a 0 + a 1 x 2 + a 2 x 4 + ......... + a n x 2n be a polynomial in a real variable x with
0 < a 0 < a 1 < a 2 < .... < a n . The function P(x) has- [JEE 1986]
(A) neither a maximum nor a minimum (B) only one maximum

R
(C) only one minimum (D) only one maximum and only one minimum
2
14. If g(x) = 7x 2 e  x  x  R , then g(x) has -

P
(A) local maximum at x = 0 (B) local minima at x = 0
(C) local maximum at x = –1 (D) two local maxima and one local minima

S
2
15. The coordinates of the point on the parabola y = 8x, which is at minimum distance from the circle
x2 + (y + 6)2 = 1 are -
(A) (2, –4) (B) (18, –2) (C) (2, 4) (D) none of these

JE E
ANSWER KEY
Que. 1 2 3 4 5 6 7 8 9 10
Ans. C D C D A,B, C,D A, B, C A,B D A C
Que. 11 12 13 14 15
Ans. B B C B,C,D A

Page 102
103
JEE Sprint Mathematics
VECTOR ALGEBRA
SELECT THE CORRECT ALTERNATIVE (ONLY ONE CORRECT ANSWER)
     
1. If ABCDEF is a regular hexagon and if AB  AC  AD  AE  AF   AD , then  is -
(A) 0 (B) 1 (C) 2 (D) 3
   
2. If a  b is along the angle bisector of a & b then -
   
(A) a & b are perpendicular (B) a  b
   
(C) angle between a & b is 60° (D) a  b

T
3. Given the points A (2, 3, 4) , B (3, 2, 5) , C (1, 1, 2) & D (3, 2, 4) . The projection of the vector AB on

the vector CD is -

N
22 21 47

I
(A) (B)  (C)  (D) –47
3 4 7
 
4. The vectors AB  3ˆi  2ˆj  2kˆ and BC   î  2kˆ are the adjacent sides of a parallelogram ABCD then the

R
angle between the diagonals is -

P
1
 1  1
 49   1   3 
(A) cos   (B)  – cos   (C) cos 1   (D) cos 1  
 85   85  2 2   10 
 

S
5. ˆ ˆi – 3ˆj – 5kˆ and aiˆ  3ˆj  kˆ
The values of a, for which the points A, B, C with position vectors 2ˆi – ˆj + k,


respectively are the vertices of a right angled triangle with C  are -

E
2
(A) –2 and 1 (B) 2 and –1 (C) 2 and 1 (D) –2 and –1
     

E
a .a a.b a .c
        
ˆ b  ˆi  ˆj  k,
ˆ c  ˆi  2ˆj  kˆ , then the value of

J
6. If a  ˆi  ˆj  k, b.a b.b b.c 
     
c.a c.b c.c

(A) 2 (B) 4 (C) 16 (D) 64

7. The area of the triangle whose vertices are A (1, –1, 2) ; B (2, 1, –1) ; C (3, –1, 2) is -
(A) 13 (B) 2 13 (C) 13 (D) none
         
8. Let a  ˆi  ˆj & b  2 ˆi  kˆ . The point of intersection of the lines r x a  b x a & r x b  a x b is -

(A)  ˆi  ˆj  kˆ (B) 3 ˆi  ˆj  kˆ (C) 3 ˆi  ˆj  kˆ (D) ˆi  ˆj  kˆ

      2        
9. If a, b, c are non-coplanar vectors and  is a real number then  (a  b )  b  c    a b  c b  for -
   
(A) exactly two values of  (B) exactly three values of 
(C) no value of  (D) exactly one value of 
10. Volume of the tetrahedron whose vertices are represented by the position vectors , A (0, 1, 2) ; B (3, 0, 1) ;
C (4, 3, 6) & D (2, 3, 2) is -
(A) 3 (B) 6 (C) 36 (D) none

Page 103
104
JEE Sprint Mathematics
11. The sine of angle formed by the lateral face ADC and plane of the base ABC of the tetrahedron ABCD where
A  (3, –2, 1) ; B  (3, 1, 5); C  (4, 0, 3) and D  (1, 0, 0) is -

2 5 3 3 2
(A) (B) (C) (D)
29 29 29 29
12. Given the vertices A (2, 3, 1), B (4, 1, –2), C (6, 3, 7) & D (–5, –4, 8) of a tetrahedron. The length of the altitude
drawn from the vertex D is -
(A) 7 (B) 9 (C) 11 (D) none
        1   
13. Let a , b and c be non-zero vectors such that a and b are non-collinear & satisfies (a  b )  c  | b|| c | a .
3
 
If  is the angle between the vectors b and c then sin equals -

T
2 2 1 2 2
(A) (B) (C) (D)
3 3 3 3

N
  
The value of ˆi  (r  ˆi )  ˆj  (r  ˆj)  kˆ  (r  kˆ) is -

I
14.
   
(A) r (B) 2 r (C) 3r (D) 4 r

R
     
15. A, B, C, D be four points in a space and if, | AB  CD  BC  AD  CA  BD| =  (area of triangle ABC)
then the value of  is -

P
(A) 4 (B) 2 (C) 1 (D) none of these

S
16. ˆ 3ˆj – k,
If the volume of the parallelopiped whose conterminous edges are represented by –12ˆi  k, ˆ 2ˆi  ˆj – 15kˆ

is 546, then  equals-


(A) 3 (B) 2 (C) –3 (D) –2

E
 
17. Let a = 2 î + 3 ĵ – k̂ and b = î – 2 ĵ + 3 k̂ . Then the value of  for which the vector
  

E
c =  î + ĵ + (2 – 1) k̂ is parallel to the plane containing a and b , is-

J
(A) 1 (B) 0 (C) –1 (D) 2
     
18. If a + 5 b = c and a – 7 b = 2 c , then-
   
(A) a and c are like but b and c are unlike vectors
   
(B) a and b are unlike vectors and so also a and c
   
(C) b and c are like but a and b are unlike vectors
   
(D) a and c are unlike vectors and so also b and c
     
  
19. If a , b , c are three non-coplanar and p , q , r are reciprocal vectors to a , b and c respectively, then
   
 
( a + m b + n c ).( p + m q + n r ) is equal to : (where , m, n are scalars)
(A) 2 + m2 + n2 (B) m + mn + n (C) 0 (D) none of these
 
20. If x & y are two non collinear vectors and a, b, c represent the sides of a ABC satisfying
   
(a  b)x  (b  c)y  (c  a)(x  y)  0 then ABC is -
(A) an acute angle triangle (B) an obtuse angle triangle
(C) a right angle triangle (D) a scalene triangle
         
21. If A , B and C are three non-coplanar vectors then ( A + B + C ).[( A + B ) × ( A + C )] equals -
        
(A) 0 (B) [ A B C ] (C) 2[ A B C ] (D) –[ A B C ]

Page 104
105
JEE Sprint Mathematics
SELECT THE CORRECT ALTERNATIVES (ONE OR MORE THAN ONE CORRECT ANSWERS)

22. ABCD is a parallelogram. E and F be the middle points of the sides AB and BC, then -
(A) DE trisect AC (B) DF trisect AC
(C) DE divide AC in ratio 2 : 3 (D) DF divide AC in ratio 3 : 2
      
23. a , b , c are mutually perpendicular vectors of equal magnitude then angle between a  b  c and a is -

FG 1 IJ F1I F1I
(A) cos 1 H3K (B) cos 1 GH 3 JK (C)  – cos 1 GH 3 JK (D) tan1 2

              
24. If (a  b)  c  a  (b  c) , where a, b and c are any three vectors such that a . b  0, b.c  0 then a and c

are -

T
(A) perpendicular (B) parallel
(C) non collinear (D) linearly dependent

      b  c
e j

N
25. If a , b & c are non coplanar unit vectors such that a  b  c = , then the angle between -
2

I
  3      3   
(A) a & b is (B) a & b is (C) a & c is (D) a & c is
4 4 4 4
     

R
26. If a, b, c, d, e, f are position vectors of 6 points A, B, C, D, E & F respectively such that
      

P
3a  4b  6c  d  4e  3 f  x , then -
 
(A) AB is parallel to CD

S
(B) line AB, CD and EF are concurrent

x
(C) is position vector of the point dividing CD in ratio 1 : 6
7

E
(D) A, B, C, D, E & F are coplanar
27. Read the following statement carefully and identify the true statement -

E
(a) Two lines parallel to a third line are parallel.

J
(b) Two lines perpendicular to a third line are parallel.
(c) Two lines parallel to a plane are parallel.

(d) Two lines perpendicular to a plane are parallel.

(e) Two lines either intersect or are parallel.


(A) a & b (B) a & d (C) d & e (D) a

1 ˆ ˆ ˆ
28. The vector (2 i  2 j  k) is -
3

(A) unit vector (B) makes an angle /3 with vector 2ˆi  4 ˆj  3kˆ

(C) parallel to the vector ˆi  ˆj  (1 / 2)kˆ (D) perpendicular to the vector 3ˆi  2ˆj  2kˆ

 
29. If a vector r of magnitude 3 6 is collinear with the bisector of the angle between the vectors a  7 i  4 j  4 k
 
& b   2 i  j  2 k , then r =

13ˆi  ˆj  10kˆ
(A) i  7 j  2 k (B) i  7 j  2 k (C) (D) i  7 j  2 k
5

Page 105
106
JEE Sprint Mathematics
30. A parallelopiped is formed by planes drawn through the points (1, 2, 3) and (9, 8, 5) parallel to the coordinate
planes then which of the following is the length of an edge of this rectangular parallelopiped -
(A) 2 (B) 4 (C) 6 (D) 8
31. If A (a ) ; B (b) ; C (c ) and D (d ) are four points such that a = –2 î + 4 ĵ + 3 k̂ ; b = 2 î – 8 ĵ ; c = î – 3 ĵ + 5 k̂ ;
d = 4 î + ĵ – 7 k̂ , d is the shortest distance between the lines AB and CD, then
  
[AB CD BD]
(A) d = 0, hence AB and CD intersect (B) d =  
AB  CD
  
23 [AB CD AC]
(C) AB and CD are skew lines and d = (D) d =  
13 AB  CD

N T
R I
S P
JE E
ANSWER KEY
Que. 1 2 3 4 5 6 7 8 9 10
Ans. D B C D C C A C C B
Que. 11 12 13 14 15 16 17 18 19 20
Ans. B C D B A C B A A A
Que. 21 22 23 24 25 26 27 28 29 30
Ans. D A,B B,D B,D A,D B,C B,D A ,C , D A,C A ,C , D
Que. 31
Ans. B,C,D

Page 106
107
JEE Sprint Mathematics
EXTRA PRACTICE QUESTIONS ON VECTOR ALGEBRA
SELECT THE CORRECT ALTERNATIVES (ONE OR MORE THAN ONE CORRECT ANSWERS)

1.
 ˆ b  ˆi  2ˆj  kˆ and c  ˆi  ˆj  2kˆ be three vectors. A vector in the plane of b and c whose
Let a  2ˆi  ˆj  k,

projection on a is magnitude 2 / 3 is -

(A) 2ˆi  3ˆj  3kˆ (B) 2ˆi  3ˆj  3kˆ (C) 2ˆi  5 ˆj  kˆ (D) 2ˆi  ˆj  5kˆ
              
2. Let a, b, c are three non-coplanar vectors such that r1  a  b  c , r2  b  c  a , r3  c  a  b ,
       
r  2a  3b  4c . If r  1 r1  2 r2   3 r3 , then -
(A) 1  7 (B) 1   3  3 (C) 1   2   3  4 (D)  3   2  2

T
  1  
3. Taken on side AC of a triangle ABC, a point M such that AM  AC . A point N is taken on the side CB such
3
   

N
that BN  CB then, for the point of intersection X of AB & MN which of the
following holds good ?

I
   1     
(A) XB  1 AB (B) AX  AB (C) XN  3 MN (D) XM  3XN
3 2 4

R

4. Vector A has components A 1, A 2, A 3 along the three axes. If the co-ordinates system is rotated by 90°
about z-axis, then the new components along the axes are -

P
(A) A1,  A2, A3 (B)  A1,  A2, A3 (C) A2,  A1, A3 (D)  A2,  A1, A3
   
5. Let p, q, r be three mutually perpendicular vectors of the same magnitude. If a vector x satisfies the
            

S
equation p  ((x  q )  p)  q  ((x  r )  q)  r  ((x  p)  r )  0 . Then x is given by -

1    1    1    1   
(A) (p  q  2r ) (B) (p  q  r ) (C) (p  q  r ) (D) (2p  q  r )
2 3 2 3

E
1 ˆ ˆ 1
6. A vector which makes equal angles with the vectors (i  2 j  2kˆ), ( 4 ˆi  3kˆ), ˆj is -
3 5

E
(A) 5 ˆi  ˆj  5kˆ (B) 5 ˆi  ˆj  5kˆ (C) 5ˆi  ˆj  5kˆ (D) 5 ˆi  ˆj  5kˆ

J
     
7.     
The triple product d  a · a  b  c  d  simplifies to -
 
              
(A) ( b. d)[d a c] (B) (b.c)[a b d] (C) (b.a)[a b d] (D) none
  
8. If the vectors a, b, c are non-coplanar and ,m,n are distinct real numbers, then
  
( a  mb  nc)
    
( b  mc  na) ( c  ma  nb)  = 0 implies -
 
(A) m + mn+ n = 0 (C) 2 + m2+ n2 = 0
(B)  + m + n = 0 (D) 3 + m3 + n3 = 0
    
9. If unit vectors ˆi & ˆj are at right angles to each other and p  3 ˆi  4 ˆj , q  5 ˆi , 4 r  p  q and
  
2 s  p  q , then -
     
(A) r  k s = r  k s for all real k (B) r is perpendicular to s
       
(C) r  s is perpendicular to r  s (D) r  s  p  q

10. ˆ kˆ  ˆi taken two at a time form three planes, The three unit vectors drawn
The three vectors ˆi  ˆj, ˆj  k,
perpendicular to these planes form a parallelopiped of volume :
1 3 3 4
(A) (B) 4 (C) (D)
3 4 3 3

Page 107
108
JEE Sprint Mathematics
11. If a, b, c are different real numbers and a ˆi  b ˆj  c kˆ ; b ˆi  c ˆj  a kˆ & c ˆi  a ˆj  b kˆ are position vectors
of three non-collinear points A, B & C then -
a bc ˆ ˆ ˆ
(A) centroid of triangle ABC is
3

i  jk 
(B) ˆi  ˆj  kˆ is equally inclined to the three vectors

(C) perpendicular from the origin to the plane of triangle ABC meet at centroid
(D) triangle ABC is an equilateral triangle.
12. Identify the statement (s) which is/are incorrect ?
      2
  
(A) a  a  a  b   a  b a  

T
         
(B) If a, b, c are non coplanar vectors and v .a  v . b  v .c  0 then v must be a null vector
       
(C) If a and b lie in a plane normal to the plane containing the vectors c and d then a  b  c  d =0    

N
           
(D) If a, b, c and a ', b ', c ' are reciprocal system of vectors then a . b ' b.c ' c .a '  3

I
  
13. Given a parallelogram OACB. The lengths of the vectors OA , OB & AB are a, b & c respectively.
 

R
The scalar product of the vectors OC & OB is -
a 2  3b 2  c 2 3a 2  b 2  c 2 3a 2  b 2  c 2 a 2  3b 2  c 2
(A) (B) (C) (D)

P
2 2 2 2
           
14. Consider ABC with A  (a) , B  (b) and C = (c) . If b .  a  c  = b.b + a .c ; b – a = 3; c – b = 4, then the


S

angle between the medians AM and BD is -

 1   1   1   1 
(A) – cos–1   (B)  – cos–1   (C) cos–1   (D) cos–1  
 5 13   13 5  5 13   13 5 

E

    
15. If the non zero vectors a & b are perpendicular to each other then the solution of the equation, r  a  b is -

E
1       
 
(A) r  xa    a  b  
1
(B) r  x b    a  b 

(C) r  x a  b  (D) none of these

J
a.a b.b
   
16. a , b , c be three non coplanar vectors and
nd r be any arbitrary vector, then
           
( a × b ) × ( r × c ) + ( b × c ) × ( r × a ) + ( c × a ) × ( r × b ) is equal to-
           
(A) [ a b c ] r (B) 2 [ a b c ] r (C) 3[ a b c ] r (D) none of these
         
17. a and b are mutually perpendicular unit vectors. r is a vector satisfying r . a = 0, r . b = 1 and [ r a b ] = 1,

then r is -
             
(A) a + ( a × b ) (B) b + ( a × b ) (C) a + b ( a × b ) (D) a – b + ( a × b )’

ANSWER KEY
Que. 1 2 3 4 5 6 7 8 9 10
Ans. A,C B,C B,C C C B,C A B A, B, C D
Que. 11 12 13 14 15 16 17
Ans. A,B, C,D A ,C , D D A A B B

Page 108
109
JEE Sprint Mathematics
3D GEOMETRY
SELECT THE CORRECT ALTERNATIVE (ONLY ONE CORRECT ANSWER)

1. The plane XOZ divides the join of (1, –1, 5) and (2, 3, 4) in the ratio  : 1, then  is -
(A) –3 (B) –1/3 (C) 3 (D) 1/3
2. Let ABCD be a tetrahedron such that the edges AB, AC and AD are mutually perpendicular. Let the area of
triangles ABC, ACD and ADB be 3, 4 and 5 sq. units respectively. Then the area of the triangle BCD, is -
5 5
(A) 5 2 (B) 5 (C) (D)
2 2
3. Which one of the following statement is INCORRECT ?
        

T
(A) If n . a = 0, n . b = 0 and n . c = 0 for some non zero vector n , then [a b c] = 0

(B) there exist a vector having direction angles  = 30° and  = 45°

N
(C) locus of point in space for which x = 3 and y = 4 is a line parallel to the z-axis whose distance from the
z-axis is 5

I
  
(D) In a regular tetrahedron OABC where 'O' is the origin, the vector OA + OB + OC is perpendicular to the

R
plane ABC.
4. Consider the following 5 statements
(I) There exists a plane containing the points (1, 2, 3) and (2, 3, 4) and perpendicular to the vector

P

V1 = î + ĵ – k̂

S
(II) There exist no plane containing the point (1, 0, 0); (0, 1, 0); (0, 0, 1) and (1, 1, 1)
   
(III) If a plane with normal vector N is perpendicular to a vector V then N · V = 0
(IV) If two planes are perpendicular then every line in one plane is perpendicular to every line on the other

E
plane
(v) Let P1 and P 2 are two perpendicular planes. If a third plane P 3 is perpendicular to P 1 then it must be

E
either parallel or perpendicular or at an angle of 45° to P2.

J
Choose the correct alternative.
(A) exactly one is false (B) exactly 2 are false (C) exactly 3 are false (D) exactly four are false
 
5. Let L1 be the line r1 = 2 î + ĵ – k̂ + ( î + 2 k̂ ) and let L2 be the line r2 = 3 î + ĵ + µ( î + ĵ – k̂ ).

Let  be the plane which contains the line L1 and is parallel to L2. The distance of the plane  from the origin
is -

(A) 2/7 (B) 1/7 (C) 6 (D) none of these


 
6. The intercept made by the plane r . n = q on the x-axis is -

q ˆi. n  q
(A) (B) (C) ( î. n ) q (D) 

ˆi. n q | n|
7. If from the point P(f, g, h) perpendiculars PL, PM be drawn to yz and zx planes then the equation to the plane
OLM is -
x y z x y z
(A) + + = 0 (B) + – =0
f g h f g h
x y z x y z
(C) – + =0 (D) – + + = 0
f g h f g h

Page 109
110
JEE Sprint Mathematics
8. The line which contains all points (x, y, z) which are of the form (x, y, z) = (2, –2, 5) + (1, –3, 2) intersects the

plane 2x – 3y + 4z = 163 at P and intersects the YZ plane at Q. If the distance PQ is a b , where


a, b  N and a > 3 then (a + b) equals -
(A) 23 (B) 95 (C) 27 (D) none of these
9. A plane passes through the point P(4, 0, 0) and Q(0, 0, 4) and is parallel to the y-axis. The distance of the
plane from the origin is -

(A) 2 (B) 4 (C) 2 (D) 2 2



10. The distance between the parallel planes given by the equations, r . (2 î – 2 ĵ + k̂ ) + 3 = 0 and

r . (4 î – 4 ĵ + 2 k̂ ) + 5 = 0 is -

T
(A) 1/2 (B) 1/3 (C) 1/4 (D) 1/6
–1
11. If the plane 2x – 3y + 6z – 11 = 0 makes an angle sin (k) with x-axis, then k is equal to -

N
3 2 2
(A) (B) (C) (D) 1
7 3

I
2

12. A variable plane forms a tetrahedron of constant volume 64K3 with the coordinate planes and the origin, then
locus of the centroid of the tetrahedron is -

R
3 3 3 2 3 2 2 2 2
(A) x + y + z = 6K (B) xyz = 6k (C) x + y + z = 4K (D) x + y + z = 4k–2
–2 –2 –2

  

P
13. The expression in the vector form for the point r1 of intersection of the plane r · n = d and the perpendicular
  
line r = r0 + t n where t is a parameter given by -

S
   
   d  r0 ·n      r0 ·n  
(A) r1 = r0 +   2  n (B) r1 = r0 –   2  n
 n   n 

E
   
   r ·n  d      r0 ·n  
(C) r1 = r0 –  0  n (D) r1 = r0 +    n
 n   n 

JE
x 1 y  3 z  2
14. The equation of the plane containing the line   and the point (0, 7, –7) is -
3 2 1
(A) x + y + z = 1 (B) x + y + z = 2 (C) x + y + z = 0 (D) none of these

SELECT THE CORRECT ALTERNATIVES (ONE OR MORE THAN ONE CORRECT ANSWERS)
   
1 5 . Consider the plane r .n1  d1 and r .n 2  d 2 , then which of the follwoing are true -
 
(A) they are perpendicular if n1 .n 2  0
 
 n .n 
(B) angle between them is cos 1   1 2 
| n1 || n 2 | 
  d   d
(C) normal form of the equation of plane are r .n 1   1 & r .n 2   2
| n1 | | n2 |
(D) none of these
 ˆ and r  ˆi  2ˆj  kˆ  µ(iˆ  ˆj  3kˆ)
16. The equation of the plane which contains the lines r  ˆi  2ˆj  kˆ  (iˆ  2ˆj  k)

must be -
 ˆ 0
(A) r.(7iˆ  4ˆj  k) (B) 7(x – 1) – 4(y – 2) – (z + 1) = 0
 ˆ 0 
(C) r.(iˆ  2ˆj  k) (D) r.(iˆ  ˆj  3kˆ)  0

Page 110
111
JEE Sprint Mathematics
     
17. The plane containing the lines r  a  ta ' and r  a ' sa -
   
(A) must be parallel to a  a ' (B) must be the perpendicular to a  a '
      
(C) must be [r, a, a '] = 0 (D) (r  a).(a  a ')  0
18. The points A(5, –1, 1), B(7, –4, 7), C(1, –6, 10) and D(–1, –3, 4) are the vertices of a -
(A) parallelogram (B) rectangle (C) rhombus (D) square
19. If P1, P2, P3 denotes the perpendicular distances of the plane 2x – 3y + 4z + 2 = 0 from the parallel planes
2x – 3y + 4z + 6 = 0, 4x – 6y + 8z + 3 = 0 and 2x – 3y + 4z – 6 = 0 respectively, then -
(A) P1 + 8P2 – P3 = 0 (B) P3 = 16P2

(C) 8P2 = P1 (D) P1 + 2P2 + 3P3 = 29

N T
R I
S P
JE E
ANSWER KEY
Que. 1 2 3 4 5 6 7 8 9 10
Ans. D A B D A A B A D D
Que. 11 12 13 14 15 16 17 18 19
Ans. B B A C A,B A,B B,C,D A,C A,B,C,D

Page 111
112
JEE Sprint Mathematics
EXTRA PRACTICE QUESTIONS ON 3D GEOMETRY

SELECT THE CORRECT ALTERNATIVES (ONE OR MORE THAN ONE CORRECT ANSWERS)


1. If the line r = 2 î – ĵ + 3 k̂ + ( î + ĵ + 2 k̂ ) makes angles , , with xy, yz and zx planes respectively then
which one of the following are not possible ?
(A) sin2 + sin2 + sin2 = 2 and cos2 + cos2 + cos2 = 1
(B) tan2 + tan2 + tan2 = 7 and cot2 + cot2 + cot2 = 5/3
(C) sin2 + sin2 + sin2 = 1 and cos2 + cos2 + cos2 = 2
(D) sec2 + sec2 + sec2 = 10 and cosec2 + cosec2 + cosec2 = 14/3

T
2. A plane meets the coordinate axes in A, B, C such that the centroid of the triangle ABC is the point (1, r, r2).
The plane passes through the point (4, –8, 15) if r is equal to -
(A) –3 (B) 3 (C) 5 (D) –5

N
3. Indicate the correct order statements -

I
x 4 y 6 z 6 x 1 y 2 z 3
(A) The lines = = and = = are orthogonal
3 1 1 1 2 2

R
(B) The planes 3x – 2y – 4z = 3 and the plane x – y – z = 3 are orthogonal.
(C) The function f(x) = n(e–2 + ex) is monotonic increasing  x  R.

P
(D) If g is the inverse of the function, f(x) = n(e–2 + ex) then g(x) = n(ex – e–2)
x 1 y 1

S
4. The coordinates of a point on the line = = z at a distance 4 14 from the point (1, –1, 0) are-
2 3

(A) (9, –13, 4) (B) ( 8 14 +1, –12 14 –1, 4 14 )

E
(C) (–7, 11, –4) (D) (– 8 14 +1, 12 14 – 1, – 4 14 )
x 9 y 4 z 5

E
5. Let 6x + 4y – 5z = 4, x – 5y + 2z = 12 and = = be two lines then-
2 1 1

J
 5
(A) the angle between them must be (B) the angle between them must be cos–1
3 6
(C) the plane containing them must be x + y – z = 0 (D) they are non-coplanar

x 1 y 1 z 3 x y z 1
6. The lines = = and = = are -
2 1  1 2 1
(A) coplanar for all  (B) coplanar for  = 19/3

 1 2 4 1 1 
(C) if coplanar then intersect at   ,  ,   (D) intersect at  ,  ,  1 
 5 5 5 2 2 

7. If two pairs of opposite edges of a tetrahedron are perpendicular then -


(A) the third is also perpendicular (B) the third pair is inclined at 60°
(C) the third pair is inclined at 45° (D) (B), (C) are false
8. The equation of a plane bisecting the angle between the plane 2x – y + 2z + 3 = 0 and 3x – 2y + 6z + 8 = 0
is -
(A) 5x – y – 4z – 45 = 0 (B) 5x – y – 4z – 3 = 0
(C) 23x – 13y + 32z + 45 = 0 (D) 23x – 13y + 32z + 5 = 0

Page 112
113
JEE Sprint Mathematics

9. A non-zero vector a is parallel to the line of intersection of the plane determined by the vectors î , î + ĵ and the


plane determined by the vectors î – ĵ , î – k̂ ,. The possible angle between a and î – 2 ĵ + 2 k̂ is -
(A) /3 (B) /4 (C) /6 (D) 3/4
10. If 1, m1, n1 and 2, m2, n2 are DCs of the two lines inclined to each other at an angle , then the DCs of the
bisector of the angle between these lines are-

1   2 m1  m 2 n n2 1   2 m1  m 2 n1  n 2
(A) , , 1 (B) , ,
2 sin  / 2 2 sin  / 2 2 sin  / 2 2 cos  / 2 2 cos  / 2 2 cos  / 2

1   2 m 1  m 2 n1  n 2 1   2 m1  m 2 n1  n 2
(C) , , (D) , ,
2 sin  / 2 2 sin  / 2 2 sin  / 2 2 cos  / 2 2 cos  / 2 2 cos  / 2

T
x 2 y 3 z 6 x 2 y 3 z 6
11. Points that lie on the lines bisecting the angle between the lines   and  
2 3 6 3 6 2

N
are -
(A) (7, 12, 14) (B) (0, –3, 14) (C) (1, 0, 10) (D) (–3, –6, –2)

R I
S P
JE E
ANSWER KEY
Que. 1 2 3 4 5 6 7 8 9 10
Ans. A,B,D B,C C,D A,C A,C B,C A,D B,C B,D B,C
Que. 11
Ans. A,B,C,D

Page 113
114
JEE Sprint Mathematics
DETERMINANTS
SELECT THE CORRECT ALTERNATIVE (ONLY ONE CORRECT ANSWER)

ab bc c a
1. The value of determinant bc ca ab is equal to -
c a ab bc

(A) abc (B) 2abc (C) 0 (D) 4abc

sin 2x cos 2 x cos 4 x


2. If cos 2 x cos 2x sin 2 x = a0 + a1 (sinx) + a2 (sin2x) +.......+ an (sinnx) then the value of a0 is -

T
cos 4 x sin 2 x sin 2x
(A) –1 (B) 1 (C) 0 (D) 2

N
a b c
3. The value of the determinant a b c is equal to -

I
a b c

(A) 0 (B) (a – b)(b – c)(c – a) (C) (a + b)(b + c)(c + a) (D) 4abc

R
sin 2 A cot A 1

P
4. For any ABC, the value of determinant sin 2 B cot B 1 is equal to -
sin 2 C cot C 1

S
(A) 0 (B) 1 (C) sin A sin B sin C (D) sin A + sin B + sin C

p 15 8
2
5. If Dp = p 35 9 , then D1 + D2 + D3 + D4 + D5 is equal to -

E
p3 25 10

(A) 0 (B) 25 (C) 625 (D) none of these

JE
sin(A  B  C ) sin B cos C
6. If A + B + C = , then  sin B 0 tan A is equal to -
cos(A  B )  tan A 0

(A) 0 (B) 2 sin B tan A cos C (C) 1 (D) none of these


x 3x  2 2x  1
7. The number of real values of x satisfying 2x  1 4x 3x  1 = 0 is -
7x  2 17x  6 12x  1
(A) 3 (B) 0 (C) 1 (D) infinite

log a p 1
8. If a, b, c are pth, qth and rth terms of a GP, then log b q 1 is equal to -
log c r 1

(A) 0 (B) 1 (C) log abc (D) pqr

log a n log a n 2 log a n  4


9. If a1, a2,.......an, an+1,...... are in GP and ai > 0 i, then log a n  6 log a n 8 log a n 10 is equal to -
log a n 12 log a n 14 log a n 16

(A) 0 (B) n log an (C) n(n + 1) log an (D) none of these

Page 114
115
JEE Sprint Mathematics
x 2  3x x 1 x3
10. 4 3 2
If px + qx + rx + sx + t = x 1 2x x 3 then t is equal to -
x 3 x4 3x
(A) 33 (B) 0 (C) 21 (D) none
1 log x y log x z
11. For positive numbers x, y and z, the numerical value of the determinant log y x 1 log y z is -
log z x log z y 1
(A) 0 (B) log xyz (C) log(x + y + z) (D) logx logy logz

(a x  a  x ) 2 (a x  a  x ) 2 1
12. If a, b, c > 0 and x, y, z  R, then the determinant (b y  b  y )2 (b y  b  y )2 1 is equal to -

T
(c z  c  z )2 (c z  c  z )2 1
(A) axbycx (B) a–xb–yc–z (C) a2xb2yc 2z (D) zero

N
a 2  b2
c c

I
c
b2  c2
13. For a non-zero real a, b and c a a =  abc, then the values of  is -
a

R
c2  a 2
b b
b

P
(A) –4 (B) 0 (C) 2 (D) 4

S
(1  x)2 (1  x)2 (2  x 2 ) (1  x) 2 2x  1 x 1
14. The equation 2x  1 3x 1  5x  (1  x)2 3x 2x = 0
x 1 2x 2  3x 1  2x 3x  2 2x  3

E
(A) has no real solution (B) has 4 real solutions
(C) has two real and two non-real solutions (D) has infinite number of solutions, real or non-real

E
a b c px qy rz

J
15. Let a determinant is given by A = p q r and suppose determinant A = 6. If B = ax by cz
x y z ap bq cr
then -
(A) det. B = 6 (B) det. B = –6 (C) det. B = 12 (D) det. B = –12
a b c
2 2
16. If a  b  c and a b c2 = 0 then -
bc ca ab
(A) a + b+ c = 0 (B) ab + bc + ca = 0
(C) a2 + b2 + c2 = ab + bc + ca (D) abc = 0
b2 c 2 bc bc
17. If a, b, & c are nonzero real numbers, then c2 a2 ca ca is equal to -
a 2 b2 ab ab
(A) a2 b 2c 2(a + b + c) (B) abc(a + b + c) 2 (C) zero (D) none of these

1 x x 1
18. If f(x) = 2x x(x  1) (x  1)x , then f (100) is equal to - [JEE 98]
3x(x  1) x(x  1)(x  2) (x  1)x (x  1)
(A) 0 (B) 1 (C) 100 (D) –100

Page 115
116
JEE Sprint Mathematics
a b 0
19. The value of the determinant 0 a b is equal to -
b 0 a
(A) a3 – b3 (B) a3 + b3 (C) 0 (D) none of these
20. An equilateral triangle has each of its sides of length 6 cm. If (x1, y1); (x2, y2) & (x3, y3) are its vertices then the
2
x1 y1 1
value of the determinant, x 2 y2 1 is equal to -
x3 y3 1

(A) 192 (B) 243 (C) 486 (D) 972


21. If the system of equations x + 2y + 3z =4, x + py + 2z = 3,  x + 4y + z = 3 has an infinite number of
solutions, then -

T
(A) p = 2, µ = 3 (B) p =2, µ= 4 (C) 3p = 2µ (D) none of these

SELECT THE CORRECT ALTERNATIVES (ONE OR MORE THAN ONE CORRECT ANSWERS)

N
a a x

I
22. If m m m  0 , then x may be equal to -
b x b

R
(A) a (B) b (C) a + b (D) m

sin 2x e x sin x  x cos x sin x  x 2 cos x

P
23. If D(x)  cos x  sin x ex  x 1  x2 , then the value of |n cos (Dx)| will be -
x
e cos x e2 x e x

S
(A) independent of x (B) dependent on x (C) 0 (D) non-existent
  
The value of the determinant  x n is

E
24.
  x
(A) independent of  (B) independent of n (C) (x – )(x – ) (D)(x – )(x – n)

E
25. If the system of linear equations x + ay + az = 0, x + by + bz = 0, x + cy + cz = 0 has a non-zero solution

J
then
(A) System has always non-trivial solutions.
(B) System is consistent only when a = b = c
(C) If a  b  c then x = 0, y = t, z=–t  t  R
(D) If a = b = c then y = t 1, z = t2, x = –a(t1+ t2)  t 1,t 2  R
26. If the system of equations x + y – 3 = 0, (1 + K ) x + (2 + K ) y – 8 = 0 & x – (1 + K) y + (2 + K) = 0 is consistent
then the value of K may be -
3 5
(A) 1 (B) (C) – (D) 2
5 3

ANSWER KEY
Que. 1 2 3 4 5 6 7 8 9 10
Ans. C A D A D A D A A C
Que. 11 12 13 14 15 16 17 18 19 20
Ans. A D D D C A C A B D
Que. 21 22 23 24 25 26
Ans. D A,B A,C B,C A ,C , D A,C

Page 116
117
JEE Sprint Mathematics
EXTRA PRACTICE QUESTIONS ON DETERMINANTS
SELECT THE CORRECT ALTERNATIVES (ONE OR MORE THAN ONE CORRECT ANSWERS)

1. Which of the following determinant(s) vanish(es) ?

1 1
1 ab 
1 bc bc(b  c) a b
1 ca ca(c  a) 1 1
(A) (B) 1 bc 
1 ab ab(a  b) b c
1 1
1 ca 
c a

0 a b a c log x xyz log x y log x z

T
ba 0 b c log y xyz 1 log y z
(C) (D)
c a c b 0 log z xyz log z y 1

N
mx mx  p mx  p

I
2. If f'(x) = n np n p , then y = f(x) represents -
mx  2n mx  2n  p mx  2n  p

R
(A) a straight line parallel to x–axis (B) a straight line parallel to y–axis
(C) parabola (D) a straight line with negative slope

P
a2 a 2  (b  c)2 bc
3. The determinant b2 b 2  (c  a)2 ca is divisible by -

S
c2 c 2  (a  b)2 ab

(A) a + b + c (B) (a + b) (b + c) (c + a) (C) a2 + b2 + c2 (D) (a – b)(b – c) (c – a)

E
a b a  b
4. The determinant b c b  c is equal to zero, if -

E
a  b b  c 0

J
(A) a, b, c are in AP (B) a, b, c are in GP
(C)  is a root of the equation ax2+bx+c=0 (D) (x–  ) is a factor of ax2 + 2bx + c

1  sin 2 x cos 2 x 4 sin 2x


5. Let f(x) = sin 2 x 1  cos 2 x 4 sin 2x , then the maximum value of f(x) =
sin 2 x cos 2 x 1  4 sin 2x

(A) 2 (B) 4 (C) 6 (D) 8

1 a a2
6. The parameter on which the value of the determinant cos(p  d)x cos px cos(p  d)x does not depend upon
sin(p  d)x sin px sin(p  d)x
is-
(A) a (B) p (C) d (D) x

1  a 2 x (1  b 2 )x (1  c 2 )x
2 2 2
7. If a2 + b2 + c2 = -2 and f(x)  (1  a )x 1  b x (1  c )x , then f(x) is a polynomial of degree-
(1  a 2 )x (1  b 2 )x 1  c 2 x

(A) 2 (B) 3 (C) 0 (D) 1

Page 117
118
JEE Sprint Mathematics

p q px  qy
8. 2
Given that q –pr < 0, p > 0, then the value of q r qx  ry is-
px  qy qx  ry 0

(A) zero (B) positive (C) negative (D) q2 + pr


  
9. The value of  lying between  & and 0  A  and satisfying the equation
4 2 2
1  sin 2 A cos 2 A 2 sin 4 
sin 2 A 1  cos 2 A 2 sin 4  = 0 are -
2 2
sin A cos A 1  2 sin 4 

T
  3    3
(A) A = ,  (B) A =   (C) A  ,  (D) A  , 
4 8 8 5 8 6 8
10. The set of equations x – y + 3z = 2, 2x – y + z = 4, x – 2y +  z = 3 has -

N
(A) unique solution only for  = 0 (B) unique solution for   8

I
(C) infinite number of solutions of  = 8 (D) no solution for  = 8

P R
E S
JE
ANSWER KEY
Que. 1 2 3 4 5 6 7 8 9 10
Ans. A,B, C,D A A ,C , D B,D C B A C A,B, C,D B,D

Page 118
119
JEE Sprint Mathematics
MATRICES
SELECT THE CORRECT ALTERNATIVE (ONLY ONE CORRECT ANSWER)

1 5   2 5 
1. If A – 2B =   and 2A – 3B =   , then matrix B is equal to -
3 7   0 7

 4 5   0 6 2  1  6 1 
(A)  (B)  (C)  (D) 
 6 7  
 3 7  3 2 

0 1 

 cos  sin  
2. If A  =   , then A A  is equal to -
  sin  cos  

T
(A) A + (B) A  (C) A (D) none of these
3. If number of elements in a matrix is 60 then how many different order of matrix are possible -

N
(A) 12 (B) 6 (C) 24 (D) none of these

I
4. Matrix A has x rows and x + 5 columns. Matrix B has y rows and 11 – y columns. Both AB and BA exist, then -
(A) x = 3, y = 4 (B) x = 4, y = 3 (C) x = 3, y = 8 (D) x = 8, y = 3

R
5. If A2 = A, then(I + A)4 is equal to -
(A) I + A (B) I + 4A (C) I + 15A (D) none of these

P
1 1  1 2  1 3  1 n  1 378 
6. If the product of n matrices   0 1  0 1  ...... 0 1  is equal to the matrix 0 then the value
 0 1       1 

S
of n is equal to -
(A) 26 (B) 27 (C) 377 (D) 378
 0 1 2

E
7. If A =   and (aI2 +bA) = A , then -
  1 0 
(A) a = b = 2 (B) a = b = 1/ 2 (C) a = b = 3 (D) a = b = 1/ 3

E
8. If A is a skew symmetric matrix such that A TA = I, then A 4n–1  n  N  is equal to -

J
(A) – AT (B) I (C) – I (D) AT
9. If A A T = I and det(A) = 1, then -
(A) Every element of A is equal to it's co-factor.
(B) Every element of A and it's co-factor are additive inverse of each other.
(C) Every element of A and it's co-factor are multiplicative inverse of each other.
(D) None of these
10. Which of the following is an orthogonal matrix -

 6 / 7 2 / 7 3 / 7  6 / 7 2 / 7 3/7 
  2 / 7 3 / 7 6 / 7 
(A) 2 / 7 3 / 7 6 / 7  (B)  
3 / 7 6 / 7 2 / 7  3 / 7 6 / 7 2 / 7 

 6 / 7 2 / 7 3 / 7   6 / 7 2 / 7 3 / 7 
  2 / 7 3 / 7 
(C)  2 / 7 3/7 6 / 7  (D)  2 / 7
 3 / 7 6 / 7 2 / 7   6 / 7 2 / 7 3 / 7 

11. If A is an orthogonal matrix & | A | = –1, then AT is equal to -


(A) –A (B) A (C) –(adj A) (D) (adj A)

Page 119
120
JEE Sprint Mathematics
1 1 1  4 2 2
12. Let A = 2 1 3  and 10B =  5
 0   . If B is the inverse of matrix A, then  is -

1 1 1   1 2 3 
(A) –2 (B) –1 (C) 2 (D) 5
3 2  3 1 
13. Let the matrix A and B be defined as A =   and B    then the value of Det.(2A 9 B –1 ), is -
2 1  7 3 
(A) 2 (B) 1 (C) –1 (D) –2

2 1   3 2  1 0 
14. If  A  
3  0 1 
, then matrix A equals -
7 4   5

 7 5  2 1   7 1  5 3

T
(A)  (B)  (C)  (D) 
  11  8  
5 3 

34 5 

13 8 

0 5  2 16
15. If A =   and (x) = 1 + x + x + ...... + x , then (A) =

N
0 0 

I
1 5  1 5  0 5 
(A) 0 (B)   (C)   (D)  
0 1  0 0  1 1 

R
1 2  2
16. If M =   and M –M – I2 = O , then  equals -
2 3 
(A) –2 (B) 2 (C) –4 (D) 4

P
1 2   1 4  4 8 
17. If A =  , B =   and ABC = 3 7  , then C equals -

S
3 0   2 3   
1 72 32  1  54 110  1  54 110  1  72 32 
(A) (B) (C) (D)
66 57 29  
66  3 11  
66  3 11  66  57 29 

E
18. If P is a two-rowed matrix satisfying PT = P–1, then P can be -
 cos   sin    cos  sin     cos  sin  
(A)   (B)   (C)   (D) none of these

E
  sin  cos     sin  cos    sin   cos  

J
 a 0 0 
 
19. If A   0 a 0  , then | A | | Adj A | is equal to -
 0 0 a 
(A) a25 (B) a27 (C) a81 (D) none of these
2 3 
20. If A    , then 19A–1 is equal to -
5 2 

1
(A) AT (B) 2A (C) A (D) A
2

SELECT THE CORRECT ALTERNATIVES (ONE OR MORE THAN ONE CORRECT ANSWERS)
21. If A and B are square matrices of same order, then which of the following is correct -
(A) A + B = B + A (B) A + B = A – B
(C) A – B = B – A (D) AB = BA
22. A square matrix can always be expressed as a
(A) sum of a symmetric matrix and skew symmetric matrix of the same order
(B) difference of a symmetric matrix and skew symmetric matrix of the same order
(C) skew symmetric matrix
(D) symmetric matrix

Page 120
121
JEE Sprint Mathematics
23. Choose the correct answer :
(A) every scalar matrix is an identity matrix.
(B) every identity matrix is a scalar matrix
(C) transpose of transpose of a matrix gives the matrix itself.
(D) for every square matrix A there exists another matrix B such that AB = I = BA.
24. Let aij denote the element of the ith row and jthcolumn in a 3 × 3 matrix and let aij = –aji for every i and j then this
matrix is an -
(A) orthogonal matrix (B) singular matrix
(C) matrix whose principal diagonal elements are all zero (D) skew symmetric matrix
25. Let A be an invertible matrix then which of the following is/are true :

T
(A) |A –1 | = |A| –1 (B) (A2)–1 = (A–1)2 (C) (AT)–1 = (A–1)T (D) none of these

1 9 7 
 n 
26. If A  i  8  , where i  1 and   is complex cube root of unity, then tr(A) will be-

N
1 6 2 n 

I
(A) 1, if n = 3k, k  N (B) 3, if n = 3k, k  N (C) 0, if n  3k, k  N (D) –1, if n  3k, n  N
27. If A is a square matrix, then -

R
(A) AAT is symmetric (B) AAT is skew-symmetric (C) ATA is symmetric (D) ATA is skew symmetric.

P
a b
28. If A =  satisfies the equation x 2 + k = 0, then -
c d 

S
(A) a + d = 0 (B) k = –|A| (C) k = a 2 + b 2 + c 2 + d 2 (D) k = |A|
29. If A and B are invertible matrices, which one of the following statement is/are correct -

E
(A) Adj(A) = |A|A –1 (B) det(A –1 ) =|det(A)| –1

(C) (A + B) –1 = B –1 + A –1 (D) (AB) –1 = B –1 A –1

E
a b  a  b  

J
 
30. Matrix  b c  b   c   is non invertible if -
2 1 0 
(A)  = 1/2 (B) a, b, c are in A.P. (C) a, b, c are in G.P. (D) a, b, c are in H.P.

ANSWER KEY
Que. 1 2 3 4 5 6 7 8 9 10
Ans. A A A C C B B D A A
Que. 11 12 13 14 15 16 17 18 19 20
Ans. C D D A B D B B D D
Que. 21 22 23 24 25 26 27 28 29 30
Ans. A A,B B,C B,C,D A, B, C B,C A,C A,D A,B,D A,C

Page 121
122
JEE Sprint Mathematics
EXTRA PRACTICE QUESTIONS ON MATRICES
SELECT THE CORRECT ALTERNATIVES (ONE OR MORE THAN ONE CORRECT ANSWERS)

1. If A and B are square matrices of same order and AAT = I then (ATBA)10 is equal to -
(A) AB10AT (B) ATB10A (C) A 10B 10 (A T) 10 (D) 10ATBA
2. If A is a invertible idempotent matrix of order n, then adj A is equal to -
(A) (adj A)2 (B)  (C) A–1 (D) none of these
x 3 2 
 
3. Matrix A = 1 y 4  , if xyz = 60 and 8x + 4y + 3z = 20, then A (adj A) is equal to -
2 2 z 

T
64 0 0  88 0 0  68 0 0  34 0 0 
       0 34 0 
(A)  0 64 0  (B)  0 88 0  (C)  0 68 0  (D)  
 0 0 64   0 0 88   0 0 68   0 0 34 

N
2 1  3 4  3 –4 

I
4. Let three matrices A =  4 1  ; B = 2 3  and C =  –2 3 
then
    
 ABC   A (BC)2   A (BC) 3 
t r (A )  t r    tr    tr    ......   

R
 2   4   8 
(A) 6 (B) 9 (C) 12 (D) none of these

P
5. Let A, B, C, D be (not necessarily square) real matrices such that AT = BCD ; BT = CDA; CT = DAB and
DT = ABC for the matrix S = ABCD, then which of the following is/are true

S
(A) S3 =S (B) S2 = S4 (C) S = S2 (D) none of these
 1 tan x 
6. If A =   tan x 1 
then let us define a function f(x) = det (ATA–1) then which of the following can be the value

E
of f(f(f(f...........f(x))))
  (n  2)
n times

E
(A) fn(x) (B) 1 (C) fn–1(x) (D) nf(x)

J
50
1 2r  1  1 2r  1 
7. For a matrix A = 0 1 
, the value of  0 1 
is equal to -
 r 1 
1 100  1 4950  1 5050  1 2500 
(A) 0 1 
(B) 0 1 
(C) 0 1 
(D) 0 1 
   
8. If A and B are two invertible matrices of the same order, then adj (AB) is equal to -
(A) adj (B) adj (A) (B) |B||A| B –1 A –1 (C) |B||A| A –1 B –1 (D) |A||B|(AB) –1
0 1 2  1 / 2 1 / 2 1 / 2 
9. If A = 1 2 3  , A 1   4 3 c  , then -
3 a 1  5 / 2 3 / 2 1 / 2 
1 1 1
(A) a = 1, c = –1 (C) a = –1, c = 1
(B) a = 2, c = – (D) a = , c =
2 2 2
10. 3 3 2 2
If A and B are different matrices satisfying A = B and A B = B A, then which of the following is/are
incorrect-
(A) det (A 2 + B 2 ) must be zero
(B) det (A – B) must be zero
(C) det (A 2 + B 2 ) as well as det (A – B) must be zero
(D) At least one of det (A 2 + B 2 ) or det (A – B) must be zero

Page 122
123
JEE Sprint Mathematics
0 0 1 
11. If A  0 1 0  , then-
1 0 0 

 0 0 1 
(A) AdjA is zero matrix (B) Adj A   0 1 0 
 1 0 0 
(C) A–1 = A 2
(D) A = I

12. If A and B are square matrices of the same order such that A2 = A, B2 = B, AB = BA, then which one of the
following may be true-

(A) A(B)2 = O (B) (A + B)2 = A + B (C) (A – B)2 = A – B (D) none of these

T
13. If B is an idempotent matrix and A = I – B, then-

(A) A2 = A (B) A2 = I (C) AB = O (D) BA = O

N
a 11 a 12 a 13

I
14. Let  0 = a 21 a 22 a 23 (where  0  0) and let  1 denote the determinant formed by the cofactors of
a 31 a 32 a 33
elements of 0 and 2 denote the determinant formed by the cofactor at 1 and so on n denotes the determinant

R
formed by the cofactors at n–1 then the determinant value of n is -
n 2

P
2n 2
(A)  0 (B)  20 (C)  0n (D)  0

E S
JE
ANSWER KEY
Que. 1 2 3 4 5 6 7 8 9 10
Ans. B A, B, C C A A,B A, B, C D A,B,D A A, B, C
Que. 11 12 13 14
Ans. B,C,D A, B, C A ,C , D B

Page 123
124
JEE Sprint Mathematics
STRAIGHT LINES

SELECT THE CORRECT ALTERNATIVE (ONLY ONE CORRECT ANSWER)


1. If (3, –4) and (–6, 5) are the extremities of a diagonal of a parallelogram and (2, 1) is its third vertex, then its
fourth vertex is -
(A) (–1, 0) (B) (–1, 1) (C) (0, –1) (D) (–5, 0)
2. The ratio in which the line joining the points (3, –4) and (–5, 6) is divided by x-axis -
(A) 2 : 3 (B) 6 : 4 (C) 3 : 2 (D) none of these
3. The circumcentre of the triangle with vertices (0, 0), (3, 0) and (0, 4) is -
(A) (1, 1) (B) (2, 3/2) (C) (3/2, 2) (D) none of these
4. The mid points of the sides of a triangle are (5, 0), (5, 12) and (0, 12), then orthocentre of this triangle is -

 13 

T
(A) (0, 0) (B) (0, 24) (C) (10, 0) (D)  ,8 
 3 
5. Area of a triangle whose vertices are (a cos , b sin), (–a sin , b cos ) and (–a cos , – b sin ) is -

N
1
(A) a b sin  cos  (B) a cos  sin  (C) ab (D) ab
2

I
6. The point A divides the join of the points (–5,1) and (3,5) in the ratio k : 1 and coordinates of points B and C are
(1,5) and (7,–2) respectively. If the area of ABC be 2 units, then k equals -

R
(A) 7,9 (B) 6,7 (C) 7,31/9 (D) 9,31/9
7. If A(cos , sin), B (sin, – cos), C (1,2) are the vertices of a ABC, then as  varies, the locus of its centroid is -
(A) x2 + y2 – 2x – 4y + 3 = 0 (B) x2 + y2 – 2x – 4y + 1 = 0

P
2 2
(C) 3(x + y ) – 2x – 4y + 1 = 0 (D) none of these
8. The points with the co-ordinates (2a, 3a), (3b, 2b) & (c, c) are collinear-

S
(A) for no value of a, b, c (B) for all values of a, b, c
c 2
(C) if a, , b are in H.P.. (D) if a, c, b are in H.P..
5 5

E
9. A stick of length 10 units rests against the floor and a wall of a room. If the stick begins to slide on the
floor then the locus of its middle point is -

E
(A) x 2 + y 2 = 2.5 (B) x 2 + y 2 = 25 (C) x 2 + y 2 = 100 (D) none
3

J
10. The equation of the line cutting an intercept of 3 units on negative y-axis and inclined at an angle tan–1 to the
5
x-axis is -
(A) 5y – 3x + 15 = 0 (B) 5y – 3x = 15 (C) 3y – 5x + 15 = 0 (D) none of these
11. The equation of a straight line which passes through the point (–3, 5) such that the portion of it between the axes
is divided by the point in the ratio 5 : 3, internally (reckoning from x-axis) will be -
(A) x + y – 2 = 0 (B) 2x + y + 1 = 0 (C) x + 2y – 7 = 0 (D) x – y + 8 = 0
 8
12. The points  0,  , (1, 3) and (82, 30) are vertices of-
 3
(A) an obtuse angled triangle (B) an acute angled triangle
(C) a right angled triangle (D) an isosceles triangle
13. The straight lines x + y = 0, 3x + y – 4 = 0, x + 3y – 4 = 0 form a triangle which is-
(A) isosceles (B) equilateral (C) right angled (D) none of these
14. The co-ordinates of the vertices P, Q, R & S of square PQRS inscribed in the triangle ABC with vertices
A  (0, 0), B (3, 0) & C  (2, 1) given that two of its vertices P, Q are on the side AB are respectively :
1  3  3 1 1 1 1  3  3 1 1 1
(A)  , 0  ,  , 0  ,  , &  , (B)  , 0  ,  , 0  ,  , &  ,
4 8 8 8  4 8  2 4 4 4  2 4 
3   3 1  1 3  9  9 3 3 3
(C) (1, 0),  , 0  ,  , &  1, (D)  , 0  ,  , 0  ,  , &  ,
2 2 2  2  2 4 4 4  2 4 

Page 124
125
JEE Sprint Mathematics
15. The equation of perpendicular bisector of the line segment joining the points (1, 2) and (–2, 0) is -
(A) 5x + 2y = 1 (B) 4x + 6y = 1 (C) 6x + 4y = 1 (D) none of these
16. The number of possible straight lines, passing through (2, 3) and forming a triangle with coordinate axes,
whose area is 12 sq. units, is -
(A) one (B) two (C) three (D) four
17. Points A & B are in the first quadrant ; point ‘O’ is the origin. If the slope of OA is 1, slope of OB is 7 and
OA = OB, then the slope of AB is -
(A) –1/5 (B) –1/4 (C) –1/3 (D) –1/2
18. A line is perpendicular to 3x + y = 3 and passes through a point (2, 2). Its y intercept is -
(A) 2/3 (B) 1/3 (C) 1 (D) 4/3

T
19. The equation of the line passing through the point (c, d) and parallel to the line ax + by + c = 0 is -
(A) a(x + c) + b(y + d) = 0 (B) a(x + c) – b(y + d) = 0 (C) a(x – c) + b(y – d) = 0 (D) none of these
20. The position of the point (8,–9) with respect to the lines 2x + 3y – 4 = 0 and 6x + 9y + 8 = 0 is -

N
(A) point lies on the same side of the lines (B) point lies on one of the lines

I
(C) point lies on the different sides of the line (D) point lies between the lines
21. If origin and (3, 2) are contained in the same angle of the lines 2x + y – a = 0, x – 3y + a = 0, then 'a' must lie

R
in the interval -
(A) (–, 0)  (8, ) (B) (–, 0) (3, ) (C) (0, 3) (D) (3, 8)

P
22. The line 3x + 2y = 6 will divide the quadrilateral formed by the lines x + y = 5, y – 2x = 8, 3y + 2x = 0 &
4y – x = 0 in -

S
(A) two quadrilaterals (B) one pentagon and one triangle
(C) two triangles (D) one triangle and one quadrilateral
23. If the point (a, 2) lies between the lines x – y – 1 = 0 and 2(x – y) – 5 = 0, then the set of values of a is -

E
(A) (–, 3)  (9/2, ) (B) (3, 9/2) (C) (–, 3) (D) (9/2, )
24. A(x1, y1) , B(x2, y2) and C(x3, y3) are three non-collinear points in cartesian plane. Number of parallelograms

E
that can be drawn with these three points as vertices is -

J
(A) one (B) two (C) three (D) four
25. If P = (1,0) ; Q = (–1,0) & R = (2,0) are three given points, then the locus of the points S satisfying the relation,
SQ2 + SR2 = 2 SP2 is -
(A) A straight line parallel to x-axis (B) A circle passing through the origin
(C) A circle with the centre at the origin (D) A straight line parallel to y-axis
26. The area of triangle formed by the lines x + y – 3 = 0, x – 3y + 9 = 0 and 3x – 2y + 1 = 0 is -

16 10
(A) sq. units (B) sq. units (C) 4 sq. units (D) 9 sq. units
7 7

27. The co-ordinates of foot of the perpendicular drawn on line 3x – 4y – 5 = 0 from the point (0, 5) is -
(A) (1, 3) (B) (2, 3) (C) (3, 2) (D) (3, 1)
28. If the sum of the distances of a point from two perpendicular lines in a plane is 1, then its locus is-

(A) square (B) circle (C) straight line (D) two intersecting lines
29. Distance of the point (2, 5) from the line 3x + y + 4 = 0 measured parallel to the line
3x – 4y + 8 = 0 is -
(A) 15/2 (B) 9/2 (C) 5 (D) none

Page 125
126
JEE Sprint Mathematics
30. Three vertices of triangle ABC are A(–1, 11), B(–9, –8) and C(15, –2). The equation of angle bisector of angle
A is -
(A) 4x – y = 7 (B) 4x + y = 7 (C) x + 4y = 7 (D) x – 4y = 7
31. Given the four lines with the equations
x + 2y – 3 = 0, 3x + 4y – 7 = 0
2x + 3y – 4 = 0, 4x + 5y – 6 = 0
then
(A) they are all concurrent (B) they are the sides of a quadrilateral
(C) only three lines are concurrent (D) none of the above
32. The co-ordinates of the point of reflection of the origin (0, 0) in the line 4x – 2y – 5 = 0 is -
4 2
(A) (1, –2) (B) (2, –1) (C)  ,   (D) (2, 5)

T
5 5
33. If the axes are rotated through an angle of 30° in the anti-clockwise direction, the coordinates of point
(4,–2 3 ) with respect to new axes are-

N
(A) (2, 3 ) (B) ( 3,  5) (C) (2, 3) (D) ( 3, 2)

I
34. If one diagonal of a square is along the line x = 2y and one of its vertex is (3, 0), then its sides through this vertex
are given by the equations -

R
(A) y – 3x + 9 = 0, x – 3y – 3 = 0 (B) y – 3x + 9 = 0, x – 3y – 3 = 0
(C) y + 3x – 9 = 0, x + 3y – 3 = 0 (D) y – 3x + 9 = 0, x + 3y – 3 = 0
35. The line (p + 2q)x + (p – 3q)y = p – q for different values of p and q passes through a fixed point whose co-

P
ordinates are -

FG 3 , 5 IJ FG 2 , 2 IJ FG 3 , 3 IJ FG 2 , 3 IJ

S
(A) H2 2K (B) H5 5K (C) H5 5K (D) H5 5K
36. The equation 2x2 + 4xy – py2 + 4x + qy + 1 = 0 will represent two mutually perpendicular straight lines, if -

E
(A) p=1 and q = 2 or 6 (B) p = – 2 and q = – 2 or 8
(C) p = 2 and q = 0 or 8 (D) p = 2 and q = 0 or 6

E
37. Equation of the pair of straight lines through origin and perpendicular to the pair of straight lines

J
5x2 – 7xy – 3y2 = 0 is -
(A) 3x2 – 7xy – 5y2 = 0 (B) 3x2 + 7xy + 5y2 = 0 (C) 3x2 – 7xy + 5y2 = 0 (D) 3x2 + 7xy – 5y2 = 0
38. If the straight lines joining the origin and the points of intersection of the curve
5x2 + 12xy – 6y2 + 4x – 2y + 3 = 0 and x + ky – 1 = 0 are equally inclined to the co-ordinate axis, then the
value of k -
(A) is equal to 1 (B) is equal to –1 (C) is equal to 2 (D) does not exist in the set of real numbers

SELECT THE CORRECT ALTERNATIVES (ONE OR MORE THAN ONE CORRECT ANSWERS)

39. Coordinates of a point which is at 3 units distance from the point (1, –3) on the line 2x + 3y + 7 = 0 is/are -

 9 6   9 6   9 6   9 6 
(A)  1  ,3   (B)  1  , 3   (C)  1  , 3   (D)  1  ,3  
 13 13   13 13   13 13   13 13 

40. The angle between the lines y – x + 5 = 0 and 3 x – y + 7= 0 is/are -


(A) 15° (B) 60° (C) 165° (D) 75°
41. If line y – x + 2 = 0 is shifted parallel to itself towards the x-axis by a perpendicular distance of 3 2 units, then
the equation of the new line is may be -

(A) y = x + 4 (B) y = x + 1 (C) y = x – (2 + 3 2 ) (D) y = x – 8

Page 126
127
JEE Sprint Mathematics
42. Three lines px + qy + r = 0, qx + ry + p = 0 and rx + py + q = 0 are concurrent if -

(A) p + q + r = 0 (B) p 2 + q 2 + r 2 = pr + qr + pq

(C) p 3 + q 3 + r 3 = 3pqr (D) none of these

43. All points lying inside the triangle formed by the points (1, 3), (5, 0) and (–1, 2) satisfy -

(A) 3x + 2y  0 (B) 2x + y – 13  0 (C) 2x – 3y – 12  0 (D) –2x + y  0

44. The diagonals of a square are along the pair of lines whose equation is 2x2 – 3xy – 2y2 = 0. If (2, 1) is a vertex
of the square, then the vertex of the square adjacent to it may be -

(A) (1, 4) (B) (–1, –4) (C) (–1, 2) (D) (1, –2)

45. Equation of two equal sides of a triangle are the lines 7x + 3y – 20 = 0 and 3x + 7y – 20 = 0 and the third side

T
passes through the point (–3, 3), then the equation of the third side can be -
(A) x + y = 0 (B) x – y + 6 = 0 (C) x + 3 = 0 (D) y = 3

R I N
S P
JE E
ANSWER KEY
Que. 1 2 3 4 5 6 7 8 9 10
Ans. D A C A D C C D B A
Que. 11 12 13 14 15 16 17 18 19 20
Ans. D D A D C C D D C A
Que. 21 22 23 24 25 26 27 28 29 30
Ans. A A B C D B D A C B
Que. 31 32 33 34 35 36 37 38 39 40
Ans. C B B D D C A B B,C A,C
Que. 41 42 43 44 45
Ans. A,D A,B,C A,C C,D A,B

Page 127
128
JEE Sprint Mathematics
EXTRA PRACTICE QUESTIONS ON STRAIGHT LINES
SELECT THE CORRECT ALTERNATIVES (ONE OR MORE THAN ONE CORRECT ANSWERS)
1. The co-ordinates of a point P on the line 2x – y + 5 = 0 such that |PA – PB| is maximum where A is (4, – 2)
and B is (2, – 4) will be -
(A) (11, 27) (B) (–11, – 17) (C) (–11, 17) (D) (0, 5)
2. The line x + y = p meets the axis of x and y at A and B respectively. A triangle APQ is inscribed in the
triangle OAB, O being the origin, with right angle at Q. P and Q lie respectively on OB and AB. If the area
AQ
of the triangle APQ is 3/8th of the area of the triangle OAB, then is equal to -
BQ
(A) 2 (B) 2/3 (C) 1/3 (D) 3
3. Lines, Li : x + 3y = 2, and L2 : ax + by = 1, meet at P and enclose an angle of 45° between them. Line L3 :

T
y = 3x, also passes through P then -
(A) a2 + b2 = 1 (B) a2 + b2 = 2 (C) a2 + b2 = 3 (D) a2 + b2 = 4
4. A triangle is formed by the lines 2x – 3y – 6 = 0 ; 3x – y + 3 = 0 and 3x + 4y – 12 = 0. If the points P(,0) and

N
Q (0,) always lie on or inside the ABC , then range of    -

I
(A)   [ 1, 2] &   [ 2, 3] (B)   [ 1, 3] &   [ 2, 4]

R
(C)   [ 2, 4] &   [ 3, 4] (D)   [ 1, 3] &   [ 2, 3]
5. The line x + 3y – 2 = 0 bisects the angle between a pair of straight lines of which one has equation
x –7y + 5 = 0. The equation of the other line is -

P
(A) 3x + 3y – 1 = 0 (B) x – 3y + 2 = 0 (C) 5x + 5y – 3 = 0 (D) none
6. A ray of light passing through the point A (1, 2) is reflected at a point B on the x-axis line mirror and then passes

S
through (5, 3). Then the equation of AB is -
(A) 5x + 4y = 13 (B) 5x – 4y = – 3 (C) 4x + 5y = 14 (D) 4x – 5y = – 6
7. Let the algebraic sum of the perpendicular distances from the points (3, 0), (0, 3) & (2, 2) to a variable

E
straight line be zero, then the line passes through a fixed point whose co-ordinates are-
3 3 5 5
(A) (3, 2) (B) (2, 3) (C)  ,  (D)  , 
5 5 3 3

E
8. 2 2
The image of the pair of lines respresented by ax + 2h xy + by = 0 by the line mirror y = 0 is :

J
(A) ax2 – 2hxy + by2 = 0 (B) bx2 – 2h xy + ay2 = 0
(C) bx2 + 2h xy + ay2 =0 (D) ax2 – 2h xy – by2 = 0
9. The pair of straight lines x2 – 4xy + y2 = 0 together with the line x + y + 4 6 = 0 form a triangle which is :
(A) right angled but not isosscles (B) right isosceles
(C) scalene (D) equilateral
10. Let A  (3, 2) and B  (5, 1). ABP is an equilateral triangle is constructed on the side of AB remote from
the origin then the orthocentre of triangle ABP is -
 1 3   1 3   1 3 1   1 3 1 
(A)  4  3,  3 
 (B)  4  3,  3 
 (C)  4 – 3, – 3
 (D)  4  3,  3

2 2 2 2 6 2 3 6 2 3
11. The line PQ whose equation is x – y = 2 cuts the x axis at P and Q is (4,2). The line PQ is rotated about P through
45° in the anticlockwise direction. The equation of the line PQ in the new position is -
(A) y = – 2 (B) y = 2 (C) x = 2 (D) x = –2
12. 2 2
Distance between two lines respresented by the line pair, x – 4xy + 4y + x – 2y – 6 = 0 is -
1
(A) (B) 5 (C) 2 5 (D) none
5
13. The circumcentre of the triangle formed by the lines, xy + 2x + 2y + 4 = 0 and x + y + 2 = 0 is -
(A) (–1, –1) (B) (–2, –2) (C) (0, 0) (D) (–1, –2)

Page 128
129
JEE Sprint Mathematics
14. Area of the rhombus bounded by the four lines, ax  by  c = 0 is -
c2 2c 2 4c 2 ab
(A) (B) (C) (D) 2
2ab ab ab 4c
15. If the lines ax + y + 1 = 0, x + by + 1 = 0 & x + y + c = 0 where a, b & c are distinct real numbers different
1 1 1
from 1 are concurrent, then the value of   
1a 1b 1c
(A) 4 (B) 3 (C) 2 (D) 1
16. If one vertex of an equilateral triangle of side ‘a’ lies at the origin and the other lies on the line x  3y  0 ,
then the co-ordinates of the third vertex are -
 3 a a  3 a a
(A) (0, a) (B)  2 ,  2  (C) (0, –a) (D)   2 , 2 
   

T
17. The area enclosed by 2 |x| + 3|y|  6 is -

(A) 3 sq. units (B) 4 sq. units (C) 12 sq. units (D) 24 sq. units

N
18. The point (4, 1) undergoes the following three transformations successively -

I
(i) Reflection about the line y = x
(ii) Translation through a distance 2 units along the positive directions of x-axis.
(iii) Rotation through an angle /4 about the origin.

R
The final position of the point is given by the coordinates :
 7 1   7 1   1 7 
(A)  ,  (B)  , (C)   , (D) none of these

P
  
2 2 2 2 2 2
19. If the equation ax2 – 6xy + y2 + bx + cx + d = 0 represents a pair of lines whose slopes are m and m2, then

S
value(s) of a is/are -
(A) a = –8 (B) a = 8 (C) a = 27 (D) a = –27
20. Given the family of lines, a(3x + 4y + 6) + b(x + y + 2) = 0. The line of the family situated at the greatest

E
distance from the point P (2,3) has equation -
(A) 4x + 3y + 8 = 0 (B) 5x + 3y + 10 = 0 (C) 15x + 8y + 30 = 0 (D) none

E
21. If the vertices P, Q, R of a triangle PQR are rational points, which of the following points of the triangle PQR

J
is/are always rational point (s) ?

(A) centriod (B) incentre (C) circumcentre (D) orthocentre

22. Let PQR be a right angled isosceles triangle, right angled at P (2, 1). If the equation of the line QR is 2x + y = 3,
then the equation representing the pair of lines PQ and PR is -

(A) 3x2 – 3y2 + 8xy + 20x + 10y + 25 = 0 (B) 3x2 – 3y2 + 8xy – 20x – 10y + 25 = 0

(C) 3x2 – 3y2 + 8xy + 10x + 15y + 20 = 0 (D) 3x2 – 3y2 – 8xy – 10x – 15y – 20 = 0

ANSWER KEY
Que. 1 2 3 4 5 6 7 8 9 10
Ans. B D B D C A D A D D
Que. 11 12 13 14 15 16 17 18 19 20
Ans. C B A B D A,B, C,D C C B,D A
Que. 21 22
Ans. A ,C , D B

Page 129
130
JEE Sprint Mathematics
CIRCLES
SELECT  THE  CORRECT  ALTERNATIVE  (ONLY  ONE  CORRECT  ANSWER)
1. The  lines  2x  –  3y  =  5  and  3x  –  4y  =  7  are  diameters  of  a  circle  of  area  154  sq.  units.  The  equation  of  the  circle  is -
(A) x2  +  y2  –  2x  –  2y  =  47 (B) x2  +  y2  –  2x  –2y  =  62
(C) x2  +  y2  –  2x  +  2y  =  47 (D) x2  +  y2  –  2x  +  2y  =  62
2. If  a  be  the  radius  of  a  circle  which  touches  x-axis  at  the  origin,  then  its  equation  is  -
(A) x2  +  y2  +  ax  =  0 (B) x2  +  y2  ±  2ya  =  0 (C) x2  +  y2  ±  2xa  =  0 (D) x2  +  y2  +  ya  =  0
3. The  equation  of  the  circle  which  touches  the  axis  of  y  at  the  origin  and  passes  through  (3,4)  is  -
(A) 4(x2  +  y2)  –  25x  =  0 (B) 3(x2  +  y2)  –  25x  =  0

T
(C) 2(x2  +  y2)  –  3x  =  0 (D) 4(x2  +  y2)  –  25x  +  10  =  0
4. The  equation  of  the  circle  passing  through    (3,6)  and  whose  centre  is  (2,–1)  is  -
(A) x2  +  y2  –  4x  +  2y  =  45 (B) x2  +  y2  –  4x  –  2y  +  45  =  0

N
(C) x2  +  y2  +  4x  –  2y  =  45 (D)  x2  +  y2  –  4x  +  2y  +  45  =  0

I
5. The  equation  to  the  circle  whose  radius  is  4  and  which  touches  the  negative  x-axis  at  a  distance  3  units  from  the
origin  is  -
(A) x2  +  y2  –  6x  +  8y  –  9  =  0 (B) x2  +  y2  ±  6x  –  8y  +  9  =  0

R
(C) x2  +  y2  +  6x  ±  8y  +  9  =  0 (D) x2  +  y2  ±  6x  –  8y  –  9  =  0

P
6. The  equation  of  a  circle  which  passes  through  the  three  points  (3,0)  (1,–6),(4,–1)  is  -
(A) 2x2  +  2y2  +  5x  –11y  +  3  =  0 (B) x2  +  y2  –  5x  +11y  –  3  =  0

S
(C) x2  +  y2  +  5x  –11y  +  3  =  0 (D) 2x2  +  2y2  –  5x  +11y  –  3  =  0

7. y  3 x  c 1   &  y  3 x  c 2   are  two  parallel  tangents  of  a  circle  of  radius  2  units,  then c1  c 2   is  equal  to -
(A) 8 (B) 4 (C) 2 (D) 1

E
8. Number  of  different  circles  that  can  be  drawn  touching  3  lines,  no  two  of  which  are  parallel  and  they  are  neither
coincident  nor  concurrent,  are  -

E
(A) 1 (B) 2 (C) 3 (D) 4
9. B  and  C  are  fixed  points  having  co-ordinates  (3,  0)  and  (–3,  0)  respectively.  If  the  vertical  angle  BAC  is  90°,

J
then  the  locus  of  the  centroid  of  the  ABC  has  the  equation  -
(A) x 2   +  y 2   =  1 (B) x 2   +  y 2   =  2 (C) 9(x 2   +  y 2 )  =  1 (D) 9(x 2   +  y 2 )  =  4
10. If  a  circle  of  constant  radius  3k  passes  through  the  origin  ‘O’    and  meets  co-ordinate  axes  at    A  and  B  then  the
locus  of  the  centroid  of  the  triangle  OAB  is  -
(A) x2  +  y2  =  (2k)2 (B) x2  +  y2  =  (3k)2 (C) x2  +  y2  =  (4k)2 (D) x2  +  y2  =  (6k)2
11. The  area  of  an  equilateral  triangle  inscribed  in  the  circle    x2  +  y2  –  2x  =  0  is  :

3 3 3 3 3 3
(A)  (B)  (C)  (D) none
2 4 8
12. The  length  of  intercept  on  y-axis,  by  a  circle  whose  diameter  is  the  line  joining  the  points  (–4,3)  and
(12, –1)  is  -

(A) 3 2 (B)  13 (C) 4 13 (D) none  of  these

13. The  gradient  of  the  tangent  line  at  the  point  (a  cos  ,  a  sin  )  to  the  circle  x2  +  y2  =  a2,  is  -
(A) tan  (  –  ) (B) tan   (C) cot  (D) –  cot 
14. x  +  my  +  n  =  0  is  a  tangent  line  to  the  circle  x2  +  y2  =  r2  ,  if  -
(A) 2  +  m2  =  n2  r2 (B) 2  +  m2  =  n2  +  r2 (C) n2  =  r2(2  +  m2) (D) none  of  these

Page 130
131
JEE Sprint Mathematics
15. Line  3x  +  4y  =  25  touches  the  circle  x2  +  y2  =  25  at  the  point  -

(A) (4,  3) (B) (3,  4) (C) (–3,  –4) (D) none  of  these
16. The  equations  of  the  tangents  drawn  from  the  point  (0,1)  to  the  circle  x2  +  y2  –  2x  +  4y  =  0  are  -
(A) 2x  –  y  +  1  =  0,  x  +  2y  –  2  =  0 (B) 2x  –  y  –  1  =  0,  x  +  2y  –  2  =  0
(C) 2x  –  y  +  1  =  0,  x  +  2y  +  2  =  0 (D) 2x  –  y  –  1  =  0,  x  +  2y  +  2  =  0
17. The  greatest  distance  of  the  point  P(10,7)  from  the  circle  x2  +  y2  –  4x  –  2y  –  20  =  0  is  -
(A) 5 (B) 15 (C) 10 (D) None  of  these
 3 3 
18. The  equation  of  the  normal  to  the  circle  x2  +  y2  =  9  at  the  point   ,    is  -
 2 2

T
2
(A) x – y  (B)  x  +  y  =  0 (C) x  –  y  =  0 (D) none  of  these
3
19. The  parametric  coordinates  of  any  point  on  the  circle  x2  +  y2  –  4x  –  4y  =  0  are-

N
(A) (–2  +  2cos,  –2  +  2  sin) (B) (2  +  2cos,  2  +  2  sin)

I
(C) (2  +  2 2 cos,  2  +  2 2 sin) (D) (–2  +  2 2 cos,  –2  +  2 2 sin)
20. The  length  of  the  tangent  drawn  from  the  point  (2,3)  to  the  circles  2(x2  +  y2)  –  7x  +  9y  –  11  =  0  -

R
(A)  18 (B)  14 (C)  14 (D)  28
21. A  pair  of  tangents  are  drawn  from  the  origin  to  the  circle  x2  +  y2  +  20(x  +  y)  +  20  =  0.  The  equation  of  the  pair

P
of  tangents  is  -
(A) x2  +  y2  +  5xy  =  0 (B) x2  +  y2  +  10xy  =  0 (C) 2x2  +  2y2  +  5xy  =  0 (D) 2x2  +  2y2  –  5xy  =  0

S
22. Tangents  are  drawn  from  (4,  4)  to  the  circle  x 2  +  y 2   –  2x  –  2y  –  7  =  0  to  meet  the  circle  at  A  and  B.
The  length  of  the  chord  AB  is  -
(A) 2 3 (B) 3 2 (C) 2 6 (D) 6 2

E
23. 2 2
The  angle  between  the  two  tangents  from  the  origin  to  the  circle    (x  –7)   +  (y  +  1)   =  25  equals  -
  
(A)  (B)  (C)  (D) none

E
2 3 4

J
24. Pair  of  tangents  are  drawn  from  every  point  on  the  line  3x  +  4y  =  12  on  the  circle  x2  +  y2  =  4.  Their  variable
chord  of  contact  always  passes  through  a  fixed  point  whose  co-ordinates  are  -
 4 3  3 3  4
(A)   ,  (B)   ,  (C) (1,  1) (D)   1, 
3 4 4 4 3
25. The  locus  of  the  mid-points  of  the  chords  of  the  circle  x 2  +  y 2  –  2x  –  4y  –  11  =  0  which  subtend  60°  at
the  centre  is -
(A) x 2   +  y 2   –  4x  –  2y  –  7  =  0 (B) x 2   +  y 2   +  4x  +  2y  –  7  =  0
(C) x 2   +  y 2   –  2x  –  4y  –  7  =  0 (D) x 2   +  y 2   +  2x  +  4y  +  7  =  0
26. The  locus  of  the  centres  of  the  circles  such  that  the  point  (2,3)  is  the  mid  point  of  the  chord  5x  +  2y  =  16  is  -
(A) 2x  –  5y  +  11  =  0 (B) 2x  +  5y  –  11  =  0 (C) 2x  +  5y  +  11  =  0 (D) none
2 2
27. The  locus  of  the  centre  of  a  circle  which  touches  externally  the  circle,  x   +  y   –  6x  –  6y  +  14  =  0  and
also  touches  the  y-axis  is  given  by  the  equation  -
(A) x 2   –  6x  –  10y  +  14  =  0 (B) x 2   –  10x  –  6y  +  14  =  0
(C) y 2   –  6x  –  10y  +  14  =  0 (D) y 2   –  10x  –  6y  +  14  =  0
28. The  equation  of  the  circle  having  the  lines  y2  –  2y  +  4x  –  2xy  =  0  as  its  normals  &  passing  through  the  point
(2,1)  is  -
(A) x2  +  y2  –  2x  –  4y  +  3  =  0 (B) x2  +  y2  –  2x  +  4y  –  5  =  0
(C) x2  +  y2  +  2x  +  4y  –13  =  0 (D) none

Page 131
132
JEE Sprint Mathematics
29. A  circle  is  drawn  touching  the  x-axis  and  centre  at  the  point  which  is  the  reflection  of  (a,  b)  in  the  line
y  –  x  =  0.  The  equation  of  the  circle  is  -
(A) x 2   +  y 2   –  2bx  –  2ay  +  a 2   =  0 (B) x 2   +  y 2   –  2bx  –  2ay  +  b 2   =  0
(C) x 2   +  y 2   –  2ax  –  2by  +  b 2   =  0 (D) x 2   +  y 2   –  2ax  –  2by  +  a 2   =  0
30. The  length  of  the  common  chord  of  circles  x2  +  y2  –  6x  –  16  =  0  and  x2  +  y2  –  8y  –  9  =  0  is  -

(A) 10 3 (B) 5 3 (C) 5 3 / 2 (D) none  of  these

31. The  number  of  common  tangents  of  the  circles  x2  +  y2  –  2x  –  1  =  0  and  x2  +  y2  –  2y  –  7  =  0  -
(A) 1 (B) 3 (C) 2 (D) 4
32. If  the  circle  x2  +  y2  =  9  touches  the  circle  x2  +  y2   +  6y  +  c  =  0,  then  c  is  equal  to  -
(A) –27 (B) 36 (C) –36 (D) 27

T
2 2 2 2
33. If  the  two  circles,  x   +  y   +  2g 1 x  +  2f 1 y  =  0  and  x   +  y   +  2g 2 x  +  2f 2 y  =  0  touches  each  other,  then -
f1 f2
(A) f 1 g 1   =  f 2 g 2 (B)  g   = (C) f 1 f 2   =  g 1 g 2 (D) none
1 g2

N
34. The  tangent  from  the  point  of  intersection  of  the  lines  2x  –  3y  +  1  =  0  and  3x  –  2y  –  1  =  0  to  the  circle

I
x 2   +  y 2   +  2x  –  4y  =  0  is  -
(A) x  +  2y  =  0,  x  –  2y  +  1  =  0 (B) 2x  –  y  –  1  =  0
(C) y  =  x,  y  =  3x  –  2 (D) 2x  +  y  +  1  =  0

R
35. The  locus  of  the  centers  of  the  circles  which  cut  the  circles  x 2   +  y 2   +  4x  –  6y  +  9  =  0  and
x 2  +  y 2  –  5x  +  4y  –  2  =  0  orthogonally  is  -

P
(A) 9x  +  10y  –  7  =  0 (B) x  –  y  +  2  =  0
(C) 9x  –  10y  +  11  =  0 (D) 9x  +  10y  +  7  =  0

S
SELECT  THE  CORRECT  ALTERNATIVES  (ONE  OR  MORE  THAN  ONE  CORRECT  ANSWERS)
x  x1 y  y 1
36. Equation    r, may  represents  -

E
cos  sin 
(A) Equation  of  straight  line,  if    is  constant  and  r  is  variable.

E
(B) Equation  of  a  circle,  if  r  is  constant  &    is  variable.

J
(C) A  straight  line  passing  through  a  fixed  point  &  having  a  known  slope.
(D) A  circle  with  a  known  centre  and  given  radius.
37. If  r  represent  the  distance  of  a  point  from  origin  &    is  the  angle  made  by  line  joining  origin  to  that  point  from
line  x-axis,  then  r  =  |cos|  represents  -

1 1   1 
(A) two  circles  of  radii   each. (B) two  circles  centred  at  , 0    &    , 0 
2 2   2 
(C) two  circles  touching  each  other  at  the  origin. (D) pair  of  straight  line
38. If  the  circle  C1  :  x2  +  y2  =  16  intersects  another  circle  C2  of  radius  5  in  such  a  manner  that  the  common  chord
3
is  of  maximum  length  8  has  a  slope  equal  to  ,  then  coordinates  of  centre  of  C2  are  -
4
9 12   9 12   9 12   9 12 
(A)   ,  (B)    , (C)   , (D)    , 
5 5   5 5 

5 5 

 5 5 
39. For  the  equation  x2  +  y2  +  2x  +  4  =  0  which  of  the  following  can  be  true  -
(A) It  represents  a  real  circle  for  all      R.
(B) It  represents  a  real  circle  for  ||  >  2.
(C) The  radical  axis  of  any  two  circles  of  the  family  is  the  y-axis.
(D) The  radical  axis  of  any  two  circles  of  the  family  is  the  x-axis.

Page 132
133
JEE Sprint Mathematics
40. If  y  =  c  is  a  tangent  to  the  circle  x2  +  y2  –  2x  +  2y  –  2  =  0,  then  the  value  of  c  can  be  -
(A) 1 (B) 3 (C) –1 (D) –3
41. For the circles S1  x2 + y2 – 4x – 6y – 12 = 0 and S2  x2 + y2 + 6x + 4y – 12 = 0 and the line  L  x  y  0
(A) L  is  common  tangent  of  S1  and  S2
(B) L  is  common  chord  of  S1  and  S2
(C) L  is  radical  axis  of  S1  and  S2
(D) L  is  perpendicular  to  the  line  joining  the  centre  of  S1  &  S2

N T
R I
S P
JE E
ANSWER  KEY
Que. 1 2 3 4 5 6 7 8 9 10
Ans. C B B A C D A D A A
Que. 11 12 13 14 15 16 17 18 19 20
Ans. B C D C B A B C C C
Que. 21 22 23 24 25 26 27 28 29 30
Ans. C B A D C A D A B B
Que. 31 32 33 34 35 36 37 38 39 40
Ans. A A B B C A,B,C,D A, B, C A,B B,C A,D
Que. 41
Ans. B,C,D

Page 133
134
JEE Sprint Mathematics
EXTRA PRACTICE QUESTIONS ON CIRCLES
SELECT  THE  CORRECT  ALTERNATIVES  (ONE  OR  MORE  THAN  ONE  CORRECT  ANSWERS)

 1   1   1  1
1. If   a,  ,   b,  ,   c,    &   d,    are  four  distinct  points  on  a  circle  of  radius  4  units  then,  abcd  =
 a   b  c  d
(A) 4 (B) 1/4 (C) 1 (D) 16
2. What  is  the  length  of  shortest  path  by  which  one  can  go  from  (–2,  0)  to  (2,  0)  without  entering  the  interior  of
circle,  x2  +  y2  =  1  ?
2 
(A)    23 (B)   3  (C)   2 3  (D)    none  of  these
3 3
3. Three  equal  circles  each  of  radius  r  touch  one  another.  The  radius  of  the  circle  touching  all  the  three  given
circles  internally  is  -

T
(A) (2  3 )r (B) 
2  3  r (C) 
2  3  r (D) (2  3 )r
3 3
4. If  a2  +  b2  =  1,  m2  +  n2  =  1,  then  which  of  the  following  is  true  for  all  values  of  m,  n,  a,  b  -

N
(A) |am  +  bn|    1 (B)  |am  –  bn|    1 (C) |am  +  bn|    1 (D) |am  –  bn|    1

I
5. Circles  are  drawn  touching  the  co-ordinate  axis  and  having  radius  2,  then  -
(A) centre  of  these  circles  lie  on  the  pair  of  lines  y2  –  x2  =  0

R
(B) centre  of  these  circles  lie  only  on  the  line  y  =  x
(C) Area  of  the  quadrilateral  whose  vertices  are  centre  of  these  circles  is  16  sq.unit

P
(D) Area  of  the  circle  touching  these  four  circles  internally  is  4 (3  2 2 )
6. The  distance  between  the  chords  of  contact  of  tangents  to  the  circle  x2  +  y2  +  2gx  +  2fy  +  c  =  0  from  the  origin  and

S
from  the  point  (g,f)  is  -

g2  f2  c g2  f2  c g2  f2  c
(A)   g 2  f 2 (B)   (C)  (D) 
2 2 g2  f 2 2 g2  f2

E
7. x2  +  y2  +  6x  =  0  and  x2  +  y2  –  2x  =  0  are  two  circles,  then  -
(A) They  touch  each  other  externally

E
(B) They  touch  each  other  internally

J
(C) Area  of  triangle  formed  by  their  common  tangents  is  33  sq.  units.
(D) Their  common  tangents  do  not  form  any  triangle.
8. Tangents  are  draw n  to  the  circle  x 2   +  y 2   =  1  at  the  poi nt s  where  it  is  met  by  the  circle s,
x2  +  y2  –  (  +  6)x  +  (8  –  2)y  –  3  =  0,   being  the  variable.  The  locus  of  the  point  of  intersection  of  these
tangents  is -
(A) 2x  –  y  +  10  =  0 (B)  x  +  2y  –  10  =  0 (C)  x  –  2y  +  10  =  0 (D)  2x  +  y  –  10  =  0
9. 3  circle  of  radii  1,  2  and  3  and  centres  at  A,  B  and  C  respectively,  touch  each  other.  Another  circle  whose  centre
is  P  touches  all  these  3  circles  externally  and  has  radius  r.  Also  PAB     &  PAC   -
3r 2r 6 6
(A) cos   (B) cos   (C) r  (D) r 
3 (1  r ) 2 (1  r ) 23 23
10. Slope  of  tangent  to  the  circle  (x  –  r)2  +  y2  =  r2  at  the  point  (x,  y)  lying  on  the  circle  is -
2 2
x rx y x y2  x2
(A)   (B)   (C)  (D)  
y r y 2 xy 2 xy
11. The  circle  passing  through  the  distinct  points  (1,t)  ,  (t,1)  &  (t,t)  for  all  values  of  ‘t’,  passes  through  the  point  -
(A) (–1,  –1) (B) (–1,  1) (C) (1,  –1) (D) (1,1)
12. AB  is  a  diameter  of  a  circle.  CD  is  a  chord  parallel  to  AB  and    2CD  =  AB.  The  tangent  at  B  meets  the  line  AC
produced  at  E    then  AE  is  equal  to  -

(A) AB (B)  2 AB (C) 2 2AB (D) 2AB

Page 134
135
JEE Sprint Mathematics
13. The  locus  of  the  mid  points  of  the  chords  of  the  circle  x2  +  y2  –  ax  –  by  =  0  which  subtend  a  right  angle  at
 a b
 2 , 2   is -
(A) ax  +  by  =  0 (B) ax  +  by  =  a2  +  b2
2 2 a 2  b2 2 2 a 2  b2
(C)  x  y  ax  by  0 (D)  x  y  ax  by  0
8 8
14. A  variable  circle  is  drawn  to  touch  the  x-axis  at  the  origin.  The  locus  of  the  pole  of  the  straight  line
x  +  my  +  n  =  0  w.r.t.  the  variable  circle  has  the  equation  -
(A) x(my  –  n)  –  y2  =  0 (B) x(my  +  n)  –  y2  =  0 (C) x(my  –  n)  +  y2  =  0 (D) none
15. (6,0)  ,  (0,6)  and  (7,7)  are  the  vertices  of  a  triangle.  The  circle  inscribed  in  the  triangle  has  the  equation  -
(A) x2  +  y2  –  9x  +  9y  +  36  =  0 (B) x2  +  y2  –  9x  –  9y  +  36  =  0
(C) x2  +  y2  +  9x  –  9y  +  36  =  0 (D) x2  +  y2  –  9x  –  9y  –  36  =  0

T
16. Number  of  points  (x,  y)  having  integral  coordinates  satisfying  the  condition  x 2   +  y 2  <  25  is -
(A) 69 (B) 80 (C) 81 (D) 77
17. The  centre(s)  of  the  circle(s)  passing  through  the  points  (0,  0),  (1,  0)  and  touching  the  circle

N
x 2   +  y 2   =  9  is/are -

I
3 1 1 3  1 1/2  1 1/2 
(A)   ,  (B)   ,  (C)   , 2  (D)   ,  2 
2 2 2 2 2 2

R
18. The  equation(s)  of  the  tangent  at  the  point  (0,  0)  to  the  circle,  making  intercepts  of  length  2a  and  2b  units
on  the  co-ordinate  axes,  is  (are)  -

P
(A) ax  +  by  =  0 (B) ax  –  by  =  0 (C) x  =  y (D) bx  +  ay  =  0
19. Tangents  are  drawn  to  the  circle  x 2  +  y 2  =  50  from  a  point  'P'  lying  on  the  x-axis.  These  tangents  meet
the  y-axis  at  points  'P1 '  and  'P 2 '.  Possible  co-ordinates  of  'P'  so  that  area  of  triangle  PP 1 P 2  is  minimum

S
is/are -
(A)  (10,  0) (B)  (10 2, 0) (C)  (–10,  0) (D)  ( 10 2, 0)

E
2 2 2
20. The  tangents  drawn  from  the  origin  to  the  circle  x   +  y   –  2rx  –  2hy  +  h   =  0  are  perpendicular  if  -
(A) h  =  r (B) h  =  –r (C) r 2   +  h 2   =  1 (D) r 2   +  h 2  =  2
21. The  common  chord  of  two  intersecting  circles  C1  and  C2  can  be  seen  from  their  centres  at  the  angles  of  90°  &

E
60°  respectively.  If  the  distance  between  their  centres  is  equal  to  3   +  1  then  the  radii  of  C1  and  C2  are -

J
(A)  3   and  3 (B)  2   and  2 2 (C)  2   and  2 (D)  2 2   and  4
22. In  a  right  triangle  ABC,  right  angled  at  A,  on  the  leg  AC  as  diameter,  a  semicircle  is  described.  The  chord
joining  A  with  the  point  of  intersection  D  of  the  hypotenuse  and  the  semicircle,  then  the  length  AC  equals  to -
AB . AD AB . AD AB . AD
(A)  (B)  (C)  AB . AD (D) 
2
AB  AD 2 AB  AD AB 2  AD 2
23. A  circle  touches  a  straight  line  x  +  my  +  n  =  0  and  cuts  the  circle  x 2  +  y 2  =  9  orthogonally.  The  locus
of  centres  of  such  circles  is  -
(A) (x  +  my  +  n) 2  =  ( 2  +  m 2 )  (x 2   +  y 2   –  9) (B) (x  +  my  –  n) 2   =  ( 2   +  m 2)  (x 2   +  y 2   –  9)
(C) (x  +  my  +  n) 2  =  ( 2   +  m 2 )  (x 2   +  y 2   +  9) (D) none  of  these

ANSWER  KEY
Que. 1 2 3 4 5 6 7 8 9 10
Ans. C C B A,D A ,C , D C A,C A A, B, C B,C
Que. 11 12 13 14 15 16 17 18 19 20
Ans. D D C A B A C,D A,B A,C A,B
Que. 21 22 23
Ans. C D A

Page 135
136
JEE Sprint Mathematics
PARABOLA

SELECT THE CORRECT ALTERNATIVE (ONLY ONE CORRECT ANSWER)

1. Latus rectum of the parabola whose focus is (3, 4) and whose tangent at vertex has the equation

x + y = 7  5 2 is -
(A) 5 (B) 10 (C) 20 (D) 15
2. Directrix of a parabola is x + y = 2. If it's focus is origin, then latus rectum of the parabola is equal to -

(A) 2 units (B) 2 units (C) 2 2 units (D) 4 units


3. Which one of the following equations represents parametrically, parabolic profile ?

T
t
(A) x = 3 cos t ; y = 4 sint (B) x2 – 2 = –cost ; y = 4 cos2
2

N
t t

I
(C) x  tan t ; y  sec t (D) x  1  sin t ; y  sin  cos
2 2

4. Let C be a circle and L a line on the same plane such that C and L do not intersect. Let P be a moving point

R
such that the circle drawn with centre at P to touch L also touches C. Then the locus of P is -
(A) a straight line parallel to L not intersecting C

P
(B) a circle concentric with C
(C) a parabola whose focus is centre of C and whose directrix is L.

S
(D) a parabola whose focus is the centre of C and whose directrix is a straight line parallel to L.
5. If (t 2, 2t) is one end of a focal chord of the parabola y 2 = 4x then the length of the focal chord will be-

E
2
 1  1  2 1  1  2 1
(A)  t   (B)  t   t  2  (C)  t   t  2  (D) none
 t  t  t   t  t 

E
6. From the focus of the parabola y2 = 8x as centre, a circle is described so that a common chord of the curves

J
is equidistant from the vertex and focus of the parabola. The equation of the circle is -
(A) (x – 2) 2 + y 2 = 3 (B) (x – 2) 2 + y 2 = 9 (C) (x + 2) 2 + y 2 = 9 (D) x 2 + y 2 – 4x = 0
7. The point of intersection of the curves whose parametric equations are x = t2+1, y = 2t and x = 2s, y = 2/s is
given by -
(A) (4, 1) (B) (2, 2) (C) ( –2, 4) (D) (1, 2)
2
8. If M is the foot of the perpendicular from a point P of a parabola y = 4ax to its directrix and SPM is an
equilateral triangle, where S is the focus, then SP is equal to -
(A) a (B) 2a (C) 3a (D) 4a
2
9. Through the vertex ‘O’ of the parabola y = 4ax, variable chords OP and OQ are drawn at right angles. If the
variable chord PQ intersects the axis of x at R, then distance OR :
(A) varies with different positions of P and Q (B) equals the semi latus rectum of the parabola
(C) equals latus rectum of the parabola (D) equals double the latus rectum of the parabola
10. The triangle PQR of area ‘A’ is inscribed in the parabola y2 = 4ax such that the vertex P lies at the vertex of the
parabola and the base QR is a focal chord. The modulus of the difference of the ordinates of the points Q and
R is -

A A 2A 4A
(A) (B) (C) (D)
2a a a a

Page 136
137
JEE Sprint Mathematics
11. Point P lies on y2 = 4ax & N is foot of perpendicular from P on its axis. A straight line is drawn parallel to the
axis to bisect NP and meets the curve in Q. NQ meets the tangent at the vertex in a point T such that
AT = k NP, then the value of k is : (where A is the vertex)
(A) 3/2 (B) 2/3 (C) 1 (D) none
12. The tangents to the parabola x = y2 + c from origin are perpendicular then c is equal to -

1 1
(A) (B) 1 (C) 2 (D)
2 4

13. The locus of a point such that two tangents drawn from it to the parabola y2 = 4ax are such that the slope of
one is double the other is -

T
2 9 2 9
(A) y  ax (B) y  ax (C) y 2  9ax (D) x 2  4ay
2 4

14. T is a point on the tangent to a parabola y2 = 4ax at its point P. TL and TN are the perpendiculars on the focal

N
radius SP and the directrix of the parabola respectively. Then -

I
(A) SL = 2 (TN) (B) 3 (SL) = 2 (TN) (C) SL = TN (D) 2 (SL) = 3 (TN)
2
15. The equation of the circle drawn with the focus of the parabola (x – 1 ) – 8y = 0 as its centre and touching the

R
parabola at its vertex is :
(A) x2 + y2 – 4 y = 0 (B) x2 + y2 – 4 y + 1 = 0

P
(C) x2 + y2 – 2x – 4 y = 0 (D) x2 + y2 – 2x – 4 y + 1 = 0

S
16. Length of the normal chord of the parabola, y 2 = 4x, which makes an angle of with the axis of x is-
4

(A) 8 (B) 8 2 (C) 4 (D) 4 2

E
17. Tangents are drawn from the point (–1, 2) on the parabola y2 = 4x . The length , these tangents will intercept
on the line x = 2 :

E
(A) 6 (B) 6 2 (C) 2 6 (D) none of these

J
18. Locus of the point of intersection of the perpendiculars tangent of the curve y2 + 4y – 6x – 2 = 0 is :
(A) 2x – 1 = 0 (B) 2x + 3 = 0 (C) 2y + 3 = 0 (D) 2x + 5 = 0
2
19. Tangents are drawn from the points on the line x – y + 3 = 0 to parabola y = 8x. Then the variable chords
of contact pass through a fixed point whose coordinates are-
(A) (3, 2) (B) (2, 4) (C) (3, 4) (D) (4, 1)
2
20. The line 4x – 7y + 10 = 0 intersects the parabola, y = 4x at the points A & B. The co-ordinates of the point
of intersection of the tangents drawn at the points A & B are :

7 5  5 7  5 7  7 5
(A)  ,  (B)   ,  (C)  ,  (D)   , 
2 2 2 2 2 2 2 2

21. From the point (4, 6) a pair of tangent lines are drawn to the parabola, y2 = 8x. The area of the triangle formed
by these pair of tangent lines & the chord of contact of the point (4, 6) is
(A) 2 (B) 4 (C) 8 (D) none
22. TP & TQ are tangents to the parabola, y2=4ax at P & Q. If the chord PQ passes through the fixed point (-a,
b) then the locus of T is -
(A) ay = 2b (x – b) (B) bx = 2a (y – a) (C) by = 2a (x – a) (D) ax = 2b (y – b)

Page 137
138
JEE Sprint Mathematics
23. If the tangent at the point P (x1,y1) to the parabola y2 = 4ax meets the parabola y2 = 4a (x + b) at Q & R, then
the mid point of QR is -
(A) (x1+ b, y1+ b) (B) (x1– b, y1– b) (C) (x1, y1) (D) (x1+ b, y1)
24. Let PSQ be the focal chord of the parabola, y = 8x. If the length of SP=6 then, l(SQ) is equal to(where S is the
2

focus) -
(A) 3 (B) 4 (C) 6 (D) none
25. Two parabolas y = 4a(x – l1) and x = 4a(y – l2) always touch one another , the quantities l1 and l2 are both
2 2

variable. Locus of their point of contact has the equation -


(A) xy = a2 (B) xy = 2a2 (C) xy = 4a2 (D) none
SELECT THE CORRECT ALTERNATIVES (ONE OR MORE THAN ONE CORRECT ANSWERS)

T
26. Equation x2 – 2x – 2y + 5 = 0 represents -
(A) a parabola with vertex (1, 2) (B) a parabola with vertex (2, 1)

3 2

N
(C) a parabola with directrix y  (D) a parabola with directrix y 
2 5

I
27. The normals to the parabola y2 = 4ax from the point (5a, 2a) are -
(A) y = –3x + 33a (B) x = –3y + 3a (C) y = x – 3a (D) y = –2x + 12a

R
28. The equation of the lines joining the vertex of the parabola y2 = 6x to the points on it whose abscissa is 24, is -
(A) 2y + x + 1 = 0 (B) 2y – x + 1 =0 (C) x + 2y = 0 (D) x – 2y = 0

P
29. The equation of the tangent to the parabola y2 = 9x which passes through the point (4, 10) is -
(A) x + 4y + 1 = 0 (B) x – 4y + 36 = 0 (C) 9x – 4y + 4 = 0 (D) 9x + 4y + 4 = 0

S
30. Consider the equation of a parabola y2 = 4ax, (a < 0) which of the following is false -
(A) tangent at the vertex is x = 0 (B) directrix of the parabola is x = 0
(C) vertex of the parabola is at the origin (D) focus of the parabola is at (–a, 0)

JE E
ANSWER KEY
Que. 1 2 3 4 5 6 7 8 9 10 11 12 13 14 15
Ans. C C B D A B B D C C B D A C D
Que. 16 17 18 19 20 21 22 23 24 25 26 27 28 29 30
Ans. B B D C C A C C A C A,C C,D C,D B,C B,D

Page 138
139
JEE Sprint Mathematics
EXTRA PRACTICE QUESTIONS ON PARABOLA
SELECT THE CORRECT ALTERNATIVES (ONE OR MORE THAN ONE CORRECT ANSWERS)
1. The straight line joining any point P on the parabola y 2 = 4ax to the vertex and perpendicular from the
focus to the tangent at P, intersect at R, then the equation of the locus of R is -
(A) x 2 + 2y 2 – ax = 0 (B) 2x 2 + y 2 – 2ax = 0 (C) 2x 2 + 2y 2 – ay = 0 (D) 2x 2 + y 2 – 2ay = 0
2. Let A be the vertex and L the length of the latus rectum of parabola, y2 – 2y – 4x – 7 = 0. The equation
of the parabola with point A as vertex, 2L as the length of the latus rectum and the axis at right angles
to that of the given curve is -
(A) x 2 + 4x + 8y – 4 = 0 (B) x 2 + 4x – 8y + 12 = 0
(C) x 2 + 4x + 8y + 12 = 0 (D) x 2 + 8x – 4y + 8 = 0

T
3. The parametric coordinates of any point on the parabola y 2 = 4ax can be -
(A) (at 2, 2at) (B) (at 2, –2at) (C) (asin2 t, 2asint) (D) (asint, 2acost)

N
2
4. PQ is a normal chord of the parabola y = 4ax at P, A being the vertex of the parabola. Through P a line is

I
drawn parallel to AQ meeting the x-axis in R. Then the length of of AR is -
(A) equal to the length of the latus rectum

R
(B) equal to the focal distance of the point P.
(C) equal to twice the focal distance of the point P.

P
(D) equal to the distance of the point P from the directrix
5. The length of the chord of the parabola y 2 = x which is bisected at the point (2, 1) is-

S
(A) 5 2 (B) 4 5 (C) 4 50 (D) 2 5

6. If the tangents and normals at the extremities of a focal chord of a parabola intersect at (x 1 , y 1) and

E
(x 2, y 2 ) respectively, then -
(A) x 1 = x 2 (B) x 1 = y 2 (C) y 1 = y 2 (D) x 2 = y 1

E
7. Locus of the intersection of the tangents at the ends of the normal chords of the parabola y 2 = 4ax is -

J
(A) (2a + x)y 2 + 4a 3 = 0 (B) (x + 2a)y 2 + 4a 2 = 0
(C) (y + 2a)x 2 + 4a 3 = 0 (D) none
8. The locus of the mid point of the focal radii of a variable point moving on the parabola, y 2 = 4ax is a
parabola whose
(A) latus rectum is half the latus rectum of the original parabola
(B) vertex is (a/2, 0)
(C) directrix is y-axis
(D) focus has the co-ordinates (a, 0)

9. The equation of a straight line passing through the point (3, 6) and cutting the curve y = x orthogonally
is -
(A) 4x + y – 18 = 0 (B) x + y – 9 = 0 (C) 4x – y – 6 = 0 (D) none
2
10. The tangent and normal at P (t), for all real positive t, to the parabola y = 4ax meet the axis of the parabola
in T and G respectively, then the angle at which the tangent at P to the parabola is inclined to the tangent at
P to the circle through the points P, T and G is -

 t 
(A) cot–1t (B) cot–1t 2 (C) tan–1t (D) sin–1  
 1  t2 

Page 139
140
JEE Sprint Mathematics
11. A variable circle is described to passes through the point (1, 0) and tangent to the curve y = tan(tan–1x).
The locus of the centre of the circle is a parabola whose -
(A) length of the latus rectum is 2 2 (B) axis of symmetry has the equation x + y = 1
(C) vertex has the co-ordinates (3/4, 1/4) (D) none of these
2
12. AB, AC are tangents to a parabola y = 4ax. p 1 p 2 and p 3 are the lengths of the perpendiculars from A,
B and C respectively on any tangent to the curve, then p 2, p 1, p 3 are in-
(A) A.P. (B) G.P. (C) H.P. (D) none of these
2
13. Through the vertex O of the parabola, y = 4ax two chords OP and OQ are drawn and the circles on
OP and OQ as diameter intersect in R. If  1, 2 and  are the angles made with the axis by the tangent
at P and Q on the parabola and by OR then the value of cot 1 + cot2 =
(A) – 2tan (B) – 2 tan( – ) (C) 0 (D) 2cot

T
14. Two parabolas have the same focus. If their directrices are the x-axis & the y-axis respectively, then the
slope of their common chord is -

N
(A) 1 (B) –1 (C) 4/3 (D) 3/4
y2

I
15. Tangent to the parabola = 4ax at point P meets the tangent at vertex A, at point B and the axis of parabola
at T. Q is any point on this tangent and N is the foot of perpendicular from Q on SP, where S is focus. M is the
foot of perpendicular from Q on the directrix then -

R
(A) B bisects PT (B) B trisects PT (C) QM = SN (D) QM = 2SN
16. If the distance between a tangent to the parabola y2 = 4 x and a parallel normal to the same parabola is 2 2 ,

P
then possible values of gradient of either of them are -

(A) –1 (B) +1 (C) – (D) +

S
5 2 5 2
17. If two distinct chords of a parabola x 2 = 4ay passing through (2a, a) are bisected on the line x + y = 1,
then length of latus rectum can be -
(A) 2 (B) 1 (C) 4 (D) 5

E
18. 2
If PQ is a chord of parabola x = 4y which subtends right angle at vertex. Then locus of centroid of triangle
PSQ (S is focus) is a parabola whose -

E
(A) vertex is (0, 3) (B) length of LR is 4/3

J
(C) axis is x = 0 (D) tangent at the vertex is x = 3
19. Identify the correct statement(s) -
(A) In a parabola vertex is the mid point of focus and foot of directrix.

 2
(B) P at1 , 2at 1  2
& Q at 2 , 2at 2  are two points on y2 = 4ax such that t1t2 = –1, then normals at P and Q aree

perpendicular.
(C) There doesn't exist any tangent of y2 = 4ax which is parallel to x-axis.
(D) At most two normals can be drawn to a parabola from any point on its plane.
20. For parabola y2 = 4ax consider three points A, B, C lying on it. If the centroid of ABC is (h1, k1) & centroid
of triangle formed by the point of intersection of tangents at A, B, C has coordinates (h2, k2), then which of the
following is always true -
4a 4a
(A) 2k1 = k2 (B) k1 = k2
2
(C) k 1 
3
 h 1  2h 2  2
(D) k 1 
3

2h 1  h 2 
ANSWER KEY
Que. 1 2 3 4 5 6 7 8 9 10
Ans. B A,B A,B C D C A A,B,C,D A C,D
Que. 11 12 13 14 15 16 17 18 19 20
Ans. B,C B A A,B A,C A,B,C,D A,B A,B,C A,B,C B,C

Page 140
JEE Sprint Mathematics
ELLIPSE
SELECT THE CORRECT ALTERNATIVE (ONLY ONE CORRECT ANSWER)

1. If distance between the directrices be thrice the distance between the foci, then eccentricity of ellipse is -
1 2 1 4
(A) (B) (C) (D)
2 3 3 5
2. If the eccentricity of an ellipse be 5/8 and the distance between its foci be 10, then its latus rectum is -
39 37
(A) (B) 12 (C) 15 (D)
4 2
3. The curve represented by x = 3(cost + sint), y = 4(cost – sint), is -

T
(A) ellipse (B) parabola (C) hyperbola (D) circle

x2 y2
4. If the distance of a point on the ellipse   1 from the centre is 2, then the eccentric angle is-
6 2

N
(A) /3 (B) /4 (C) /6 (D) /2

I
5. An ellipse having foci at (3, 3) and (–4, 4) and passing through the origin has eccentricity equal to-
3 2 5 3

R
(A) (B) (C) (D)
7 7 7 5
4 x2 y2
  1 intersects the major & minor axes in points A & B

P
6. A tangent having slope of – to the ellipse
3 18 32
respectively. If C is the centre of the ellipse then the area of the triangle ABC is :

S
(A) 12 sq. units (B) 24 sq. units (C) 36 sq. units (D) 48 sq. units
x2 y2
7. The equation to the locus of the middle point of the portion of the tangent to the ellipse + = 1 included
16 9

E
between the co-ordinate axes is the curve-
(A) 9x 2 + 16y 2 = 4x 2 y 2 (B) 16x 2 + 9y 2 = 4x 2 y 2

E
(C) 3x 2 + 4y 2 = 4x 2 y 2 (D) 9x 2 + 16y 2 = x 2 y 2
8. An ellipse is drawn with major and minor axes of lengths 10 and 8 respectively. Using one focus as centre,

J
a circle is drawn that is tangent to the ellipse, with no part of the circle being outside the ellipse. The radius
of the circle is-
(A) 3 (B) 2 (C) 2 2 (D) 5

x2 y2 x2 y2
9. Which of the following is the common tangent to the ellipses 2  2 1 &  1 ?
a b 2
b a2 a2  b2

(A) ay = bx + a 4  a 2 b2  b4 (B) by = ax – a 4  a 2 b 2  b 4

(C) ay = bx – a 4  a 2 b 2  b 4 (D) by = ax – a 4  a 2 b 2  b 4

x2 y2
10. Angle between the tangents drawn from point (4, 5) to the ellipse   1 is -
16 25
 5  
(A) (B) (C) (D)
3 6 4 2
x2 y2
11. The point of intersection of the tangents at the point P on the ellipse + = 1, and its corresponding
a2 b2
point Q on the auxiliary circle meet on the line -
(A) x = a/e (B) x = 0 (C) y = 0 (D) none

Page 141
JEE Sprint Mathematics
12. An ellipse is such that the length of the latus rectum is equal to the sum of the lengths of its semi principal axes.
Then -
(A) Ellipse becomes a circle (B) Ellipse becomes a line segment between the two foci
(C) Ellipse becomes a parabola (D) none of these
2 2
x y
13. The equation of the normal to the ellipse 2
 2  1 at the positive end of latus rectum is -
a b
(A) x + ey + e2a = 0 (B) x – ey – e3a = 0 (C) x – ey – e2a = 0 (D) none of these
x2 y2
14. The eccentric angle of the point where the line, 5x – 3y = 8 2 is a normal to the ellipse + = 1 is -
25 9
3  
(A) (B) (C) (D) tan –1 2
4 4 6
15. PQ is a double ordinate of the ellipse x 2 + 9y 2 = 9, the normal at P meets the diameter through Q at R,

T
then the locus of the mid point of PR is -
(A) a circle (B) a parabola (C) an ellipse (D) a hyperbola
2 2
16. The equation of the chord of the ellipse 2x + 5y = 20 which is bisected at the point (2, 1) is -

N
(A) 4x + 5y + 13 = 0 (B) 4x + 5y = 13 (C) 5x + 4y + 13 = 0 (D) 4x + 5y = 13

I
x2 y2
17. If F1 & F2 are the feet of the perpendiculars from the foci S1 & S2 of an ellipse   1 on the tangent at
5 3

R
any point P on the ellipse , then (S1F1) . (S2F2) is equal to
(A) 2 (B) 3 (C) 4 (D) 5
a2 x2 y2

P
18. If tan 1. tan 2 = – then the chord joining two points  &  on the ellipse   1 will subtend a right
b2 1 2
a 2 b2
angle at -

S
(A) focus (B) centre (C) end of the major axis (D) end of the minor axis
19. The number of values of c such that the straight line y = 4x + c touches the curve (x2 / 4) + y2 = 1 is -
(A) 0 (B) 1 (C) 2 (D) infinite [JEE 98]

E
SELECT THE CORRECT ALTERNATIVES (ONE OR MORE THAN ONE CORRECT ANSWERS)

E
x2 y2
20. If x – 2y + k = 0 is a common tangent to y 2 = 4x &   1 (a > 3 ), then the value of a, k and

J
a2 3
other common tangent are given by -
(A) a = 2 (B) a = –2 (C) x + 2y + 4 = 0 (D) k = 4

x2 y2
21. All ellipse   1 (0 < b < a) has fixed major axis. Tangent at any end point of latus rectum meet
a2 b2

at a fixed point which can be -


(A) (a, a) (B) (0, a) (C) (0, –a) (D) (0, 0)

22. Eccentric angle of a point on the ellipse x2 + 3y2 = 6 at a distance 3 units from the centre of the ellipse
is -
5  3 2
(A) (B) (C) (D)
3 3 4 3
23. For the ellipse 9x 2 + 16y 2 – 18x + 32y – 119 = 0, which of the following is/are true -
(A) centre is (1, –1)
(B) length of major and minor axis are 8 and 6 respectively
7
(C) e 
4
(D) foci are (1  7,  1)

Page 142
JEE Sprint Mathematics
24. With respect to the ellipse 4x 2 + 7y 2 = 8, the correct statement(s) is/are -
8 2
(A) length of latus rectum
7

(B) the distance between the directrix 4 7


3
1 
(C) tangent at  , 1  is 2x + 7y = 8
2 
4 3
(D) Area of  formed by foci and one end of minor axis is
7
25. On the ellipse, 4x 2 + 9y 2 = 1, the points at which the tangents are parallel to the line 8x = 9y are -

T
[JEE 99]

2 1  2 1  2 1 2 1
(A)  , (B)   , (C)   ,   (D)  ,  
5 5  5 5  5 5 5 5

R I N
S P
JE E
ANSWER KEY
Que. 1 2 3 4 5 6 7 8 9 10
Ans. C A A B C B A B B D
Que. 11 12 13 14 15 16 17 18 19 20
Ans. C A B B C B B B C A,B, C,D
Que. 21 22 23 24 25
Ans. B,C A,B,D A,B, C,D A ,C , D B,D

Page 143
JEE Sprint Mathematics
EXTRA PRACTICE QUESTIONS ON ELLIPSE

SELECT THE CORRECT ALTERNATIVES (ONE OR MORE THAN ONE CORRECT ANSWERS)

x2 y2
1. x – 2y + 4 = 0 is a common tangent to y2 = 4x &   1 . Then the value of b and the other common
4 b2
tangent are given by -

(A) b  3 ; x  2 y  4  0 (B) b  3 ; x  2y  4  0

(C) b  3 ; x  2 y  4  0 (D) b  3 ; x  2y  4  0

2. The tangent at any point P on a standard ellipse with foci as S & S' meets the tangents at the vertices A & A' in

T
the points V & V', then -

(A) l( AV ). l( A ' V ')  b 2 (B) l  AV  . l  A ' V '  a 2

I N
(C)  V ' SV  90  (D) V'S' VS is a cyclic quadrilateral

3. The area of the rectangle formed by the perpendiculars from the centre of the standard ellipse to the tangent

R
and normal at its point whose eccentric angle is /4 is -

P
(a 2  b 2 )ab (a 2  b 2 )ab (a 2  b 2 ) (a 2  b 2 )
(A) (B) (C) (D) 2
a 2  b2 a 2  b2 ab(a 2  b 2 ) (a  b 2 )ab

S
4. Q is a point on the auxiliary circle of an ellipse. P is the corresponding point on ellipse. N is the foot of
perpendicular from focus S, to the tangent of auxiliary circle at Q. Then -
(A) SP = SN (B) SP = PQ (C) PN = SP (D) NQ = SP

E
x2 y2
5. The line, lx + my + n = 0 will cut the ellipse   1 in points whose eccentric angles differ by /2 if -
a 2 b2

E
(A) x2 l2 + b2 n2 = 2m2 (B) a2 m2 + b2 l = 2n2

J
(C) a2 l2 + b2m2 = 2n2 (D) a2 n2 + b2 m2 = 2l

6. A circle has the same centre as an ellipse & passes through the foci F1 & F2 of the ellipse, such that the two
curves intersect in 4 points. Let ‘P’ be any one of their point of intersection. If the major axis of the ellipse is 17
& the area of the triangle PF1F2 is 30, then the distance between the foci is -

(A) 11 (B) 12 (C) 13 (D) none


2 2
x y
7. The normal at a variable point P on an ellipse 2
 2  1 of eccentricity e meets the axes of the ellipse in Q
a b
and R then the locus of the mid-point of QR is a conic with an eccentricity e' such that -
(A) e' is independent of e (B) e' = 1
(C) e' = e (D) e' = 1/e

x2 y2
8. The length of the normal (terminated by the major axis) at a point of the ellipse 2
+ 2 = 1 is -
a b
b b b
(A) (r + r 1 ) (B) |r – r 1 | (C) rr1 (D) independent of r, r 1
a a a
where r and r 1 are the focal distance of the point.

Page 144
JEE Sprint Mathematics
9. Point 'O' is the centre of the ellipse with major axis AB and minor axis CD. Point F is one focus of the ellipse.
If OF = 6 and the diameter of the inscribed circle of triangle OCF is 2, then the product (AB)(CD) is equal
to -
(A) 65 (B) 52 (C) 78 (D) none

x2 y2
10. If P is a point of the ellipse   1 , whose foci are S and S'. Let PSS '   and PS ' S   , then -
a 2 b2
(A) PS + PS' = 2a, if a > b (B) PS + PS' = 2b, if a < b

  1e   a 2  b2
(C) tan tan  (D) tan tan  [a  a 2  b 2 ] when a > b
2 2 1e 2 2 b2

T
x2 y2
11. If the chord through the points whose eccentric angles are    on the ellipse,   1 passes through
a 2 b2
the focus, then the value of tan (/2) tan (/2) is -

N
e 1 e 1 1e 1e

I
(A) (B) (C) (D)
e 1 e 1 1e 1e
12. If point P( + 1, ) lies between the ellipse 16x 2 + 9y 2 – 16x = 0 and its auxiliary circle, then -

R
(A) [] = 0 (B) [] = –1
(C) no such real  exist (D) [] = 1
where [.] denotes greatest integer function.

P
x2 y
13. If latus rectum of an ellipse  2  1 {0< b < 4}, subtend angle 2 at farthest vertex such that

S
16 b
cosec = 5 , then -
1
(A) e = (B) no such ellipse exist

E
2
(C) b = 2 3 (D) area of  formed by LR and nearest vertex is 6 sq. units

E
14. If x 1, x 2 , x 3 as well as y 1 , y 2 , y 3 are in G.P. with the same common ratio, then the points (x 1 , y 1 ),

J
(x 2, y 2 ) & (x 3 , y 3 ) - [JEE 99]
(A) lie on a straight line (B) lie on an ellipse
(C) lie on a circle (D) are vertices of a triangle.

ANSWER KEY
Que. 1 2 3 4 5 6 7 8 9 10
Ans. A A ,C , D A A C C C C A A, B, C
Que. 11 12 13 14
Ans. A,B A,B A ,C , D A

Page 145
JEE Sprint Mathematics
HYPERBOLA
SELECT THE CORRECT ALTERNATIVE (ONLY ONE CORRECT ANSWER)
1. The eccentricity of the hyperbola 4x2 – 9y2 – 8x = 32 is -
5 13 4 3
(A) (B) (C) (D)
3 3 3 2
2. The locus of the point of intersection of the lines 3x  y  4 3k  0 and 3kx  ky  4 3  0 for different
values of k is -
(A) ellipse (B) parabola (C) circle (D) hyperbola
3
3. If the latus rectum of an hyperbola be 8 and eccentricity be then the equation of the hyperbola can be -
5

T
(A) 4x2 – 5y2 = 100 (B) 5x2 – 4y2 = 100 (C) 4x2 + 5y2 = 100 (D) 5x2 + 4y2 = 100
4. If the centre, vertex and focus of a hyperbola be (0,0), (4, 0) and (6,0) respectively, then the equation of the
hyperbola is –

N
(A) 4x2 – 5y2 = 8 (B) 4x2 – 5y2 = 80 (C) 5x2 – 4y2 = 80 (D) 5x2 – 4y2 = 8
5. The equation of the hyperbola whose foci are (6,5), (–4, 5) and eccentricity 5/4 is–

I
(x  1)2 (y  5)2 x2 y 2 (x  1)2 (y  5)2 (x  1)2 (y  5)2
(A)  1 (B)  1 (C)   1 (D)  1
16 9 16 9 16 9 4 9

R
6. The vertices of a hyperbola are at (0, 0) and (10,0) and one of its foci is at (18,0). The possible equation of the
hyperbola is –

P
x2 y2 (x  5)2 y2 x 2 (y  5)2 (x  5) 2 (y  5) 2
(A)  1 (B)  1 (C)  1 (D)  1
25 144 25 144 25 144 25 144

S
7. The length of the transverse axis of a hyperbola is 7 and it passes through the point (5, –2). The equation of the
hyperbola is –
4 2 196 2 49 2 51 2 4 2 51 2
x  y 1 x  y 1 x  y 1

E
(A) (B) (C) (D) none of these
49 51 4 196 49 196
x2 y2
8. AB is a double ordinate of the hyperbola – = 1 such that AOB (where 'O' is the origin) is an equilateral

E
a2 b2
triangle, then the eccentricity e of the hyperbola satisfies -

J
2 2 2
(A) e > 3 (B) 1 < e < (C) e = (D) e >
3 3 3
9. The equation of the tangent lines to the hyperbola x 2 – 2y 2 = 18 which are perpendicular to the line
y = x are -
(A) y = x ± 3 (B) y = – x ± 3 (C) 2x + 3y + 4 = 0 (D) none of these
x2 y2 y2 x2
10. The equations to the common tangents to the two hyperbolas 2
 2  1 and 2  2  1 are -
a b a b
(A) y = ± x ± b2  a 2 (B) y = ± x ± (a2 – b2) (C) y = ± x ± a 2  b 2 (D) y = ± x ± a 2  b 2
11. Locus of the feet of the perpendiculars drawn from either foci on a variable tangent to the hyperbola
16y 2 – 9x 2 = 1 is -
(A) x 2 + y 2 = 9 (B) x 2 + y 2 = 1/9 (C) x 2 + y 2 = 7/144 (D) x 2 + y 2 = 1/16
2 2 2 2
12. The ellipse 4x + 9y = 36 and the hyperbola 4x – y = 4 have the same foci and they intersect at right
angles then the equation of the circle through the points of intersection of two conics is -
(A) x 2 + y 2 = 5 (B) 5 (x 2 + y 2 ) – 3x – 4y = 0
(C) 5 (x 2 + y 2 ) + 3x + 4y = 0 (D) x 2 + y 2 = 25
13. The equation of the common tangent to the parabola y2 = 8x and the hyperbola 3x2 – y2 = 3 is -
(A) 2x ± y + 1 = 0 (B) x ± y + 1 = 0 (C) x ± 2y + 1 = 0 (D) x ± y + 2 = 0

Page 146
JEE Sprint Mathematics
14. Equation of the chord of the hyperbola 25x2 – 16y2 = 400 which is bisected at the point (6, 2) is -
(A) 16x–75y=418 (B) 75x–16y=418 (C) 25x–4y = 400 (D) none of these
15. The asymptotes of the hyperbola xy – 3x – 2y = 0 are-
(A) x – 2 = 0 and y – 3 = 0 (B) x – 3 = 0 and y – 2 = 0
(C) x + 2 = 0 and y + 3 = 0 (D) x + 3 = 0 and y + 2 = 0
x2 y2
16. If the product of the perpendicular distances from any point on the hyperbola – = 1 of eccentricity
a2 b2
e = 3 on its asymptotes is equal to 6, then the length of the transverse axis of the hyperbola is -
(A) 3 (B) 6 (C) 8 (D) 12
17. If the normal to the rectangular hyperbola xy = c 2 at the point 't' meets the curve again at 't1 ' then t3 t1 has
the value equal to -
(A) 1 (B) –1 (C) 0 (D) none

T
18. Area of triangle formed by tangent to the hyperbola xy = 16 at (16, 1) and co-ordinate axes equals -
(A) 8 (B) 16 ( C) 32 (D) 64
19. Locus of the middle points of the parallel chords with gradient m of the rectangular hyperbola
xy = c2 is -

N
(A) y + mx = 0 (B) y – mx = 0 (C) my – x = 0 (D) my + x = 0

I
 x2 y2
20. Let P (asec, btan) and Q (asec, btan), where     , be two points on the hyperbola   1.
2 a2 b2

R
If (h,k) is the point of intersection of the normals at P & Q, then k is equal to - [JEE 99]
a 2  b2  a 2  b2  a 2  b2  a 2  b2 
(A) (B)   a  (C) (D)   b 

P
a b
21. If x = 9 is the chord of contact of the hyperbola x2–y 2 = 9, then the equation of the corresponding pair of
tangents, is - [JEE 99]

S
2 2 2 2 2 2 2 2
(A) 9x –8y + 18x –9 = 0 (B) 9x –8y –18x +9 = 0 (C) 9x –8y –18x –9 = 0 (C) 9x –8y + 18x +9 = 0

SELECT THE CORRECT ALTERNATIVES (ONE OR MORE THAN ONE CORRECT ANSWERS)

E
2 2 . Consider the hyperbola 3x 2 – y 2 – 24x + 4y – 4 = 0 -
(A) its centre is (4, 2) (B) its centre is (2, 4)
(C) length of latus rectum = 24 (D) length of latus rectum = 12

JE
x2 y2
23. Let an incident ray L1 = 0 gets reflected at point A(–2, 3) on hyperbola 2
  1 & passes through focus
a b2
S(2, 0), then -
(A) equation of incident ray is x + 2 = 0 (B) equation of reflected ray is 3x + 4y = 6
(C) eccentricity, e = 2 (D) length of latus rectum = 6
24. 2 2 2
For the curve 5(x – 1) + 5(y – 2) = 3(2x + y – 1) which of the following is true -

(A) a hyperbola with eccentricity 3 (B) a hyperbola with directrix 2x + y – 1 = 0


(C) a hyperbola with focus (1, 2) (D) a hyperbola with focus (2, 1)
25. The equation of common tangent of hyperbola 9x 2 – 9y 2 = 8 and the parabola y 2 = 32x is/are -
(A) 9x + 3y – 8 = 0 (B) 9x – 3y + 8 = 0 (C) 9x + 3y + 8 = 0 (D) 9x – 3y – 8 = 0

ANSWER KEY
Que. 1 2 3 4 5 6 7 8 9 10
Ans. B D A C A B C D B C
Que. 11 12 13 14 15 16 17 18 19 20
Ans. D A A B A B B C A D
Que. 21 22 23 24 25
Ans. B A,C A,B, C,D A, B, C B,C

Page 147
JEE Sprint Mathematics
EXTRA PRACTICE QUESTIONS ON HYPERBOLA
SELECT THE CORRECT ALTERNATIVES (ONE OR MORE THAN ONE CORRECT ANSWERS)
1. Variable circles are drawn touching two fixed circles externally, then locus of centre of variable circle is -
(A) parabola (B) ellipse ( C) hyperbola (D) circle
x2 y2
2. The locus of the mid points of the chords passing through a fixed point ( ) of the hyperbola,   1 is -
a 2 b2
     
(A) a circle with centre  ,  (B) an ellipse with centre  , 
2 2 2 2
     
(C) a hyperbola with centre  ,  (D) straight line through  , 
2 2 2 2

T
3. The locus of the foot of the perpendicular from the centre of the hyperbola xy = c2 on a variable tangent is :
(A) (x2 – y2)2 = 4c2 xy (B)(x2 + y2)2 = 2c2 xy (C) (x2 – y2) = 4c2xy (D) (x2 + y2)2 = 4c2 xy

N
4. The equation to the chord joining two points (x1, y1) and (x2, y2) on the rectangular hyperbola xy = c2 is -

I
x y x y
(A) x  x  y  y  1 (B) x  x  y  y  1
1 2 1 2 1 2 1 2

x y x y

R
(C) y  y  x  x  1 (D) y  y  x  x  1
1 2 1 2 1 2 1 2

5. The equation 9x2 – 16y2 – 18x + 32y – 151 = 0 represent a hyperbola -

P
(A) The length of the transverse axes is 4 (B) Length of latus rectum is 9
21 11

S
(C) Equation of directrix is x = and x = – (D) none of these
5 5
6. From the points of the circle x2 + y2 = a2, tangents are drawn to the hyperbola x2 – y2 = a2; then the locus of the
middle points of the chords of contact is -

E
(A) (x2 – y2)2 = a2 (x2 + y2) (B) (x2 – y2)2 = 2a2 (x2 + y2)
(C) (x2 + y2)2 = a2 (x2 – y2) (D) 2(x2 – y2)2 = 3a2 (x2 + y2)

E
7. The tangent to the hyperbola xy = c2 at the point P intersects the x-axis at T and the y-axis at T'. The normal

J
to the hyperbola at P intersects the x-axis at N and the y-axis at N'. The areas of the triangles PNT and

1 1
PN ' T ' are  and ' respectively, then + is -
 '
(A) equal to 1 (B) depends on t (C) depends on c (D) equal to 2

8. The tangent to the hyperbola, x 2 – 3y 2 = 3 at the point ( 3, 0) when associated with two asymptotes

constitutes -
(A) isosceles triangle (B) an equilateral triangle

(C) a triangles whose area is 3 sq. units (D) a right isosceles triangle.

x2 y2
9. The asymptote of the hyperbola   1 form with any tangent to the hyperbola a triangle whose area is
a 2 b2
a2 tan  in magnitude then its eccentricity is -
(A) sec  (B) cosec  (C) sec2  (D) cosec2 
10. From any point on the hyperbola H 1 : (x 2 /a 2 ) – (y 2 /b 2 ) = 1 tangents are drawn to the hyperbola
2 2 2 2
H2 : (x /a ) – (y /b ) = 2. The area cut-off by the chord of contact on the asymptotes of H2 is equal to -
(A) ab/2 (B) ab (C) 2 ab (D) 4 ab

Page 148
JEE Sprint Mathematics
x y
11. The tangent at P on the hyperbola (x2/a2) – (y2/b2) = 1 meets the asymptote   0 at Q. If the locus of the
a b
mid point of PQ has the equation (x2/a2) – (y2/b2) = k, then k has the value equal to -
(A) 1/2 (B) 2 (C) 3/4 (D) 4/3

x2 y2 
12. If  is the angle between the asymptotes of the hyperbola   1 with eccentricity e, then sec can be -
a2 b2 2

e
(A) e (B) e/2 (C) e/3 (D) 2
e 1
13. If (5, 12) and (24, 7) are the focii of a conic passing through the origin, then the eccentricity of conic is -

T
(A) 386 / 12 (B) 386 / 13 (C) 386 / 25 (D) 386 / 38

14. The point of contact of line 5x + 12y = 9 and hyperbola x2 – 9y2 = 9 will lie on

N
(A) 4x + 15y = 0 (B) 7x + 12y = 19 (C) 4x + 15y + 1 = 0 (D) 7x – 12y = 19

I
15. Equation (2 + )x2 – 2xy + ( – 1)y2 – 4x – 2 = 0 represents a hyperbola if -
(A)  = 4 (B)  = 1 (C)  = 4/3 (D)  = –1

R
16. If a real circle will pass through the points of intersection of hyperbola x2 – y2 = a 2 & parabola y = x 2 ,
then -

P
 1 1
(A) a  (–1, 1) (B) a    ,  – {0}
 2 2

S
 1 1
(C) area of circle =  – a 2 ; a   ,   {0} (D) area of circle =  – 4a 2
 2 2
x2 y2 3

E
17. If least numerical value of slope of line which is tangent to hyperbola   1 is , a  R0
a (a  a  a) 2 3 2 2 4
is obtained at a = k. For this value of 'a', which of the following is/are true -

E
1 1 9 5

J
(A) a = – (B) a = (C) LR = (D) e =
2 2 16 4
18. If the normal at point P to the rectangular hyperbola x 2 – y 2 = 4 meets the transverse and conjugate axes
at A and B respectively and C is the centre of the hyperbola, then -
(A) PA = PC (B) PA = PB (C) PB = PC (D) AB = 2PC
19. If the circle x2 + y2 = a2 intersects the hyperbola xy = c2 in four points P (x1, y 1), Q(x2,y 2), R(x3,y 3), S(x 4, y4),
then - [JEE 98]
(A) x1+ x2 + x3 + x4 = 0 (B) y1 + y2 +y3 + y4 = 0 (C) x1 x2 x3 x4 = c4 (D) y1 y2 y3 y4 = c4
20. The curve described parametrically by, x = t2 + t + 1, y = t2 – t +1 represents - [JEE 99]
(A) a pair of straight lines (B) an ellipse (C) a parabola (D) a hyperbola

ANSWER KEY
Que. 1 2 3 4 5 6 7 8 9 10
Ans. C C D A C A C A, B, C A D
Que. 11 12 13 14 15 16 17 18 19 20
Ans. C A,D A,D A,B B,D B,C A ,C , D A,B, C,D A,B, C,D C

Page 149
JEE Sprint Mathematics
INDEFINITE INTEGRATION

SELECT THE CORRECT ALTERNATIVE (ONLY ONE CORRECT ANSWER)


2 sin x  sin 2 x
1. If f(x) =  dx , x  0 then lim f'(x) is equal to-
x3 x 0

(A) 0 (B) 1 (C) 2 (D) 1/2


x 3x
2.  4 sin x cos 2 cos 2 dx is equal to -

1 1 1 1
(A) cos x  cos 2x  cos 3x  c (B) cos x  cos 2x  cos 3x  c
2 3 2 3

T
1 1 1 1
(C) cos x  cos 2x  cos 3x  c (D) cos x  cos 2x  cos 3x  c
2 3 2 3
8 x  13
3.  dx is equal to -

N
4x 7

I
1 1
(A) (8x + 11) 4 x  7 + c (B) (8x + 13) 4 x  7 + c
6 6

R
1 1
(C) (8x + 9) 4 x  7 + c (D) (8x + 15) 4 x  7 + c
6 6

P
 cos 8 x  sin 8 x 
4.   2 2  dx equals -
 1  2 sin x cos x 

S
sin 2 x sin 2 x cos 2 x cos 2 x
(A)  c (B) c (C) c (D)  c
2 2 2 2
x

E
5. Primitive of 3 4
w.r.t. x is -
x 4
1 

E
1 1 1 1
3 1 3 3 1 3 4 1 3 4 1 3

J
(A)  1  4  c (B)   1  4  c (C)  1  4  c (D)   1  4  c
4 x 1  4 x 1  3 x 1  3 x 1 
2
6.  (1  2x  3x  4x 3 ......) dx (|x| < 1) -

(A) (1 + x)–1 + c (B) (1 – x)–1 + c (C) (1 + x)–2 + c (D) none of these


x dx
7.  3
is equal to -
1x  2
1  x  2

1
(A)
2

ln 1  1  x 2  c  (B) 2 1  1  x 2  c


(C) 2 1  1  x 2  c  (D) none of these

8.
n x
x dx equals -
1  n x

2 2
(A) 1  n x ( ln x  2)  c (B) 1  n x ( ln x  2)  c
3 3
1
(C) 1  n x ( ln x  2)  c (D) 2 1  n x (3 n x  2)  c
3

Page 150
JEE Sprint Mathematics
x4  1 B
9. If  x x 2
dx  A  n x  + c, where c is the constant of integration then :
2
1  1  x2

(A) A = 1; B = –1 (B) A = –1; B = 1 (C) A = 1; B = 1 (D) A = –1 ; B = –1


3/2
 x 
10.
  5 
1  x 
dx equals -

2 x5 2 x 2 1
(A) c (B) c (C) c (D) none of these
5 1  x5 5 1  x5 5 1  x5

11.
 sin x.cos x.cos 2x.cos 4 x.cos 8x.cos16x dx equals -

T
sin 16 x cos 32 x cos 32x cos 32 x
(A) c (B)  c (C) c (D)  c
1024 1024 1096 1096
12. Identify the correct expression

N
(A) x  nx dx  x 2 n| x| x 2  c (B) x  n x dx  xe x  c

I
dx 1 x
(C) x  e x dx  xe x  c x (D)   tan 1    c
2
a x 2 a a 

R

n x  1  x 2  dx equals -
13.  x.

P
2
1x

S
x x
(A) 1  x n x  1  x
2
 2
x c (B) 2

. n 2 x  1  x 2 
1  x2
c 
x x
  

E
(C) 2 . n x  1  x 
2 2

1  x2
c (D) 
1  x 2 n x  1  x 2  x  c

E
dx 1
 (x  2) (x 2  1) = a  n(1 + x ) + btan x + 5  n|x + 2| + C then -
2 –1
14. If

J
1 2 1 2 1 2 1 2
(A) a = – , b = – (B) a = , b = – (C) a = – , b = (D) a = , b =
10 5 10 5 10 5 10 5

2
x  1
15. x 4
 2x 2  1
dx equals -

x3 x x5  x3  x  3 x 5  4 x 3  3x  3
(A) x 2 c (B) c (C) c (D) None of these
3 x 1 
3 x2  1  
3 x2  1 
16. z x2  4
x 4  24 x 2  16
dx equals -

1
tan
 x2  4
1 
    c 
1
cot
 x2  4
1 
    c
(A) 4  4x  (B) 4  x 
   

1  4 x2  4
1 
    c 1  x2  4
1 
    c
(C)  4 cot  x  (D) 4 cot  x 
   

Page 151
JEE Sprint Mathematics
x4  4
17.
x 2
4  x2  x 4
dx equals-

4  x2  x 4 4  x2  x 4 4  x2  x 4
(A) c (B) 2
4x x c 4 (C) c (D) c
x 2 2 x

x9
18.  (x 2  4 ) 6 dx is equal to -
5 5
1  1  1  1 
(A)  4 2  + c (B)  4 2  + c
5x  x  5  x 
1 1

T
(C) (1 + 4x 2) –5 +c (D) (1 + 4x –2 ) –5 + c
10x 40
dx –1  x
19. If  5  4 cos x = a tan  b tan  + c, then-

N
 2

I
2 1 2 1
(A) a = , b = – (B) a = , b =
3 3 3 3
2 1 2 1

R
(C) a = – , b = (D) a = – , b = –
3 3 3 3
SELECT THE CORRECT ALTERNATIVES (ONE OR MORE THAN ONE CORRECT ANSWERS)

P
20. Primitive of 1  2 tan x  sec x  tan x  w.r.t.x is -

(A) n sec x  n sec x  tan x  c (B) n sec x  tan x  n sec x  c

S
x x
(C) 2 n sec  tan  c (D) n 1  tan x(sec x  tan x)  c
2 2

E
21.  sin 2x dx equals -

cos 2 x sin 2 x cos 2 x cos 2x

E
(A)  c (B) c (C)  c (D) c
2 2 2 2

J
dx
22.   1  equals-
x 3 1  2 
 2x 

(A) n 2x  1  2 n| x|  c (B) n 2x  1  2 n| x|  c


2 2

1
(C) n 2x  1  n(x )  n2  c (D) n 1  c
2 2
2x 2
3x 3x
23. If  e cos 4 x dx  e (A sin 4 x  B cos 4 x)  c , then -
(A) 4A = 3B (B) 2A = 3B (C) 3A = 4B (D) 4A + 3B = 1

ANSWER
ANSWER KEY
KEY
Que. 1 2 3 4 5 6 7 8 9 10
Ans. B B A B B B B A C A
Que. 11 12 13 14 15 16 17 18 19 20
Ans. B C A C D A A D B A,B,D
Que. 21 22 23
Ans. A,B,C B,C,D C,D

Page 152
JEE Sprint Mathematics
EXTRA PRACTICE QUESTIONS ON INDEFINITE INTEGRATION

SELECT THE CORRECT ALTERNATIVES (ONE OR MORE THAN ONE CORRECT ANSWERS)

1.
z cot x  tan x
b
2 cos x  sin x g dx
equals -

–1 –1
(A) sec  sin x  cos x   c (B) sec  sin x  cos x   c

(C) n  sin x  cos x   sin 2x  c (D) n  sin x  cos x   sin 2x  c

z sin x  4 sin 3 x  6 sin 5 x  3 sin 7 x

T
2. dx equals -
sin 2 x  3 sin 4 x  3 sin 6 x

(A) –2sinx+c (B) 2sinx+c (C) –2cosx+c (D) 2cosx+c

N
1  x7
3.  dx equals -

I
x(1  x 7 )
2 2
(A) n| x|  n 1  x  c
7
(B) n| x|  n 1  x 7  c
7

R
4
2 2
(C) n| x|  n 1  x  c
7
(D) n| x|  n 1  x 7  c
7 4

P
x 3 dx

S
4.  2
is equal to -
1x

1 1
1  x 2 (2 + x 2) + c 1  x 2 (x 2 – 1) + c

E
(A) (B)
3 3

1 1

E
(C) (1 + x 2 ) 3/2 + c (D) 1  x 2 (x 2 – 2) + c
3 3

J
 sin (  n x) dx is equal to -
2
5.

x x
(A) (5 + 2sin(2nx) + cos(2nx)) + c (B) (5 + 2sin(2nx) – cos(2nx)) + c
10 10

x x
(C) (5 – 2sin(2nx) – cos(2nx)) + c (D) (5 – 2sin(2nx) + cos(2nx)) + c
10 10

x 2  cos 2 x
6.  1  x2 .cosec 2x dx is equal to -

(A) cotx + tan–1x + c (B) cotx – tan–1x + c (C) –cotx – tan–1x + c (D) tan–1 x – cotx + c

 x2  3 
7.  ex   dx , equals -
  x  3 2 
 

x x x 6  x 6  x 3
(A) e . c (B) e  2  c (C) e  1  c (D) e . c
x 3  x  3   x  3  x 3

Page 153
JEE Sprint Mathematics
tan 1 x
8. e (1  x  x 2 ). d(cot 1 x ) is equal to -

1 1 1 1
(A) – e tan x
+ c (B) e tan x
+ c (C) –x. e tan x
+ c (D) x. e tan x
+ c

x (1  n.x n 1  x 2 n )
9. e dx is equal to -
(1  x n ) 1  x 2 n

1  xn 1  xn 1  xn 1  xn
(A) e x + c (B) e x + c (C) – e x + c (D) – e x + c
1  xn 1  xn 1  xn 1  xn

x4 2
10. e (x  x 3  2x 5 ) e x dx is equal to -

T
1 x2 x 4 1 2 x4 1 x2 x 4 1 2 x2 x 4
(A) xe . e  c (B) x e c (C) e .e c (D) x e .e c
2 2 2 2

I N
3x 4  1
11. Primitive of 2
w.r.t. x is -
x 4
 x 1 

R
x x x 1 x 1
(A) 4
c (B)  4
c (C) 4
c (D)  4
c
x  x 1 x  x 1 x  x 1 x  x 1

P
dx
12. x 4
[x (x 5  1)]1 / 3
equals -

S
2/3 2/3 2/3 2/3
3  x5  1  3  x5  1  3  x5  1  3  x 5  1 
(A)   c (B)   c (C)   c (D)  5  c
2  x5  10  x 5  4  x5  5  x 

E
sin x
13.  sin 4 x dx is equal to -

JE
1 1  2 sin x 1 1  sin x 1 1  2 sin x 1 1  sin x
(A) n + n + c (B) n – n +c
2 2 1  2 sin x 8 1  sin x 2 2 1  2 sin x 8 1  sin x

1 1  2 sin x 1 1  sin x 1 1  2 sin x 1 1  sin x


(C) n + n + c (D) n – n +c
4 2 1  2 sin x 8 1  sin x 4 2 1  2 sin x 8 1  sin x

d
14. The value of integral  cos 3
 sin 2 
can be expressed as irrational function of tan as -

2 2
(A)
5
 tan 2   5  tan   c (B)
5
tan 2   5  tan   c

2 2
(C)
5

tan 2   5  tan   c (D)
5
 tan 2
5  tan   c

3 sin x  2 cos x
15. If  3 cos x  2 sin x dx = ax + bn [2sinx + 3cosx| + c, then -

12 15 17 6 12 15 17 1
(A) a = – , b = (B) a = , b = (C) a = , b = – (D) a = – , b = –
13 39 13 13 13 39 13 192

Page 154
JEE Sprint Mathematics
x 1
16.  dx is equal to -
x x 1

(A) n x  x 2  1 – tan –1 x + c (B) n x  x 2  1 – tan –1 x + c

(C)  n x  x 2  1 – sec –1 x + c (D)  n x  x 2  1 – sec –1 x + c

dx
17.  is equal to -
(1  x ) x  x 2

2( x  1) 2(1  x ) 2( x  1) 2(1  x )
(A) + c (B) + c (C) + c (D) + c
1x 1x x 1

T
x 1

1 1 1 
18. Let f'(x) = 3x 2 .sin – xcos , x  0, f(0) = 0, f   = 0, then which of the following is/are not correct.
x x 

I N
(A) f(x) is continuous at x = 0 (B) f(x) is non-differentiable at x = 0
(C) f'(x) is discontinuous at x = 0 (D) f'(x) is differentiable at x = 0

R
1 x 1
19.  2
n dx equals -
x 1 x 1

P
1 2 x 1 1 2 x 1 1 2 x 1 1 2 x 1
(A) n c (B) n c (C) n c (D) n c

S
2 x 1 4 x 1 2 x 1 4 x 1

dx
20.  equals, where x   1 ,1 -
xx 2 2 

E
1
(A) 2 sin 1 x  c (B) sin (2 x  1)  c

E
1
(C) c  cos (2 x  1) (D) cos 1 2 x  x 2  c

J
sin 2x
21.  sin 4
x  cos 4 x
dx is equal to -

1 2

(A) cot cot x  c  1 2

(B)  cot tan x  c  1 2

(C) tan tan x  c  (D)  tan 1  cos 2 x   c

ANSWER
ANSWER KEY
KEY
Que. 1 2 3 4 5 6 7 8 9 10
Ans. A B C D C C C C B C
Que. 11 12 13 14 15 16 17 18 19 20
Ans. B B D C C D A B,C,D B,D A,B, C,D
Que. 21
Ans. A,B,C,D

Page 155
JEE Sprint Mathematics
DEFINITE INTEGRATION
SELECT  THE  CORRECT  ALTERNATIVE  (ONLY  ONE  CORRECT  ANSWER)
/3
cos x 3 2 3
1. If   3  4 sin x
dx  k log 
 3   then  k is-

0

1 1 1 1
(A)  (B)  (C)  (D) 
2 3 4 8
ee
ee
dx
2.  e x nx.n( nx).n( n( nx))
  equals  -
ee

T
(A) 1 (B) 1/e (C) e  –  1 (D) 1  +  e


x 1
3. The  value  of  the  definite  integral   (e  e 3  x ) 1 dx  is

N
1

I
  1  1 1  
(A) (B)  (C)    tan  (D)
4e 2 4e e2 2 e 2e 2

R
e

  (x  1)e .nx  dx   is  -


x
4. The  value  of  the  definite  integral 
1

P
(A) e (B) ee  +  1 (C) ee(e  –  1) (D) ee(e  –  1)  +  e

S
1 2
5. Let  a,  b,  c    be  non-zero  real  numbers  such  that  ;  (1  cos 8 x)(ax 2  bx  c)dx  (1  cos 8 x)(ax 2  bx  c)dx ,
 
0 0

then  the  quadratic  equation  ax2  +  bx  +  c  =  0  has  -

E
(A) no  root  in  (0,2) (B) atleast  one  root  in  (0,2)
(C) a  double  root  in  (0,2) (D) none

E
1
2A

J
 x  1
6. If  f(x)  =  A  sin    + B,  f'    2   and   f(x)dx  ,  then  the  constant  A  and  B  are-
 2   2 0

  2 4 4
(A)   and  (B)    and  3 (C) 0  and (D)    and  0
2 2   

/4
7. If  In  =   tan n xdx then lim n  (In  +  In–2)  =
n 
0

(A) 1 (B) 1/2 (C)  (D) 



x tan 1 x
8.  (1  x 2 2
dx
0 )
   
(A)  (B)  (C)  (D) 
2 4 6 8
e
f(x) 1
9. Suppose  f,  f'  and  f''  are  continuous  on  [0,  e]  and  that  f'(e)  =  f(e)  =  f(1)  =  1  and   x 2
dx  ,    then  the  value  of
2
1

 f ''(x) n x dx   equals  -
1
(A) 0 (B) 1 (C) 2 (D) none  of  these

Page 156
JEE Sprint Mathematics
2
1  1
10.  sin  x   dx   has  the  value  equal  to  -
x  x
1/2

3 5
(A) 0 (B)  (C)  (D) 2
4 4
2
4   log x 2  
11.  log x 2  n2  dx =

2  
(A) 0 (B) 1 (C) 2 (D) 4
3
sin x sin 2x
12. Suppose  that  F(x)  is  an  antiderivative  of  f(x)  =  ,  x  >  0  then   dx  can  be  expressed  as  -
x 1
x

T
1 1
(A) F(6)  –  F(2) (B)  (F(6)  –  F(2)) (C)  (F(3)  –  F(1)) (D) 2(F(6)  –  F(2))
2 2

N

 1  nx
13.  f  x  x  . dx

I
0
x

1
(A) is  equal  to  zero (B) is  equal  to  one (C) is  equal  to (D) can  not  be  evaluated

R
2
1
14. Integral  | sin 2 x| dx   is  equal  to  -

P
0

1 1 2
(A) 0 (B)   (C)  (D) 

S
  
3
x
15.  dx 
2 (5  x)  x

E
1 1 1
(A)  (B)  (C)  (D) none
2 3 5

E
 2
For  any  integer  n  the  integral   e cos x
cos 3 (2n  1) xdx   has  the  value

J
16.
0

(A)  (B) 1 (C) 0 (D) none  of  these


3
(x  2)2
17.  2x 2  10x  53 dx is  equal  to  -
2

(A) 2 (B) 1 (C) 1/2 (D) 5/2


1 2
x
18. The  value  of  the  definite  integral   (1  e ) dx   is -
0

(A) –1 (B) 2 (C) 1  +  e –1 (D) none  of  these



2
19.  (cos ax  sin bx) dx   where  a  and  b  are  integer  is  equal  to  -


(A) – (B) 0 (C)  (D) 2



2
20. The  value  of   (1  x ) sin x cos 2 xdx   is  -


3 7
(A) 0 (B)    (C)  2  –  3 (D)   2 3
3 2

Page 157
JEE Sprint Mathematics
2

21. The  value  of   [2 sin x]dx ,  where  [  ]  represents  the  greatest  integer  function  is  -

5 5
(A)   (B)  – (C)  (D) –2
3 3
f(x)

22. If   t 2 dt  x cos x ,  then  f'(9)


0

1 1 1
(A) is  equal  to  – (B) is  equal  to  – (C) is  equal  to (D) is  non  existent
9 3 3
ƒ( x )
2t
23. Let  ƒ  :  R    R  be  a  differentiable  function  and  ƒ  (1)  =  4.  Then  the  value  of  lim 4 dt   is  -
x 1 x 1

T
[JEE  1990]
(A) 8ƒ '(1) (B) 4ƒ '(1) (C) 2ƒ '(1) (D) ƒ '(1)
x

N
24. If  g(x)  =  cos 4 t dt ,  then  g(x  +  )  equals  - [JEE  1997]
0

I
g(x)
(A) g(x)  +  g() (B) g(x)  –  g() (C) g(x)g() (D) 
g( )

R
n  V
1  cos 2x 
25. For  n    N,  the  value  of  the  definite  integral   dx   where   V     is  -
0
2 2

P
(A) 2n  +  1  –  cosV (B) 2n  –  sinV (C) 2n  +  2  –  sinV (D) 2n  +  1  –  sinV

S
SELECT  THE  CORRECT  ALTERNATIVES  (ONE  OR  MORE  THAN  ONE  CORRECT  ANSWERS)

x
26.  (1  x)(1  x 2
dx
0 )

E
 
(A)  (B) 
4 2

E

dx
 (1  x)(1  x

J
(C) is  same  as 2 (D) cannot  be  evaluated
0 )
27. Which  of  the  following  are  true ?
 a  a a a
 2
(A)   x.f(sin x)dx  .  f(sin x)dx (B)   f(x )dx  2. f(x 2 )dx
a
2 a a 0
n  bc b
2 2
(C)   f(cos x)dx  n. f(cos x)dx (D)   f(x  c)dx   f(x)dx
0 0 0 c
x
dt
28. Let  f(x)  =   1  t4
and  g  be  the  inverse  of  f.  Then  the  value  of  g'(0)  is  -
2

(A) 1 (B) 17 (C)  17 (D) none  of  these


x
nt
29. If  f(x)  =   1  t dt  where  x  >  0  then  the  value(s)  of  x  satisfying  the  equation,  f(x)  +  f(1/x)  =  2  is  -
1

(A) 2 (B) e (C) e–2 (D) e2


r 4 n
n
30. The  value  of  Lim  2
is  equal  to  -
r 3 r  4 n 
n 
r 1

1 1 1 1
(A)  (B)  (C)  (D) 
35 14 10 5

Page 158
JEE Sprint Mathematics
1 1 2 2
2 3 2 3
31. If  I1   2 x dx, I2   2 x dx I 3   2 x dx  and   I 4   2 x dx  then -
0 0 1 1

(A) I3    >  I4 (B) I3  =  I4 (C) I1    >  I2 (D) I2    >  I1
n n n 1
32. Let  Sn  =     ........  ,  then  Lim S n   is  -
(n  1)(n  2) (n  2)(n  4) (n  3)(n  6) 6n n 

3 9
(A)  n (B)  n (C)  greater  than  one (D) less  than  two
2 2

/ 2
cot x
33. The  value  of  the  integral   dx   is-
0 cot x  tan x

T
3/ 8 /2
cot x dx
(A) /4 (B) /2 (C)  
/ 8 cot x  tan x
dx (D)  
0 1  tan 3 x

N
k k I1

I
34. Let  ƒ  be  a  positive  function,  let  I1   x ƒ [x(1  x)]dx , I2   ƒ [x(1  x)]dx ,  where  2k  –  1  >  0.  Then I
2
1 k 1 k
is  -

R
1
(A) 2 (B) k (C)  (D) less  than  1
2

S P
JE E
ANSWER  KEY
Que. 1 2 3 4 5 6 7 8 9 10
Ans. C A A D B D A D D A
Que. 11 12 13 14 15 16 17 18 19 20
Ans. A A A D A C C D D A
Que. 21 22 23 24 25 26 27 28 29 30
Ans. A A A A C A,C A,B, C,D C C,D C
Que. 31 32 33 34
Ans. C A,D A,D C,D

Page 159
JEE Sprint Mathematics
EXTRA PRACTICE QUESTIONS ON DEFINITE INTEGRATION

SELECT  THE  CORRECT  ALTERNATIVES  (ONE  OR  MORE  THEN  ONE  CORRECT  ANSWERS)

1
2x 2  3x  3
1. The  value  of   (x  1)(x 2  2x  2) dx   is  -
0

  1 
(A)  2 n2  tan 1 2 (B)   2 n2  tan 1 (C) 2 n2  cot 1 3 (D)   n4  cot 1 2
4 4 3 4
1
dx
2. If  I n   (1  x ;  n    N,  then  which  of  the  following  statements  hold  good ?
2 n
0 )

 1  1  5

T
(A)  2nI n+1  =2 –n  +  (2n – 1)I n (B)  I 2   (C)  I 2   (D) I 3  
8 4 8 4 16 48
3. If  a,  b,  c    R  and  satisfy  3a  +  5b  +  15c  =  0,  the  equation  ax4  +  bx2  +  c  =  0  has  -

N
(A) atleast  one  root  in  (–1,  0) (B) atleast  one  root  in  (0,  1)

I
(C) atleast  two  roots  in  (–1,  1) (D) no  root  in  (–1,  1)
 
dx x 2 dx
4. Let  u  =  x 4
 7x 2  1
  &  v  =   4 2   then  -

R
0 0 x  7x  1

(A) v  >  u (B) 6v  =   (C) 3u  +  2v  =  5/6 (D) u  +  v  =  /3

P
5. Let  f(x)  be  a  function  satisfying  f'(x)  =  f(x)  with  f(0)  =  1  and  g  be  the  function  satisfying  f(x)  +  g(x)  =  x2.  The  value
1

S
of  the  integral   f(x)g(x)dx   is  -
0

1 2 5 1 1 2 3
(A) e  – e  (B) e  –  e2  –  3 (C)  (e  3) (D)  e  e 
2 2 2 2 2

E
 /2
I1
6. For  f(x)  =  x4  +|x|,  let  I1  =   f(cos x)dx   and  I2  =   f(sin x)dx then    has  the  value  equal  to  -
I2

E
0 0

J
(A) 1 (B) 1/2 (C) 2 (D) 4

7. Number  of  values  of  x  satisfying  the  equation 


x
 2 28   23  x  1 ,  is  -
  8 t 
3
t  4  dt 
 log ( x 1) x  1
1

(A) 0 (B) 1 (C) 2 (D) 3


0
ze  z
8. The  value  of  definite  integral   dz
 1  e 2 z

  1
(A) – n2 (B)  n2 (C)  –n2 (D) n
2 2 2
/4

9.  (cos 2x) 3 / 2 .cos x dx 


0

3 3 3 3 2
(A)  (B)  (C)  (D) 
16 32 16 2 16
1
 n  n 1 
10. The  value  of     (x  r)   dx equals

0 r 1
  k 1 x  k 

(A) n (B) n! (C) (n+1)! (D) n.n!

Page 160
JEE Sprint Mathematics
2 e4
x2
11. If  the  value  of  the  integral   e dx   is  ,  then  the  value  of  nx dx   is  -
1 e

(A) e4 – e –  (B) 2e4  –  e  –   (C) 2(e4  –  e)  –   (D) 2e4  –  1  –  


x
d r3
12. The  value  of  Lim
x  dx  (r  1)(r  1) dr is  -
3

1
(A) 0 (B) 1 (C)  (D) non  existent
2


x
13.  [2e ]dx where  [x]  denotes  the  greatest  interger  function  is  -

T
0

(A) 0 (B) n2 (C) e2 (D) 2/e


/2

N
sin x  
14. Let  f(x)  = ,  then   f(x)f   x  dx =
x 2 

I
0

   
2 1
(A)   f(x)dx (B)   f(x)dx (C)    f(x)dx (D)   f(x)dx

R
0 0 0 0
2
1 1

P
15. If  for  a  non-zero  x,  af(x)  +  bf     5 ,  where  a    b,  then   f(x)dx   =
x x 1

S
1  7b  1  7b 
(A) 2  a log 2  5a  2  (B) 2  a log 2  5a  2 
a  b2 a  b2

1  7b 
(C)   a log 2  5a  2  (D) none  of  these

E
a  b2
2

1 t 
If  a,  b  and  c  are  real  numbers  then  the  value  of  Lim n   (1  a sin bx) c / x dx    equals  -

E
16.
t 0 t0 

J
ab bc ca
(A) abc (B)  (C)  (D) 
c a b

x / 4
x2 2 2 ƒ (x)  x 9  x 3  x  1
17. Let  y  =  ƒ  (x)  be  a  differentiable  curve  satisfying   ƒ (t)dt   t ƒ (t)dt   ,  then   dx
2 2 x  / 4 cos 2 x

equals -
(A) 0 (B) 1 (C) 2 (D) 4

18. If  y  =  ƒ(x)  is  a  linear  function  satisfying  the  relation  ƒ(xy)  =  ƒ(x).ƒ(y)   x, y  R ,  then  the  curve
x
y 2   (sin t  a 2 t 3  bt)dt  ,  R    cuts  y  =  ƒ–1(x)  at  -
0

(A) no  point (B) exactly  one  point (C) atleast  two  points (D) infinite  points

4 8
19. If ƒ (8 – t) = ƒ (t) and   ƒ (  )d   8 , then  ƒ (  )d  is  -
0 0

(A) 4 (B) 8 (C) 16 (D) 32

Page 161
JEE Sprint Mathematics
t2 2 t2
z  2 tan z  1  tan z  z
1  tan 2 z  2 tan z 
20. If  x  e   dz  &  y  e   dz .
0  2 z sec 2 z  0  2 z sec 2 z 

Then  the  inclination  of  the  tangent  to  the  curve  at  t  =    is  -
4
   3
(A)  (B)  (C)  (D) 
4 3 2 4

21. The  value  of  integral   x ƒ(sin x)dx =
0

 /2 /2
 
(A)   ƒ(sin x)dx (B)    ƒ(sin x)dx (C)   ƒ(cos x)dx (D)  ƒ(cos x)dx
20 0 0 20

N T
R I
S P
JE E
ANSWER  KEY
Que. 1 2 3 4 5 6 7 8 9 10
Ans. A,C,D A,B A,B,C B,C,D D C B A,D C D
Que. 11 12 13 14 15 16 17 18 19 20
Ans. B C B A B A C C C D
Que. 21
Ans. A,B,C

Page 162
JEE Sprint Mathematics
AREA UNDER THE CURVES

SELECT THE CORRECT ALTERNATIVE (ONLY ONE CORRECT ANSWER)


1. The area of the region bounded by the curves y = | x – 2| , x = 1 , x = 3 and the x-axis is -
(A) 3 (B) 2 (C) 1 (D) 4
2. The area enclosed between the curve y = loge(x + e) and the coordinate axes is -
(A) 4 (B) 3 (C) 2 (D) 1
3. The area of the figure bounded by the curves y = nx & y = (nx) is - 2

(A) e + 1 (B) e – 1 (C) 3 – e (D) 1


4. Suppose y = f(x) and y = g(x) are two functions whose grahps intersect at three points (0, 4), (2, 2) and (4, 0) with

T
f(x) > g(x) for 0 < x < 2 and f(x) < g(x) for 2 < x < 4.
4 4
If  [f(x)  g(x)]dx  10 and  [g(x)  f(x)]dx  5 , the area between two curves for 0 < x < 2, is -
0 2

N
(A) 5 (B) 10 (C) 15 (D) 20

I
5. The area bounded by the curves y = –  x and x = –  y where x, y  0
(A) cannot be determined (B) is 1/3 (C) is 2/3

R
(D) is same as that of the figure bounded by the curves y =  x ; x  0 and x = y ; y  0
6. The area of the closed figure bounded by y = x, y = –x & the tangent to the curve y = x 2  5 at the point (3, 2)

P
is -
5
(A) 5 (B) 2 5 (C) 10 (D)

S
2
7. The area of the region(s) enclosed by the curves y = x2 and y = | x| is -
(A) 1/3 (B) 2/3 (C) 1/6 (D) 1

E
–x
8. The area bounded by the curve y = xe ; xy = 0 and x = c, where c is the x-coordinate of the curve's inflection
point, is -

E
–2 –2
(A) 1 – 3e (B) 1 – 2e (C) 1 – e–2 (D) 1

J
3 3
9. The area enclosed by the curves y = cos x, y = 1 + sin2x and x = as x varies from 0 to , is -
2 2
3 3 3 3
(A) –2 (B) (C) 2 + (D) 1 +
2 2 2 2
10. The area enclosed by the curve y2 + x4 = x2 is -
2 4 8 10
(A) (B) (C) (D)
3 3 3 3
1
11. Consider two curves C1 : y = and C2 : y = nx on the xy plane. Let D1 denotes the region surrounded by
x
C1, C2 and the line x = 1 and D2 denotes the region surrounded by C1, C2 and the line x = a. If D1 = D2 then the
value of 'a' -
e
(A) (B) e (C) e – 1 (D) 2(e – 1)
2
2
12. The area of the region for which 0 < y < 3 – 2x – x & x > 0 is -
3 3 1 3
2 2 2 2
(A)  (3  2x  x )dx (B)  (3  2x  x )dx (C)  (3  2x  x )dx (D)  (3  2x  x )dx
1 0 0 1

13. The area bounded by the curves y = x(1 – nx) and positive x-axis between x = e–1 and x = e is -

Page 163
JEE Sprint Mathematics
 e 2  4e 2   e 2  5e 2   4e 2  e 2   5e 2  e 2 
(A)   (B)   (C)   (D)  
5 4 5 4
14. The curve f(x) = Ax2 + Bx + C passes through the point (1, 3) and line 4x + y = 8 is tangent to it at the point
(2, 0). The area enclosed by y = f(x), the tangent line and the y-axis is -
(A) 4/3 (B) 8/3 (C) 16/3 (D) 32/3
3
15. Let y = g(x) be the inverse of a bijective mapping f : R  Rf(x) = 3x + 2x. The area bounded by graph of
g(x), the x-axis and the ordinate at x = 5 is -
5 7 9 13
(A) (B) (C) (D)
4 4 4 4
dy
16. A function y = f(x) satisfies the differential equation,– y = cos x – sin x, with initial condition that y is
dx
bounded when x  . The area enclosed by y = f(x), y = cosx and the y-axis in the 1st quadrant is-

T
1
(A) 2 –1 (B) 2 (C) 1 (D)
2

N
SELECT THE CORRECT ALTERNATIVES (ONE OR MORE THAN ONE CORRECT ANSWERS)

I
17. Let 'a' be a positive constant number. Consider two curves C1 : y = ex, C2 : y = ea – x. Let S be the area of the
part surrounding by C1, C2 and the y-axis, then -

R
S 1
(A) aLim S 1 (B) Lim
2

 a 4
a 0

(C) Range of S is [0,) (D) S(a) is neither odd nor even

P
18. Area enclosed by the curve y = sinx between x =2n to x = 2(n+1) is-
2  /2

S
(A)  sin x dx
0
(B) 2  sin x dx
0
(C) 4 0
sin x dx (D) 4

19. If (a, 0) & (b,0) [a,b > 0] are the points where the curve y = sin2x – 3 sinx cuts the positive x-axis first &

E
second time, A & B are the areas bounded by the curve & positive x-axis between x=0 to x=a and x = a to x=b
respectively, then -

E
1
(A) 4A + 8 cosa = 7 (B) AB  (C) 4A + 4B + 14cosb = 0 (D) B – A = 4cos a
16

J
20. For which of the following values of m, is the area of the region bounded by the curve y = x – x2 and the
line y = mx equals to 9/2 ?
(A) –4 (B) –2 (C) 2 (D) 4
21. Let f(x) = |x|– 2 and g(x) = |f(x)|.
Now area bounded by x-axis and f(x) is A1 and area bounded by x-axis and g(x) is A2 then –
(A) A1 = 3 (B) A1 = A2 (C) A2 = 4 (D) A1 + A2 = 8

ANSWER KEY
Que. 1 2 3 4 5 6 7 8 9 10
Ans. C D C C B A B A C B
Que. 11 12 13 14 15 16 17 18 19 20
Ans. B C B B D A A,B, C,D B,C,D A,B, C,D B,D
Que. 21
Ans. B,C,D

Page 164
JEE Sprint Mathematics
EXTRA PRACTICE QUESTIONS ON AREA UNDER THE CURVE
SELECT THE CORRECT ALTERNATIVES (ONE OR MORE THAN ONE CORRECT ANSWERS)
1 x2
1. If C1  y = 2 and C2  y = are two curves lying in the XY plane. Then -
1x 2
 1
(A) area bounded by curve C1 and y = 0 is  (B) area bounded by C1 and C2 is 
2 3
 
(C) area bounded by C1 and C2 is 1 – (D) area bounded by curve C1 and x-axis is
2 2
2. Area enclosed by the curves y = nx ; y = n|x| ; y = |nx| and y = |n|x|| is equal to -
(A) 2 (B) 4 (C) 8 (D) cannot be determined
3. y = f(x) is a function which satisfies-
(i) f(0) = 0 (ii) f''(x) = f'(x) and (iii) f'(0) = 1
then the area bounded by the graph of y = f(x), the lines x = 0, x – 1 = 0 and y + 1 = 0, is -

T
(A) e (B) e – 2 (C) e – 1 (D) e + 1
4. Let T be the triangle with vertices (0, 0), (0, c2) and (c, c2) and let R be the region between y = cx and y = x2
where c > 0 then -

N
c3 c3 Area(T) Area(T) 3
(A) Area (R) = (B) Area of R = (C) Lim 3 (D) Lim 

I
6 3 c  0 
Area(R ) c  0 
Area(R ) 2
5. Suppose g(x) = 2x + 1 and h(x) = 4x 2 + 4x + 5 and h(x) = (fog)(x). The area enclosed by the graph of the
function y = f(x) and the pair of tangents drawn to it from the origin, is -

R
(A) 8/3 (B) 16/3 (C) 32/3 (D) none
6. Let f(x) = x2 + 6x + 1 and R denote the set of points (x, y) in the coordinate plane such that f(x) + f(y)  0 and
f(x) – f(y)  0. The area of R is equal to -

P
(A) 16 (B) 12 (C) 8 (D) 4
x 1
7. The value of 'a' (a > 0) for which the area bounded by the curves y =  , y = 0, x = a and x = 2a has the
6 x2

S
least value, is -
(A) 2 (B) 2 (C) 21/3 (D) 1
8. Consider the following regions in the plane :

E
R1 = {(x, y) : 0  x  1 and 0  y  1} and R2 = {(x, y) : x2 + y2 4/3}
a 3  b
The area of the region R1  R2 can be expressed as , where a and b are integers, then -

E
9
(A) a = 3 (B) a = 1 (C) b = 1 (D) b = 3

J
1
9. The area of the region of the plane bounded by (| x| ,| y| )  1 & xy  is -
2
15
(A) less then 4n 3 (B) (C) 2 + 2n2 (D) 3 + n2
4
3
10. The line y = mx bisects the area enclosed by the curve y = 1 + 4x – x2 & the lines x = 0, x = & y = 0. Then
2
the value of m is -
13 6 3
(A) (B) (C) (D) 4
6 13 2
11. Area of the region enclosed between the curves x = y2 – 1 and x = |y| 1  y 2 is -
(A) 1 (B) 4/3 (C) 2/3 (D) 2
12. If the tangent to the curve y = 1 – x2 at x = , where 0 <  < 1, meets the axes at P and Q. As  varies, the
minimum value of the area of the triangle OPQ is k times the area bounded by the axes and the part of the
curve for which 0 < x < 1, then k is equal to -
2 75 25 2
(A) (B) (C) (D)
3 16 18 3
ANSWER KEY
Que. 1 2 3 4 5 6 7 8 9 10 11 12
Ans. A,B B C A,C B C D A,C A,D A D A

Page 165
JEE Sprint Mathematics
DIFFERENTIAL EQUATION
SELECT  THE  CORRECT  ALTERNATIVE  (ONLY  ONE  CORRECT  ANSWER)
2
 dy  3 d3 y
1. The  order  and  degree  of  the  differential  equation   1  3   4 3   are  -
 dx  dx
2
(A) 1 , (B) 3  ,  1 (C) 1,  2 (D) 3,  3
3
2. The  degree  and  order  of  the  differential  equation  of  the  family  of  all  parabolas  whose  axis  is  x-axis  are  respectively
(A) 2  , 1 (B) 1  ,  2 (C) 3  , 2 (D) 2  ,  3

dy d2 y
3. The  order  and  degree  of  the  differential  equation  3  4 2  7x  0   are  a  and  b,  then  a  +  b  is  -
dx dx

T
(A) 3 (B) 4 (C) 5 (D) 6

4. The  order  of  the  differential  equation  whose  general  solution  is  given  by  y  (C 1  C 2 ) cos(x  C 3 )  C 4 e x  c5

I N
where  C1,  C2,  C3,  C4,  C5  are  arbitrary  constants,  is  -   [JEE  98]
(A) 5 (B)   4 (C)  3 (D)    2
5. The  differential  equation  of  the  family  of  curves  represented  by  y  =  a  +  bx  +  ce–x  (where  a,  b,  c  are  arbitrary

R
constants)  is  -
(A) y''' = y' (B) y''' + y'' = 0 (C) y''' – y'' + y' = 0 (D) y''' + y'' – y' = 0

P
6. The  differential  equation  for  the  family  of  curves  x2  +  y2  –  2ay  =  0,  where  a  is  an  arbitrary  constant  is  -
(A) (x2  –  y2  )  y'  =  2xy (B) 2(x2  +  y2)  y'  =  xy

S
(C) 2(x2–y2)  y'  =  xy (D)  (x2  +  y2)y'  =  2xy
7. Number  of  values  of  m    N  for  which  y  =  emx  is  a  solution  of  the  differential  equation
D3y  –  3D2y  –  4Dy  +  12y  =  0  is  -

E
(A) 0 (B) 1 (C) 2 (D) more  than  2

d2 y dy

E
8. If  y  =  e(K  +  1)x  is  a  solution  of  differential  equation  2
4  4y  0 ,  then  k  =
dx dx

J
(A) –1 (B) 0 (C) 1 (D) 2
dy 1  x
9. The  general  solution  of  the  differential  equation   is  a  family  of  curves  which  looks  most  like  which  of
dx y
the  following  ?

(A)  (B)  (C)  (D) 

10. The  solution  to  the  differential  equation  yny  +  xy'  =  0,  where  y(1)  =  e,  is  -
 x2 
(A) x(ny)  =  1 (B) xy(ny)  =  1 (C) (ny)2  =  2 (D) ny  +   y  =  1
 2
dy
11. The  equation  of  the  curve  passing  through  origin  and  satisfying  the  differential  equation    =  sin  (  10x  +  6y)
dx
is -
1  5 tan 4 x  5 x 1  5 tan 4 x  5 x
(A)  y  tan 1   (B)  y  tan 1  
3  4  3 tan 4 x  3 3  4  3 tan 4 x  3
1  3  tan 4 x  5 x
(C)  y  tan 1   (D)  none  of  these
3  4  3 tan 4 x  3

Page 166
JEE Sprint Mathematics
12. Which  one  of  the  following  curves  represents  the  solution  of  the  initial  value  problem  Dy  =  100  –  y,  where
y(0)  =  50

y y y y
100 100 100 100
50 50 50 50
(A)  (B)  (C)  (D) 
O x O x O x O x

x
13. A  curve  passing  through  (2,  3)  and  satisfying  the  differential  equation   ty(t)dt  x 2 y(x), (x  0)   is  -
0

2 9 x2 y2
(A) x2  +  y2  =  13 (B)  y  x (C)   1 (D) xy  =  6
2 8 18

T
  y  y
14. A curve passes through  the point   1,   & its slope at any point is given by – cos2   . Then the  curve has
 4 x x
the  equation  -

N
 e  e

I
1
(A) y  =  xtan–1  n  (B) y  =  xtan–1(n  +  2) (C) y  = tan–1  n  (D) none
x x x
dy

R
15. The  solution  of  the  differential  equation  (2x  –  10y3)  + y  =  0  is  -
dx
(A) x  +  y  =  ce2x (B) y2  =  2x3  +  c (C) xy2  =  2y5  +  c (D) x  (  y2  +  xy)  =  0

P
16.  
Solution  of  differential  equation  1  y 2 dx  x  e tan  1
y
 dy  0 is  -

S
1 tan 1
y 1 2 tan 1 y
(A) y e tan x
 tan 1
xc (B)  x e  e c
2
1
1  tan x
(C) 2x  e tan y
c (D) y  x e c

E
17. The  general  solution  of  the  differential  equation,  y'  +  y'(x)  –  (x)  .  '(x)  =  0  where  (x)  is  a  known  function  is -
(A) y  =  ce–(x)  +  (x)  –1 (B) y  =  ce(x)  +  (x)  +K (C) y  =  ce–(x)  –  (x)  +1 (D) y  =  ce–(x)  +  (x)  +K

E
18. The  solution  of  the  differential  equation,  ex(x  +  1)dx  +  (yey  –  xex)dy  =  0  with  initial  condition  f(0)  =  0,  is  -

J
(A) xex  +  2y2ey  =  0 (B) 2xex  +  y2ey  =  0 (C) xex  –  2y2ey  =  0 (D) 2xex  –  y 2ey  =  0
19. The  solution  of  the  differential  equation  ydx  +  (  x  +  x2  y)  dy  =  0  is  -
1 1 1
(A)   log y  c (B)  log  y  =  cx (C)   c (D)    log y  c
xy xy xy
dy
20. The solution of y5 x  +  y – x   = 0   is -
dx
(A) x4/4  +  1/5  (x/y)5  =  C (B) x5/5  +  (1/4)  (x/y)4  =  C
(C) (x/y)5  +  x4/4  =  C (D) (xy)4+  x5/5  =  C
xdy  y 
21. The  solution  of  2 2
 2 2
 1 dx  is  -
x y x y 
(A) y  =  x cot  (  c–  x) (B) cos–1  y/x  =  –x  +  c
(C) y  =  x  tan  (c  –  x) (D) y2/x2  =  x  tan  (  c  –  x)

SELECT  THE  CORRECT  ALTERNATIVES  (ONE  OR  MORE  THAN  ONE  CORRECT  ANSWERS)

22. The  value  of  the  constant  'm'  and  'c'  for  which  y  =  mx  +  c  is  a  solution  of  the  differential  equation
D2y  –  3Dy  –  4y  =  –  4x
(A) is  m  =  –1 (B) is  c  =  3/4 (C) is  m  =  1 (D) is  c  =  –3/4

Page 167
JEE Sprint Mathematics
dy
23. If  x =  y  (log  y  –  log  x  +  1),  then  the  solution  of  the  equation  is  -
dx

 x  y cx
(A)  log    cy (B)  log    cx (C) y  xe cx (D) x  ye
 y  x
2
 dy 2  dy  2
24. Solutions  of  the  differential  equation  x    xy    6 y  0   -
 dx   dx 
(A) y  =  cx2 (B) x3  y  =  c (C) xy3  =  c (D) y  =  cx
2
 dy  dy
25. A  solution  of  the  differential  equation,     x  y  0 is    -
 dx  dx

T
(A) y  =  2 (B) y  =  2x (C) y  =  2x  –  4 (D) y  =  2x2  –  4
2
 dy  dy x
26. The  solution  the  differential  equation    
 dx  dx
 
e  e  x  1  0   is  are    -

N
(A) y  +  e–x  =  c (B) y  –  e–x  =  c (C) y  +  ex  =  c (D) y  –  ex  =  c

I
dy ax  h
27. The  solution  of   represent  a  parabola  if  -
dx by  k

R
(A) a  =  –2,  b  =  0 (B) a  =  –2,  b  =  2 (C) a  =  0,  b  =  2 (D) a  =  0,  b  =  0
28. A  normal  is  drawn  at  a  point  P(x,  y)  of  a  curve.  It  meets  the  x-axis  and  the  y-axis  in  point  A  and  B,  respectively,

P
1 1
such  that    1 ,  where  O  is  the  origin,  the  equation  of  such  a  curve    is  a  circle  which  passes  through
OA OB

S
(5, 4)  and  has  -
(A) centre  (1,  1) (B) centre  (2,  1) (C) radius  5 (D) radius  4

JE E
ANSWER  KEY
Que. 1 2 3 4 5 6 7 8 9 10
Ans. D B C C B A C C B A
Que. 11 12 13 14 15 16 17 18 19 20
Ans. A B D A C B A B D B
Que. 21 22 23 24 25 26 27 28
Ans. C C,D B,C,D A,B C A,D A,C A,C

Page 168
JEE Sprint Mathematics
EXTRA PRACTICE QUESTIONS ON DIFFERENTIAL EQUATION
SELECT  THE  CORRECT  ALTERNATIVES  (ONE  OR  MORE  THAN  ONE  CORRECT  ANSWERS)
1. Which  one  of  the  following  is  homogeneous  function ?
1 2
xy  x
(A) f(x,  y)  =  (B)  f(x,  y)= x 3 .y 3
tan 1
x2  y2 y

 2x 2  y 2  x  2y
(C) f(x,  y)  =  x ( n x 2  y 2  ny )  ye x / y (D) f(x,  y)  =  x n  n(x  y )   y 2 tan
 x  3x  y

2. The  graph  of  the  function  y  =  f(x)  passing  through  the  point  (0,  1)  and  satisfying  the  differential  equation

T
dy
 y cos x  cos x   is  such  that  -
dx

N
(A) it  is  a  constant  function (B) it  is  periodic

I
(C) it  is  neither  an  even  nor  an  odd  function (D) it  is  continuous  &  differentiable  for  all  x.

3. Water  is  drained  from  a  vertical  cylindrical  tank  by  opening  a  valve  at  the  base  of  the  tank.  It  is  known  that

R
the  rate  at  which  the  water  level  drops  is  proportional  to  the  square  root  of  water  depth  y,  where  the  constant
of  proportionality  k  >  0  depends  on  the  acceleration due to  gravity  and  the geometry  of the hole.  If  t  is  measured

P
in  minutes  and  k  =  1/15  then  the  time  to  drain  the  tank  if  the  water  is  4  meter  deep  to  start  with  is  -
(A) 30  min (B) 45  min (C) 60  min (D) 80  min

S
dy 1 1
4. The  solution  of  the  differential  equation,  x 2 .cos  y sin  1 ,  where  y    –1  as  x     is  -
dx x x

E
1 1 x 1 1 1 x 1
(A) y  = sin  cos (B)  y  = (C) y  = cos  sin (D)  y  =
x x 1 x x 1
x sin x cos
x x

JE
x dy y  x
5. If  y   (where  c is an arbitrary  constant)  is  the general solution of the  differential equation     
n| cx| dx x  y

 x
then  the  function       is  -
 y

x2 x2 y2 y2
(A)  2 (B) – 2 (C)  (D) –
y y x2 x2

x
6. If   ty(t)dt  x 2  y(x)   then  y  as  a  function  of  x  is  -
a

x 2 a2 x2 a 2
x2 a 2
(A) y  2  2  a 2  e 2
(B) y  1  2  a  e 2 2 (C) y  2  1  a 2  e 2
(D) none
1

7. A  function  f(x)  satisfying   f(tx)dt  nf(x) ,  where  x  >  0,  is  -


0

1n n 1
(A)  f(x)  c.x n
(B)  f(x)  c.x n 1 (C)  f(x)  c.x n (D) f(x )  c.x (1  n )

Page 169
JEE Sprint Mathematics
d 2 y dy
8. The  differential  equation    sin y  x 2  0   is  of  the  following  type  -
dx 2 dx
(A) linear (B)  homogeneous (C)  order  two (D)  degree  one
9. A  curve  C  passes  through  origin  and  has  the  property  that  at  each  point  (x,  y)  on  it  the  normal  line  at  that  point
passes  through  (1,  0).  The  equation  of  a  common  tangent  to  the  curve  C  and  the  parabola  y2  =  4x  is  -
(A) x  =  0 (B) y  =  0 (C) y  =  x  +  1 (D) x  +  y  +  1  =  0
10. The  function  f(x)  satisfying  the  equation,  f 2(x)  +  4f'(x)  .  f(x)  +  [f'(x)] 2   =  0  is  -

(A) f(x)  =  c  .  e (2  3 )x
(B) f(x)  =  c  .  e (2  3 )x
(C) f(x)  =  c  .  e ( 3 2 ) x
(D) f(x)  =  c  .  e  (2  3 )x

11. The  equation  of  the  curve  passing  through  (3,  4)  &  satisfying  the  differential  equation,
2
 dy  dy
y    (x  y )  x  0   can  be  -

T
 dx  dx
(A) x  –  y  +  1  =  0 (B) x2  +  y2  =  25 (C) x2  +  y2  –  5x  –  10  =  0 (D)  x  +  y  –  7  =  0
2

N
dy  dy 
12. Number  of  straight  lines  which  satisfy  the  differential  equation   x    y  0   is  -
dx  dx 

I
(A) 1 (B) 2 (C) 3 (D) 4

d2 y dy

R
13. Let  y  =  (A  +  Bx)e 3x   be  a  solution  of  the  differential  equation  2
m  ny  0 ,  m,  n    I,  then  -
dx dx
(A) m  +  n  =  3 (B) n 2 – m 2 = 64 (C) m  =  –6 (D) n  =  9

P
14. The  differential  equation  2xy  dy  =  (x2  +  y2  +  1)  dx  determines  -

(A) A  family  of  circles  with  centre  on  x-axis

S
(B) A  family  of  circles  with  centre  on  y-axis

(C) A  family  of  rectangular  hyperbola  with  centre  on  x-axis

E
(D) A family of rectangular hyperbola with centre on y-axis

15. If  ƒ ''(x)  + ƒ '(x)  + ƒ 2(x)  = x2 be  the differential  equation  of  a curve and  let  P  be  the point of  maxima then number

E
of  tangents  which  can be  drawn from  point P to x2 –  y2 = a2,  a   0  is  -

J
(A) 2 (B) 1 (C) 0 (D) either  1  or  2
2 2 2
16. The  solution  of  x dy  –  y dx  +  xy (x  –  y)dy  =  0  is  -

xy y2 xy x2 xy x2 xy


(A)  n  c (B)  n  c (C)  n  c (D)  n  xc
xy 2 xy 2 xy 2 xy
2
 dy  4
17. The  orthogonal  trajectories  of  the  system  of  curves       are  -
 dx  x

x 3 / 2 x3 / 2
(A) 9(y  +  c)2  =  x3 (B) y  +  c  = (C) y  +  c  = (D) all  of  these
3 3

ANSWER  KEY
Que. 1 2 3 4 5 6 7 8 9 10
Ans. A, B, C A,B,D C A D A A C,D A C,D
Que. 11 12 13 14 15 16 17
Ans. A,B B A ,C , D C A A A,B, C,D

Page 170
PREVIOUS YEAR

JEE MAIN
QUESTIONS
JEE MAIN 2018 OFFLINE
QUESTION PAPER
This booklet contains 20 printed pages.

PAPER - 1 : PHYSICS, CHEMISTRY & MATHEMATICS Test Booklet Code


Do not open this Test Booklet until you are asked to do so.
Read carefully the Instructions on the Back Cover of this Test Booklet.
Important Instructions :
1. Immediately fill in the particulars on this page of the Test Booklet with only Black Ball Point
A
Pen provided in the examination hall.
2. The Answer Sheet is kept inside this Test Booklet. When you are directed to open the Test
Booklet, take out the Answer Sheet and fill in the particulars carefully.
3. The test is of 3 hours duration.
4. The Test Booklet consists of 90 questions. The maximum marks are 360.
5. There are three parts in the question paper A, B, C consisting of Physics, Chemistry and
Mathematics having 30 questions in each part of equal weightage. Each question is allotted
4 (four) marks for correct response.
6. Candidates will be awarded marks as stated above in instruction No. 5 for correct response of each question.
¼ (one-fourth) marks of the total marks allotted to the question (i.e. 1 mark) will be deducted for indicating
incorrect response of each question. No deduction from the total score will be made if no response is indicated
for an item in the answer sheet.
7. There is only one correct response for each question. Filling up more than one response in any
question will be treated as wrong response and marks for wrong response will be deducted
accordingly as per instruction 6 above.
8. For writing particulars/marking responses on Side-1 and Side–2 of the Answer Sheet use only
Black Ball Point Pen provided in the examination hall.
9. No candidate is allowed to carry any textual material, printed or written, bits of papers, pager,
mobile phone, any electronic device, etc. except the Admit Card inside the examination room/
hall.
10. On completion of the test, the candidate must hand over the Answer Sheet to the Invigilator on
duty in the Room/Hall. However, the candidates are allowed to take away this Test Booklet
with them.
11. The CODE for this Booklet is A. Make sure that the CODE printed on Side–2 of the Answer
Sheet is same as that on this Booklet. Also tally the serial number of the Test Booklet and
Answer Sheet are the same as that on this booklet. In case of discrepancy, the candidate should
immediately report the matter to the Invigilator for replacement of both the Test Booklet and
the Answer Sheet.
12. Do not fold or make any stray mark on the Answer Sheet.

Name of the Candidate (in Capital letters ) :

Roll Number : in figures

: in words

Examination Centre Number :

Name of Examination Centre (in Capital letters) :

Candidate’s Signature : 1. Invigilator’s Signature :

2. Invigilator’s Signature :
PART A — PHYSICS 3. Two masses m 1=5 kg and m 2=10 kg,
ALL THE GRAPHS/DIAGRAMS GIVEN ARE connected by an inextensible string over a
SCHEMATIC AND NOT DRAWN TO SCALE. frictionless pulley, are moving as shown
1. The density of a material in the shape of a in the figure. The coefficient of friction of
cube is determined by measuring three horizontal surface is 0.15. The minimum
sides of the cube and its mass. If the weight m that should be put on top of m2
relative errors in measuring the mass and to stop the motion is :
length are respectively 1.5% and 1%, the
maximum error in determining the density
is :
(1) 2.5%
(2) 3.5%
(3) 4.5%
(4) 6%
2. All the graphs below are intended to
represent the same motion. One of them
does it incorrectly. Pick it up. (1) 18.3 kg

(2) 27.3 kg

(3) 43.3 kg
(1)
(4) 10.3 kg

4. A particle is moving in a circular path of


(2) radius a under the action of an attractive
k
potential U=− . Its total energy is :
2 r2

k
(1) −
(3) 4 a2

k
(2)
2 a2

(3) Zero
(4)
3 k
(4) −
2 a2

A/Page 2 SPACE FOR ROUGH WORK


5. In a collinear collision, a particle with an 7. From a uniform circular disc of radius R
initial speed vo strikes a stationary particle R
and mass 9 M, a small disc of radius
is
of the same mass. If the final total kinetic 3
energy is 50% greater than the original removed as shown in the figure. The
kinetic energy, the magnitude of the moment of inertia of the remaining disc
relative velocity between the two particles, about an axis perpendicular to the plane
after collision, is : of the disc and passing through centre of
vo disc is :
(1)
4
(2) 2 vo
vo
(3)
2
vo
(4)
2

6. Seven identical circular planar disks, each


of mass M and radius R are welded (1) 4 MR2
symmetrically as shown. The moment of
40
inertia of the arrangement about the axis (2) MR 2
9
normal to the plane and passing through
the point P is : (3) 10 MR2

37
(4) MR 2
9

8. A particle is moving with a uniform speed


in a circular orbit of radius R in a central
force inversely proportional to the n th
power of R. If the period of rotation of the
19 particle is T, then :
(1) MR 2
2
(1) T∝R3/2 for any n.
55
(2) MR 2
2 n +1
(2) T∝R 2
73
(3) MR 2
2 (3) T∝R(n+1)/2
181
(4) MR 2 (4) T∝Rn/2
2

A/Page 3 SPACE FOR ROUGH WORK


9. A solid sphere of radius r made of a soft 11. The mass of a hydrogen molecule is
material of bulk modulus K is surrounded 3.32×10 −27 kg. If 10 23 hydrogen
by a liquid in a cylindrical container. A molecules strike, per second, a fixed wall
massless piston of area a floats on the of area 2 cm 2 at an angle of 458 to the
surface of the liquid, covering entire cross normal, and rebound elastically with a
section of cylindrical container. When a speed of 103 m/s, then the pressure on the
mass m is placed on the surface of the wall is nearly :
piston to compress the liquid, the fractional (1) 2.35×103 N/m2
decrement in the radius of the sphere,
(2) 4.70×103 N/m2
 dr  , is : (3) 2.35×102 N/m2
 
 r  (4) 4.70×102 N/m2

Ka 12. A silver atom in a solid oscillates in simple


(1) mg harmonic motion in some direction with a
frequency of 1012/sec. What is the force
Ka
(2) constant of the bonds connecting one atom
3 mg
with the other ? (Mole wt. of silver=108 and
Avagadro number=6.02×1023 gm mole−1)
mg
(3) (1) 6.4 N/m
3 Ka
(2) 7.1 N/m
mg
(4) (3) 2.2 N/m
Ka
(4) 5.5 N/m

10. Two moles of an ideal monoatomic gas 13. A granite rod of 60 cm length is clamped
occupies a volume V at 278C. The gas at its middle point and is set into
expands adiabatically to a volume 2 V. longitudinal vibrations. The density of
Calculate (a) the final temperature of the granite is 2.7×103 kg/m3 and its Young’s
gas and (b) change in its internal energy. modulus is 9.27×1010 Pa. What will be
the fundamental frequency of the
(1) (a) 189 K (b) 2.7 kJ
longitudinal vibrations ?
(2) (a) 195 K (b) −2.7 kJ (1) 5 kHz
(2) 2.5 kHz
(3) (a) 189 K (b) −2.7 kJ
(3) 10 kHz
(4) (a) 195 K (b) 2.7 kJ
(4) 7.5 kHz

A/Page 4 SPACE FOR ROUGH WORK


14. Three concentric metal shells A, B and C 16. In an a.c. circuit, the instantaneous e.m.f.
of respective radii a, b and c (a < b < c) and current are given by
have surface charge densities +σ, −σ and e=100 sin 30 t
+σ respectively. The potential of shell B
i=20 sin  30 t − 
π
is :  4
In one cycle of a.c., the average power
σ  a 2 − b2  consumed by the circuit and the wattless
(1)  + c
o  a  current are, respectively :
(1) 50, 10

σ  a 2 − b2 
(2)
1000
, 10
(2)  + c 2
o  b 
50
(3) ,0
2
σ  b2 − c2 
(3)  + a (4) 50, 0
o  b 

17. Two batteries with e.m.f. 12 V and 13 V


σ  b2 − c2  are connected in parallel across a load
(4)  + a
o  c  resistor of 10 Ω. The internal resistances of
the two batteries are 1 Ω and 2 Ω
respectively. The voltage across the load
lies between :
15. A parallel plate capacitor of capacitance (1) 11.6 V and 11.7 V
90 pF is connected to a battery of emf (2) 11.5 V and 11.6 V
20 V. If a dielectric material of dielectric (3) 11.4 V and 11.5 V
5 (4) 11.7 V and 11.8 V
constant K = is inserted between the
3
plates, the magnitude of the induced
18. An electron, a proton and an alpha
charge will be :
particle having the same kinetic energy are
moving in circular orbits of radii re, rp, rα
(1) 1.2 n C
respectively in a uniform magnetic field B.
The relation between re, rp, rα is :
(2) 0.3 n C
(1) re > rp = rα
(3) 2.4 n C (2) re < rp = rα
(3) re < rp < rα
(4) 0.9 n C (4) re < rα < rp

A/Page 5 SPACE FOR ROUGH WORK


19. The dipole moment of a circular loop 21. An EM wave from air enters a medium.
carrying a current I, is m and the magnetic The electric fields are
field at the centre of the loop is B1. When → ∧  z 
E 1= E01 x cos  2 πν  − t   in air and
the dipole moment is doubled by keeping  c 
the current constant, the magnetic field at → ∧
E 2= E02 x cos [ k (2 z−ct)] in medium,
B1
the centre of the loop is B2. The ratio where the wave number k and
B2
frequency ν refer to their values in air. The
is :
medium is non-magnetic. If ȏ r1 and ȏr
2
(1) 2 refer to relative permittivities of air and
medium respectively, which of the
(2) 3 following options is correct ?
r
1
(3) (1) =4
2 r
2
r
1 1
(2) =2
(4) r
2 2
r 1
1
(3) =
r
2
4

20. For an RLC circuit driven with voltage of r 1


1
(4) =
1 r
2
2
amplitude vm and frequency ωo=
LC
the current exibits resonance. The quality 22. Unpolarized light of intensity I passes
factor, Q is given by : through an ideal polarizer A. Another
identical polarizer B is placed behind A.
ωo L The intensity of light beyond B is found to
(1)
R I
be . Now another identical polarizer C
2
ωo R is placed between A and B. The intensity
(2) I
L
beyond B is now found to be . The angle
8
R between polarizer A and C is :
(3) ( ωo C) (1) 08
(2) 308
CR (3) 458
(4) ωo (4) 608

A/Page 6 SPACE FOR ROUGH WORK


23. The angular width of the central 25. If the series limit frequency of the Lyman
maximum in a single slit diffraction series is νL, then the series limit frequency
pattern is 608. The width of the slit is of the Pfund series is :
1 µm. The slit is illuminated by (1) 25 νL
monochromatic plane waves. If another
(2) 16 νL
slit of same width is made near it, Young’s
fringes can be observed on a screen placed (3) νL/16
at a distance 50 cm from the slits. If the (4) νL/25
observed fringe width is 1 cm, what is slit
separation distance ?
26. It is found that if a neutron suffers an
(i.e. distance between the centres of each elastic collinear collision with deuterium
slit.) at rest, fractional loss of its energy is pd ;
while for its similar collision with carbon
(1) 25 µm
nucleus at rest, fractional loss of energy
(2) 50 µm is p c . The values of p d and p c are
(3) 75 µm respectively :
(1) (⋅89, ⋅28)
(4) 100 µm
(2) (⋅28, ⋅89)
(3) (0, 0)
24. An electron from various excited states of
(4) (0, 1)
hydrogen atom emit radiation to come to
the ground state. Let λ n , λ g be the
de Broglie wavelength of the electron in 27. The reading of the ammeter for a silicon
the n th state and the ground state diode in the given circuit is :
respectively. Let Λn be the wavelength of
the emitted photon in the transition from
the nth state to the ground state. For large
n, (A, B are constants)

B
(1) Λn ≈ A + 2
λn

(2) Λn ≈ A +B λn (1) 0
(2) 15 mA
(3) Λ2n ≈ A+B λn2
(3) 11.5 mA
(4) Λn2 ≈λ (4) 13.5 mA

A/Page 7 SPACE FOR ROUGH WORK


28. A telephonic communication service is PART B — CHEMISTRY
working at carrier frequency of 10 GHz.
Only 10% of it is utilized for transmission.
31. The ratio of mass percent of C and H of an
How many telephonic channels can be
organic compound (CXHYOZ) is 6 : 1. If
transmitted simultaneously if each channel
one molecule of the above compound
requires a bandwidth of 5 kHz ?
(CXHYOZ) contains half as much oxygen
(1) 2×10 3
as required to burn one molecule of
(2) 2×10 4
compound CXHY completely to CO2 and
(3) 2×10 5
H2O. The empirical formula of compound
(4) 2×10 6 CXHYOZ is :

(1) C 3H6O 3
29. In a potentiometer experiment, it is found
that no current passes through the (2) C 2H 4O
galvanometer when the terminals of the (3) C 3H4O 2
cell are connected across 52 cm of the (4) C 2H4O 3
potentiometer wire. If the cell is shunted
by a resistance of 5 Ω, a balance is found
when the cell is connected across 40 cm of 32. Which type of ‘defect’ has the presence of
the wire. Find the internal resistance of cations in the interstitial sites ?
the cell. (1) Schottky defect
(1) 1Ω
(2) Vacancy defect
(2) 1.5 Ω
(3) Frenkel defect
(3) 2Ω
(4) 2.5 Ω (4) Metal deficiency defect

30. On interchanging the resistances, the 33. According to molecular orbital theory,
balance point of a meter bridge shifts to which of the following will not be a viable
the left by 10 cm. The resistance of their molecule ?
series combination is 1 kΩ. How much was
2+
the resistance on the left slot before (1) He2
interchanging the resistances ?
+
(2) He2
(1) 990 Ω
(2) 505 Ω (3) H−
2
(3) 550 Ω
(4) 910 Ω (4) H22−

A/Page 8 SPACE FOR ROUGH WORK


34. Which of the following lines correctly show 36. For 1 molal aqueous solution of the
the temperature dependence of following compounds, which one will
equilibrium constant, K, for an exothermic show the highest freezing point ?
reaction ?
(1) [Co(H2O)6]Cl3

(2) [Co(H2O)5Cl]Cl2 . H2O

(3) [Co(H2O)4Cl2]Cl . 2H2O

(4) [Co(H2O)3Cl3] . 3H2O

37. An aqueous solution contains 0.10 M H2S


and 0.20 M HCl. If the equilibrium
constants for the formation of HS− from
(1) A and B H2S is 1.0×10 −7 and that of S 2− from
HS − ions is 1.2×10 −13 then the
(2) B and C
concentration of S 2− ions in aqueous
(3) C and D solution is :

(1) 5×10 −8
(4) A and D
(2) 3×10 −20

(3) 6×10 −21

35. The combustion of benzene (l) gives CO2(g) (4) 5×10 −19
and H2O(l). Given that heat of combustion
of benzene at constant volume is
38. An aqueous solution contains an unknown
−3263.9 kJ mol −1 at 258 C; heat of
concentration of Ba2+. When 50 mL of a
combustion (in kJ mol−1) of benzene at
1 M solution of Na2SO4 is added, BaSO4
constant pressure will be :
just begins to precipitate. The final volume
(R=8.314 JK−1 mol−1) is 500 mL. The solubility product of BaSO4
is 1×10 −10 . What is the original
(1) 4152.6 concentration of Ba2+ ?

(1) 5×10−9 M
(2) −452.46
(2) 2×10−9 M
(3) 3260
(3) 1.1×10−9 M
(4) −3267.6 (4) 1.0×10−10 M

A/Page 9 SPACE FOR ROUGH WORK


39. At 5188 C, the rate of decomposition of a 42. Which of the following compounds
sample of gaseous acetaldehyde, initially contain(s) no covalent bond(s) ?
at a pressure of 363 Torr, was
KCl, PH3, O2, B2H6, H2SO4
1.00 Torr s−1 when 5% had reacted and
0.5 Torr s−1 when 33% had reacted. The (1) KCl, B2H6, PH3
order of the reaction is : (2) KCl, H2SO4
(1) 2 (3) KCl
(2) 3 (4) KCl, B2H6
(3) 1

(4) 0
43. Which of the following are Lewis acids ?

(1) PH3 and BCl3


40. How long (approximate) should water be
(2) AlCl3 and SiCl4
electrolysed by passing through
100 amperes current so that the oxygen (3) PH3 and SiCl4
released can completely burn 27.66 g of (4) BCl3 and AlCl3
diborane ?

(Atomic weight of B=10.8 u)


44. Total number of lone pair of electrons in
(1) 6.4 hours

(2) 0.8 hours 3 ion is :


I−

(3) 3.2 hours (1) 3

(4) 1.6 hours (2) 6

(3) 9

41. The recommended concentration of (4) 12


fluoride ion in drinking water is up to
1 ppm as fluoride ion is required to make
teeth enamel harder by converting 45. Which of the following salts is the most

[3Ca3(PO4)2⋅Ca(OH)2] to : basic in aqueous solution ?

(1) [CaF2] (1) Al(CN)3

(2) [3(CaF2)⋅Ca(OH)2] (2) CH 3COOK

(3) [3Ca 3(PO 4)2⋅CaF 2] (3) FeCl3

(4) [3{Ca(OH)2}⋅CaF2] (4) Pb(CH3COO)2

A/Page 10 SPACE FOR ROUGH WORK


46. Hydrogen peroxide oxidises [Fe(CN)6]4− 49. When metal ‘M’ is treated with NaOH, a
to [Fe(CN) 6 ] 3− in acidic medium but white gelatinous precipitate ‘X’ is obtained,
reduces [Fe(CN) 6] 3− to [Fe(CN) 6]4− in which is soluble in excess of NaOH.
alkaline medium. The other products Compound ’X’ when heated strongly gives
formed are, respectively : an oxide which is used in chromatography
as an adsorbent. The metal ‘M’ is :
(1) (H2O+O2) and H2O
(1) Zn
(2) (H2O+O2) and (H2 O+OH−)
(2) Ca
(3) H2O and (H2O+O2)
(3) Al
(4) H2O and (H2O+OH−) (4) Fe

47. The oxidation states of 50. Consider the following reaction and
Cr in [Cr(H 2 O) 6 ]Cl 3 , [Cr(C 6 H 6 ) 2 ], and statements :
K2[Cr(CN) 2(O) 2(O 2)(NH 3)] respectively
[Co(NH3)4Br2]++Br−→[Co(NH3)3Br3]+NH3
are :
(I) Two isomers are produced if the
(1) +3, +4, and +6 reactant complex ion is a cis-isomer.

(2) +3, +2, and +4 (II) Two isomers are produced if the
reactant complex ion is a trans-
(3) +3, 0, and +6 isomer.

(4) +3, 0, and +4 (III) Only one isomer is produced if the


reactant complex ion is a trans-
isomer.
48. The compound that does not produce (IV) Only one isomer is produced if the
nitrogen gas by the thermal decomposition reactant complex ion is a cis-isomer.
is :
The correct statements are :
(1) Ba(N3)2
(1) (I) and (II)
(2) (NH4)2Cr2O7
(2) (I) and (III)
(3) NH4NO2 (3) (III) and (IV)

(4) (NH4)2SO4 (4) (II) and (IV)

A/Page 11 SPACE FOR ROUGH WORK


51. Glucose on prolonged heating with HI 54. Phenol on treatment with CO 2 in the
gives : presence of NaOH followed by
acidification produces compound X as the
(1) n-Hexane
major product. X on treatment with
(2) 1-Hexene (CH 3CO)2O in the presence of catalytic
(3) Hexanoic acid amount of H2SO4 produces :

(4) 6-iodohexanal

52. The trans-alkenes are formed by the (1)


reduction of alkynes with :

(1) H2 - Pd/C, BaSO4

(2) NaBH4

(3) Na/liq. NH3

(4) Sn - HCl
(2)

53. Which of the following compounds will


be suitable for Kjeldahl’s method for
nitrogen estimation ?

(1)
(3)

(2)

(3)

(4)

(4)

A/Page 12 SPACE FOR ROUGH WORK


55. An alkali is titrated against an acid with 57. Phenol reacts with methyl chloroformate
methyl orange as indicator, which of the in the presence of NaOH to form product
following is a correct combination ? A. A reacts with Br2 to form product B.
A and B are respectively :
Base Acid End point

(1) Weak Strong Colourless to


pink (1)

(2) Strong Strong Pinkish red


to yellow
(2)
(3) Weak Strong Yellow to
pinkish red

(4) Strong Strong Pink to (3)


colourless

(4)
56. The predominant form of histamine
present in human blood is (pK a ,
Histidine=6.0)
58. The increasing order of basicity of the
following compounds is :
(1) (a)

(b)

(2)
(c)

(d)

(3)
(1) (a) < (b) < (c) < (d)
(2) (b) < (a) < (c) < (d)
(3) (b) < (a) < (d) < (c)
(4)
(4) (d) < (b) < (a) < (c)

A/Page 13 SPACE FOR ROUGH WORK


59. The major product formed in the following 60. The major product of the following
reaction is : reaction is :

(1) (1)

(2) (2)

(3) (3)

(4) (4)

A/Page 14 SPACE FOR ROUGH WORK


PART C — MATHEMATICS 65. If the system of linear equations
x+ky+3z=0
61. Two sets A and B are as under :
3x+ky−2z=0
A={(a, b) e R×R : ?a−5? < 1 and
?b−5? < 1}; 2x+4y−3z=0

B={(a, b) e R×R : 4(a−6) 2 +9(b−5) 2 xz


has a non-zero solution (x, y, z), then
≤ 36}. Then : y2
(1) B⊂A is equal to :
(2) A⊂B (1) −10
(3) A ∩ B = φ (an empty set) (2) 10
(4) neither A ⊂ B nor B ⊂ A (3) −30

62. Let S={x e R : x / 0 and (4) 30

2? x −3?+ x ( x −6)+6=0}. Then S :


(1) is an empty set. 66. From 6 different novels and 3 different
dictionaries, 4 novels and 1 dictionary are
(2) contains exactly one element.
to be selected and arranged in a row on a
(3) contains exactly two elements.
shelf so that the dictionary is always in
(4) contains exactly four elements. the middle. The number of such
arrangements is :
63. If α, β e C are the distinct roots, of the
equation x2−x+1=0, then α101+β107 is (1) at least 1000
equal to : (2) less than 500
(1) −1 (3) at least 500 but less than 750
(2) 0 (4) at least 750 but less than 1000
(3) 1
(4) 2 67. The sum of the co-efficients of all odd
degree terms in the expansion of
x−4 2x 2x
5 5
64. If 2 x x−4
2x
2 x = (A + Bx )(x − A)2 ,
2 x x−4
(x + x3 − 1 ) + (x − x3 − 1 ) , (x > 1)
is :
then the ordered pair (A, B) is equal to :
(1) −1
(1) (−4, −5)
(2) (−4, 3) (2) 0

(3) (−4, 5) (3) 1

(4) (4, 5) (4) 2

A/Page 15 SPACE FOR ROUGH WORK


68. Let a1, a2, a3, ....., a49 be in A.P. such that 72. If the curves y 2 =6x, 9x 2 +by 2 =16
12 intersect each other at right angles, then
∑ a4k+1 = 416 and a 9 +a 43 =66. If the value of b is :
k=0
(1) 6
a12 + a 22 2
+ ..... + a17 = 140 m , then m is 7
(2)
equal to : 2
(1) 66 (3) 4
(2) 68 9
(3) 34 (4)
2
(4) 33

1 1
69. Let A be the sum of the first 20 terms and 73. Let f ( x ) = x 2 + and g( x ) = x − ,
2
x x
B be the sum of the first 40 terms of the
series f (x )
x e R−{−1, 0, 1}. If h( x ) = , then the
12+2⋅22+32+2⋅4 2+52+2⋅62+..... g(x )
If B−2A=100λ, then λ is equal to : local minimum value of h(x) is :
(1) 232 (1) 3
(2) 248 (2) −3
(3) 464 (3) −2 2
(4) 496
(4) 2 2
70. For each t e R, let [t] be the greatest integer
less than or equal to t. Then 74. The integral

lim x   1  +  2  + ..... +  15   sin 2 x cos 2 x


x→0+   x   x   x   ∫ (sin x+cos x sin x+sin x cos x+cos x) dx
5 3 2 3 2 5 2

(1) is equal to 0. is equal to :


(2) is equal to 15.
1
(3) is equal to 120. (1) +C
3(1 + tan 3 x )
(4) does not exist (in R).
−1
(2) +C
71. Let S={t e R : f (x)=?x−π? ⋅ (e?x?−1) sin?x? 3(1 + tan 3 x )
is not differentiable at t}. Then the set S is
1
equal to : (3) +C
1 + cot 3 x
(1) φ (an empty set)
(2) {0} −1
(4) +C
(3) {π} 1 + cot 3 x
(4) {0, π} (where C is a constant of integration)

A/Page 16 SPACE FOR ROUGH WORK


π
2 77. Let y=y(x) be the solution of the
sin 2 x
75. The value of
∫ 1+2
−π2
x
dx is : differential equation

sin x
dy
+ y cosx = 4x , x  (0, π) . If
dx
π π
y   = 0 , then y   is equal to :
π 2 6
(1)
8 4
(1) π2
9 3
π −8 2
(2) (2) π
2 9 3
8 2
(3) − π
9
(3) 4π
4 2
(4) − π
9
π
(4)
4 78. A straight line through a fixed point (2, 3)
intersects the coordinate axes at distinct
points P and Q. If O is the origin and the
76. Let g(x)=cos x 2 , f (x) = x , and
rectangle OPRQ is completed, then the
α, β (α < β) be the roots of the quadratic
locus of R is :
equation 18x 2 −9πx+π 2 =0. Then the
(1) 3x+2y=6
area (in sq. units) bounded by the curve
(2) 2x+3y=xy
y=(gof )(x) and the lines x=α, x=β and
y=0, is : (3) 3x+2y=xy
(4) 3x+2y=6xy

1
(1)
2
( 3 −1 ) 79. Let the orthocentre and centroid of a
triangle be A(−3, 5) and B(3, 3)
respectively. If C is the circumcentre of
1 this triangle, then the radius of the circle
(2)
2
( 3 +1 ) having line segment AC as diameter, is :
(1) 10
1 (2) 2 10
(3)
2
( 3− 2 )
5
(3) 3
2
1 3 5
(4)
2
( 2 −1 ) (4)
2

A/Page 17 SPACE FOR ROUGH WORK


80. If the tangent at (1, 7) to the curve 83. If L1 is the line of intersection of the planes
x 2 =y−6 touches the circle 2x−2y+3z−2=0, x−y+z+1=0 and
x2+y2+16x+12y+c=0 then the value of L2 is the line of intersection of the planes
c is : x+2y−z−3=0, 3x−y+2z−1=0, then
the distance of the origin from the plane,
(1) 195
containing the lines L1 and L2, is :
(2) 185
1
(1)
(3) 85 4 2
1
(4) 95 (2)
3 2
1
(3)
81. Tangent and normal are drawn at 2 2
P(16, 16) on the parabola y2=16x, which 1
(4)
intersect the axis of the parabola at A and 2
B, respectively. If C is the centre of the
84. The length of the projection of the line
circle through the points P, A and B and
segment joining the points (5, −1, 4) and
∠CPB=θ, then a value of tan θ is :
(4, −1, 3) on the plane, x+y+z=7 is :
1 2
(1) (1)
2 3
(2) 2 2
(2)
3
(3) 3 1
(3)
4 3
(4)
3 2
(4)
3

82. Tangents are drawn to the hyperbola →


85. Let u be a vector coplanar with the vectors
4x2−y2=36 at the points P and Q. If these
→ ∧ ∧ ∧ → ∧ ∧ →
tangents intersect at the point T(0, 3) then a = 2 i + 3 j − k and b = j + k . If u
the area (in sq. units) of ∆PTQ is : → → →
is perpendicular to a and u ⋅ b = 24 ,
(1) 45 5 →2
then u is equal to :
(2) 54 3 (1) 336
(3) (2) 315
60 3
(3) 256
(4) 36 5 (4) 84

A/Page 18 SPACE FOR ROUGH WORK


86. A bag contains 4 red and 6 black balls. A 88. If sum of all the solutions of the equation
ball is drawn at random from the bag, its  π  π  1
8 cos x ⋅  cos  + x  ⋅ cos  − x  −  = 1
colour is observed and this ball along with  6  6  2
two additional balls of the same colour are in [0, π] is kπ, then k is equal to :
returned to the bag. If now a ball is drawn 2
at random from the bag, then the (1)
3
probability that this drawn ball is red, is : 13
(2)
9
3
(1) 8
10 (3)
9
2 20
(2) (4)
5 9

1 89. PQR is a triangular park with


(3) PQ=PR=200 m. A T.V. tower stands at
5
the mid-point of QR. If the angles of
3 elevation of the top of the tower at P, Q
(4)
4 and R are respectively 458, 308 and 308,
then the height of the tower (in m) is :
(1) 100
9 9 (2) 50
87. If ∑ (xi − 5) = 9 and ∑ (xi − 5)2 = 45 , (3) 100 3
i=1 i=1
(4) 50 2
then the standard deviation of the 9 items
x1, x2, ....., x9 is :
90. The Boolean expression
(1) 9 ~(p ∨ q) ∨ (~ p ∧ q) is equivalent to :
(1) ~p
(2) 4 (2) p
(3) q
(3) 2
(4) ~q
(4) 3
-o0o-
SPACE FOR ROUGH WORK

A/Page 19 SPACE FOR ROUGH WORK


Read the following instructions carefully :

1. The candidates should fill in the required particulars on the Test Booklet and Answer
Sheet (Side–1) with Black Ball Point Pen.
2. For writing/marking particulars on Side–2 of the Answer Sheet, use Black Ball Point Pen
only.
3. The candidates should not write their Roll Numbers anywhere else (except in the specified
space) on the Test Booklet/Answer Sheet.
4. Out of the four options given for each question, only one option is the correct answer.
5. For each incorrect response, ¼ (one–fourth) marks of the total marks allotted to the question
(i.e. 1 mark) will be deducted from the total score. No deduction from the total score,
however, will be made if no response is indicated for an item in the Answer Sheet.
6. Handle the Test Booklet and Answer Sheet with care, as under no circumstances (except for
discrepancy in Test Booklet Code and Answer Sheet Code), another set will be provided.
7. The candidates are not allowed to do any rough work or writing work on the Answer
Sheet. All calculations/writing work are to be done in the space provided for this purpose
in the Test Booklet itself, marked ‘Space for Rough Work’. This space is given at the
bottom of each page and in four pages (Page 20-23) at the end of the booklet.
8. On completion of the test, the candidates must hand over the Answer Sheet to the
Invigilator on duty in the Room/Hall. However, the candidates are allowed to take
away this Test Booklet with them.
9. Each candidate must show on demand his/her Admit Card to the Invigilator.
10. No candidate, without special permission of the Superintendent or Invigilator, should
leave his/her seat.
11. The candidates should not leave the Examination Hall without handing over their Answer
Sheet to the Invigilator on duty and sign the Attendance Sheet again. Cases where a
candidate has not signed the Attendance Sheet second time will be deemed not to have
handed over the Answer Sheet and dealt with as an unfair means case. The candidates
are also required to put their left hand THUMB impression in the space provided in
the Attendance Sheet.
12. Use of Electronic/Manual Calculator and any Electronic device like mobile phone, pager
etc. is prohibited.
13. The candidates are governed by all Rules and Regulations of the Examination body with
regard to their conduct in the Examination Hall. All cases of unfair means will be dealt
with as per Rules and Regulations of the Examination body.
14. No part of the Test Booklet and Answer Sheet shall be detached under any circumstances.
15. Candidates are not allowed to carry any textual material, printed or written, bits of
papers, pager, mobile phone, electronic device or any other material except the Admit
Card inside the examination room/hall.
Online Test JEE (Main) –2018/15-04-2018 (Morning)
ONLINE TEST PAPER OF JEE(MAIN) – 2018
(Held On Sunday 15th April, 2018)(Morning)
MATHEMATICS
1. The value of the integral
1 2

  2  sin x   6. Let A be a matrix such that A.   is a scalar
2
sin 4 x 1  log  0 3
    dx is :

2   2  sin x   matrix and |3A|= 108. Then A2 equals :
3 36 32   4 0
(1)  (2) 0 (1)  (2) 
8 0 4  
 32 36 
3 3
(3)  (4)  4 32   36 0 
16 4 (3)   (4)  
Ans. (1)  0 36   32 4 
2. If (P  ~q)  (p  r)  ~p  q is false, then the Ans. (1)
truth values of p,q and r are respectively 7. A box 'A' contains 2 white, 3 red and 2 black balls.
(1) T,T,T (2) F,F,F Another box 'B' contains 4 white, 2 red and 3 black
(3) T,F,T (4) F,T,F balls. If two balls are drawn at random, without
Ans. (3) replacement, from a randomly selected box and
one ball turns out to be white while the other ball
3. In a triangle ABC, coordinates of A are (1,2) and
turns out to be red, then the probability that both
the equations of the medians through B and C are
balls are drawn from box 'B' is :
respectively, x+y = 5 and x = 4. Then area of
ABC (in sq. units) is: 7 7 9 9
(1) (2) (3) (4)
(1) 12 (2) 9 (3) 4 (4) 5 16 8 16 32
Ans. (2) Ans. (1)
4. If tan A and tanB are the roots of the quadratic 8. If  is one of the angles between the normals to
equation, 3x2–10x–25=0, then the value of 3 the ellipse, x 2 +3y 2 = 9 at the points
sin2(A+B)–10 sin(A+B).cos(A+B)–25 cos2(A+B) (3 cos, 3 sin) and
is :
(1) 10 (2) –10
  2 cot 
(3sin , 3 cos );    0,  ; then is
(3) 25 (4) –25
 2 sin 2
equal to
Ans. (4)
5. An aeroplane flying at a constant speed, parallel 1 2 3
(1) (2) (3) 2 (4)
to the horizontal ground, 3 km above it, is 3 3 4
observed at an elevation of 60° from a point on Ans. (2)
the ground. If, after five seconds, its elevation 9. If a right circular cone, having maximum volume,
from the same point, is 30°, then the speed is inscribed in a sphere of radius 3cm, then the
(in km/hr) of the aeroplane, is : curved surface area (in cm2) of this cone is :
(1) 750 (2) 1440 (3) 1500 (4) 720 (1) 6 3  (2) 6 2 
Ans. (2)
(3) 8 2  (4) 8 3 
Ans. (4)
Online Test JEE (Main)–2018/15-04-2018 (Morning)

  
10. If a, b, and c are unit vectors such that e2  1 e2  1 e2  1 1
(1) (2) (3) (4)
      2e 3
e 3
2e 4
2e
a  2b  2c  0 , then a  c is equal to :
Ans. (1)
1 15 15 15 15. If R is such that the sum of the cubes of the
(1) (2) (3) (4) roots of the equation, x2+(2–)x+(10–) = 0 is
4 16 16 4
minimum, then the magnitude of the difference
Ans. (4)
of the roots of this equation is :
11. If the tangents drawn to the hyperbola 4y2=x2+1
intersect the co-ordinate axes at the distinct points (1) 4 2 (2) 20 (3) 2 7 (4) 2 5
A and B, then the locus of the mid point of AB Ans. (4)
is :
(1) 4x2 – y2 – 16x2y2 = 0 cos x x 1
(2) 4x2 – y2 + 16x2y2 = 0 2sin x x 2 2x , then lim f '(x)
16. If f(x) = x 0
(3) x2– 4y2 + 16x2y2 = 0 tan x x 1 x
(4) x2– 4y2– 16x2y2 = 0
Ans. (4) (1) exists and is equal to 0
12. Consider the following two binary relations on the (2) exists and is equal to –2
set A={a,b,c}: (3) exists and is equal to 2
R 1 ={(c,a),(b,b),(a,c),(c,c),(b,c),(a,a)}and (4) does not exist
R2 = {(a,b),(b,a),(c,c),(c,a),(a,a),(b,b),(a,c)}.Then Ans. (2)
(1) R1 is not symmetric but it is transitive.
1  (1  8)z
(2) both R1 and R2 are transitive 17. The set of all  R, for which w  is
(3) R2 is symmetric but it is not transitive 1 z
(4) both R1 and R2 are not symmetric a purely imaginary number, for all z  C satisfying
Ans. (3) |z| = 1 and Re z # 1, is :
13. A variable plane passes through a fixed (1) an ampty set (2) equal to R
point(3,2,1) and meets x,y and z axes at A,B and
 1 1
C respectively. A plane is drawn parallel to yz- (3) {0} (4) 0, ,  
plane through A, a second plane is drawn parallel  4 4
zx-plane through B a third plane is drawn parallel Ans. (3)
to xy-plane through C. Then the locus of the point 18. If b is the first term of an infinite G.P. whose sum
of intersection of these three planes, is : is five, then b lies in the interval :
(1) (– , –10] (2) (–10, 0)
1 1 1 11 x y z
(1)    (2)   1 (3) (0,10) (4) [10, )
x y z 6 3 2 1 Ans. (3)
19. An angle between the plane, x+y+z = 5 and the
3 2 1
(3)   1 (4) x + y + z = 6 line of intersection of the planes, 3x+4y+z–1=0
x y z and 5x+8y+2z+14 = 0, is
Ans. (3)
14. Let y = y(x) be the solution of the differential (1) sin
1
3/ 17  (2) cos–1  3/17 
dy
equation
dx
 2y  f (x) , where f(x) (3) sin
1
 3/17  (4) cos
1
3/ 17 
Ans. (3)
1, x[0,1] 3
= If y(0) = 0, then y   is :
0,otherwise 2
Online Test JEE (Main)–2018/15-04-2018 (Morning)

20. A circle passes through the points (2,3) and (4,5).


If its centre lies on the line, y–4x+3 = 0, then its
d2y
26. If x2+y2 + sin y = 4, then the value of at the
radius is equal to dx 2
(4) point (–2,0) is
(1) 1 (2) 2 (3) 5 2
(1) –34 (2) –32 (3) –2 (4) 4
Ans. (2) Ans. (1)
21. n-digit numbers are formed using only three digits 27. Two parabolas with a common vertex and with
2,5 and 7. The smallest value of n for which 900
axes along x-axis and y-axis, respectively,
such distinct numbers can be formed, is
intersect each other in the first quadrant. If the
(1) 9 (2) 6 (3) 8 (4) 7
length of the latus rectum of each parabola is 3,
Ans. (4)
then the equation of the common tangent to the
22. Let S be the set of all real values of k for which
two parabolas is
the system of linear equations
(1) 4(x + y) + 3 = 0 (2) 8(2x + y) + 3 = 0
x+y+z=2 (3) 3(x + y) + 4 = 0 (4) x + 2y + 3 = 0
2x+y–z=3 Ans. (1)
3x+2y+kz = 4 28. The mean of a set of 30 observations is 75. If each
has a unique solution.Then S is observation is multiplied by a non-zero number
(1) an empty set (2) equal to R  and then each of them is decreased by 25, their
(3) equal to {0} (4) equal to R–{0} mean remains the same. Then  is equal to
Ans. (4)
23. The area (in sq. units) of the region 2 10 1 4
(1) (2) (3) (4)
{x  R : x  0, y 0, y  x–2 and y  x },is 3 3 3 3
Ans. (4)
10 13 5 8
(1) (2) (3) (4) 1 1 1
3 3 3 3 If x1, x2,....,xn and , ,...,
29.
h1 h 2 h n are two A.P.s
Ans. (1)
24. Let S = {,)  R×R: f(t) = (||) e|t|–. sin(2|t|), such that x3 = h2 = 8 and x8 = h7 = 20, then x5.h10
t  R, is a differentiable function}.Then S is a equals :
subset of : (1) 2560 (2) 2650 (3) 3200 (4) 1600
(1) [0, ) × R (2) R × (– , 0) Ans. (1)
(3) R × [0, ) (4) (–, 0) × R 30. If n is the degree of the polynomial,
Ans. (3) 8
 2 
 x4  3 3  
25. If f   =2x + 1, (xR–{1,–2}), then  5x  1  5x  1 
 x2
8
 f (x)dx is equal to :(where C is a constant of  2 
 3 3  and m is the coefficient
integration)  5x  1  5x  1 
(1) 12 loge |1–x|–3x+c of xn in it, then the ordered pair (n,m) is equal to:
(2) –12 loge |1–x|–3x+c (1) (24, (10)8) (2) (12, (20)4)
(3) –12 loge |1–x|+3x+c (3) (12, 8(10)4) (4) (8, 5(10)4)
(4) 12 loge |1–x|+ 3x+c Ans. (2)
Ans. (2)
ONLINE TEST PAPER OF JEE(MAIN) – 2018
(Held On Sunday 15th April, 2018)(Evening)
MATHEMATICS
ONLINE TEST PAPER OF JEE(MAIN) – 2018
(Held On Monday 16th April, 2018) (Morning)
MATHEMATICS
–2018/16-04-2018 (Morning)
Solved---·�-.�·
Paper-2017--� 41
� � -

1 1 1 1 - 1 1 (a) 8.00 (b) 8.25


(a) - - - + -
x2 + y2 + 22 =1. x2 y2 + 22 3
(b) - (c) 9.00 (d) 8.50 (Online)

1 -- 1 - 1 1 1 1 Probability
(c) - (d) - +-+- 1
x2 + y2 + 22 =9 x2 y2 z2 =-
9 77. A box contains 15 green and 10 yellow balls. If 10 balls are
(Online) randomly drawn, one-by-one, with replacement, then the
70. If x = a, y = b, z = c is a solution of the system of linear variance of the number of green balls drawn is
equations x + Sy+ 7z = 0, 9x + 2y + 3z = 0, x + y + z = 0 (a) 6 (b) 4
such that the point (a, b, c) lies on the plane x + 2y + z = 6, 6 12
then 2a + b + c equals (c) (d)
25
(Offline)
(a) 1 5
(b) 2
(c) -1 (d) 0 (Online) 78. For three events A, B and C,
x- 3 P(Exactly one of A or B occurs)
71. If the line, + 2 2 + 'A lies in the plane,
=Y = =P(Exactly one of B or C occurs)
1 -1 -2
2x - 4y + 3z = 2, then the shortest distance between this 1
= P(Exactly one of C or A occurs) = -
. . x-1 y 2 . 4
!me and the !me, -- =-=- 1s and P(All the three events occur simultaneously)
12 9 4
(a) o (b) 3 1
(c) 1 (d) 2 = - . Then the probability that at least one of the events
(Online) 16
occurs, is
Vector Algebra
A A A - A A (a) 7 (b) 7
72. Let a= 2 i + j - 2 k and b = i + j. Let c be a vector such 16 64
c a c
that I - I= 3, I (ax b) x I= 3 and the angle between 3
(c) (d) 7
c and ax b be 30°. Then a· c is eqnal to 16 32
(Offline)

(a) 2 (b) 5 79. If two different numbers are taken from the set
25 {0, 1, 2, 3, ........, 10}; then the probability that their sum as
(c) ]_ (d) (Offline)
8 8 well as absolute difference are both multiples of 4, is
14
73. The area (in sq. units) ofthe parallelogram whose diagonals (a) 12 (b)
are along the vectors 8i - 6 j and 3i + 4 j - 12 /c, is 55 45
I\ I\ I\ I\ I\

(a) 65 (b) 52 6
(c) 26 (d) 20 (c) 7 (d) (Offline)
55
(Online)
55
74. If the vector b =3 j+ 4 Jc is written as the sum of a vector 80. Three persons, P, Q and R independently try to hit a
b1 , parallel to a =i+ J and a vector b2, perpendicular to target. If the probabilities of their hitting the target are
a, then b1 x b2 is equal to �,4 .!.2 and �8 respectively, then the probability that the
(a) 3i- 3f+9k (b) -3 i + 3j 9k
A A A

target is hit by P or Q but not by R is


(d) 6i 6j+-k 9 39 21
(a) (b)
A A A
(Online)
2 64 64
1
Statistics (c) 5 (d) 9
64 64 (Online)
75. The mean age of 25 teachers in a school is 40 years. A
teacher retires at the age of 60 years and a new teacher 81. An unbiased coin is tossed eight times. The probability of
is appointed in his place. If now the mean age of the obtaining at least one head and at least one tail is
63
teachers in this school is 39 years, then the age (in years) (a) (b) 255
of the newly appointed teacher is 64 2 6
5
(a) 25 (b) 35 127 1
(c) (d) -
(c) 30 (d) 40 (Online) 128 2 (Online)

76. The sum of 100 observations and the sum of their 82. Let E and F be two independent events. The probability
squares are 400 and 2475, respectively. Later on, three
that both E and F happeri is l. and the probability that

;i;;
observations, 3, 4 and 5 were found to be incorrect. If the 12
incorrect observations are omitted, then the variance of
the remaining observations is
neither E nor F happens is ½, then a value of is
42
~ --·-·-··-------

-1 5 90. Contrapositive of the statement 'If two numbers are not


(a) 3 (b)
12 equal, then their squares are not equal', is
3 (a) If the squares of two numbers are not equal, then the
(c) 2 (d) (Online)
3 numbers are equal.
Trigonometry (b) If the squares of two numbers are equal, then the
numbers are not equal.
83. If 5(tan x cos x) = 2cos 2x + 9, then the value of cos4x
2 2
(c) If the squares of two numbers are equal, then the
is numbers are equal.
1 2 (d) If the squares of two numbers are not equal, then the
(a) (b)
3 9 numbers are not equal. (Online)
7 3
(c) (d) (Ojftine)
9 5
X
84. Let a vertical tower AB have its end A on the level ground. 1. (b):We have j(x)=--
Let C be the mid-point of AB and P be a point on the l+x2
ground such that AP = 2AB. If LBPC = �, then tan� is 2
, x (l+x )·l-x·2x
equal to f(=
(1+X2 ) 2
)
1 2
(a) (b) l-x2 (l-x )(l+x)
4 9
=---=-----
6
(c) 4 (d) (Ojftine)
7 The sign ofj'(x) is given as

[J,5 +f� ],
9
-ve +ve -ve
85. The value of tan-1 -1
l+x2 - l-x2 Now f can be graphed as under
y 1
I x I < �-, x ;1= 0, is equal to . . . .y=2
2
n l -1 2 n -1 2l __-...,1,______,_____ x
(a) ---COS X (b) -+-cos x 0
4 2 4 2 1
......... y=-2
(c) _:: _ cos -1 x 2 (d) .:: + cos-I x2 (Online)
4 4 Clearly function is surjective but not injective, as a
86. A value of x satisfying the equation horizontal line meet the curve in two points.
sin[cot-1(1 + x)] = cos[tan-1x], is 2. (d): We have,J(g(x)) = x
(a) I. (b) o f( 3 10x - 1) = 2 10 (3 10 -x - 1) + 1 = x
2 1
⇒ 2 10.3 10X - 2 10 + 1 = X
(c) -1 (d) (Online) ⇒ x(2 103 10 1) = 2 10 - 1
2
87. The two adjacent sides of a cyclic quadrilateral are _1 2 101 2 10( -TlO)
⇒ X=--- ⇒x=----�
2 and 5 and the angle between them is 60°. If the area 2 10310 -1 2 10(310 -TIO )
of the quadrilateral is 4 ✓ , then the perimeter of the
3
1-TlO
quadrilateral is ⇒ X=---
310 -z-10
(a) 12 (b) 12.5
3. (c)
(c) 13 (d) 13.2 (Online)
4. (d): We have, z = 1 + 2(J)
Mathematical Reasoning
i.e., i'\Jr,:;
j = 1+2co
-1 +i
co= ---
:.
✓3
88. The following statement (p➔q) ➔ [ ( ~p➔q)➔q] is 2
(a) equivalent to ~p➔q Then (J) is a cube root of unity.
(b) equivalent to p➔ ~q Also, 1 + (J) + (J)2 = O
(c) a fallacy
(d) a tautology (Ojftine) 1 1

89. The proposition ( ~p) v (p I\ ~ q) is equivalent to Now 1 -co2 - 1 co2 =3k


(a) j)A~q (b) pv~q
(c) p ➔ ~ q (d) q ➔ p (Online)

Вам также может понравиться